neuro, NURIV- neuro lecture practice questions, NU310: Neurologic Alterations, Prepu Neuro, cv prep u, Ch. 67: Management of Patients with Cerebrovascular Disorders, Lacharity ch. 9, 253 - Neuro Questions (51-100 with rationales and strategies), PEDS...

Ace your homework & exams now with Quizwiz!

What is the priority nursing concern for a client experiencing a migraine headache? 1. Pain 2. Anxiety 3. Hopelessness 4. Risk for brain injury

1 The priority for interdisciplinary care for the client experiencing a migraine headache is pain management. All of the other problems are accurate, but none of them is as urgent as the issue of pain, which is often incapacitating.

The nurse is assessing the cranial nerves and begins testing the facial nerve (cranial nerve VII). Which direction should the nurse give the client to test this cranial nerve? 1. "Close your eyes and identify this smell." 2. "Follow my finger with your eyes without moving your head." 3. "Look straight ahead and let me know when you can see my finger." 4. "Raise your eyebrows, smile, and frown."

4

The nurse is assigned to care for an 8-year-old child with a diagnosis of a basilar skull fracture. The nurse reviews the health care provider's (HCP's) prescriptions and should contact the HCP to question which prescription? 1. Obtain daily weight. 2. Provide clear liquid intake. 3. Nasotracheal suction as needed. 4. Maintain a patent intravenous line.

3 A basilar skull fracture is a type of head injury. Nasotracheal suctioning is contraindicated in a child with a basilar skull fracture: Because of the nature of the injury, there is a possibility that the catheter will enter the brain through the fracture, creating a high risk of secondary infection. Fluid balance is monitored closely by daily weight determination, intake and output measurement, and serum osmolality determination to detect early signs of water retention, excessive dehydration, and states of hypertonicity or hypotonicity. The child is maintained on NPO (nothing by mouth) status or restricted to clear liquids until it is determined that vomiting will not occur. An intravenous line is maintained to administer fluids or medications, if necessary.

The nurse is reviewing the record of a child with increased intracranial pressure and notes that the child has exhibited signs of decerebrate posturing. On assessment of the child, the nurse expects to note which characteristic of this type of posturing? 1. Flaccid paralysis of all extremities 2. Adduction of the arms at the shoulders 3. Rigid extension and pronation of the arms and legs 4. Abnormal flexion of the upper extremities and extension and adduction of the lower extremities

3 Decerebrate (extension) posturing is characterized by the rigid extension and pronation of the arms and legs. Option 1 is incorrect. Options 2 and 4 describe decorticate (flexion) posturing.

The nurse is providing instructions to the parents of an infant with a ventriculoperitoneal shunt. The nurse should include which instruction? 1. Expect an increased urine output from the shunt. 2. Notify the health care provider if the infant is fussy. 3. Call the health care provider if the infant has a high-pitched cry. 4. Position the infant on the side of the shunt when the infant is put to bed.

3 If the shunt is malfunctioning, the fluid from the ventricle part of the brain will not be diverted to the peritoneal cavity. The cerebrospinal fluid will build up in the cranial area. The result is increased intracranial pressure, which then causes a high-pitched cry in the infant. The infant should not be positioned on the side of the shunt because this will cause pressure on the shunt and skin breakdown. This type of shunt affects the gastrointestinal system, not the genitourinary system, and an increased urinary output is not expected. Being fussy is a concern only if other signs indicative of a complication are occurring.

After a client has a seizure, which action can the nurse delegate to the unlicensed assistive personnel (UAP)? 1. Documenting the seizure 2. Performing neurologic checks 3. Checking the client's vital signs 4. Restraining the client for protection

3 Measurement of vital signs is within the education and scope of practice of UAPs. The nurse should perform neurologic checks and document the seizure. Clients with seizures should not be restrained; however, the nurse may guide the client's movements if necessary to prevent injury.

Which client in the neurologic intensive care unit should the charge nurse assign to an RN who has been floated from the medical unit? 1. A 26-year-old client with a basilar skull fracture who has clear drainage coming out of the nose 2. A 42-year-old client admitted several hours ago with a headache and a diagnosis of a ruptured berry aneurysm 3. A 46-year-old client who was admitted 48 hours ago with bacterial meningitis and has an antibiotic dose due 4. A 65-year-old client with an astrocytoma who has just returned to the unit after undergoing craniotomy

3 Of the clients listed, the client with bacterial meningitis is in the most stable condition and likely the least complex. An RN from the medical unit would be familiar with administering IV antibiotics. The other clients require assessments and care from RNs more experienced in caring for clients with neurologic diagnoses.

A client is having the dosage of clonazepam (Klonopin) adjusted. The nurse should plan to perform which action? 1. Weigh the client daily. 2. Monitor blood glucose levels. 3. Institute seizure precautions. 4. Observe for areas of ecchymosis.

3 Rationale: Clonazepam is a benzodiazepine that is used as an anticonvulsant. During initial therapy and during periods of dosage adjustment, the nurse should initiate seizure precautions for the client. Weight, glucose levels, and ecchymosis are unrelated to this medication.

A nurse preparing to administer carbamazepine (Tegretol) notices each of the following items on the client's breakfast tray. Which item should be a cause for concern and should be removed from the tray? 1. Carton of milk 2. Scrambled eggs 3. Grapefruit juice 4. Toast with honey

3 Rationale: Grapefruit juice can increase peak and trough levels of carbamazepine by 40%. Accordingly, clients taking the medication should be advised to avoid grapefruit juice. The other foods can be taken with this medication.

A client with Guillain-Barré syndrome (GBS) is to undergo plasmapheresis to remove circulating antibodies thought to be responsible for the disease. Which client care action should the nurse delegate to the experienced unlicensed assistive personnel (UAP)? 1. Observe the access site for ecchymosis or bleeding. 2. Instruct the client that there will be three or four treatments. 3. Weigh the client before and after the procedure. 4. Assess the access site for bruit and thrill every 2 to 4 hours.

3 The scope of practice for an experienced UAP would include weighing clients. Observing, assessing, and providing instructions all require additional educational preparation and are appropriate to the scope of practice for a professional nurse.

Which information will the nurse reinforce to the client scheduled for a lumbar puncture?

An informed consent will be required. Client preparation for lumbar puncture includes obtaining informed consent from the client. No dietary or food restrictions are required before the test. The client is told that the test will take approximately 15 to 60 minutes. The nurse needs to inform the client about the need for bed rest following the test.

A nurse is assessing a client with meningitis. The nurse places the client in a supine position and flexes the client's leg at the hip and knee. The nurse notes resistance when straightening the knee and the client reports pain. The nurse should document what neurologic sign as positive? a) Lichtheim's sign b) Kernig's sign c) Babinski's reflex d) Brudzinski's sign

B

Assessment of a client taking a nonsteroidal anti-inflammatory drug (NSAID) for pain management should include specific questions regarding which body system? a) cardiac b) gastrointestinal c) pulmonary d) renal

B

halo sign is an indication of

CSF leakage

The nurse is discussing with a parent the difference between a breath-holding spell and a seizure. The nurse would be correct in telling the parent which of the following with regards to seizures? a) Convulsive activity occurs. b) The EEG is normal. c) Cyanosis occurs at the onset of the seizure. d) The patient is bradycardiac.

Convulsive activity occurs.

widening pulse pressure and bradycardia are signs of

IICP

The nurse is observing an infant who may have acute bacterial meningitis. Which finding might the nurse look for? a) Irritability, fever, and vomiting b) Negative Kernig's sign c) Flat fontanel d) Jaundice, drowsiness, and refusal to eat

Irritability, fever, and vomiting

The nurse is caring for a client admitted with a stroke. Imaging studies indicate an embolus partially obstructing the right carotid artery. What type of stroke does the nurse know this client has?

Ischemic Ischemic strokes occur when a thrombus or embolus obstructs an artery carrying blood to the brain; about 80% of strokes are the ischemic variety. The other options are incorrect.

Any individual taking phenobarbital for a seizure disorder should be taught:

Never to discontinue the drug abruptly. Phenobarbital should always be tapered, not stopped abruptly, or seizures from the child's dependency on the drug can result.

The client with spinal cord injury suddenly experiences an episode of autonomic dysreflexia. After checking vital signs, which immediate action should the nurse take?

Raise the head of the bed and remove the noxious stimulus. Key nursing actions are to sit the client up in bed, remove the noxious stimulus, and bring the blood pressure under control with antihypertensive medication per protocol. The nurse can also clearly label the client's chart identifying the risk for autonomic dysreflexia. Client and family should be taught to recognize, and later manage, the signs and symptoms of this syndrome.

Which statement about cerebral palsy would be accurate? a) "Cerebral palsy is a condition that runs in families." b) "Cerebral palsy means there will be many disabilities." c) "Cerebral palsy is a condition that doesn't get worse." d) "Cerebral palsy occurs because of too much oxygen to the brain."

"Cerebral palsy is a condition that doesn't get worse."

Which statement about cerebral palsy would be accurate? a) "Cerebral palsy means there will be many disabilities." b) "Cerebral palsy is a condition that doesn't get worse." c) "Cerebral palsy is a condition that runs in families." d) "Cerebral palsy occurs because of too much oxygen to the brain."

"Cerebral palsy is a condition that doesn't get worse." Correct Explanation: By definition, cerebral palsy is a nonprogressive neuromuscular disorder. It can be mild or quite severe and is believed to be the result of a hypoxic event during pregnancy or the birth process and doesn't run in families.

The mother of a newborn with a caput succedaneum asks the nurse how this happened to her baby. Which response by the nurse would be most appropriate? a) "Your baby's head became blocked inside your vagina while you were pushing." b) "It's normal for this to happen, but they don't really know why." c) "The forceps used during delivery caused this to happen." d) "During delivery, your vaginal wall put pressure on the baby's head."

"During delivery, your vaginal wall put pressure on the baby's head."

The mother of a newborn with a caput succedaneum asks the nurse how this happened to her baby. Which response by the nurse would be most appropriate? a) "It's normal for this to happen, but they don't really know why." b) "During delivery, your vaginal wall put pressure on the baby's head." c) "Your baby's head became blocked inside your vagina while you were pushing." d) "The forceps used during delivery caused this to happen."

"During delivery, your vaginal wall put pressure on the baby's head." Correct Explanation: Caput succedaneum results from pressure from the uterus or vaginal wall during a head-first delivery The use of forceps is associated with a cephalohematoma. Caput succedaneum is not due to the baby's head becoming blocked inside the vagina. The cause of caput succedaneum is known; it is caused by pressure from the uterus or vaginal wall during a head-first delivery.

An otherwise healthy 18-month-old child with a history of febrile seizures is in the wellchild clinic. Which statement by the father would indicate to the nurse that additional teaching should be done? a) "I always keep phenobarbital with me in case of a fever." b) "My child will likely outgrow these seizures by age 5." c) "The most likely time for a seizure is when the fever is rising." d) "I have ibuprofen available in case it's needed."

"I always keep phenobarbital with me in case of a fever." (Phenobarbital is used for prolonged seizures or neurologic abnormalities)

A halo vest is applied to a client following a cervical spine fracture. The nurse reinforces instructions to the client regarding safety measures related to the vest. Which statement by the client indicates a need for further teaching?

"I will bend at the waist, keeping the halo vest straight to pick up items." The client with a halo vest should avoid bending at the waist because the halo vest is heavy and the client's trunk is limited in flexibility. It is helpful for the client to scan the environment visually because the client's peripheral vision is diminished from keeping the neck in a stationary position. Use of a walker and rubber-soled shoes may help prevent falls and injury, so these items are also helpful.

A child is scheduled to have an electroencephalogram (EEG). Which statement by the parent indicates understanding of the teaching? 1. "I will let my child drink cocoa as usual the morning of the procedure." 2. "I will wash my child's hair using shampoo the morning of the procedure." 3. "My child may have scalp tenderness where the electrodes were applied." 4. "My child will not remember the procedure."

2

A 1-year-old has just undergone surgery to correct craniosynostosis. Which comment is the best psychosocial intervention for the parents?

"The surgery was successful. Do you have any questions?" Often what parents need most is someone to listen to their concerns. Although this is a good time for education, let the parents adjust to their baby's appearance and adapt your teaching to their questions, comments, and knowledge level.

The nurse is providing education to the parents of a 2-year-old boy with hydrocephalus who has just had a ventriculoperitoneal shunt placed. Which information is most important for the parents to be taught? a) "Limit the amount of television he watches." b) "Call the doctor if he gets a headache." c) "Watch for changes in his behavior or eating patterns." d) "Always keep his head raised 30 degrees."

"Watch for changes in his behavior or eating patterns."

A 6-month-old is admitted with bacterial meningitis. Which action is the priority of care? 1. Administering antibiotics 2. Avoiding environmental stimuli 3. Initiating seizure precautions 4. Measuring head circumference

1

The nurse notes documentation that a child is exhibiting an inability to flex the leg when the thigh is flexed anteriorly at the hip. Which condition does the nurse suspect? 1. Meningitis 2. Spinal cord injury 3. Intracranial bleeding 4. Decreased cerebral blood flow

1 Meningitis is an infectious process of the central nervous system caused by bacteria and viruses. The inability to extend the leg when the thigh is flexed anteriorly at the hip is a positive Kernig's sign, noted in meningitis. Kernig's sign is not seen specifically with spinal cord injury, intracranial bleeding, or decreased cerebral blood flow.

A patient is in the acute phase of an ischemic stroke. How long does the nurse know that this phase may last?

1 to 3 days The acute phase of an ischemic stroke may last 1 to 3 days, but ongoing monitoring of all body systems is essential as long as the patient requires care.

All of the following nursing care activities are included in the care plan for a 78-year-old man with Parkinson disease who has been referred to the home health agency. Which activities will the nurse delegate to the unlicensed assistive personnel (UAP)? Select all that apply. 1. Checking for orthostatic changes in pulse and blood pressure 2. Assessing for improvement in tremor after levodopa is given 3. Reminding the client to allow adequate time for meals 4.Monitoring for signs of toxic reactions to anti-Parkinson medications 5. Assisting the client with prescribed strengthening exercises 6. Adapting the client's preferred activities to his level of function

1, 3, 5 UAP education and scope of practice include checking pulse and blood pressure measurements. The nurse would be sure to instruct the UAP to report heart rate and blood pressure findings. In addition, UAPs can reinforce previous teaching or skills taught by the RN or personnel in other disciplines, such as speech or physical therapists. Evaluating client response to medications and developing and individualizing the plan of care require RN-level education and scope of practice

A client is diagnosed with right-sided Bell's palsy. What instructions should the nurse give this client for care at home? Select all that apply. 1. Apply a patch to the right eye at night 2. Avoid driving 3. Chew on the left side 4. Maintain meticulous oral hygiene 5. Use a cane on the left side

1,3,4

At a well-child visit, hydrocephalus may be suspected in an infant if upon assessment the nurse finds: 1. Narrow sutures 2. Sunken fontanels 3. A rapid increase in head circumference 4. Increase in weight since last visit

A rapid increase in head circumference

The nurse is creating a plan of care for a client with dysphagia following a stroke (brain attack). Which should the nurse include in the plan? Select all that apply. 1. Thicken liquids. 2. Assist the client with eating. 3. Assess for the presence of a swallow reflex. 4. Place the food on the affected side of the mouth. 5. Provide ample time for the client to chew and swallow.

1. Thicken liquids. 2. Assist the client with eating. 3. Assess for the presence of a swallow reflex. 5. Provide ample time for the client to chew and swallow. Liquids are thickened to prevent aspiration. The nurse should assist the client with eating and place food on the unaffected side of the mouth. The nurse should assess for gag and swallowing reflexes before the client with dysphagia is started on a diet. The client should be allowed ample time to chew and swallow to prevent choking.

Cerebral palsy (CP) is suspected in a child and the parents ask the nurse about the potential warning signs of CP. The nurse should provide which information? Select all that apply. 1. The infant's arms or legs are stiff or rigid. 2. A high risk factor for CP is very low birth weight. 3. By 8 months of age, the infant can sit without support. 4. The infant has strong head control but a limp body posture. 5. The infant has feeding difficulties, such as poor sucking and swallowing. 6. If the infant is able to crawl, only one side is used to propel himself or herself.

1.2.5.6 Cerebral palsy (CP) is a term applied to a disorder that impairs movement and posture. The effects on perception, language, and intellect are determined by the type that is diagnosed. Stiff, rigid arms and legs, low birth weight, poor sucking and swallowing, and inability to crawl properly are potential warning signs of CP. By 8 months of age, if the infant cannot sit up without support, this would be considered a potential warning sign, because this developmental task should be completed by this time. The infant with a potential diagnosis of CP has poor head control by 3 months of age, when head control should be strong.

A 7-month-old infant is admitted to the unit with suspected bacterial meningitis after receiving an initial dose of antibiotics in the emergency department. Frequent assessment of which of the following is most important in the plan of care? 1. Babinski reflex 2. Fontanel assessment 3. Pulse pressure 4. Pupillary light response

2

The nurse is preparing a plan of care for a client with a diagnosis of amyotrophic lateral sclerosis (ALS). On assessment, the nurse notes that the client is severely dysphagic. Which intervention should be included in the care plan for this client? Select all that apply. 1.Provide oral hygiene after each meal. 2.Assess swallowing ability frequently. 3.Allow the client sufficient time to eat. 4. Maintain a suction machine at the bedside. 5. Provide a full liquid diet for ease in swallowing.

1234 Rationale: A client who is severely dysphagic is at risk for aspiration. Swallowing is assessed frequently. The client should be given a sufficient amount of time to eat. Semisoft foods are easiest to swallow and require less chewing. Oral hygiene is necessary after each meal. Suctioning should be available for clients who experience dysphagia and are at risk for aspiration.

The nurse has just admitted a client with bacterial meningitis who reports a severe headache with photophobia and has a temperature of 102.6°F (39.2°C) orally. Which prescribed intervention should be implemented first? 1. Administer codeine 15 mg orally for the client's headache. 2. Infuse ceftriaxone 2000 mg IV to treat the infection. 3. Give acetaminophen 650 mg orally to reduce the fever. 4. Give furosemide 40 mg IV to decrease intracranial pressure.

2 Bacterial meningitis is a medical emergency, and antibiotics are administered even before the diagnosis is confirmed (after specimens have been collected for culture). The other interventions will also help to reduce central nervous system stimulation and irritation and should be implemented as soon as possible but are not as important as starting antibiotic therapy.

A child with developmental dysplasia of the hip is placed in a Pavlik harness. The nurse should demonstrate to the parents how to place the child in this harness by placing the child's legs in which position? 1. Prone 2. Abduction 3. Adduction 4. Extension

2 The Pavlik harness consists of chest and shoulder straps and foot stirrups. The device, which is used to correct hip dislocations in infants with developmental dysplasia of the hip, consists of a set of straps that hold the hips in flexion and abduction. Therefore, the remaining options are incorrect positions.

A nurse is caring for a 3-month-old infant who has bacterial meningitis. Which clinical findings support this diagnosis? Select all that apply. 1. Depressed anterior fontanelle 2. Frequent seizures 3. High-pitched cry 4. Poor feeding 5. Presence of the Babinski sign 6. Vomiting

2,3,4,6

The nurse enters a child's room and discovers that the child is having a seizure. Which actions should the nurse take? Select all that apply. 1. Call a code. 2. Run to get the crash cart. 3. Turn the child on her side. 4. Loosen any restrictive clothing. 5. Check the child's respiratory status. 6. Place an airway into the child's mouth.

2.3.4 During a seizure the child is placed on his or her side in a lateral position. Positioning on the side will prevent aspiration because saliva will drain out the corner of the child's mouth. The child is not restrained because this could cause injury to the child. The nurse loosens clothing around the child's neck and ensures a patent airway by checking respiratory status. A code is called if the child is not breathing or the heart is not beating. There are no data in the question indicating that this is the case. The nurse stays with the child to reduce the risk of injury and to allow for observation and timing of the seizure. Nothing is placed into the child's mouth during a seizure because this could injure the child's mouth, gums, or teeth.

A client has been diagnosed with Alzheimer's disease. The nurse concludes that the client has a pathological condition of which components of the nervous system? 1.Glia 2.Peripheral nerves 3. Neuronal dendrites 4. Monoamine oxidase

3 Rationale: Alzheimer's disease is characterized by changes in the dendrites of the neurons. The decrease in the number and composition of the dendrites is responsible for the symptoms of the disease. The components in the other options are not related to the pathology of Alzheimer's disease.

The nurse is caring for a child with a head injury. The nurse observes decerebrate posturing. What is the nurse's best action? 1. Document the finding. 2. Complete a head-to-toe examination. 3. Notify the health care provider. 4. Inform the family of the improved status.

3 Decorticate posturing indicates a lesion in the cerebral hemisphere or disruption of the corticospinal tracts. Decerebrate posturing indicates damage in the diencephalon, midbrain, or pons. The progression from flexion to extension posturing usually indicates deteriorating neurological function, not improvement, and warrants physician notification. A focused neurological examination is priority at this time, not a complete head to toe.

A nurse is transcribing a prescription for antibiotic therapy for a client with bacterial meningitis. The nurse understands that the medication that will be prescribed for the client will have which characteristic? 1. Metabolizes slowly 2. Acts within minutes to hours 3. Crosses the blood-brain barrier 4. Excretes readily and easily in the urine

3 Rationale: A key consideration in prescribing medications that will affect the brain is the ability of the medication to cross the blood-brain barrier. If the medication cannot pass into the brain, it will not be effective. The duration of action, onset, and excretion (options 1, 2, and 4) apply to most medications and most uses but are not specific to those needed to treat meningitis.

An infant is brought to the child care clinic for a follow-up visit. The nurse notes that the infant is wearing this apparatus. The nurse documents that the infant is wearing which device? Refer to Figure. (Figure shows harness that is secured on chest with straps going to lower legs and feet, baby is shown with hips and knees bent) 1. A back brace for the treatment of scoliosis 2. Bilateral foot braces for the treatment of clubfoot 3. A shoulder brace for the treatment of shoulder dystocia 4. A Pavlik harness for the treatment of congenital hip dislocation

4 A Pavlik harness is a device that is used to treat congenital hip dislocation. It keeps the hips and knees flexed, the hips abducted, and the femoral head in the acetabulum. The Pavlik harness is worn continuously for 3 to 6 months. It promotes the development of muscle and cartilage, resulting in a stable hip.

A child has a right femur fracture caused by a motor vehicle crash and is placed in skin traction temporarily until surgery can be performed. During assessment, the nurse notes that the dorsalis pedis pulse is absent on the right foot. Which action should the nurse take? 1. Administer an analgesic. 2. Release the skin traction. 3. Apply ice to the extremity. 4. Notify the health care provider (HCP).

4 An absent pulse to an extremity of the affected limb after a bone fracture could mean that the child is developing or experiencing compartment syndrome. This is an emergency situation, and the HCP should be notified immediately. Administering analgesics would not improve circulation. The skin traction should not be released without an HCP's prescription. Applying ice to an extremity with absent perfusion is incorrect. Ice may be prescribed when perfusion is adequate to decrease swelling.

nurse is assessing a client with a brain injury. What is a client's cerebral perfusion pressure (CPP) when the blood pressure (BP) is 90/50 mm Hg and the intracranial pressure (ICP) is 21? Round to the nearest whole number.

42

A patient is admitted via ambulance to the emergency room of a stroke center at 1:30 p.m. with symptoms that the patient said began at 1:00 p.m. Within 1 hour, an ischemic stroke had been confirmed and the doctor ordered tPA. The nurse knows to give this drug no later than what time?

4pm Tissue plasminogen activator (tPA) must be given within 3 hours after symptom onset. Therefore, since symptom onset was 1:00 pm, the window of opportunity ends at 4:00 pm.

The nurse is caring for a client with a cerebral aneurysm who is on aneurysm precautions and is monitoring the client for signs of aneurysm rupture. The nurse understands that an early sign of rupture is which?

A decline in the level of consciousness Rupture of a cerebral aneurysm usually results in increased intracranial pressure (ICP). The first sign of pressure is a change in the level of consciousness because of compression of the reticular formation in the brain. This change in level of consciousness can be as subtle as drowsiness or restlessness. Because centers that control blood pressure are located lower in the brainstem than those that control consciousness, changes in pulse pressure are a later sign. Options 1 and 2 are not early signs of ICP. These signs may occur later if the ICP has led to neurological damage.

A patient had a carotid endarterectomy yesterday and when the nurse arrived in the room to perform an assessment, the patient states, "All of a sudden, I am having trouble moving my right side." What concern should the nurse have about this complaint?

A thrombus formation at the site of the endarterectomy Formation of a thrombus at the site of the endarterectomy is suspected if there is a sudden new onset of neurologic deficits, such as weakness on one side of the body.

A client with spinal cord injury becomes angry and belligerent whenever the nurse tries to administer care. Which is the best response by the nurse?

Acknowledge the client's anger and continue to encourage participation in care. Adjusting to paralysis is difficult both physically and psychosocially for the client and family. The nurse recognizes that the client goes through the grieving process in adjusting to the loss and may move back and forth among the stages of grief. The nurse acknowledges the client's feelings while continuing to meet the client's physical needs and encouraging independence.

A 6-year-old child with hydrocephalus had a ventriculoperitoneal (VP) shunt placed 6 weeks ago and now has experienced a seizure, vomiting, and loss of appetite. Which of the following interventions will target the child's most pressing need? a) Prepare a menu with the child's favorite foods. b) Pad and raise the rails on the child's bed. c) Administer intravenous antibiotics as ordered. d) Educate the parents about seizure precautions.

Administer intravenous antibiotics as ordered. Explanation: It is likely the child's VP shunt has become infected. Intravenous antibiotics are required. The symptoms of seizures and vomiting should diminish once the infection is brought under control. Eradicating the likely central nervous system infection takes precedence over poor appetite.

A client has a neurological deficit involving the limbic system. On assessment, which finding is specific to this type of deficit? 1. Is disoriented to person, place, and time 2. Affect is flat, with periods of emotional lability 3. Cannot recall what was eaten for breakfast today 4. Demonstrates inability to add and subtract; does not know who is the president of the United States

Affect is flat, with periods of emotional lability The limbic system is responsible for feelings (affect) and emotions. Calculation ability and knowledge of current events relate to function of the frontal lobe. The cerebral hemispheres, with specific regional functions, control orientation. Recall of recent events is controlled by the hippocampus.

Which of the following terms refer to the failure to recognize familiar objects perceived by the senses? a) Agraphia b) Perseveration c) Agnosia d) Apraxia

Agnosia Auditory agnosia is failure to recognize significance of sounds.

A client with a spinal cord injury at the level of C5 has a weakened respiratory effort and ineffective cough and is using accessory neck muscles in breathing. The nurse carefully monitors the client and suspects the presence of which problem? 1. Altered breathing pattern 2. Increased likelihood of injury 3. Ineffective oxygen consumption 4. Increased susceptibility to aspiration

Altered breathing pattern Altered breathing pattern indicates that the respiratory rate, depth, rhythm, timing, or chest wall movements are insufficient for optimal ventilation of the client. This is a risk for clients with spinal cord injury in the lower cervical area. Ineffective oxygen consumption occurs when oxygenation or carbon dioxide elimination is altered at the alveolar-capillary membrane. Increased susceptibility to aspiration and increased likelihood of injury are unrelated to the subject of the question.

The nurse is taking care of a client with a headache. In addition to administering medications, the nurse takes which measure to assist the client in reducing the pain associated with the headache?

Apply warm or cool cloths to the forehead or back of the neck. Warmth promotes vasodilation; cool stimuli reduce blood flow.

The nurse is taking care of a client with a headache. In addition to administering medications, the nurse takes measures to assist the client in reducing the pain associated with his headache. Which of the following appropriate nursing interventions may be provided by the nurse to assist this client in reducing or eliminating his pain? a) Apply warm or cool cloths to the forehead or back of the neck. b) Use pressure-relieving pads or a similar type of mattress. c) Maintain hydration by drinking eight glasses of fluid a day. d) Perform the Heimlich maneuver.

Apply warm or cool cloths to the forehead or back of the neck. Warmth promotes vasodilation; cool stimuli reduce blood flow. This will not reduce or eliminate the pain associated with this headache.

Which term refers to the inability to perform previously learned purposeful motor acts on a voluntary basis?

Apraxia Verbal apraxia refers to difficulty forming and organizing intelligible words although the musculature is intact. Agnosia is a failure to recognize familiar objects perceived by the senses. Agraphia refers to disturbances in writing intelligible words. Perseveration is the continued and automatic repetition of an activity or word or phrase that is no longer appropriate.

What information is most correct regarding the nervous system of the child? a) The child's nervous system is fully developed at birth. b) The child has underdeveloped fine motor skills and well-developed gross motor skills. c) The child has underdeveloped gross motor skills and well-developed fine motor skills. d) As the child grows, the gross and fine motor skills increase.

As the child grows, the gross and fine motor skills increase.

The eyes of a 9-year-old who suffered a head injury are crossed. Besides checking ICP, which intervention would be most important for the nurse to perform? a) Monitor core body temperature. b) Assess the child's level of consciousness. c) Pull up the side rails on the bed. d) Help the child cope with an altered appearance.

Assess the child's level of consciousness

The eyes of a 9-year-old who suffered a head injury are crossed. Besides checking ICP, which intervention would be most important for the nurse to perform?

Assess the child's level of consciousness. Decreased level of consciousness is frequently the first sign of major neurologic problems after head trauma. While body temperature is an important indicator of infection, it is not a priority here. Preventing harm by setting the side rails is more important for a seizure client. The child's eyes will correct themselves when ICP is reduced.

The nurse is caring for a client who is in the chronic phase of stroke (brain attack) and has a right-sided hemiparesis. The nurse identifies that the client is unable to feed self. Which is the appropriate nursing intervention? 1. Assist the client to eat with the left hand to build strength. 2. Provide a pureed diet that is easy for the client to swallow. 3. Inform the client that a feeding tube will be placed if progress is not made. 4. Provide a variety of foods on the meal tray to stimulate the client's appetite.

Assist the client to eat with the left hand to build strength. Right-sided hemiparesis is weakness of the right arm and leg. The nurse should teach the client to use both sides of the body to increase strength and build endurance. Providing a pureed diet is incorrect. The question does not mention swallowing difficulty, so there is no need to puree the food. Informing the client that a feeding tube may need to be placed is incorrect. That information would come from the health care provider. Providing a variety of foods is also incorrect because the problem is not the food selection but the client's ability to eat the food independently.

The nurse is caring for a child who had a seizure, fell to the ground, and hit and injured his face, head, and shoulders. This information indicates the child likely had which type of seizures? a) Myoclonic b) Atonic c) Absence d) Infantile

Atonic Atonic or akinetic seizures cause a sudden momentary loss of consciousness, muscle tone, and postural control and can cause the child to fall. They can result in serious facial, head, or shoulder injuries. In absence seizures the child loses awareness and stares straight ahead but does not fall. Myoclonic seizures are characterized by a sudden jerking of a muscle or group of muscles, often in the arms or legs, without loss of consciousness. With infantile spasms, muscle contractions are sudden, brief, symmetrical, and accompanied by rolling eyes.

A client has experienced an ischemic stroke that has damaged the temporal (lateral and superior portions) lobe. Which of the following deficits would the nurse expect during assessment of this client?

Auditory agnosia Damage to the occipital lobe can result in visual agnosia, whereas damage to the temporal lobe can cause auditory agnosia. If damage has occurred to the frontal lobe, learning capacity, memory, or other higher cortical intellectual functions may be impaired. Such dysfunction may be reflected in a limited attention span, difficulties in comprehension, forgetfulness, and lack of motivation. Damage to motor neurons may cause hemiparesis, hemaplegia, and a change in reflexes.

A client has just been diagnosed with an aneurysm. In planning discharge teaching for this client, what instructions should be delivered by the nurse to the client? a) Take opioid analgesics. b) Avoid heavy lifting. c) Take an herbal form of feverfew. d) Include peanut butter, bread, or tart foods in the diet.

Avoid heavy lifting. A client with an aneurysm should be advised to avoid heavy lifting, extreme emotional situations, or straining of stools because they may increase intracranial pressure and thereby headaches.

a client recovering from SCI has spasticity. which medication controls spasticity?

Baclofen

The nurse is reviewing the record of a client with a suspected diagnosis of Huntington's disease. The nurse should expect to note documentation of which early symptom of this disease? 1. Aphasia 2. Agnosia 3. Difficulty with swallowing 4. Balance and coordination problems

Balance and coordination problems Early symptoms of Huntington's disease include restlessness, forgetfulness, clumsiness, falls, balance and coordination problems, altered speech, and altered handwriting. Difficulty with swallowing occurs in the later stages. Aphasia and agnosia do not occur.

A child has been diagnosed with a basilar skull fracture. The nurse identifies ecchymosis behind the patient's ear. This would be documented as which of the following? a) Battle sign b) Otorrhea c) Rhinorrhea d) Raccoon eyes

Battle sign

Which of the following is the most common side effect of tissue plasminogen activator (tPA)?

Bleeding Bleeding is the most common side effect of tPA. The patient is closely monitored for bleeding (at IV insertion sites, gums, urine/stools, and intracranially by assessing changes in level of consciousness). Headache, increased ICP, and hypertension are not side effects of tPA.

The nurse in the neurological unit is monitoring a client for signs of increased intracranial pressure (ICP). The nurse reviews the assessment findings for the client and notes documentation of the presence of Cushing's reflex. The nurse determines that the presence of this reflex is obtained by assessing which item? Blood pressure 2. Motor response 3. Pupillary response 4. Level of consciousness

Blood pressure Cushing's reflex is a late sign of increased ICP and consists of a widening pulse pressure (systolic pressure rises faster than diastolic pressure) and bradycardia. Options 2, 3, and 4 are unrelated to monitoring for Cushing's reflex.

The nurse is monitoring a client with a C5 spinal cord injury for spinal shock. Which findings would be associated with spinal shock in this client? Select all that apply.

Bowel sounds are absent. The client's abdomen is distended. Respiratory excursion is diminished. Accessory muscles of respiration are areflexic. During the period of areflexia that characterizes spinal shock, the blood pressure may fall when the client sits up. The bowel and bladder often become flaccid, may become distended, and fail to empty spontaneously. Bowel sounds would be absent. Accessory muscles of respiration may become areflexic as well, diminishing respiratory excursion and oxygenation.

A mother arrives at an emergency department with her 5-year-old child and states that the child fell off a bunk bed. A head injury is suspected, and the nurse checks the child's airway status and assesses the child for early and late signs of increased intracranial pressure (ICP). Which is a late sign of increased ICP? Nausea 2. Irritability 3. Headache 4. Bradycardia

Bradycardia

The nurse assesses a child and finds that the child's pupils are pinpoint. What does this finding indicate? a) Intracranial mass b) Seizure activity c) Brain stem herniation d) Brain stem dysfunction

Brain stem dysfunction

A 73-year-old client is visiting the neurologist. The client reports light-headedness, speech disturbance, and left-sided weakness that have lasted for several hours. In the examination, an abnormal sound is auscultated in an artery leading to the brain. What is the term for the auscultated discovery? a) TIA b) Diplopia c) Atherosclerotic plaque d) Bruit

Bruit A neurologic examination during an attack reveals neurologic deficits. Auscultation of the artery may reveal a bruit (abnormal sound caused by blood flowing over a rough surface within one or both carotid arteries). The term for the auscultated discovery is bruit.

Which technique is appropriate when the nurse is irrigating an adult client's ear to move cerumen? a) After instilling the solution, pack the ear canal tightly with a cotton ball. b) Use sterile solution and equipment. c) Allow the irrigating solution to run down the wall of the ear canal. d) Make sure the irrigating solution is cool.

C

A client who has had a brain attack (stroke) is being managed on the medical nursing unit. At 0800, the client was awake and alert with vital signs of temperature 98° F orally, pulse 80 beats/min, respirations 18 breaths/min, and blood pressure 138/80 mm Hg. At noon, the client is confused and only responsive to tactile stimuli, and vital signs are temperature 99° F orally, pulse 62 beats/min, respirations 20 breaths/min, and blood pressure 166/72 mm Hg. The nurse should take which action? Reorient the client. 2. Retake the vital signs. 3. Call the health care provider (HCP). 4. Administer an antihypertensive PRN.

Call the health care provider (HCP). Rationale: The important nursing action is to call the HCP. The deterioration in neurological status, decreasing pulse, and increasing blood pressure with a widening pulse pressure all indicate that the client is experiencing increased intracranial pressure, which requires immediate treatment to prevent further complications and possible death. The nurse should retake the vital signs and reorient the client to surroundings. If the client's blood pressure falls within parameters for PRN antihypertensive medication, the medication also should be administered. However, options 1, 2, and 4 are secondary nursing actions.

The provider diagnoses the patient as having had an ischemic stroke. The etiology of an ischemic stroke would include which of the following?

Cardiogenic emboli Aneurysms, hemorrhages, and malformations are all examples of a hemorrhagic stroke. An embolism can block blood flow, leading to ischemia.

To detect complications as early as possible in a child with meningitis who's receiving I.V. fluids, monitoring for which condition should be the nurse's priority? a) Cerebral edema b) Cardiogenic shock c) Left-sided heart failure d) Renal failure

Cerebral edema

The nurse has a prescription to administer a medication to a client who is experiencing shivering as a result of hyperthermia. Which medication should the nurse anticipate to be prescribed? 1. Buspirone 2. Fluphenazine 3. Chlorpromazine 4. Prochlorperazine

Chlorpromazine Chlorpromazine is used to control shivering in hyperthermic states. It is a phenothiazine and has antiemetic and antipsychotic uses, especially when psychosis is accompanied by increased psychomotor activity. Buspirone is an anxiolytic. Prochlorperazine is a phenothiazine that is an antiemetic and antipsychotic. Fluphenazine is a phenothiazine that is used as an antipsychotic.

The nurse is in the room when a child with a seizure disorder is having a seizure. The child is having generalized jerking muscle movement, and the nurse notes the bed appears to be wet with urine. The child is in which stage of the generalized seizure? a) Clonic b) Tonic c) Prodromal d) Postictal

Clonic

The nurse is in the room when a child with a seizure disorder is having a seizure. The child is having generalized jerking muscle movement, and the nurse notes the bed appears to be wet with urine. The child is in which stage of the generalized seizure? a) Postictal b) Tonic c) Prodromal d) Clonic

Clonic Explanation: The initial rigidity of the tonic phase changes rapidly to generalized jerking muscle movements in the clonic phase. The child may bite the tongue or lose control of bladder and bowel functions. The jerking movements gradually diminish and then disappear, and the child relaxes.

The nurse is preparing to give the postcraniotomy client medication for incisional pain. The family asks the nurse why the client is receiving codeine sulfate and not "something stronger." The nurse should formulate a response based on which understanding of codeine?

Codeine does not alter respirations or mask neurological signs as do other opioids. Codeine sulfate is the opioid analgesic often used for clients after craniotomy. It is frequently combined with a nonopioid analgesic such as acetaminophen for added effect. It does not alter the respiratory rate or mask neurological signs as do other opioids. Side effects of codeine include gastrointestinal upset and constipation. The medication can lead to physical and psychological dependence with chronic use. It is not the strongest opioid analgesic available.

Antibiotic therapy to treat meningitis should be instituted immediately after which event? a) Identification of the causative organism b) Initiation of I.V. therapy c) Admission to the nursing unit d) Collection of cerebrospinal fluid (CSF) and blood for culture

Collection of cerebrospinal fluid (CSF) and blood for culture

The nurse is trying to communicate with a stroke (brain attack) client with aphasia. Which action by the nurse would be least helpful to the client?

Completing the sentences that the client cannot finish Clients with aphasia after stroke often fatigue easily and have a short attention span. General guidelines when trying to communicate with the aphasic client include speaking more slowly and allowing adequate response time, listening to and watching attempts to communicate, and trying to put the client at ease with a caring and understanding manner. The nurse should avoid shouting (because the client is not deaf), appearing impatient for a response, and completing responses for the client.

A client with Parkinson's disease "freezes" while ambulating, increasing the risk for falls. Which suggestion should the nurse include in the client's plan of care to alleviate this problem?

Consciously think about walking over imaginary lines on the floor. Clients with Parkinson's disease can develop bradykinesia (slow movement) or akinesia (freezing or no movement). Having these individuals imagine lines on the floor to step over can keep them moving forward. Although standing erect and using a cane can help prevent falls, these measures will not help a person with akinesia move forward. Clients with Parkinson's disease should walk with a wide gait, not with the feet close together. A wheelchair should be used only when the client can no longer ambulate with assistive devices such as canes or walkers.

The nurse is assessing the adaptation of a client to changes in functional status after a stroke (brain attack). Which observation indicates to the nurse that the client is adapting most successfully? 1. Gets angry with family if they interrupt a task 2. Experiences bouts of depression and irritability 3. Has difficulty with using modified feeding utensils 4. Consistently uses adaptive equipment in dressing self

Consistently uses adaptive equipment in dressing self Clients are evaluated as coping successfully with lifestyle changes after a stroke if they make appropriate lifestyle alterations, use the assistance of others, and have appropriate social interactions. Options 1 and 2 are not adaptive behaviors; option 3 indicates a not yet successful attempt to adapt.

A client with a suspected overdose of an unknown drug is admitted to the emergency department. Arterial blood gas values indicate respiratory acidosis. What should the nurse do first? a) Monitor the client's heart rhythm. b) Prepare for gastric lavage. c) Obtain a urine specimen for drug screening. d) Prepare to assist with ventilation.

D

A client with a tentative diagnosis of myasthenia gravis is admitted for a diagnostic workup. Myasthenia gravis is confirmed by: a) Kernig's sign. b) a positive sweat chloride test. c) Brudzinski's sign. d) a positive edrophonium test.

D

To assess a client's cranial nerve function, a nurse should assess: a) arm drifting. b) hand grip. c) orientation to person, time, and place. d) gag reflex.

D

An adult client with suspected meningitis has undergone lumbar puncture to obtain cerebrospinal fluid (CSF) for analysis of a bacterial infection. The nurse checks for which value indicating a bacterial infection of the CSF?

Decreased glucose level Findings that indicate a bacterial infection of the cerebrospinal fluid include presence of a bacterial organism, elevated WBC count, elevated protein level, and decreased glucose level. Red blood cells should not be present in CSF.

The nurse is preparing for the admission of a client with a suspected diagnosis of Guillain-Barré syndrome. Which sign/symptom is considered a primary symptom of this syndrome?

Development of muscle weakness A hallmark symptom of Guillain-Barré syndrome is muscle weakness that develops rapidly. The client does not have symptoms such as a fever or headache. Cerebral function, level of consciousness, and pupillary responses are normal. Seizures are not normally associated with this disorder.

The nurse is caring for a client with a history of transient ischemic attacks (TIAs) and moderate carotid stenosis who has undergone a carotid endarterectomy. Which postoperative finding would cause the nurse the most concern?

Difficulty swallowing The client's inability to swallow without difficulty would cause the nurse the most concern. Difficulty swallowing, hoarseness, or other signs of cranial nerve dysfunction must be assessed. The nurse focuses on assessment of the following cranial nerves: facial (VII), vagus (X), spinal accessory (XI), and hypoglossal (XII). Some edema in the neck after surgery is expected; however, extensive edema and hematoma formation can obstruct the airway. Emergency airway supplies, including those needed for a tracheostomy, must be available. The client's neck pain and mildly elevated blood pressure need to be addressed but would not cause the nurse the most concern. Hypotension is avoided to prevent cerebral ischemia and thrombosis. Uncontrolled hypertension may precipitate cerebral hemorrhage, edema, hemorrhage at the surgical incision, or disruption of the arterial reconstruction.

The nurse is assessing a client with a brainstem injury. In addition to obtaining the client's vital signs and determining the Glasgow Coma Scale score, what priority intervention should the nurse plan to implement? 1. Check cranial nerve functioning. 2. Determine the cause of the accident. 3. Draw blood for arterial blood gas analysis. 4. Perform a pulmonary wedge pressure measurement.

Draw blood for arterial blood gas analysis. Assessment should be specific to the area of the brain involved. The respiratory center is located in the brainstem. Assessing the respiratory status is the priority for a client with a brainstem injury. The actions in the remaining options are not priorities, although they may be a component in the assessment process, depending on the injury and client condition.

Which assessment finding should the nurse expect to note in the client hospitalized with a diagnosis of stroke who has difficulty chewing food? 1. Dysfunction of vagus nerve (cranial nerve X) 2. Dysfunction of trigeminal nerve (cranial nerve V) 3. Dysfunction of hypoglossal nerve (cranial nerve XII) 4. Dysfunction of spinal accessory nerve (cranial nerve XI)

Dysfunction of trigeminal nerve (cranial nerve V) The motor branch of cranial nerve V is responsible for the ability to chew food. The vagus nerve is active in parasympathetic functions of the autonomic nervous system. The hypoglossal nerve aids in swallowing. The spinal accessory nerve is responsible for shoulder movement, among other things.

test to find out the electrical activity of the brain

EEG

The nurse is admitting a client with Guillain-Barré syndrome to the nursing unit. The client has an ascending paralysis to the level of the waist. Knowing the complications of the disorder, the nurse should bring which items into the client's room?

Electrocardiographic monitoring electrodes and intubation tray The client with Guillain-Barré syndrome is at risk for respiratory failure because of ascending paralysis. An intubation tray should be available for use. Another complication of this syndrome is cardiac dysrhythmia, which necessitates the use of ECG monitoring. Because the client is immobilized, the nurse should routinely assess for deep vein thrombosis and pulmonary embolism.

After a stroke, a client is admitted to the facility. The client has left-sided weakness and an absent gag reflex. He's incontinent and has a tarry stool. His blood pressure is 90/50 mm Hg, and his hemoglobin is 10 g. Which nursing intervention is a priority for this client?

Elevating the head of the bed to 30 degrees Because the client's gag reflex is absent, elevating the head of the bed to 30 degrees helps minimize the client's risk of aspiration. Checking the stools, performing ROM exercises, and keeping the skin clean and dry are important, but preventing aspiration through positioning is the priority.

A client who had cranial surgery 5 days earlier to remove a brain tumor has a few cognitive deficits and does not seem to be progressing as quickly as the client or family hoped. The nurse plans to implement which approach as most helpful to the client and family at this time? Emphasize progress in a realistic manner. 2. Set high goals to give the client something to "aim for." 3. Tell the family to be extremely optimistic with the client. 4. Inform the client and family of standardized goals of care.

Emphasize progress in a realistic manner. Rationale: The most helpful approach by the nurse is to emphasize progress that is being made in a realistic manner. The nurse does not offer false hope but does provide factual information in a clear and positive manner. The nurse encourages the family to be realistic in their expectations and attitudes. The plan of care should be individualized for each client.

Which statement reflects nursing management of the client with expressive aphasia?

Encourage the client to repeat sounds of the alphabet Nursing management of the client with expressive aphasia includes encouraging the client to repeat sounds of the alphabet. Nursing management of the client with global aphasia includes speaking clearly to the client in simple sentences and using gestures or pictures when able. Nursing management of the client with receptive aphasia includes speaking slowing and clearly to assist the client in forming the sounds. Nursing management of the client with cognitive deficits, such as memory loss, includes frequently reorienting the client to time, place, and situation.

A child is diagnosed with aseptic meningitis. The child's mother states, "I don't know where she would have picked this up." The nurse prepares to respond to the mother, based on the understanding that this disorder is most likely caused by which of the following? a) Haemophilus influenza type B b) Escherichia coli c) Streptococcus group B d) Enterovirus

Enterovirus Correct Explanation: Aseptic meningitis is the most common type of meningitis, and if a causative organism can be identified, it is usually a virus such as enterovirus. E. coli is a cause of bacterial meningitis. H. influenza type B is a cause of bacterial meningitis. Streptococcus group B is a cause of bacterial meningitis.

A client with a neurological impairment experiences urinary incontinence. Which nursing action should help the client adapt to this alteration?

Establishing a toileting schedule A bladder retraining program, such as use of a toileting schedule, may be helpful to clients experiencing urinary incontinence. A Foley catheter should be used only when necessary because of risk of infection. Use of diapers or pads is the least acceptable alternative because the risk of skin breakdown exists.

A client with a neurological impairment experiences urinary incontinence. Which nursing action would be most helpful in assisting the client to adapt to this alteration? 1. Using adult diapers 2. Inserting a Foley catheter 3. Establishing a toileting schedule 4. Padding the bed with an absorbent cotton pad

Establishing a toileting schedule A bladder retraining program, such as use of a toileting schedule, may be helpful to clients experiencing urinary incontinence. A Foley catheter should be used only when necessary because of the associated risk of infection. Use of diapers or pads is the least acceptable alternative because of the risk of skin breakdown.

The nurse is caring for a client with aphasia. Which strategy will the nurse use to facilitate communication with the client?

Establishing eye contact The following strategies should be used by the nurse to encourage communication with a client with aphasia: face the client and establish eye contact, speak in your usual manner and tone, use short phrases, and pause between phrases to allow the client time to understand what is being said; limit conversation to practical and concrete matters; use gestures, pictures, objects, and writing; and as the client uses and handles an object, say what the object is. It helps to match the words with the object or action. Be consistent in using the same words and gestures each time you give instructions or ask a question, and keep extraneous noises and sounds to a minimum. Too much background noise can distract the client or make it difficult to sort out the message being spoken.

The nurse is assessing the function of cranial nerve XII in a client who sustained a stroke. To assess function of this nerve, which action should the nurse ask the client to perform? 1. Extend the arms. 2. Extend the tongue. 3. Turn the head toward the nurse's arm. 4. Focus the eyes on the object held by the nurse.

Extend the tongue. Impairment of cranial nerve XII can occur with a stroke. To assess the function of cranial nerve XII (the hypoglossal nerve), the nurse would assess the client's ability to extend the tongue. The maneuvers noted in the remaining options do not test the function of cranial nerve XII.

A doctor orders the placement of an ICP monitor in a patient with cerebral edema. The nurse is aware that this surgery will take place in the infratentorial region of the brain. a) True b) False

False Explanation: Surgical procedures in the infratentorial region are usually indicated for tumor or cyst resection. Most surgical procedures in the supratentorial region of the brain are indicated for resection of epileptogenic cortex (seizure foci), placement of ventricular catheters to drain CSF, draining collected blood following head injury, placement of ICP monitors, and also resection or biopsy of tumors or cysts

The nurse is evaluating the neurological signs of a client in spinal shock following spinal cord injury. Which observation indicates that spinal shock persists? 1. Hyperreflexia 2. Positive reflexes 3. Flaccid paralysis 4. Reflex emptying of the bladder

Flaccid paralysis Resolution of spinal shock is occurring when there is return of reflexes (especially flexors to noxious cutaneous stimuli), a state of hyperreflexia rather than flaccidity, and reflex emptying of the bladder.

During assessment of cognitive impairment, post-stroke, the nurse documents that the patient was experiencing memory loss and impaired learning capacity. The nurse knows that brain damage has most likely occurred in which lobe?

Frontal Frontal lobe damage results in impaired learning capacity, memory, and other higher cortical intellectual functions.

A stroke victim is experiencing memory loss and impaired learning capacity. The nurse knows that brain damage has most likely occurred in which lobe?

Frontal If damage has occurred to the frontal lobe, learning capacity, memory, or other higher cortical intellectual functions may be impaired. Such dysfunction may be reflected in a limited attention span, difficulties in comprehension, forgetfulness, and a lack of motivation.

A client with a stroke (brain attack) has residual dysphagia. When a diet prescription is initiated, the should nurse avoid which action?

Giving the client thin liquids Before the client with dysphagia is started on a diet, the gag and swallow reflexes must have returned. The client is assisted with meals as needed and is given ample time to chew and swallow. Food is placed on the unaffected side of the mouth. Liquids are thickened to avoid aspiration.

A nurse is caring for a child who sustained a head injury after falling from a tree. On assessment of the child, the nurse notes the presence of a watery discharge from the child's nose. The nurse should immediately test the discharge for the presence of which substance? Protein 2. Glucose 3. Neutrophils 4. White blood cells

Glucose

The nurse is caring for the client who suffered a spinal cord injury 48 hours ago. What should the nurse assess for when monitoring for gastrointestinal complications? 1. A history of diarrhea 2. A flattened abdomen 3. Hyperactive bowel sounds 4. Hematest-positive nasogastric tube drainage

Hematest-positive nasogastric tube drainage Development of a stress ulcer also can occur after spinal cord injury and can be detected by Hematest-positive nasogastric tube aspirate or stool. The client is also at risk for paralytic ileus, which is characterized by the absence of bowel sounds and abdominal distention. A history of diarrhea is irrelevant.

A client undergoes cerebral angiography for evaluation after an intracranial computed tomography scan revealed a subarachnoid hemorrhage. Afterward, the nurse checks frequently for signs and symptoms of complications associated with this procedure. Which findings indicate spasm or occlusion of a cerebral vessel by a clot? a) Tachycardia, tachypnea, and hypotension b) Nausea, vomiting, and profuse sweating c) Hemiplegia, seizures, and decreased level of consciousness (LOC) d) Difficulty breathing or swallowing

Hemiplegia, seizures, and decreased level of consciousness (LOC) Spasm or occlusion of a cerebral vessel by a clot causes signs and symptoms similar to those of a stroke: hemiplegia, seizures, decreased LOC, aphasia, hemiparesis, and increased focal symptoms.

During physical assessment of a 2-month-old infant, the nurse suspects the child may have a lesion on the brain stem. Which symptom was observed? a) Sudden increase in head circumference b) Closed posterior fontanel c) Only one eye is dilated and reactive d) Horizontal nystagmus

Horizontal nystagmus

A physician orders several drugs for a client with hemorrhagic stroke. Which drug order should the nurse question? a) Dexamethasone (Decadron) b) Heparin sodium c) Phenytoin (Dilantin) d) Methyldopa (Aldomet)

Heparin sodium Administering heparin, an anticoagulant, could increase the bleeding associated with hemorrhagic stroke. Therefore, the nurse should question this order to prevent additional hemorrhage in the brain

The nurse practitioner advises a patient who is at high risk for a stroke to be vigilant in his medication regimen, to maintain a healthy weight, and to adopt a reasonable exercise program. This advice is based on research data that shows the most important risk factor for stroke is:

Hypertension Hypertension is the most modifiable risk factor for either ischemic or hemorrhagic stroke. Unfortunately, it remains under-recognized and undertreated in most communities.

A client with Parkinson's disease is experiencing a parkinsonian crisis. The nurse should immediately place the client where?

In a quiet, dim room with respiratory and cardiac support available Parkinsonian crisis can occur with emotional trauma or sudden withdrawal of medications. The client exhibits severe tremors, rigidity, and bradykinesia. The client also displays anxiety, is diaphoretic, and has tachycardia and hyperpnea. The client should be placed in a quiet, dim room, and respiratory and cardiac support should be available.

Preterm infants have more fragile capillaries in the periventricular area than term infants. This put these infants at risk for which problem? a) Congenital hydrocephalus b) Early closure of the fontanels c) Moderate closed-head injury d) Intracranial hemorrhaging

Intracranial hemorrhaging

A nurse is caring for an older client who has had a hemorrhagic stroke. The client has exhibited impulsive behavior and, despite reminders from the nurse, doesn't recognize his limitations. Which priority measure should the nurse implement to prevent injury? a) Install a bed alarm to remind the client to ask for assistance and to alert staff that the client is getting out of bed. b) Encourage the client to do as much as possible without assistance, and to use the call light only in emergencies. c) Encourage the family to reprimand the client if he doesn't ask for help with transfers and mobility. d) Ask a physician to order a vest and wrist restraints.

Install a bed alarm to remind the client to ask for assistance and to alert staff that the client is getting out of bed. The bed alarm will alert staff that the client is attempting to transfer, so they can come to assist.

Preterm infants have more fragile capillaries in the periventricular area than term infants. This put these infants at risk for which problem?

Intracranial hemorrhaging Fragile capillaries in the periventricular area of the brain put preterm infants at risk for intracranial hemorrhage. Closure of the fontanels has nothing to do with fragile capillaries within the brain. Larger head size gives children a higher center of gravity which causes them to hit their head more readily. Congenital hydrocephalus may be caused by abnormal intrauterine development or infection

The nurse is performing an assessment on a client with the diagnosis of Brown-Séquard syndrome. The nurse would expect to note which assessment finding? 1. Bilateral loss of pain and temperature sensation 2. Ipsilateral paralysis and loss of touch and vibration 3. Contralateral paralysis and loss of touch, pressure, and vibration 4. Complete paraplegia or quadriplegia, depending on the level of injury

Ipsilateral paralysis and loss of touch and vibration Brown-Séquard syndrome results from hemisection of the spinal cord, resulting in ipsilateral paralysis and loss of touch, pressure, vibration, and proprioception. Contralaterally, pain and temperature sensation are lost because these fibers decussate after entering the cord. The remaining options are not assessment findings in this syndrome.

The nurse is caring for a client with a diagnosis of right (nondominant) hemispheric stroke. The nurse notes that the client is alert and oriented to time and place. On the basis of these assessment findings, the nurse should make which interpretation? Had a very mild stroke 2. Most likely suffered a transient ischemic attack 3. May have difficulty with language abilities only 4. Is likely to have perceptual and spatial disabilities

Is likely to have perceptual and spatial disabilities The client with a right (nondominant) hemispheric stroke may be alert and oriented to time and place. These signs of apparent wellness often suggest that the client is less disabled than is the case. However, impulsivity and confusion in carrying out activities may be very real problems for these clients as a result of perceptual and spatial disabilities. The right hemisphere is considered specialized in sensory-perceptual and visual-spatial processing and awareness of body space. The left hemisphere is dominant for language abilities

Which is the initial diagnostic test for a stroke?

Noncontrast computed tomography The initial diagnostic test for a stroke is nonconstrast computed tomography performed emergently to determine whether the event is ischemic or hemorrhagic. Further diagnostics include a carotid Doppler, electrocardiogram, and transcranial Doppler.

The nurse is planning discharge teaching for a client started on acetazolamide for a supratentorial lesion. Which information about the primary action of the medication should be included in the client's education? 1. It will prevent hypertension. 2. It will prevent hyperthermia. 3. It decreases cerebrospinal fluid production. 4. It maintains adequate blood pressure for cerebral perfusion.

It decreases cerebrospinal fluid production. Acetazolamide is a carbonic anhydrase inhibitor and a diuretic. It is used in the client with or at risk for increased intracranial pressure to decrease cerebrospinal fluid production. The remaining options are not actions of this medication.

Members of the family of an unconscious client with increased intracranial pressure are talking at the client's bedside. They are discussing the client's condition and wondering whether the client will ever recover. The nurse intervenes on the basis of which interpretation? 1. It is possible the client can hear the family. 2. The family needs immediate crisis intervention. 3. The client might have wanted a visit from the hospital chaplain. 4. The family could benefit from a conference with the health care provider.

It is possible the client can hear the family. Some clients who have awakened from an unconscious state have remembered hearing specific voices and conversations. Family and staff should assume that the client's sense of hearing is intact and act accordingly. In addition, positive outcomes are associated with coma stimulation-that is, speaking to and touching the client. The remaining options are incorrect interpretations.

A client has a high level of carbon dioxide (CO2) in the bloodstream, as measured by arterial blood gases. The nurse anticipates that which underlying pathophysiology can occur as a result of this elevated CO2? 1. It will cause arteriovenous shunting. 2. It will cause vasodilation of blood vessels in the brain. 3. It will cause blood vessels in the circle of Willis to collapse. 4. It will cause hyperresponsiveness of blood vessels in the brain

It will cause vasodilation of blood vessels in the brain. CO2 is one of the metabolic end products that can alter the tone of the blood vessels in the brain. High CO2 levels cause vasodilation, which may cause headache, whereas low CO2 levels cause vasoconstriction, which may cause lightheadedness. The statements included in the other options are incorrect effects.

What clinical manifestations does the nurse recognize when a patient has had a right hemispheric stroke?

Left visual field deficit A left visual field deficit is a common clinical manifestation of a right hemispheric stroke. Aphasia, slow, cautious behavior, and altered intellectual ability are all clinical manifestations of a left hemispheric stroke.

The nurse is assisting in caring for a client with a supratentorial lesion. The nurse monitors which criterion as the critical index of central nervous system (CNS) dysfunction?

Level of consciousness Level of consciousness is the most critical index of CNS dysfunction. Changes in level of consciousness can indicate clinical improvement or deterioration. Although blood pressure, temperature, and ability to speak may be components of the assessment, the client level of consciousness is the most critical index of CNS dysfunction.

The nurse is performing an assessment on a client with Guillain-Barré syndrome. The nurse determines that which finding would be of most concern? 1. Difficulty articulating words 2. Lung vital capacity of 10 mL/kg 3. Paralysis progressing from the toes to the waist 4. A blood pressure (BP) decrease from 110/78 mm Hg to 102/70 mm Hg

Lung vital capacity of 10 mL/kg Respiratory compromise is a major concern in clients with Guillain-Barré syndrome. Clients often are intubated and mechanically ventilated when the vital capacity is less than 15 mL/kg. Difficulty articulating words and paralysis progressing from the toes to the waist are expected, depending on the degree of paralysis that occurs. Although orthostatic hypotension is a problem with these clients, the BP drop in option 4 is less than 10 mm Hg and is not significant.

The nurse is preparing a toddler for a lumbar puncture. For this procedure, the nurse should place the child in which position? a) Lying prone, with the feet higher than the head b) Lying on one side, with the back curved c) Lying prone, with the neck flexed d) Sitting up, with the back straight

Lying on one side, with the back curved

A client who complains of recurring headaches, accompanied by increased irritability, photophobia, and fatigue is asked to track the headache symptoms and occurrence on a calendar log. Which is the best nursing rationale for this action?

Migraines often coincide with menstrual cycle. Changes in reproductive hormones as found during menstrual cycle can be a trigger for migraine headaches and may assist in the management of the symptoms. Cluster headaches can cause severe pain but is not the reason for tracking. Tension headaches can be managed but is not associated with a monthly calendar. Headaches are common but not the reason for tracking.

A thymectomy via a median sternotomy approach is performed on a client with a diagnosis of myasthenia gravis. The nurse has assisted in developing a plan of care for the client and includes which nursing action in the plan?

Monitor the chest tube drainage. A thymectomy may be performed to improve the condition in clients with myasthenia gravis. The procedure is performed by a median sternotomy or a transcervical approach. Postoperatively, the client will have a chest tube in the mediastinum. Pain medication is administered as needed, but the client is monitored closely for respiratory depression. Lactated Ringer's intravenous solutions are usually avoided because they can increase weakness. There is no reason to restrict visitors.

The nurse develops a plan of care for a client following a lumbar puncture. Which interventions should be included in the plan? Select all that apply.

Monitor the client's ability to void. Maintain the client in a flat position. Monitor the client's ability to move the extremities. Inspect the puncture site for swelling, redness, and drainage. Following a lumbar puncture, the client remains flat in bed for 6 to 24 hours, depending on the health care provider's prescriptions. A liberal fluid intake (not NPO status) is encouraged to replace cerebrospinal fluid removed during the procedure, unless contraindicated by the client's condition. The nurse checks the puncture site for redness and drainage, and monitors the client's ability to void and move the extremities.

The nurse is planning care for a client in spinal shock. Which action would be least helpful in minimizing the effects of vasodilation below the level of the injury?

Moving the client quickly as one unit Reflex vasodilation below the level of spinal cord injury places the client at risk of orthostatic hypotension, which may be profound. Actions to minimize this include measuring vital signs before and during position changes, use of a tilt table in early mobilization, and changing the client's position slowly. Venous pooling can be reduced by using compression stockings, if prescribed. Vasopressor medications are used as per protocol and as prescribed.

The nurse caring for an infant with craniosynostosis, specifically positional plagiocephaly, should prioritize which activity? a) Massaging the scalp gently every 4 hours b) Giving the infant small feedings whenever he is fussy c) Moving the infant's head every 2 hours d) Measuring the intake and output every shift

Moving the infant's head every 2 hours

The nurse is assessing the motor function of an unconscious client. The nurse should plan to use which technique to test the client's peripheral response to pain? Sternal rub 2. Nail bed pressure 3. Pressure on the orbital rim 4. Squeezing of the sternocleidomastoid muscle

Nail bed pressure

The nurse is assessing the motor and sensory function of an unconscious client. The nurse should use which technique to test the client's peripheral response to pain? 1. Sternal rub 2. Nail bed pressure 3. Pressure on the orbital rim 4. Squeezing of the sternocleidomastoid muscle

Nail bed pressure Nail bed pressure tests a basic motor and sensory peripheral response. Cerebral responses to pain are tested using a sternal rub, placing upward pressure on the orbital rim, or squeezing the clavicle or sternocleidomastoid muscle.

The nurse is monitoring a client who has returned to the nursing unit after a myelogram. Which client complaint would indicate the need to notify the health care provider (HCP)? 1. Backache 2. Headache 3. Neck stiffness 4. Feelings of fatigue

Neck stiffness Headache is relatively common after the procedure, but neck stiffness, especially on flexion, and pain should be reported because they signal meningeal irritation. The client also is monitored for evidence of allergic reactions to the dye such as confusion, dizziness, tremors, and hallucinations. Feelings of fatigue may be normal, and back discomfort may occur because of the positions required for the procedure.

The nurse is caring for a client diagnosed with Alzheimer's disease. The nurse should anticipate that the client has changes in which component of the nervous system? 1. Glia 2. Peripheral nerves 3. Neuronal dendrites 4. Monoamine oxidase

Neuronal dendrites Alzheimer's disease is characterized by changes in the dendrites of the neurons. The decrease in the number and composition of the dendrites is responsible for the symptoms of the disease. The components in the other options are not related to the pathology of Alzheimer's disease.

Which of the following is the initial diagnostic in suspected stroke? a) Cerebral angiography b) Magnetic resonance imaging (MRI) c) Noncontrast computed tomography (CT) d) CT with contrast

Noncontrast computed tomography (CT) An initial head CT scan will determine whether or not the patient is experiencing a hemorrhagic stroke. An ischemic infarction will not be readily visible on initial CT scan if it is performed within the first few hours after symptoms onset; however, evidence of bleeding will almost always be visible.

The nurse caring for a client with a head injury is monitoring for signs of increased intracranial pressure. The nurse reviews the record and notes that the intracranial pressure (cerebrospinal fluid) is averaging 8 mm Hg. The nurse plans care, knowing that these results are indicative of which condition? 1. Normal condition 2. Increased pressure 3. Borderline situation 4. Compensating condition

Normal condition Rationale: The normal intracranial pressure is 5 to 10 mm Hg. A pressure of 8 mm Hg is within normal range. (This is what Saunders says, however we are learning it as 5-15 is normal)

The nurse is assessing the nasal dressing on a client who had a transsphenoidal resection of the pituitary gland. The nurse notes a small amount of serosanguineous drainage that is surrounded by clear fluid on the nasal dressing. Which nursing action is most appropriate? Document the findings. 2. Reinforce the dressing. 3. Notify the health care provider (HCP). 4. Mark the area of drainage with a pen and monitor for further drainage.

Notify the health care provider (HCP) Rationale: Cerebrospinal fluid (CSF) leakage after cranial surgery may be detected by noting drainage that is serosanguineous surrounded by an area of straw-colored or pale drainage. The physical appearance of CSF drainage is that of a halo. If the nurse notes the presence of this type of drainage, the HCP needs to be notified. Options 1, 2, and 4 are inappropriate nursing actions.

A client with multiple sclerosis tells a home health care nurse that she is having increasing difficulty in transferring from the bed to a chair. What is the initial nursing action? 1. Observe the client demonstrating the transfer technique. 2. Start a restorative nursing program before an injury occurs. 3. Seize the opportunity to discuss potential nursing home placement. 4. Determine the number of falls that the client has had in recent weeks.

Observe the client demonstrating the transfer technique. Observation of the client's transfer technique is the initial intervention. Starting a restorative program is important but not unless an assessment has been completed first. Discussing nursing home placement would be inappropriate in view of the information provided in the question. Determining the number of falls is another important intervention, but observing the transfer technique should be done first.

The nurse is caring for a client with bacterial meningitis. The nurse should anticipate that an antibiotic with which characteristics will be prescribed for the client? 1. One that has a long half-life 2. One that acts within minutes to hours 3. One that can be easily excreted in the urine 4. One that is able to cross the blood-brain barrier

One that is able to cross the blood-brain barrier A primary consideration regarding medications to treat bacterial meningitis is the ability of the medication to cross the blood-brain barrier. If the medication cannot cross, it will not be effective. The duration, onset, and excretion of the medication are also of general concern but apply to all medications and not specifically to those that are used to treat meningitis.

A nurse demonstrates understanding of the various levels of consciousness as they progress from most alert to least alert. Place the levels of consciousness in the order that reflects this progression.

Oriented to person, place, and time Disorientation Obtundation Stupor Coma

Question: A nurse demonstrates understanding of the various levels of consciousness as they progress from most alert to least alert. Place the levels of consciousness in the order that reflects this progression. 1 Oriented to person, place, and time 2 Stupor 3 Disorientation 4 Obtundation 5 Coma

Oriented to person, place, and time Disorientation Obtundation Stupor Coma

A client with a traumatic brain injury is on mechanical ventilation. The nurse promotes normal intracranial pressure (ICP) by ensuring that the client's arterial blood gas (ABG) results are within which ranges? 1. PaO2 60 to 100 mm Hg (60 to 100 mm Hg), PaCo2 25 to 30 mm Hg (25 to 30 mm Hg) 2. PaO2 60 to 100 mm Hg (60 to 100 mm Hg), PaCo2 30 to 35 mm Hg (30 to 35 mm Hg) 3. PaO2 80 to 100 mm Hg (80 to 100 mm Hg), PaCo2 25 to 30 mm Hg (25 to 30 mm Hg) 4. PaO2 80 to 100 mm Hg (80 to 100 mm Hg), PaCo2 35 to 38 mm Hg (35 to 38 mm Hg)

PaO2 80 to 100 mm Hg (80 to 100 mm Hg), PaCo2 35 to 38 mm Hg (35 to 38 mm Hg) The goal is to maintain the partial pressure of arterial carbon dioxide (PaCo2) at 35 to 38 mm Hg (35 to 38 mm Hg). Carbon dioxide is a very potent vasodilator that can contribute to increases in ICP. The PaO2 is not allowed to fall below 80 mm Hg (80 mm Hg), to prevent cerebral vasodilation from hypoxemia, which can also result in an increase in ICP. Therefore, the remaining options are incorrect.

A client with a traumatic brain injury is able, with eyes closed, to identify a set of keys placed in his or her hands. On the basis of this assessment finding, the nurse determines that there is appropriate function of which lobe of the brain? 1. Frontal 2. Parietal 3. Occipital 4. Temporal

Parietal The ability to distinguish an object by touch is called stereognosis, which is a function of the right parietal area. The parietal lobe of the brain is responsible for spatial orientation and awareness of sizes and shapes. The left parietal area is responsible for mathematics and right-left orientation. The other lobes of the brain are not responsible for this function.

The nurse is assisting in checking for Tinel's sign in a client suspected of having carpal tunnel syndrome (CTS). Which technique should the nurse expect to be used to elicit this sign?

Percuss the medial nerve at the wrist as it enters the carpal tunnel, and monitor for tingling sensations. The presence of Tinel's sign is determined by percussing the medial nerve at the wrist as it enters the carpal tunnel. A tingling sensation over the distribution of the nerve occurs in CTS. The presence of Phalen's sign is determined by asking the client to flex the wrist at a 90-degree angle for 1 minute. Numbness and tingling over the distribution of the median nerve, the palmar surface of the thumb, and the index and middle fingers suggest CTS. Phalen's sign is also an indication of CTS. Options 1 and 2 are incorrect.

Which intervention prevents a 17-month-old child with spastic cerebral palsy from going into a scissoring position? a) Keeping the child in leg braces 23 hours per day b) Letting the child lie down as much as possible c) Trying to keep the child as quiet as possible d) Placing the child on your hip

Placing the child on your hip

The nurse develops a plan of care for a client with a brain aneurysm who will be placed on aneurysm precautions. Which interventions should be included in the plan? Select all that apply.

Place a blood pressure cuff at the client's bedside. 3. Close the shades in the client's room during the day.

A nurse is caring for a newborn with anencephaly. Which of the following interventions will the nurse use? a) Place a cap or similar covering on the infant's head. b) Closely monitor neurologic status. c) Monitor for increased intracranial pressure (ICP). d) Refer the family to an agency to assist with long-term care.

Place a cap or similar covering on the infant's head.

The nurse is caring for a client who is brought to the hospital emergency department with a spinal cord injury. The nurse minimizes the risk of compounding the injury by performing which action? 1. Keeping the client on a stretcher 2. Logrolling the client onto a soft mattress 3. Logrolling the client onto a firm mattress 4. Placing the client on a bed that provides spinal immobilization

Placing the client on a bed that provides spinal immobilization Spinal immobilization is necessary after spinal cord injury to prevent further damage and insult to the spinal cord. Whenever possible, the client is placed on a special bed, such as a Stryker frame, which allows the nurse to turn the client to prevent complications of immobility while maintaining alignment of the spine. If a Stryker frame is not available, a firm mattress with a bed board under it should be used. The remaining options are incorrect and potentially harmful interventions.

most common type of hematoma seen in elderly a. cerebral aneurysm b. chronic subdural hematoma c. intracranial hematoma d. acute subarachnoid hemorrhage

b. chronic subdural hematoma

when CT is preformed over MRI in a client with spinal tumor? a. fear of closed space b. patient with mechanical heart valve c. patient with IICP d. patients with increased BP

b. patient with mechanical heart valve

An adult client had a cerebrospinal fluid (CSF) analysis after lumbar puncture. The nurse interprets that a negative value of which is consistent with normal findings?

Red blood cells The adult with normal cerebrospinal fluid has no red blood cells in the CSF. The client may have small levels of white blood cells (0 to 3 per mm3). Protein (15 to 45 mg/dL) and glucose (40 to 80 mg/dL) are normally present in CSF.

The client is admitted to the hospital with a diagnosis of Guillain-Barré syndrome. Which past medical history finding makes the client most at risk for this disease? 1. Meningitis or encephalitis during the last 5 years 2. Seizures or trauma to the brain within the last year 3. Back injury or trauma to the spinal cord during the last 2 years 4. Respiratory or gastrointestinal infection during the previous month

Respiratory or gastrointestinal infection during the previous month Guillain-Barré syndrome is a clinical syndrome of unknown origin that involves cranial and peripheral nerves. Many clients report a history of respiratory or gastrointestinal infection in the 1 to 4 weeks before the onset of neurological deficits. On occasion, the syndrome can be triggered by vaccination or surgery.

is eye swelling a normal finding after craniotomy?

yes for 2-3 days

A patient is exhibiting classic signs of a hemorrhagic stroke. What complaint from the patient would be an indicator of this type of stroke? a) Dizziness and tinnitus b) Numbness of an arm or leg c) Severe headache d) Double vision

Severe headache The patient with a hemorrhagic stroke can present with a wide variety of neurologic deficits, similar to the patient with ischemic stroke. The conscious patient most commonly reports a severe headache.

The nurse is documenting nursing observations in the record of a client who experienced a tonic-clonic seizure. Which clinical manifestation did the nurse most likely note in the clonic phase of the seizure? Body stiffening 2. Spasms of the entire body 3. Sudden loss of consciousness 4. Brief flexion of the extremities

Spasms of the entire body

During the trial period to determine the efficacy of an anticonvulsant drug, which caution should be explained to the parents? a) The child shouldn't participate in activities that could be hazardous if a seizure occurs b) Plasma levels of the drug will be monitored on a daily basis c) Drug dosage will be adjusted depending on the frequency of seizure activity d) The drug must be discontinued immediately if even the slightest problem occurs

The child shouldn't participate in activities that could be hazardous if a seizure occurs

When compared with adults, why are infants and children at an increased risk of head trauma? 1. The head of the infant and young child is large in proportion to the body and the neck muscles are not well developed. 2. The development of the nervous system is complete at birth but remains immature. 3. The spine is very immobile in infants and young children. 4. The skull is more flexible due to the presence of sutures and fontanels.

The head of the infant and young child is large in proportion to the body and the neck muscles are not well developed.

The nurse is caring for a client with a head injury. The client's intracranial pressure reading is 8 mm Hg. Which condition should the nurse document? 1. The intracranial pressure reading is normal. 2. The intracranial pressure reading is elevated. 3. The intracranial pressure reading is borderline. 4. An intracranial pressure reading of 8 mm Hg is low.

The intracranial pressure reading is normal. The normal intracranial pressure is 5 to 15 mm Hg. A pressure of 8 mm Hg is within normal range.

After a difficult birth, the nurse observes that a newborn has swelling on part of his head. The nurse suspects caput succedaneum based on what evidence? a) The swelling crosses the midline of the infant's scalp. b) The infant had low-set ears and facial abnormalities. c) The swelling is limited to one small area without crossing the sagittal suture. d) The infant had a low birthweight when born at term.

The swelling crosses the midline of the infant's scalp.

After a difficult birth, the nurse observes that a newborn has swelling on part of his head. The nurse suspects caput succedaneum based on what evidence?

The swelling crosses the midline of the infant's scalp. The fact that the swelling crosses the midline of the infant's scalp indicates caput succedaneum. If the swelling is limited and does not cross the midline or suture lines, it would suggest cephalohematoma. Low birthweight does not suggest caput succedaneum. Low-set ears may be seen in infants with chromosomal abnormalities. Facial abnormalities may accompany encephalocele

The treatment for children with seizures disorders is most often which of the following? a) Restricted fat diet b) Use of anticonvulsant medications c) Surgical intervention d) Strict exercise regimen

Use of anticonvulsant medications

The nurse is assessing a toddler for motor function. Which of the following activities will be most valuable? a) Watch the child reach for a toy. b) Give the child some potato chips. c) Have the child catch a ball. d) Let the child look at a picture book.

Watch the child reach for a toy.

The nurse is planning care for the client with hemiparesis of the right arm and leg. Where should the nurse plan to place objects needed by the client?

Within the client's reach, on the left side Hemiparesis is a weakness of the face, arm, and leg on one side. The client with one-sided hemiparesis benefits from having objects placed on the unaffected side and within reach. Other helpful activities with hemiparesis include range-of-motion exercises to the affected side and muscle-strengthening exercises to the unaffected side.

A client diagnosed with a stroke is ordered to receive warfarin. Later, the nurse learns that the warfarin is contraindicated and the order is canceled. The nurse knows that the best alternative medication to give is

aspirin. If warfarin is contraindicated, aspirin is the best option, although other medications may be used if both are contraindicated.

immediate nursing action if a head injury patient started showing agitation

call the physician

The nurse is caring for a client who has homonymous hemianopsia following an acute stroke. Which nursing diagnosis is the most appropriate for this client? a. Risk for ineffective airway maintenance b. Risk for knowledge deficit c. Risk for poor fluid intake d. Risk for self-neglect

d. Risk for self-neglect

injury to CN 8. what deficit is anticipated

hearing and balance

Name two main drugs used after brain and spinal cord surgery

mannitol corticosteroids

if opioid and sedatives do not work in a patient with IICP, which drug will be the next choice

neuromuscular blocking agents

early indication of metastatic cancer to spine

pain

A client with an acute head injury cannot accurately identify the sensation felt when the nurse touches the intact skin with a cotton ball or paper clip. The nurse is aware that the deficit reflects injury to which area of the brain?

parietal lobe

SIADH stands for

syndrome of inappropriate antidiuretic hormone

RN is assisting client with right hemiparesis. which side should the patient get up on

the left side

When assessing a neonate for seizures, what would the nurse expect to find? Select all that apply. a) Ocular deviation b) Jitteriness c) Tonic-clonic contractions d) Elevated blood pressure e) Tachycardia

• Tachycardia • Elevated blood pressure • Jitteriness • Ocular deviation

The nurse provides instructions to a client who is scheduled for an electroencephalogram (EEG). Which statement by the client indicates a need for further instruction?

"All medications need to be withheld on the day of the test." Rationale: The client is informed that the test will take 45 minutes to 2 hours and that medications usually are not withheld before the test unless specifically prescribed. Preprocedural instructions include informing the client that the procedure is painless. Cola, tea, and coffee are stimulants and need to be restricted on the morning of the test. The hair should be washed the evening before the test, and gels, hairsprays, and lotion should be avoided.

The mother of a toddler tells the nurse during a routine well-child appointment that she is concerned because, "It seems like my son is falling and hitting his head all of the time." What is the best response by the nurse?

"Due to the size of their heads and immature neck muscles falling is common, but I will let the physician know your concerns." The head of the infant and young child is large in proportion to the body, and is the fastest-growing body part during infancy and continues to grow until the child is 5 years old. In addition, the infant's and child's neck muscles are not well developed. Both of these differences lead to an increased incidence of head injury from falls. The nurse should still let the physician know the mother's concerns in case there is another issue causing the falls.

A female client with myasthenia gravis comes to the health care provider's office for a scheduled office visit. The client is very concerned and tells the nurse that her husband seems to be avoiding her because she is very unattractive. Which is the appropriate nursing response?

"Have you thought about sharing your feelings with your husband?" Encouraging the client to share feelings with her husband directly addresses the subject of the question. Advising the client to join a support group will not address the client's immediate and individual concerns. The remaining options are blocks to communication and avoid the client's concerns.

The nurse is educating parents of a male infant with Chiari type II malformation. Which of the following statements about their child's condition is most accurate? a) "Take your time feeding your baby." b) "You'll see a big difference after the surgery." c) "Lay him down after feeding." d) "You won't need to change diapers often."

"Take your time feeding your baby."

A client with massive trauma and possible spinal cord injury is admitted to the emergency department following a dirt bike accident. Which clinical manifestation does the nurse assess to help best confirm a diagnosis of neurogenic shock? 1. Apical heart rate 48/min 2. Blood pressure 186/92 mm Hg 3. Cool, clammy skin 4. Temperature 100 F (37.7 C) tympanic

1

The office nurse, while reviewing a client's health information, notices that the client has recently started taking St. John's wort for symptoms of depression. What additional information is most important for the nurse to obtain? 1. Ask if the client is currently taking any prescription antidepressant medications 2. Ask if the client has been diagnosed with depression by a mental health care provider (HCP) 3. Ask if the client takes a multivitamin with iron 4. Ask if the client uses tanning beds

1

A client with multiple sclerosis tells the unlicensed assistive personnel (UAP) after physical therapy that she is too tired to take a bath. What is the priority nursing concern at this time? 1. Fatigue 2. Inability to perform activities of daily living (ADLs) 3. Decreased mobility 4. Muscular weakness

1 At this time, based on the client's statement, the priority is inability to perform ADLs most likely related to being tired (fatigue) after physical therapy. The other three nursing concerns are appropriate to a client with MS but are not related to the client's statement

A client is receiving phenobarbital sodium (Luminal) for the treatment of a seizure disorder. Which finding on the nursing assessment would indicate that the client is experiencing a common side effect of this medication? 1. Drowsiness 2. Hypocalcemia 3. Blurred vision 4. Seizure activity

1 Rationale: Drowsiness is a common side effect of phenobarbital, which is a barbiturate and antiseizure medication. Hypocalcemia is a rare toxic reaction. Blurred vision is not an associated side effect of this medication. Seizure activity could occur from abrupt withdrawal of this medication therapy or as a toxic reaction. Test-Taking Strategy: Note the subject, common side effects of this medication. Focus on the word common in the question, and recall that this medication is a barbiturate and antiseizure medication.

The nurse provides instructions to the parents of an infant with hip dysplasia regarding care of the Pavlik harness. Which statement by one of the parents indicates an understanding of the use of the harness? 1. "I can remove the harness to bathe my infant." 2. "I need to remove the harness to feed my infant." 3. "I need to remove the harness to change the diaper." 4. "My infant needs to remain in the harness at all times."

1 The harness should be worn 23 hours a day and can be removed only to check the skin and for bathing. The hips and buttocks should be supported carefully when the infant is out of the harness. The harness does not need to be removed for diaper changes or feedings.

Which nursing action will be implemented first if a client has a generalized tonic-clonic seizure? 1. Turn the client to one side. 2. Give lorazepam 2 mg IV. 3. Administer oxygen via nonrebreather mask. 4. Assess the client's level of consciousness.

1 The priority action during a generalized tonic-clonic seizure is to protect the airway by turning the client to one side to prevent aspiration. Administering lorazepam should be the next action because it will act rapidly to control the seizure. Although oxygen may be useful during the postictal phase, the hypoxemia during tonic-clonic seizures is caused by apnea, which cannot be corrected by oxygen administration. Checking level of consciousness is not appropriate during the seizure because generalized tonic-clonic seizures are associated with a loss of consciousness.

The nurse is performing an assessment on a client with a diagnosis of Bell's palsy. The nurse should expect to observe which finding in the client? 1.Facial drooping 2.Periorbital edema 3. Ptosis of the eyelid 4. Twitching on the affected side of the face

1 Rationale: Bell's palsy is a one-sided facial paralysis caused by the compression of the facial nerve (cranial nerve VII). Assessment findings include facial droop from paralysis of the facial muscles; increased lacrimation; painful sensations in the eye, face, or behind the ear; and speech or chewing difficulty. Options 2, 3, and 4 are not associated findings in Bell's palsy.

The nurse caring for a client with a head injury is monitoring for signs of increased intracranial pressure. The nurse reviews the record and notes that the intracranial pressure (cerebrospinal fluid) is averaging 8 mm Hg. The nurse plans care, knowing that these results are indicative of which condition? 1.Normal condition 2.Increased pressure 3.Borderline situation 4. Compensating condition

1 Rationale: The normal intracranial pressure is 5 to 10 mm Hg. A pressure of 8 mm Hg is within normal range.

The neurological unit staff is composed of an experienced registered nurse (RN), a new graduate RN, a licensed practical nurse (LPN), and an unlicensed assistive personnel (UAP). Which are appropriate assignments for the charge nurse to give to the new graduate RN? Select all that apply. 1. Client with multiple sclerosis who has ataxia and is awaiting discharge placement 2. Discharge a client who had a stroke to the rehabilitation unit 3. New admission with Guillain-Barré syndrome with paralysis to the thigh 4. New admission with head injury and Glasgow Coma Scale (GCS) score of 8 5. Provide initial teaching for a client beginning prednisone therapy

1,2,5

The nurse caring for a client with a head injury is monitoring for signs of increased intracranial pressure. The nurse reviews the record and notes that the intracranial pressure (cerebrospinal fluid) is averaging 8 mm Hg. The nurse plans care, knowing that these results are indicative of which condition? 1.Normal condition 2.Increased pressure 3.Borderline situation 4.Compensating condition

1. The normal intracranial pressure is 5 to 10 mm Hg. A pressure of 8 mm Hg is within normal range. *Focus on the subject, normal intracranial pressure. Knowledge about normal intracranial pressure is needed to answer this question. Recalling that the normal pressure is 5 to 10 mm Hg will direct you to the correct option.

A girl who is playing in the playroom experiences a tonic-clonic seizure. During the seizure, the nurse should take which actions? Select all that apply. 1.Remain calm. 2.Time the seizure. 3.Ease the child to the floor. 4.Loosen restrictive clothing. 5.Keep the child on her back. 6.Place a pillow under the child's head.

12346Rationale: When a child is having a seizure, the nurse should remain calm, time the seizure, ease the child to the floor if the child is standing or seated, keep the child on the side, loosen restrictive clothing, and place a pillow under the child's head.

An elderly client with dementia frequently exhibits sundowning behavior while living in a community-based residential facility. When the nurse finds the client wandering at night, which of the following statements is most appropriate? 1. "Don't you know it's not morning yet?" 2. "It's time to get back to bed now." 3. "You might fall if you wander in the dark." 4. "You should not leave your room without assistance."

2

The emergency department nurse assesses a client involved in a motor vehicle accident who sustained a coup-contrecoup head injury. Which assessment finding is consistent with injury to the occipital lobe? 1. Decreased rate and depth of respirations 2. Deficits in visual perception 3. Expressive aphasia 4. Inability to recognize touch

2

A client who has been taking phenytoin (Dilantin) for seizure control has a serum phenytoin drug level of 8 mcg/mL. The nurse should make which interpretation about this value? 1. A toxic level 2. An inadequate drug level 3. The low end of therapeutic range 4. The high end of therapeutic range

2 Rationale: The therapeutic serum drug level range for phenytoin is 10 to 20 mcg/mL. A laboratory value of 8 mcg/mL is below the therapeutic range, indicating an inadequate drug level.

For which client with severe migraine headaches would the nurse question an order for sumatriptan? 1. A 58-year-old client with gastrointestinal reflux disease 2. A 48-year-old client with hypertension 3. A 65-year-old client with mild emphysema 4. A 72-year-old client with hyperthyroidism

2 Sumatriptan is a triptan preparation developed to treat migraine headaches. Most are contraindicated in clients with actual or suspected ischemic heart disease, cerebrovascular ischemia, hypertension, and peripheral vascular disease and in those with Prinzmetal angina because of the potential for coronary vasospasm.

Which client should the charge nurse assign to a new graduate RN who is orientating to the neurologic unit? 1. A 28-year-old newly admitted client with a spinal cord injury 2. A 67-year-old client who had a stroke 3 days ago and has left-sided weakness 3. An 85-year-old client with dementia who is to be transferred to long-term care today 4. A 54-year-old client with Parkinson disease who needs assistance with bathing

2 The new graduate RN who is on orientation to the unit should be assigned to care for clients with stable, noncomplex conditions, such as the client with stroke. The task of helping the client with Parkinson disease to bathe is best delegated to the unlicensed assistive personnel (UAP). The client being transferred to the nursing home, and the newly admitted client with spinal cord injury should be assigned to experienced nurses.

Which client should the charge nurse assign to the traveling nurse, new to neurologic nursing care, who has been on the neurologic unit for 1 week? 1. A 34-year-old client with newly diagnosed multiple sclerosis (MS) 2. A 68-year-old client with chronic amyotrophic lateral sclerosis (ALS) 3. A 56-year-old client with Guillain-Barré syndrome (GBS) in respiratory distress 4. A 25-year-old client admitted with a C4-level spinal cord injury (SCI)

2 The traveling nurse is relatively new to neurologic nursing and should be assigned clients whose condition is stable and not complex, such as the client with chronic ALS. The newly-diagnosed client with MS will need a lot of teaching and support. The client with respiratory distress will need frequent assessments and may need to be transferred to the intensive care unit. The client with a C4-level SCI is at risk for respiratory arrest. All three of these clients should be assigned to nurses experienced in neurologic nursing care.

A nurse is preparing for a medical relief trip to West Africa and is concerned about a disruption in circadian rhythm from traveling across several time zones. Which herbal supplement might help synchronize the body to environmental time? 1. Evening primrose 2. Ginseng 3. Melatonin 4. St. John's wort

3

The nurse is assisting a health care provider (HCP) examining a 3-week-old infant with developmental dysplasia of the hip. What test or sign should the nurse expect the HCP to assess? 1. Babinski's sign 2. The Moro reflex 3. Ortolani's maneuver 4. The palmar-plantar grasp

3 In developmental dysplasia of the hip, the head of the femur is seated improperly in the acetabulum or hip socket of the pelvis. Ortolani's maneuver is a test to assess for hip instability and can be done only before 4 weeks of age. The examiner abducts the thigh and applies gentle pressure forward over the greater trochanter. A "clicking" sensation indicates a dislocated femoral head moving into the acetabulum. Babinski's sign is abnormal in anyone older than 2 years of age and indicates central nervous system abnormality. The Moro reflex is normally present at birth but is absent by 6 months; if still present at 6 months, there is an indication of neurological abnormality. The palmar-plantar grasp is present at birth and lessens within 8 months.

The nurse is preparing a nursing care plan for a client with a spinal cord injury (SCI) for whom problems of decreased mobility and inability to perform activities of daily living (ADLs) have been identified. The client tells the nurse, "I don't know why we're doing all this. My life's over." Based on this statement, which additional nursing concern takes priority? 1. Risk for injury 2. Decreased nutrition 3. Difficulty with coping 4. Impairment of body image

3 The client's statement indicates difficulty with coping in adjusting to the limitations of the injury and the need for additional counseling, teaching, and support. The other three nursing problems may be appropriate for a client with SCI but are not related to the client's statement.

The nurse is reviewing the record of a child with increased intracranial pressure and notes that the child has exhibited signs of decerebrate posturing. On assessment of the child, the nurse expects to note which characteristic of this type of posturing? 1.Flaccid paralysis of all extremities 2.Adduction of the arms at the shoulders 3.Rigid extension and pronation of the arms and legs 4.Abnormal flexion of the upper extremities and extension and adduction of the lower extremities

3 Rationale: Decerebrate (extension) posturing is characterized by the rigid extension and pronation of the arms and legs. Option 1 is incorrect. Options 2 and 4 describe decorticate (flexion) posturing.

A client with multiple sclerosis is experiencing muscle weakness, spasticity, and an ataxic gait. On the basis of this information, the nurse should include which client problem in the plan of care? 1.Inability to care for self 2.Interruption in skin integrity 3.Interruption in physical mobility 4.Inability to perform daily activities

3 Rationale: Multiple sclerosis is a chronic, nonprogressive, noncontagious degenerative disease of the central nervous system characterized by demyelinization of the neurons. Interruption in physical mobility is most appropriate for the client with multiple sclerosis experiencing muscle weakness, spasticity, and ataxic gait. Options 1, 2, and 4 are not related to the data in the question.

A client has suffered a head injury affecting the occipital lobe of the brain. The nurse anticipates that the client may experience difficulty with which sense? 1. Smell 2. Taste 3. Vision 4. Hearing

3 Rationale: The occipital lobe is responsible for reception of vision and contains visual association areas. This area of the brain helps the individual to visually recognize and understand the surroundings. The other senses listed are not a function of the occipital lobe.

The nurse overhears a neurologist saying that a client has an aneurysm located in the circle of Willis. The nurse understands that which blood vessels are part of the circle of Willis? Select all that apply. 1. Basilar artery 2. Vertebral artery 3. Anterior cerebral artery 4. Internal carotid arteries 5. Posterior cerebral artery

3. Anterior cerebral artery 4. Internal carotid arteries 5. Posterior cerebral artery The circle of Willis is a ring of blood vessels located at the base of the brain. It is referred to as the anterior circulation to the brain and is composed of the anterior and middle cerebral arteries, posterior cerebral arteries, posterior communicating arteries, internal carotid arteries, and anterior communicating branches. The basilar artery and vertebral artery are not part of the circle of Willis. Rather, they are part of the vertebral-basilar system, which is known as the posterior circulation to the brain. Other parts of the posterior circulation are the posterior inferior cerebellar artery and the spinal arteries.

The nurse receives a physician's order to administer 1,000 mL of intravenous (IV) normal saline solution over 8 hours to a client who recently had a stroke. What should the drip rate be if the drop factor of the tubing is 15 gtt/mL? Record your answer using a whole number.

31

The nurse enters a child's room and discovers that the child is having a seizure. Which actions should the nurse take? Select all that apply. 1. Call a code. 2. Run to get the crash cart. 3. Turn the child on her side. 4. Loosen any restrictive clothing. 5. Check the child's respiratory status. 6. Place an airway into the child's mouth.

345 Rationale: During a seizure the child is placed on his or her side in a lateral position. Positioning on the side will prevent aspiration because saliva will drain out the corner of the child's mouth. The child is not restrained because this could cause injury to the child. The nurse would loosen clothing around the child's neck and ensure a patent airway by checking respiratory status. A code would be called if the child was not breathing or the heart is not beating. There are no data in the question indicating that this is the case. The nurse would stay with the child to reduce the risk of injury and allow for observation and timing of the seizure. Nothing is placed into the child's mouth during a seizure because this could injure the child's mouth, gums, or teeth.

A client diagnosed with trigeminal neuralgia is given a prescription of carbamazepine by the health care provider. Which intervention does the nurse add to this client's care plan? 1. Encourage client to drink cold beverages 2. Encourage client to eat a high-fiber diet 3. Encourage client to perform facial massage 4. Encourage client to report any fever or sore throat

4

A client with multiple sclerosis is voicing concerns to the nurse about incoordination when walking. Which of the following instructions by the nurse would be most appropriate at this time? 1. "Avoid excess stretching of your lower extremities." 2. "Build strength by increasing the duration of daily exercise." 3. "Let me speak with your health care provider about getting a wheelchair." 4. "You should keep your feet apart and use a cane when walking."

4

A nursing student is teaching a client and family about epilepsy before the client's discharge. For which statement should the nurse intervene? 1. "You should avoid consumption of all forms of alcohol." 2. "Wear your medical alert bracelet at all times." 3. "Protect your loved one's airway during a seizure." 4. "It's OK to take over-the-counter medications."

4 A client with a seizure disorder should not take over-the-counter medications without consulting with the health care provider first. The other three statements are appropriate teaching points for clients with seizure disorders and their families.

The nurse caring for a child who has sustained a head injury in an automobile crash is monitoring the child for signs of increased intracranial pressure (ICP). For which early sign of increased ICP should the nurse monitor? 1. Increased systolic blood pressure 2. Abnormal posturing of extremities 3. Significant widening pulse pressure 4. Changes in level of consciousness

4 An altered level of consciousness is an early sign of increased intracranial pressure (ICP). Late signs of increased ICP include tachycardia leading to bradycardia, apnea, systolic hypertension, widening pulse pressure, and posturing.

The nurse is told that the result of a serum carbamazepine (Tegretol) level for a child who is receiving the medication for the control of seizures is 10 mcg/mL. Based on this laboratory result, the nurse anticipates that the health care provider (HCP) will document which prescription? 1. Discontinuation of the medication 2. A decrease of the dosage of the medication 3. An increase of the dosage of the medication 4. Continuation of the presently prescribed dosage

4 Rationale: When carbamazepine is administered, blood levels need to be monitored periodically to check for the child's absorption of the medication. The amount of the medication prescribed is based on the blood level achieved. The therapeutic serum range of carbamazepine is 4 to 12 mcg/mL. The nurse would anticipate that the HCP will continue the presently prescribed dosage.

A client is experiencing delirium. The nurse concludes that which areas of the nervous system are affected? 1. Temporal lobe and frontal lobe 2. Hippocampus and frontal lobe 3. Limbic system and cerebral hemispheres 4. Reticular activating system and cerebral hemispheres

4 Insomnia, agitation, mania, and delirium are caused by excessive arousal of the reticular activating system in conjunction with the cerebral hemispheres. The temporal lobe, hippocampus, and frontal lobe are responsible for memory. The limbic system is responsible for feelings and affect.

The nurse is providing home care instructions to the mother of a child who is recovering from Reye's syndrome. Which instruction should the nurse provide to the mother? 1.Increase stimuli in the home environment. 2.Avoid daytime naps so that the child will sleep at night 3.Give the child frequent small meals, if vomiting occurs. 4. Check the skin and eyes every day for a yellow discoloration.

4 Rationale: Checking for jaundice will assist in identifying the presence of liver complications, which are characteristic of Reye's syndrome. Decreasing stimuli and providing rest decrease stress on the brain tissue. If vomiting occurs in Reye's syndrome, it is caused by cerebral edema and is a sign of intracranial pressure.

A nurse is caring for a client with an intracranial pressure (ICP) monitoring device. The nurse should become most concerned if the ICP readings drifted to and stayed in the vicinity of which finding? 1.5 mm Hg 2.8 mm Hg 3.14 mm Hg 4. 22 mm Hg

4 Rationale: Normal ICP readings range from 5 to 15 mm Hg pressure. Pressures greater than 20 mm Hg are considered to represent increased ICP, which seriously impairs cerebral perfusion.

The home care nurse is preparing to visit a client with a diagnosis of trigeminal neuralgia (tic douloureux). When performing the assessment, the nurse should plan to ask the client which question to elicit the most specific information regarding this disorder? 1."Do you have any visual problems?" 2. "Are you having any problems hearing?" 3. "Do you have any tingling in the face region?" 4. "Is the pain experienced a stabbing type of pain?"

4 Rationale: Trigeminal neuralgia is characterized by spasms of pain that start suddenly and last for seconds to minutes. The pain often is characterized as stabbing or as similar to an electric shock. It is accompanied by spasms of facial muscles that cause twitching of parts of the face or mouth, or closure of the eye. Options 1, 2, and 3 do not elicit data specifically related to this disorder.

A client is being admitted for a potential cerebellar pathology. Which tasks should the nurse ask the client to perform to assess if cerebellar function is within the defined limits? Select all that apply. 1. Identify the number "8" traced on the palm 2. Shrug the shoulders against resistance 3. Swallow water 4. Touch each finger of one hand to the hand's thumb 5. Walk heel-to-toe

4,5

The nurse is preparing for the admission of a client with a suspected diagnosis of Guillain-Barré syndrome. When the client arrives at the nursing unit, the nurse reviews the health care provider's documentation. The nurse expects to note documentation of which hallmark clinical manifestation of this syndrome? 1.Multifocal seizures 2.Altered level of consciousness 3.Abrupt onset of a fever and headache 4.Development of progressive muscle weakness

4. A hallmark clinical manifestation of Guillain-Barré syndrome is progressive muscle weakness that develops rapidly. Seizures are not normally associated with this disorder. The client does not have symptoms such as a fever or headache. Cerebral function, level of consciousness, and pupillary responses are normal. * Focus on the subject, Guillain-Barré syndrome. Think about the pathophysiology that occurs with this disorder. Recalling that progressive muscle weakness occurs will direct you to the correct option.

A client had a transsphenoidal resection of the pituitary gland. The nurse notes drainage on the nasal dressing. Suspecting cerebrospinal fluid (CSF) leakage, the nurse should look for drainage that is of which characteristic? 1.Serosanguineous only 2.Bloody with very small clots 3.Sanguineous only with no clot formation 4.Serosanguineous, surrounded by clear to straw-colored fluid

4. CSF leakage after cranial surgery may be detected by noting drainage that is serosanguineous (from the surgery) and surrounded by an area of clear or straw-colored drainage. The typical appearance of CSF drainage is that of a "halo." The nurse also would further verify actual CSF drainage by testing the drainage for glucose, which would be positive. * Eliminate options 1 and 3 first because of the closed-ended word only. Regarding the remaining options, recall that CSF is clear to straw-colored.

A client is about to undergo a lumbar puncture. The nurse describes to the client that which position will be used during the procedure? 1.Side-lying with a pillow under the hip 2.Prone with a pillow under the abdomen 3.Prone in slight Trendelenburg's position 4. Side-lying with the legs pulled up and the head bent down onto the chest

4Rationale: A client undergoing lumbar puncture is positioned lying on the side, with the legs pulled up to the abdomen and the head bent down onto the chest. This position helps open the spaces between the vertebrae and allows for easier needle insertion by the health care provider. The nurse remains with the client during the procedure to help the client maintain this position. The other options identify incorrect positions for this procedure.

A 154-pound woman has been prescribed tPA (0.9 mg/kg) for an ischemic stroke. The nurse knows to give how many mg initially?

6.3 mg A person who weighs 154 lbs weighs 70 kg. To calculate dosage, multiply 70 × 0.9 mg/kg = 63 mg. The nurse gives 10% (6.3 mg) over 1 minute.

A nurse is caring for a client with a brain tumor and increased intracranial pressure (ICP). Which intervention should the nurse include in the care plan to reduce ICP? a) Administer stool softeners. b) Position the client with the head turned toward the side of the brain tumor. c) Provide sensory stimulation. d) Encourage coughing and deep breathing.

A

A parent of a child with a moderate head injury asks the nurse, "How will you know if my child is getting worse?" The nurse should tell the parents that best indicator of the child's brain function is: a) level of consciousness. b) reactions of the pupils. c) the vital signs. d) motor strength.

A

The client with retinal detachment in the right eye is extremely apprehensive and tells the nurse, "I am afraid of going blind. It would be so hard to live that way." What factor should the nurse consider before responding to this statement? a) Optimism is justified because surgical treatment has a 90% to 95% success rate. b) More and more services are available to help newly blind people adapt to daily living. c) Repeat surgery is impossible, so if this procedure fails, vision loss is inevitable. d) The surgery will only delay blindness in the right eye, but vision is preserved in the left eye.

A

The nurse is assessing a client with a head injury. On admission, the pupils were equal; now the right pupil is fully dilated and nonreactive, and the left pupil is 4 mm and reacts to light. What would this change in neurologic status of the client suggest to the nurse? a) Increased intracranial pressure b) Decreased intracranial pressure c) This is a normal response after a head injury, and the pupils will be expected to return to normal. d) The test was not performed accurately; there was too much light in the examination room.

A

The nurse is teaching a client with multiple sclerosis about prevention of urinary tract infection (UTI) and renal calculi. Which of the following nutrition recommendations by the nurse would be the most likely to reduce the risk of these conditions? a) Increase fluids (2500 mL/day) and maintain urine acidity by drinking cranberry juice. b) Eat foods containing vitamins C, D, and E, and drink at least 2 L of fluid a day. c) Drink a large amount of fluids, especially milk products, and eat a diet that includes multiple sources of vitamin D. d) Eat foods and ingest fluids that will cause the urine to be less acidic.

A

The nurse is caring for a near-term pregnant woman who has not taken prenatal vitamins or folic acid supplements. Which congenital defect is most likely to occur based on the mother's prenatal history? a) Incomplete myelinization b) Neonatal conjunctivitis c) Facial deformities d) A neural tube defect

A neural tube defect

The nurse is performing the oculocephalic response (doll's eyes maneuver) on an unconscious client. The nurse turns the client's head and notes movement of the eyes in the same direction as the head. How should the nurse document these findings? 1. Normal 2. Abnormal 3. Insignificant 4. Inconclusive

Abnormal In an unconscious client, eye movements are an indication of brainstem activity and are tested by the oculocephalic response. When the doll's eyes maneuver is intact, the eyes move in the opposite direction when the head is turned. Abnormal responses include movement of the eyes in the same direction as the head and maintenance of a midline position of the eyes when the head is turned. An abnormal response indicates a disruption in the processing of information through the brainstem.

After teaching a group of students about neural tube disorders, the instructor determines that additional teaching is needed when the students identify this as a neural tube defect. a) Anencephaly b) Spina bifida occulta c) Arnold-Chiari malformation d) Encephalocele

Arnold-Chiari malformation

A group of nursing students are reviewing cerebral vascular disorders and risk factors in children. The students demonstrate understanding of the material when they identify which as a risk factor for hemorrhagic stroke? a) Sickle cell disease b) Meningitis c) Congenital heart defect d) Arteriovenous malformations (AVMs)

Arteriovenous malformations (AVMs)

A client is undergoing testing to confirm a diagnosis of myasthenia gravis. The nurse explains that a diagnosis is made if muscle function improves after the client receives an I.V. injection of a medication. What is the medication the nurse tells the client he'll receive during this test? a) Cyclosporine b) Edrophonium c) Azathioprine d) Immunoglobulin G

B

The nurse is assessing the level of consciousness for a client who just had open heart surgery. When asked, the client can give his name but is not sure about where he is or the time of day. What should the nurse do? a) Notify the surgeon. b) Encourage the client's wife to orient the client. c) Tell the client where he is and the time of day. d) Rub the client's sternum to arouse the client.

C

The nurse is in the room when a child with a seizure disorder is having a seizure. The child is having generalized jerking muscle movement, and the nurse notes the bed appears to be wet with urine. The child is in which stage of the generalized seizure? a) Tonic b) Postictal c) Prodromal d) Clonic

Clonic

A client who suffered a stroke is prepared for discharge from the hospital. The health care provider has prescribed range-of-motion (ROM) exercises for the client's right side. What action should the nurse include in the client's plan of care? 1. Implement ROM exercises to the point of pain for the client. 2. Consider the use of active, passive, or active-assisted exercises in the home. 3. Encourage the client to be dependent on the home care nurse to complete the exercise program. 4. Develop a schedule of ROM exercises every 2 hours while awake even if the client is fatigued.

Consider the use of active, passive, or active-assisted exercises in the home. The home care nurse must consider all forms of ROM for the client. Even a client with hemiplegia can participate in some components of rehabilitative care. In addition, the goal in home care nursing is for the client to assume as much self-care and independence as possible. The nurse needs to teach home care measures so that the client becomes self-reliant. The options of performing ROM exercises to the point of pain and performing ROM exercises every 2 hours while the client is awake even if fatigued are incorrect from a physiological standpoint.

The nurse is discussing with a parent the difference between a breath-holding spell and a seizure. The nurse would be correct in telling the parent what information in regard to seizures?

Convulsive activity occurs. During seizures convulsive activity is typically noted. During a breath-holding spell, the child is bradycardiac, cyanosis occurs at the onset, and the EEG is normal.

The nurse assesses a client who is diagnosed with a stroke (brain attack). On assessment, the client is unable to understand the nurse's commands. Which condition should the nurse document? 1. Occipital lobe impairment 2. Damage to the auditory association areas 3. Frontal lobe and optic nerve tracts damage 4. Difficulty with concept formation and abstraction areas

Damage to the auditory association areas Auditory association and storage areas are located in the temporal lobe and relate to understanding spoken language. The occipital lobe contains areas related to vision. The frontal lobe controls voluntary muscle activity, including speech, and an impairment can result in expressive aphasia. The parietal lobe contains association areas for concept formation, abstraction, spatial orientation, body and object size and shape, and tactile sensation.

A client is diagnosed with Bell's palsy. The nurse assessing the client expects to note which symptom? 1. A symmetrical smile 2. Difficulty closing the eyelid on the affected side 3. Narrowing of the palpebral fissure on the affected side 4. Paroxysms of excruciating pain in the lips and cheek on the affected side

Difficulty closing the eyelid on the affected side The facial drooping associated with Bell's palsy makes it difficult for the client to close the eyelid on the affected side. A widening of the palpebral fissure (the opening between the eyelids) and an asymmetrical smile are seen with Bell's palsy. Paroxysms of excruciating pain are characteristic of trigeminal neuralgia.

Which type of cells lines the ventricles in the brain?

Ependymal cells

The nurse caring for a patient with a cranial injury knows that broad-spectrum antibiotics are used to reduce cerebral edema. a) False b) True

False

The nurse is caring for a client who has undergone a craniotomy and has a supratentorial incision. The nurse should place the client in which position postoperatively? 1. Head of bed flat, head and neck midline 2. Head of bed flat, head turned to the nonoperative side 3. Head of bed elevated 30 to 45 degrees, head and neck midline 4. Head of bed elevated 30 to 45 degrees, head turned to the operative side

Head of bed elevated 30 to 45 degrees, head and neck midline After supratentorial surgery, the head is kept at a 30- to 45-degree angle. The head and neck should not be angled either anteriorly or laterally but rather should be kept in a neutral (midline) position. This promotes venous return through the jugular veins, which will help prevent a rise in intracranial pressure.

The nurse is caring for a client who has undergone craniotomy with a supratentorial incision. The nurse should plan to place the client in which position postoperatively?

Head of bed elevated 30 to 45 degrees, head and neck midline Following supratentorial surgery, the head of the bed is kept at a 30- to 45-degree angle. The head and neck should not be angled either anteriorly or laterally, but rather should be kept in a neutral (midline) position. This will promote venous return through the jugular veins, which will help prevent a rise in intracranial pressure.

A 6-month-old infant is admitted to the hospital with suspected bacterial meningitis. She is crying, irritable, and lying in the opisthotonic position. The priority nursing intervention would be: 1. Educate the family on ways to prevent bacterial meningitis. 2. Initiate appropriate isolation precautions and begin intravenous antibiotics. 3. Assess the infant's fontanels. 4. Encourage the mother to hold the infant and feed her.

Initiate appropriate isolation precautions and begin intravenous antibiotics.

A 6-month-old infant is admitted with suspected bacterial meningitis. She is crying, irritable, and lying in the opisthotonic position. Which intervention should the nurse take initially? a) Institute droplet precautions in addition to standard precautions. b) Educate the family about preventing bacterial meningitis. c) Encourage the mother to hold and comfort the infant. d) Palpate the child's fontanels.

Institute droplet precautions in addition to standard precautions.

A 6-month-old infant is admitted with suspected bacterial meningitis. She is crying, irritable, and lying in the opisthotonic position. Which of the following interventions should the nurse take initially? a) Institute droplet precautions in addition to standard precautions. b) Palpate the child's fontanels. c) Encourage the mother to hold and comfort the infant. d) Educate the family about preventing bacterial meningitis.

Institute droplet precautions in addition to standard precautions. Correct Explanation: Bacterial meningitis is a medical emergency. The child must be placed on droplet precautions until 24 hours of antibiotics have been given. Encouraging the mother to hold and comfort the child is an intervention but not the priority one: the focus is to get the infant the appropriate medications to fight the infection and to prevent its spread. Educating the family about preventing bacterial meningitis would be appropriate later on once the initial infection has been controlled. Palpating the fontanels is used to assess for hydrocephalus.

Preterm infants have more fragile capillaries in the periventricular area than term infants. Which problem does this put these infants at risk for? a) Moderate closed-head injury b) Congenital hydrocephalus c) Early closure of the fontanels d) Intracranial hemorrhaging

Intracranial hemorrhaging

A client is admitted with weakness, expressive aphasia, and right hemianopia. The brain MRI reveals an infarct. The nurse understands these symptoms to be suggestive of which of the following findings? a) Left-sided cerebrovascular accident (CVA) b) Right-sided cerebrovascular accident (CVA) c) Completed Stroke d) Transient ischemic attack (TIA)

Left-sided cerebrovascular accident (CVA) When the infarct is on the left side of the brain, the symptoms are likely to be on the right, and the speech is more likely to be involved. If the MRI reveals an infarct, TIA is no longer the diagnosis.

A client has experienced an ischemic stroke that has damaged the frontal lobe of his brain. Which of the following deficits does the nurse expect to observe during assessment?

Limited attention span and forgetfulness Damage to the frontal lobe may impair learning capacity, memory, or other higher cortical intellectual functions. Such dysfunction may be reflected in a limited attention span, difficulties in comprehension, forgetfulness, and a lack of motivation. Damage to the motor neurons may cause hemiparesis, hemiplegia, and a change in reflexes. Damage to the occipital lobe can result in visual agnosia, whereas damage to the temporal lobe can cause auditory agnosia.

The client with spinal cord injury is prone to experiencing autonomic dysreflexia. The least appropriate measure to minimize the risk of autonomic dysreflexia is which action?

Limiting bladder catheterization to once every 12 hours The most frequent cause of autonomic dysreflexia is a distended bladder. Straight catheterization should be performed every 4 to 6 hours, and indwelling bladder catheters should be checked frequently for kinks in the tubing. It is not appropriate to catheterize the client every 12 hours. Constipation and fecal impaction are other causes, so maintaining bowel regularity is important. Other causes include stimulation of the skin from tactile, thermal, or painful stimuli. The nurse administers care to minimize risk in these areas.

The nurse in the health care clinic is providing medication instructions to a client with a seizure disorder who will be taking divalproex sodium. The nurse should instruct the client about the importance of returning to the clinic for monitoring of which laboratory study? 1. Electrolyte panel 2. Liver function studies 3. Renal function studies 4. Blood glucose level determination

Liver function studies Divalproex sodium, an anticonvulsant, can cause fatal hepatotoxicity. The nurse should instruct the client about the importance of monitoring the results of liver function studies and ammonia level determinations. The studies in the remaining options are not required with the use of this medication.

The nurse is reinforcing instructions to a client taking divalproex sodium (Depakote). The nurse tells the client to return to the clinic for follow-up laboratory studies related to which test?

Liver function studies Divalproex sodium, an anticonvulsant, can cause hepatotoxicity, which is potentially fatal. The nurse instructs the client to return to the clinic for follow-up liver function studies, such as lactate dehydrogenase (LDH), serum glutamic-oxaloacetic transaminase (SGOT), serum glutamate pyruvate transaminase (SGPT), and ammonia levels. This is especially indicated in the first 6 months of therapy. The laboratory studies identified in the other options are not specifically related to the administration of this medication

Absence seizures are marked by which of the following clinical manifestations? a) Loss of muscle tone and loss of consciousness b) Brief, sudden onset of increased tone of the extensor muscle c) Sudden, brief jerks of a muscle group d) Loss of motor activity accompanied by a blank stare

Loss of motor activity accompanied by a blank stare Correct Explanation: An absence seizure consists of a sudden, brief arrest of the child's motor activity accompanied by a blank stare and loss of awareness. A tonic seizure consists of a brief onset of increased tone or muscle. A myoclonic seizure is characterized by sudden, brief jerks of muscle groups. An atonic seizure involves a sudden loss of muscle tone and loss of consciousness.

The nurse is collecting data from a child who may have a seizure disorder. Which of the following is a description of an absence seizure? a) Sudden, momentary loss of muscle tone, with a brief loss of consciousness b) Muscle tone maintained and child frozen in position c) Brief, sudden contracture of a muscle or muscle group d) Minimal or no alteration in muscle tone, with a brief loss of consciousness

Minimal or no alteration in muscle tone, with a brief loss of consciousness Explanation: Absence seizures are characterized by a brief loss of responsiveness with minimal or no alteration in muscle tone. They may go unrecognized because the child's behavior changes very little. A sudden loss of muscle tone describes atonic seizures. A frozen position describes the appearance of someone having akinetic seizures. A brief, sudden contraction of muscles describes a myoclonic seizure.

A client has experienced an episode of myasthenic crisis. The nurse collects data to determine whether the client has experienced which precipitating factor?

Omitted doses of medication Myasthenic crisis is often caused by undermedication and responds to administration of cholinergic medications such as neostigmine (Prostigmin) and pyridostigmine (Mestinon). Cholinergic crisis (the opposite problem) is caused by excess medication and responds to withholding of medications. Too little exercise and fatty food intake are incorrect options. Overexertion and overeating could trigger myasthenic crisis.

A nurse is caring for a newborn with anencephaly. Which intervention will the nurse use?

Place a cap or similar covering on the infant's head. Using an infant cap can help parents deal with the malformed appearance of their child. Because the child was born with a small or missing brain, the baby will likely die within hours or days. Monitoring for increased intracranial pressure (ICP) or neurologic status are not necessary interventions.

A nurse is assessing a 3-year-old child for possible bacterial meningitis. Which sign would indicate irritation of the meninges? a) Negative Brudzinski's sign b) Positive Homans' sign c) Negative Kernig's sign d) Positive Kernig's sign

Positive Kernig's sign

A nurse is assessing a 3-year-old child with nuchal rigidity. Which sign would be documented on the chart to support this condition? a) Positive Homans' sign b) Positive Kernig's sign c) Negative Kernig's sign d) Negative Brudzinski's sign

Positive Kernig's sign Correct Explanation: A positive Kernig's sign indicates nuchal rigidity, caused by an irritative lesion of the subarachnoid space. A positive Brudzinski's sign also is indicative of the condition. A positive Homans' sign may indicate venous inflammation of the lower leg.

A 10-year-old boy has been experiencing complex partial seizures and has not responded well to medication. Surgery is planned to remove brain tissue at the seizure foci. Which diagnostic test would be the most accurate in identifying the seizure foci? a) Brain scan b) Echoencephalography c) Positron emission tomography (PET) d) Myelography

Positron emission tomography (PET)

The nurse is assessing a client who is experiencing seizure activity. The nurse understands that it is necessary to determine information about which items as part of routine assessment of seizures? Select all that apply. Postictal status 2. Duration of the seizure 3. Changes in pupil size or eye deviation 4. Seizure progression and type of movements 5. What the client ate in the 2 hours preceding seizure activity

Postictal status 2. Duration of the seizure 3. Changes in pupil size or eye deviation 4. Seizure progression and type of movements Typically seizure assessment includes the time the seizure began, parts of the body affected, type of movements and progression of the seizure, change in pupil size or eye deviation or nystagmus, client condition during the seizure, and postictal status. Option 5 is not a component of seizure assessment.

Dexamethasone (Decadron) intravenously is prescribed for the client with cerebral edema. The nurse prepares the medication for administration and plans to perform which action? Mix the medication in 1000 mL of 5% dextrose. 2. Prepare an undiluted direct injection of the medication. 3. Mix the medication in 100 mL of lactated Ringer's solution. 4. Dilute the medication in lactated Ringer's solution and administer as a direct injection.

Prepare an undiluted direct injection of the medication.

The nurse has a prescription to give dexamethasone (Decadron) by the intravenous (IV) route to a client with cerebral edema. How should the nurse prepare this medication? Diluting the medication in 500 mL of 5% dextrose 2. Preparing an undiluted direct injection of the medication 3. Diluting the medication in 1 mL of lactated Ringer's solution for direct injection 4. Diluting the medication in 10% dextrose in water and administering it as a direct injection

Preparing an undiluted direct injection of the medication

A client with neck and upper extremity pain has been diagnosed with cervical radiculitis. What does the nurse anticipate as being the cause of these clinical manifestations? 1. Pressure on a spinous process 2. Pressure on a spinal nerve root 3. Pressure on a central spinal cord 4. Pressure on a posterior facet joints

Pressure on a spinal nerve root Radiculitis is a term used to describe spinal nerve root compression at the intervertebral foramen. Radiculitis can be caused by a number of factors, such as whiplash or ruptured intervertebral disk. In many cases, it is caused by malalignment that occurs with degenerative disease or bone spur formation. Options 1, 3, and 4 are not associated with cervical radiculitis.

The nurse has a prescription to begin aneurysm precautions for a client with a subarachnoid hemorrhage secondary to aneurysm rupture. The nurse would plan to incorporate which intervention in controlling the environment for this client? 1. Keep the window blinds open. 2. Turn on a small spotlight above the client's head. 3. Make sure the door to the room is open at all times. 4. Prohibit or limit the use of a radio or television and reading.

Prohibit or limit the use of a radio or television and reading. Environmental stimuli are kept to a minimum with subarachnoid precautions to prevent or minimize increases in intracranial pressure. For this reason, lighting is reduced by closing window blinds and keeping the door to the client's room shut. Overhead lighting also is avoided for the same reason. The nurse prohibits television, radio, and reading unless this is so stressful for the client that it would be counterproductive. In that instance, minimal amounts of stimuli by these means are allowed with approval of the health care provider.

The nurse is preparing to care for a client after a lumbar puncture. The nurse should plan to place the client in which best position following the procedure? 1. Prone in semi Fowler's position 2. Supine in semi Fowler's position 3. Prone with a small pillow under the abdomen 4. Lateral with the head slightly lower than the rest of the body

Prone with a small pillow under the abdomen After the procedure, the client assumes a flat position. If the client is able, a prone position with a pillow under the abdomen is the best position. This position helps reduce cerebrospinal fluid leakage and decreases the likelihood of post-lumbar puncture headache. The remaining options are incorrect.

The nurse is providing care to a client with increased intracranial pressure (ICP). Which approaches would be beneficial in controlling the client's ICP from an environmental viewpoint? Select all that apply.

Reducing environmental noise Maintaining a calm atmosphere Allowing the client uninterrupted time for sleep Nursing interventions should be spaced out over the shift to minimize the risk of a rise in ICP. If possible, activities known to raise ICP should be avoided when possible. Other interventions to control the ICP include keeping the lighting in the room dim or off; maintaining a calm, quiet environment; and avoiding emotional stress and interruption of sleep.

At the end of the work shift, the nurse is reviewing the respiratory status of a client admitted with a stroke (brain attack) earlier in the day. The nurse determines that the client's airway is patent if which data are identified? 1. Respiratory rate 24 breaths/min, oxygen saturation 94%, breath sounds clear 2. Respiratory rate 18 breaths/min, oxygen saturation 98%, breath sounds clear 3. Respiratory rate 16 breaths/min, oxygen saturation 85%, wheezes bilaterally 4. Respiratory rate 20 breaths/min, oxygen saturation 92%, diminished breath sounds in lung bases

Respiratory rate 18 breaths/min, oxygen saturation 98%, breath sounds clear The client's airway is most protected if all of the respiratory parameters measured fall within normal limits. Therefore, the respiratory rate should ideally be 16 to 20 breaths/min, the oxygen saturation should be greater than 95%, and the breath sounds should be clear. The correct option is the only one that meets all 3 criteria.

The nurse is caring for a client who begins to experience seizure activity while in bed. Which action by the nurse is contraindicated? Loosening restrictive clothing 2. Restraining the client's limbs 3. Removing the pillow and raising padded side rails 4. Positioning the client to the side, if possible, with the head flexed forward

Restraining the client's limbs Nursing actions during a seizure include providing for privacy, loosening restrictive clothing, removing the pillow and raising padded side rails in the bed, and placing the client on one side with the head flexed forward, if possible, to allow the tongue to fall forward and facilitate drainage. The limbs are never restrained because the strong muscle contractions could cause the client harm. If the client is not in bed when seizure activity begins, the nurse lowers the client to the floor, if possible, protects the head from injury, and moves furniture that may injure the client.

The student nurse develops a plan of care for a client after a lumbar puncture. The nursing instructor corrects the student if the student documents which incorrect intervention in the plan? 1. Maintain the client in a flat position. 2. Restrict fluid intake for a period of 2 hours. 3. Assess the client's ability to void and move the extremities. 4. Inspect the puncture site for swelling, redness, and drainage.

Restrict fluid intake for a period of 2 hours. After the lumbar puncture the client remains flat in bed for at least 2 hours, depending on the health care provider's prescriptions. A liberal fluid intake is encouraged to replace the cerebrospinal fluid removed during the procedure, unless contraindicated by the client's condition. The nurse checks the puncture site for redness and drainage and assesses the client's ability to void and move the extremities.

The nurse is preparing a care plan for a child who has a seizure disorder. The child experiences tonic-clonic seizures. Which nursing diagnosis will the nurse identify as having the highest priority?

Risk for injury A seizure disorder is caused by a disruption in the electrical impulses in the brain. Tonic-clonic seizures is the most dramatic seizure disorder. It is characterized by a loss of consciousness, along with the entire body experiencing tonic contractions followed by rhythmic clonic contractions alternating with relaxation of all muscle groups. Cyanosis may be noted due to apnea, and saliva may collect in the mouth due to inability to swallow. All of these symptoms would make Risk for injury the highest priority.

In caring for the child with meningitis, the nurse recognizes that which nursing diagnosis would be the most important to include in this child's plan of care?

Risk for injury related to seizure activity The child's risk for injury would be an appropriate nursing diagnosis. Surgery is not indicated for the child with meningitis, and the history of seizures does not impact the airway clearance. Growth and development issues are a concern but not likely delayed due to this diagnosis.

The nurse caring for a client following craniotomy who has a supratentorial incision understands that the client should most likely be maintained in which position? 1. Prone position 2. Supine position 3. Semi Fowler's position 4. Dorsal recumbent position

Semi Fowler's position In supratentorial surgery (surgery above the brain's tentorium), the client's head is usually elevated 30 degrees to promote venous outflow through the jugular veins. The client's head or the head of the bed is not lowered in the acute phase of care after supratentorial surgery. An exception to this is the client who has undergone evacuation of a chronic subdural hematoma, but a health care provider's (HCP's) prescription is required for positions other than those involving head elevation. In addition, the HCP's prescription regarding positioning is always checked and agency procedures are always followed.

The nurse is caring for a patient having a hemorrhagic stroke. What position in the bed will the nurse maintain this patient?

Semi-Fowler's The head of the bed is elevated 15 to 30 degrees (semi-Fowler's position) to promote venous drainage and decrease intracranial pressure.

The client has clear fluid leaking from the nose after a basilar skull fracture. The nurse determines that this is cerebrospinal fluid (CSF) if the fluid meets which criteria?

Separates into concentric rings and tests positive for glucose Leakage of CSF from the ears or nose may accompany basilar skull fracture. It can be distinguished from other body fluids because the drainage will separate into bloody and yellow concentric rings on dressing material, which is known as the halo sign. It also tests positive for glucose. Options 1, 2, and 3 are not characteristics of CSF.

Which nursing assessment data should be given the highest priority for a child with clinical findings related to meningitis? a) Degree and extent of nuchal rigidity b) Signs of increased intracranial pressure (ICP) c) Occurrence of urine and fecal contamination d) Onset and character of fever

Signs of increased intracranial pressure (ICP)

Which nursing assessment data should be given the highest priority for a child with clinical findings related to meningitis?

Signs of increased intracranial pressure (ICP) Assessment of fever and evaluation of nuchal rigidity are important aspects of care, but assessment for signs of increasing ICP should be the highest priority due to the life-threatening implications. Urinary and fecal incontinence can occur in a child who's ill from nearly any cause but doesn't pose a great danger to life

The nurse is performing a neurological assessment on a client with a head injury. The nurse should use which technique to assess the plantar reflex? Stroking the foot from the heel to the toe 2. Gently inserting a gloved finger in the rectum 3. Directing a flashlight onto the pupils of the eyes 4. Using a tongue depressor and stimulating the back of the throat

Stroking the foot from the heel to the toe

The nurse is teaching the client with myasthenia gravis about prevention of myasthenic and cholinergic crises. The nurse tells the client that this is most effectively done by which activity?

Taking medications on time to maintain therapeutic blood levels Clients with myasthenia gravis are taught to space out activities over the day to conserve energy and restore muscle strength. It is very important to take medications correctly to maintain blood levels that are not too low or too high. Muscle-strengthening exercises are not helpful and can fatigue the client. Overeating is a cause of exacerbation of symptoms as are exposure to heat, crowds, erratic sleep habits, and emotional stress.

The nurse is providing education to the parents of a female toddler with hydrocephalus who has just had a shunt placed. Which of the following statements is the best to use for a teaching session? a) Tell me your concerns about your child's shunt. b) Always keep her head raised 30º. c) Her autoregulation mechanism to absorb spinal fluid has failed. d) Call the doctor if she gets a persistent headache.

Tell me your concerns about your child's shunt. Correct Explanation: Always start by assessing the family's knowledge. Ask them what they feel they need to know. Knowing when to call the doctor and how to raise the child's head are important, but they might not be listening if they have another question on their minds. "Autoregulation" is too technical—base information on the parents' level of understanding.

The nurse is caring for a child admitted with simple partial motor seizures. Which of the following clinical manifestations would likely have been noted in the child with this diagnosis? a) The child had jerking movements in the legs and facial muscles. b) The child was rubbing the hands and smacking the lips. c) The child had shaking movements on one side of the body. d) The child was dizzy and had decreased coordination.

The child had shaking movements on one side of the body. Correct Explanation: Simple partial motor seizures cause a localized motor activity, such as shaking of an arm, leg, or other part of the body. These may be limited to one side of the body. Myoclonic seizures are characterized by a sudden jerking of a muscle or group of muscles, often in the arms or legs, without loss of consciousness. Complex partial seizures may cause nonpurposeful movements, such as hand rubbing and lip smacking. During the prodromal period of the tonic-clonic seizure, the child might have a lack of coordination.

The nurse is interviewing the caregivers of a child brought to the emergency unit. The caregiver states, "She has a history of seizures but this time it lasted more than 30 minutes and she just keeps having them." The most accurate description of this child's condition would be: a) The child is having generalized seizures. b) The child's history indicates she has infantile seizures. c) The child may begin to have absence seizures every day. d) The child is in status epilepticus.

The child is in status epilepticus.

In caring for a child with a seizure disorder, the highest priority goal is which of the following? a) The child will have an understanding of the disorder. b) The child will be free from injury during a seizure. c) The family will understand seizure precautions. d) The family caregivers anxiety will be reduced.

The child will be free from injury during a seizure.

When should the nurse plan the rehabilitation of a patient who is having an ischemic stroke

The day the patient has the stroke Although rehabilitation begins on the day the patient has the stroke, the process is intensified during convalescence and requires a coordinated team effort.

A client who experienced a brain attack (stroke) several months ago still exhibits some difficulty with chewing food. The nurse plans care, knowing that the client has residual dysfunction of which cranial nerve? Vagus (cranial nerve X) 2. Trigeminal (cranial nerve V) 3. Hypoglossal (cranial nerve XII) 4. Spinal accessory (cranial nerve XI)

Trigeminal (cranial nerve V) The motor branch of cranial nerve V is responsible for the ability to chew food. The vagus nerve is active in parasympathetic functions of the autonomic nervous system. The hypoglossal nerve aids in swallowing. The spinal accessory nerve is responsible for shoulder movement, among other things.

The nurse is caring for an 8-year-old girl who was in a car accident. Which symptom suggests the child has a cerebral contusion? a) Vomiting b) Trouble focusing when reading c) Difficulty concentrating d) Bleeding from the ear

Trouble focusing when reading

The best way to evaluate a child's level of consciousness is through conversation. a) False b) True

True

The nurse is caring for a client with intracranial pressure (ICP) monitoring. Which intervention is appropriate to include in the plan of care? 1. Place the client in Sims' position. 2. Change the drainage tubing every 48 hours. 3. Level the transducer at the lowest point of the ear. 4. Use strict aseptic technique when touching the monitoring system.

Use strict aseptic technique when touching the monitoring system. Because there is a foreign body embedded in the client's brain, vigilant aseptic technique should be implemented. Sims' is a side-lying, flat position. With a client who has increased ICP, the head of the bed should be elevated at least 30 degrees to improve jugular outflow. The drainage tubing should not be routinely changed. It should remain for the duration of the monitoring. To obtain accurate ICP pressure readings, the transducer is zeroed at the level of the foramen of Monro, which is approximated by placing the transducer 1 inch above the level of the ear. Serial ICP readings should be done with the client's head in the same position.

The nurse is caring for a premature infant diagnosed with intraventricular hemorrhage (IVH). Which of the following interventions best serves the needs of this client? a) Using a squeak toy to attract the child's gaze b) Stroking the child's cheek with a finger c) Placing the crib in a room by itself d) Removing toys from the crib when not in use

Using a squeak toy to attract the child's gaze

The nurse is caring for a 3-year-old boy who is experiencing seizure activity. Which diagnostic test will determine the seizure area in the brain? a) Video electroencephalogram b) Computed tomography c) Cerebral angiography d) Lumbar puncture

Video electroencephalogram

A client has suffered a head injury affecting the occipital lobe of the brain. What is the focus of the nurse's immediate assessment? 1. Taste 2. Smell 3. Vision 4. Hearing

Vision The occipital lobe is responsible for reception of vision and contains visual association areas. This area of the brain helps the individual to visually recognize and understand the surroundings. The other senses listed are not a function of the occipital lobe.

The nurse is monitoring a 3-year-old child for signs and symptoms of increased intracranial pressure (ICP) after a craniotomy. The nurse plans to monitor for which early sign or symptom of increased ICP? Vomiting 2. Bulging anterior fontanel 3. Increasing head circumference 4. Complaints of a frontal headache

Vomiting he brain, although well protected by the solid bony cranium, is highly susceptible to pressure that may accumulate within the enclosure. Volume and pressure must remain constant within the brain. A change in the size of the brain, such as occurs with edema or increased volume of intracranial blood or cerebrospinal fluid without a compensatory change, leads to an increase in intracranial pressure (ICP), which may be life-threatening. Vomiting, an early sign of increased ICP, can become excessive as pressure builds up and stimulates the medulla in the brainstem, which houses the vomiting center. Children with open fontanels (posterior fontanel closes at 2 to 3 months; anterior fontanel closes at 12 to 18 months) compensate for ICP changes by skull expansion and subsequent bulging fontanels. When the fontanels have closed, nausea, excessive vomiting, diplopia, and headaches become pronounced, with headaches becoming more prevalent in older children.

The nurse plays a critical role in the initial work-up of a patient with acute stroke symptoms. An immediate decision is to determine if the stroke is ischemic or hemorrhagic. Although there is overlap in some motor, sensory, and cognitive changes, hemorrhagic strokes can be identified by some specific signs. Which of the following signs are consistent with a hemorrhagic stroke? Select all that apply.

Vomiting Sudden, severe headache Seizures these three signs are usually diagnostic of a hemorrhagic stroke. The other signs can occur with both hemorrhagic and ischemic stroke.

A client has a cerebellar lesion. The nurse would plan to obtain which item for use by this client? 1. Walker 2. Slider board 3. Raised toilet seat 4. Adaptive eating utensils

Walker The cerebellum is responsible for balance and coordination. A walker provides stability for the client during ambulation. A raised toilet seat is useful if the client has sufficient mobility or ability to flex the hips. A slider board is used in transferring a client with weak or paralyzed legs from a bed to stretcher or wheelchair. Adaptive eating utensils are beneficial if the client has partial paralysis of the hand.

A client with diplopia has been taught to use an eye patch to promote better vision and prevent injury. The nurse determines that the client understands how to use the patch if the client states that he or she will do which?

Wear the patch continuously, alternating eyes each day. Placing an eye patch over one eye in the client with diplopia removes the second image and restores more normal vision. The patch is worn continuously and is alternated on a daily basis to maintain the strength of the extraocular muscles of the eyes.

An older client is brought to the hospital emergency department by a neighbor who heard the client talking and found him wandering in the street at 3 a.m. The nurse should first determine which data about the client? 1. His insurance status 2. Blood toxicology levels 3. Whether he ate his evening meal 4. Whether this is a change in usual level of orientation

Whether this is a change in usual level of orientation The nurse should first determine whether this behavior represents a change in the client's neurological status. The next item to determine is when the client last ate. Blood toxicology levels may or may not be needed, but the health care provider would likely prescribe these. Insurance information must be obtained at some point but is not the priority from a clinical care viewpoint.

A client with blunt head injury is admitted for observation, including hourly neurologic checks. At 01:00 AM, the client reports a headache; the nurse obtains a normal neurologic assessment and administers the PRN acetaminophen. At 02:00 AM, the client appears to be sleeping. Which action should the nurse take? a. Arouse the client and ask what the current month is b. Document "relief apparently obtained" and recheck at 03:00 AM c. Let the client sleep but verify respiratory rate d. Wake the client up and check for paresthesia

a. Arouse the client and ask what the current month is

The client has increased intracranial pressure with cerebral edema, and mannitol is administered. Which assessment should the nurse make to evaluate if a complication from the mannitol is occurring? a. Auscultate breath sounds to assess for crackles b. Monitor for >50 mL/hr urine output c. Monitor Glasgow Coma Scale increasing from 8/15 to 9/15 d. Press over the tibia to assess for pitting edema

a. Auscultate breath sounds to assess for crackles

one way to prevent rupture of non-ruptured cerebral aneurysm is a. dim lighting, minimize noise, limit visitors b. neuro checks q 15 min c. complete bed rest d. valsalva to prevent constipation

a. dim lighting, minimize noise, limit visitors

The nurse is caring for a client after a motor vehicle accident. The client's injuries include 2 fractured ribs and a concussion. The nurse notes which of the following as expected neurological changes for the client with a concussion? Select all that apply. a. Asymmetrical pupillary constriction b. Brief loss of consciousness c. Headache d. Loss of vision e. Retrograde amnesia

b. Brief loss of consciousness c. Headache e. Retrograde amnesia

A client is brought to the emergency department with stroke symptoms that began 7 hours ago. A CT scan confirms the presence of an ischemic stroke. The client's current blood pressure is 202/108 mm Hg. Which nursing action is most appropriate? a. Anticipate IV labetalol to keep blood pressure <140/90 mm Hg b. Document the current findings in the client's chart c. Prepare to administer thrombolytic therapy d. Request a prescription for IV antiseizure medication

b. Document the current findings in the client's chart

The nurse is caring for a client with a history of headaches. The client has talked to the nurse, smiled at guests, and maintained stable vital signs. The nurse notes the following changes in the client's status. Which assessment finding is critical to report to the health care provider (HCP)? a. Blood pressure 136/88 mm Hg b. Flat affect and drowsiness c. Poor appetite d. Respiratory rate 12/min

b. Flat affect and drowsiness

A client was struck on the head by a baseball bat during a robbery attempt. The nurse gives this report to the oncoming nurse at shift change and conveys that the client's current Glasgow Coma Scale (GCS) score is a "10." Which client assessment is most important for the reporting nurse to include? a. Belief that the current surrounding are a racetrack b. GCS score was "11" one hour ago c. Recent vital signs show blood pressure of 120/80 mm d. Hg and pulse of 82/min e. Reported allergy to penicillin and vancomycin

b. GCS score was "11" one hour ago

An adult client with altered mental status and fever has suspected bacterial meningitis with sepsis. Blood pressure is 80/60 mm Hg. Which prescribed intervention should the nurse implement first? a. Administer IV antibiotics b. Infuse bolus of IV normal saline c. Prepare to assist with lumbar puncture d. Transport client for head CT scan

b. Infuse bolus of IV normal saline

The nurse is caring for a client with an acute ischemic stroke who has a blood pressure of 178/95 mm Hg. The health care provider prescribes as-needed antihypertensives to be given if the systolic pressure is >200 mm Hg. Which action by the nurse is most appropriate? a. Give the antihypertensive medication b. Monitor the blood pressure c. Notify the health care provider d. Question the prescription

b. Monitor the blood pressure

Best test to rule out metastasis of cancer to any part of the body a. CTA b. PET scan c. MRI d. LP

b. PET scan

which client on the rehabilitation unit is most likely to develop autonomic dysreflexia? a. a client with brain injury b. a client with high cervical spine injury c. a client with herniated disc d. a client with shock

b. a client with high cervical spine injury

what is stereotatic gamma knife treatment? a. surgeon create a opening in the head b. focus the tumor deep in the brain tissue c. multiple radiations are ordered by radiation oncologist d. linear accelerator and a robotic arm is used

b. focus the tumor deep in the brain tissue

client with head injury develop muscle weakness, decreased urine output, what does it indicate a. deficiency of ADH b. increase release of ADH c. increased ICP d. kidney failure

b. increase release of ADH

A client with TI injury (traumatic injury) came to the ER with BP 82/40, pulse 34, dry skin. condition suspected? a. autonomic dysreflexia b. neurogenic shock c. hypovolemia d. sepsis

b. neurogenic shock

A client is receiving several adjunctive professional therapies while rehabilitating after a stroke. Which client statements indicate an understanding of the services? Select all that apply. a. "Occupational therapy will help me learn how to properly use my walker." b. "Physical therapy will help me learn how to dress myself again." c. "Social services can help me find resources for affording my medications." d. "Speech therapy will teach me how to eat my meals properly." e. "Wound care will teach me how to properly dress this wound on my knee."

c. "Social services can help me find resources for affording my medications." d. "Speech therapy will teach me how to eat my meals properly." e. "Wound care will teach me how to properly dress this wound on my knee."

The nurse receives the assigned clients for today on a neurology unit. The nurse should check on which client first? a. Client with history if head injury whose Glasgow Coma Scale (GCS) changes from 13 to 14 b. Client with history of myasthenia gravis who has ptosis in the evening c. Client with history of T2 spinal injury who has diaphoresis, pulse 54/min, and hypertension d. Client with history of transverse myelitis with 2+ bilateral lower extremity muscle strength

c. Client with history of T2 spinal injury who has diaphoresis, pulse 54/min, and hypertension

A client is brought to the emergency department by emergency medical services with a flaccid right arm and leg and lack of verbal response. The stroke alert team is initiated. The nurse takes which priority action? a. Determine onset of symptoms b. Ensure that the client has 2 large-bore intravenous (IV) lines c. Maintain patent airway d. Prepare for head CT scan

c. Maintain patent airway

a client with C7 injury is flushed, anxious, and complaints of headache. symptoms anticipated a. oliguria b. respiratory depression c. hypertension and bradycardia d. symptoms of shock

c. hypertension and bradycardia

first action to be done at the accident site if the patient alert oriented a. open airway with head-tilt chin lift method b. CT scan c. immobilize head and neck d. give fluids

c. immobilize head and neck

The nurse is caring for a client with increased intracranial pressure (ICP). Which statement by the unlicensed assistive personnel would require immediate intervention by the nurse? a. "I will raise the head of the bed so it is easier to see the television" b. "I will turn down the lights when I leave" c. "Let me move your belongings closer so you can reach them" d. "You should do deep breathing and coughing exercises"

d. "You should do deep breathing and coughing exercises"

The emergency department nurse is triaging clients. Which neurologic presentation is most concerning for a serious etiology and should be given priority for definitive treatment? a. History of bell's palsy wit unilateral facial droop and drooling b. History of multiple sclerosis and reporting recent blurred vision c. Reports unilateral facial pain when consuming hot foods d. Temple region hit by ball, loss of consciousness, but Glasgow score is now 14

d. Temple region hit by ball, loss of consciousness, but Glasgow score is now 14

which is not a cause of brain absess a. otitis media b. meningitis c. endocarditis d. arthritis

d. arthritis

a client with C7 injury. which condition should the nurse anticipate at the acute phase? a. absent corneal reflex b. movement of only right or left half of the body c. decerebrate posturing d. the need for mechanical ventilation

d. the need for mechanical ventilation

person at risk for developing hemorrhagic shock after the rupture of cerebral aneurysm a. person with hep c b. person with controlled HTN c. person with pancreatitis d. women with arteriovenous malformation

d. women with arteriovenous malformation

The nurse is assigned an infant with a possible neurological disorder. Which of the following assessment findings should you communicate to the physician as a late sign of increased intracranial pressure? a) headache and sunset eyes. b) dizziness and irritability. c) decorticate posturing and fixed and dilated pupils d) decreased pupil reaction and decreased respiration.

decorticate posturing and fixed and dilated pupils

The nurse has developed a nursing plan for the care of a 6-year-old girl with congenital hydrocephalus whose shunt has become infected. The most important discharge teaching point for this family is: a) ensuring the parents know how to properly give antibiotics. b) encouraging development of motor skills. c) establishing seizure precautions for the child. d) maintaining effective cerebral perfusion.

ensuring the parents know how to properly give antibiotics.

A child is diagnosed with aseptic meningitis. The child's mother states, "I don't know where she would have picked this up." The nurse prepares to respond to the mother based on the understanding that this disorder is most likely caused by: a) Escherichia coli. b) Haemophilus influenza type B. c) enterovirus. d) Streptococcus group B.

enterovirus

Which clinical manifestation would be exhibited by a client following a hemorrhagic stroke of the right hemisphere?

neglect to left side This client would have deficits on the left side. Expressive aphasia typically occurs with left-hemisphere stroke.

Any individual taking phenobarbital for a seizure disorder should be taught a) never to go swimming. b) never to discontinue the drug abruptly. c) to brush his or her teeth four times a day. d) to avoid foods containing caffeine.

never to discontinue the drug abruptly. Correct Explanation: Phenobarbital should always be tapered, not stopped abruptly, or seizures from the child's dependency on the drug can result.

The client comes to the emergency department status post fall. The client is squinting both eyes and reports sudden blurry vision. The nurse is aware that this deficit reflects injury to which area of the brain? Left-clicking the mouse will put an X to show the answer before submitting the question. Hotspot Question

occipital lobe

When assisting a child while she is having a tonic-clonic seizure, it would be important to a) place a tongue blade between the child's teeth. b) restrain the child from all movement. c) turn the child onto her back and observe her. d) protect the child from hitting her arms against furniture.

protect the child from hitting her arms against furniture.

client having autonomic dysreflexia becomes HTN. important first intervention by the RN a. check the foley cath b. put the client in trendelenburg position c. put the client in flat bed d. put the client in high fowlers position

put the client in high fowlers position

A school-aged girl with seizures is prescribed phenytoin sodium, 75 mg four times a day. An instruction you would want to give her parents regarding this is a) even small doses may cause noticeable dizziness. b) watching television while taking the drug may cause seizures. c) numbness of the fingers is common while taking this drug. d) their child will have to practice good tooth brushing.

their child will have to practice good tooth brushing.

A school-aged girl with seizures is prescribed phenytoin sodium, 75 mg four times a day. An instruction you would want to give her parents regarding this is a) their child will have to practice good tooth brushing. b) even small doses may cause noticeable dizziness. c) watching television while taking the drug may cause seizures. d) numbness of the fingers is common while taking this drug.

their child will have to practice good tooth brushing. Explanation: A side effect of phenytoin sodium is hypertrophy of the gumline. Good tooth brushing helps prevent inflammation under the hypertrophied tissue.

An 8-year-old girl is diagnosed as having tonic-clonic seizures. You would want to teach her parents that: a) their daughter should be kept quiet late in the day when she is most likely to have a seizure. b) if their daughter shows symptoms of beginning a seizure, immediately give her medication. c) their daughter should maintain an active lifestyle. d) their daughter should carry a padded tongue blade with her at all times.

their daughter should maintain an active lifestyle.

mechanism of action of nimodipine

works on blood vessel to prevent spasm after hemorrhage its a CCB

The nurse determines that a child is experiencing late signs of increased intracranial pressure based on assessment of which of the following? Select all that apply. a) Bradycardia b) Fixed dilated pupils c) Increased blood pressure d) Irregular respirations e) Sunset eyes

• Bradycardia • Fixed dilated pupils • Irregular respirations Explanation: Late signs of increased intracranial pressure include bradycardia, fixed and dilated pupils, and irregular respirations. Increased blood pressure and sunset eyes are early signs of increased intracranial pressure.

The nurse is caring for a 6-year-old child with an external ventricular drainage device. The nurse is concerned about the minimal drainage in the past few hours. What actions by the nurse are indicated?

• Check tubing clamps to ensure they are open. • Ensure the tubing is not kinked. Nursing care of an external ventricular drainage device requires the nurse ensure all connections are secure and labeled. The amount of drainage requires close observation. If drainage is absent or minimal the nurse must assess the tubing to make certain it is not clamped or kinked. The level of the drip chamber must be set at the height of the child (at the clavicle). Taking the temperature will be useful to assess for the presence of infection but that is not currently a concern. Asking the child to cough and deep breathe should not be done. Deep breathing is beneficial for all postoperative clients, but coughing may increase pressures and should be avoided.

An 11-year-old child was recently diagnosed with chickenpox. His parents gave him aspirin for a fever and the child is now hospitalized. Which nursing interventions are appropriate for this child? Select all that apply. a) Assess child's skin for the development of distinctive rash every 4 hours b) Assess intake and output every shift c) Request order for an antiemetic d) Request order for anticonvulsant e) Monitor the child's laboratory values related to pancreatic function

• Request order for an antiemetic • Assess intake and output every shift • Request order for anticonvulsant

When assessing a neonate for seizures, what would the nurse expect to find?

• Tachycardia • Elevated blood pressure • Jitteriness • Ocular deviation Neonatal seizures may be difficult to recognize but may be manifested by tremors, jitteriness, tachycardia and elevated blood pressure, and ocular deviation. Tonic-clonic contractions typically are more common in older children.

When caring for a child who has a history of seizures, which nursing interventions would be appropriate? Select all that apply. a) The nurse positions the child on the side during a seizure. b) The nurse has oxygen available to use during a seizure. c) The nurse pads the crib or side rails before a seizure. d) The nurse teaches the caregivers regarding seizure precautions. e) The nurse places a washcloth in the mouth to prevent injury during seizure. f) The nurse goes for help as soon as a seizure begins.

• The nurse pads the crib or side rails before a seizure. • The nurse positions the child on the side during a seizure. • The nurse goes for help as soon as a seizure begins. • The nurse has oxygen available to use during a seizure. • The nurse teaches the caregivers regarding seizure precautions.

The nurse is using the pediatric Glasgow Coma Scale to assess a child's level of consciousness. Which of the following would the nurse assess? Select all that apply. a) Verbal response b) Motor response c) Posture d) Eye opening e) Fontanels

• Verbal response • Motor response • Eye opening Correct Explanation: The pediatric Glasgow Coma Scale assesses level of consciousness using three parts: eye opening, verbal response, and motor response.

The home health nurse is visiting a client with a diagnosis of multiple sclerosis. The client has been taking oxybutynin. The nurse evaluates the effectiveness of the medication by asking the client which assessment question? 1. "Are you consistently fatigued?" 2. "Are you having muscle spasms?" 3. "Are you getting up at night to urinate?" 4. "Are you having normal bowel movements?"

"Are you getting up at night to urinate?" Oxybutynin is an antispasmodic used to relieve symptoms of urinary urgency, frequency, nocturia, and incontinence in clients with uninhibited or reflex neurogenic bladder. Expected effects include improved urinary control and decreased urinary frequency, incontinence, and nocturia. The questions in the remaining options are unrelated to the use of this medication.

The home care nurse is performing an assessment on a client with a diagnosis of Bell's palsy. Which assessment question will elicit specific information regarding this client's disorder? 1. "Do your eyes feel dry?" 2. "Do you have any spasms in your throat?" 3. "Are you having any difficulty chewing food?" 4. "Do you have any tingling sensations around your mouth?"

"Are you having any difficulty chewing food?" Bell's palsy is a one-sided facial paralysis caused by compression of the facial nerve. Manifestations include facial droop from paralysis of the facial muscles; increased lacrimation; painful sensations in the eye, face, or behind the ear; and speech or chewing difficulties.

During a class on stroke, a junior nursing student asks what the clinical manifestations of stroke are. What would be the instructor's best answer? a) "Clinical manifestations of a stroke generally include aphasia, one-sided flaccidity, and trouble swallowing." b) "Clinical manifestations of a stroke are highly variable, depending on the cardiovascular health of the client." c) "Clinical manifestations of a stroke depend on the area of the cortex, the affected hemisphere, the degree of blockage, and the availability of collateral circulation." d) "Clinical manifestations of a stroke depend on how quickly the clot can be dissolved."

"Clinical manifestations of a stroke depend on the area of the cortex, the affected hemisphere, the degree of blockage, and the availability of collateral circulation." Clinical manifestations following a stroke are highly variable and depend on the area of the cerebral cortex and the affected hemisphere, the degree of blockage (total, partial), and the presence or absence of adequate collateral circulation. (Collateral circulation is circulation formed by smaller blood vessels branching off from or near larger occluded vessels.)

Transcutaneous electrical nerve stimulation (TENS) is prescribed for a client with pain, and the nurse instructs the client about the purpose of the TENS unit. Which statement by the client indicates the need for further teaching? 1. "The unit relieves pain." 2. "Electrodes are attached to the skin." 3. "The unit will reduce the need for analgesics." 4. "Hospitalization is required because the unit is not portable."

"Hospitalization is required because the unit is not portable." The TENS unit is portable and the client controls the system for relieving pain and reducing the need for analgesics. It is attached to the skin of the body by electrodes. Hospitalization is not required.

The nurse is reinforcing discharge instructions to a client who has undergone transsphenoidal surgery for a pituitary adenoma. Which statement by the client indicates the client understands the discharge instructions?

"I need to call the doctor if I develop frequent swallowing or postnasal drip. The client should report frequent swallowing or postnasal drip after transsphenoidal surgery because it could indicate cerebrospinal fluid (CSF) leakage. The surgeon removes the nasal packing, usually after 24 hours. The client should deep breathe, but coughing is contraindicated because it could cause increased intracranial pressure (ICP). The client should also report a severe headache because it could indicate increased ICP.

The nurse has given suggestions to the client with trigeminal neuralgia about strategies to minimize episodes of pain. The nurse determines that the client needs further teaching if the client made which statement?

"I will try to eat my food either very warm or very cold." Facial pain can be minimized by using cotton pads and room temperature water to wash the face. The client should chew on the unaffected side of the mouth, eat a soft diet, and take in foods and beverages at room temperature. If toothbrushing triggers pain, sometimes an oral rinse after meals is more helpful.

The parents of a child with a history of seizures who has been taking phenytoin (Dilantin) ask the nurse why it's difficult to maintain therapeutic plasma levels of this medication. Which statement by the nurse would be most accurate? a) "Small increments in dosage lead to sharp increases in plasma drug levels." b) "The capacity to metabolize the drug becomes overwhelmed over time." c) "A drop in the plasma drug level will lead to a toxic state." d) "Large increments in dosage lead to a more rapid stabilizing therapeutic effect."

"Small increments in dosage lead to sharp increases in plasma drug levels."

The nurse is observing a new nursing graduate who is preparing an intermittent intravenous (IV) infusion of phenytoin (Dilantin) for a client with a diagnosis of seizures. Which solution used by the nursing graduate should indicate to the nurse an understanding of proper preparation of this medication? 1. 5% dextrose in water 2. Lactated Ringer's solution 3. 0.9% sodium chloride (normal saline) 4. 5% dextrose and 0.45% sodium chloride

0.9% sodium chloride (normal saline) Rationale: Intermittent IV infusion of phenytoin is administered by injection into a large vein, using normal saline solution. Dextrose solutions are avoided because the medication will precipitate in these solutions. Options 1, 2, and 4 identify incorrect solutions for IV administration with this medication.

A client having an ischemic stroke arrives at the emergency department. The health care provider prescribes tissue plasminogen activator (tPA). Which client statement would be most important to clarify before administering tPA? 1. "I can't believe this is happening right after my stomach surgery." 2. "I had a concussion after a car accident a year ago." 3. "I started noticing my right arm becoming weak approximately an hour ago." 4. "I stopped taking my warfarin 4 weeks ago."

1

A client with a ventriculoperitoneal shunt has a dazed appearance and grunting and has not responded to the caregiver for 10 minutes. Status epilepticus is suspected. Which nursing intervention should be performed first? 1. Administer rectal diazepam 2. Assess for neck stiffness and Brudzinski sign 3. Draw blood for laboratory studies 4. Transport the client to CT for assessment of shunt malfunction

1

The nurse is providing care for a client newly diagnosed with early Alzheimer disease (AD). On assessment, which finding would the nurse expect to discover? 1. Short-term memory impairment 2. Rapid mood swings 3. Physical aggressiveness 4. Increased confusion at night

1 One of the first symptoms of AD is short-term memory impairment. Behavioral changes that occur late in the disease progression include rapid mood swings, tendency toward physical and verbal aggressiveness, and increased confusion at night (when light is inadequate) or when the client is excessively fatigued.

A client has suffered damage to Broca's area of the brain. The nurse providing care for this client anticipates that which area will be affected? 1.Speech 2.Hearing 3.Balance 4.Level of consciousness

1 Rationale: Broca's area in the brain is responsible for the motor aspects of speech, through coordination of the muscular activity of the tongue, mouth, and larynx. The term assigned to damage in this area is aphasia. The items listed in the other options are not the responsibility of Broca's area.

A client has a cerebellar lesion. The nurse determines that the client is adapting successfully to this problem if the client demonstrates proper use of which item? 1.Walker 2.Slider board 3.Raised toilet seat 4. Adaptive eating utensils

1 Rationale: The cerebellum is responsible for balance and coordination. A walker would provide stability for the client during ambulation. Adaptive eating utensils may be beneficial if the client has partial paralysis of the hand. A raised toilet seat is useful if the client does not have the mobility or ability to flex the hips. A slider board is used in transferring a client from a bed to a stretcher or wheelchair.

A client who is experiencing an inferior wall myocardial infarction has had a drop in heart rate into the range of 50 to 56 beats/minute. The client also is complaining of nausea. The nurse understands that these symptoms are caused by stimulation of which cranial nerve (CN)? 1. Vagus (CN X) 2.Hypoglossal (CN XII) 3. Spinal accessory (CN XI) 4. Glossopharyngeal (CN IX)

1 Rationale: The vagus nerve is responsible for sensations in the thoracic and abdominal viscera. It also is responsible for the decrease in heart rate because approximately 75% of all parasympathetic stimulation is carried by the vagus nerve. CN XII is responsible for tongue movement. CN XI is responsible for neck and shoulder movement. CN IX is responsible for taste in the posterior two thirds of the tongue, pharyngeal sensation, and swallowing.

The client with a head injury is experiencing signs of increased intracranial pressure (ICP), and mannitol (Osmitrol) is prescribed. The nurse administering this medication expects which as an intended effects of this medication? Select all that apply. 1. Increased diuresis 2. Reduced intracranial pressure 3. Increased osmotic pressure of glomerular filtrate 4. Reduced tubular reabsorption of water and solutes 5. Reabsorption of sodium and water in the loop of Henle

1,2,3,4 Rationale: Mannitol is an osmotic diuretic that induces diuresis by raising the osmotic pressure of glomerular filtrate, there by inhibiting tubular reabsorption of water and solutes. It is used to reduce intracranial pressure in the client with head trauma. The incorrect option would cause fluid retention through reabsorption, thereby increasing the intracranial pressure.

The nurse has just admitted to the nursing unit a client with a basilar skull fracture who is at risk for increased intracranial pressure (ICP). Pending specific health care provider prescriptions, the nurse should plan to place the client in which positions? Select all that apply. 1. Head midline 2. Neck in neutral position 3. Flat, with head turned to the side 4. Head of bed elevated 30 to 45 degrees 5. Head of bed elevated with the neck extended

1. Head midline 2. Neck in neutral position 4. Head of bed elevated 30 to 45 degrees The client who is at risk for or who has increased ICP should be positioned so that the head is in a neutral, midline position. The nurse should avoid flexing or extending the client's neck or turning the head from side to side. The head of the bed should be raised to 30 to 45 degrees. Use of proper positions promotes venous drainage from the cranium to keep ICP down.

A nurse knows that, for a patient with an ischemic stroke, tPA is contraindicated if the blood pressure reading is:

190 mm Hg/120 mm Hg Elevated blood pressure (systolic >185; diastolic >110 mm Hg) is a contraindication to tPA.

The nurse is performing an assessment on a client admitted to the nursing unit with a diagnosis of stroke (brain attack). On assessment, the nurse notes that the client is unable to understand spoken language. The nurse plans care, understanding that the client is experiencing impairment of which areas? 1.The occipital lobe 2.The auditory association areas 3.The frontal lobe and optic nerve tracts 4.Concept formation and abstraction areas

2. Auditory association and storage areas are located in the temporal lobe and relate to understanding spoken language. The occipital lobe contains areas related to vision. The frontal lobe controls voluntary muscle activity, including speech, and an impairment can result in expressive aphasia. The parietal lobe contains association areas for concept formation, abstraction, spatial orientation, body and object size and shape, and tactile sensation. * Focus on the subject, that the client is unable to understand spoken language. Note the relationship of these words and the words auditory association areas in the correct option.

At the end of the work shift, the nurse is reviewing the respiratory status of a client admitted with acute brain attack (stroke) earlier in the day. The nurse determines that the client's airway is patent if which data are identified? 1.Respiratory rate 24 breaths/min, oxygen saturation 94%, breath sounds clear 2.Respiratory rate 18 breaths/min, oxygen saturation 98%, breath sounds clear 3.Respiratory rate 16 breaths/min, oxygen saturation 85%, wheezes bilaterally 4.Respiratory rate 20 breaths/min, oxygen saturation 92%, diminished breath sounds in lung bases

2. The client's airway is most protected if all of the respiratory parameters measured fall within normal limits. Therefore the respiratory rate should ideally be 16 to 20 breaths/min, the oxygen saturation should be greater than 95%, and the breath sounds should be clear. The correct option is the only one that meets all three criteria. * Use the ABCs-airway, breathing, and circulation-to answer the question. Note that the correct option identifies the highest oxygen saturation level.

The clinic nurse is assessing a child suspected of having juvenile rheumatoid arthritis (JRA). Which assessment findings should the nurse expect to note in a child who has been diagnosed with JRA? Select all that apply. 1. Hematuria 2. Morning stiffness 3. Painful, stiff, and swollen joints 4. Limited range of motion of the joints 5. Stiffness that develops later in the day 6. History of late-afternoon temperature

2.3.4.6. Clinical manifestations associated with JRA include intermittent joint pain that lasts longer than 6 weeks and painful, stiff, and swollen joints that are warm to the touch, with limited range of motion. The child will complain of morning stiffness and may protect the affected joint or refuse to walk. Systemic symptoms include malaise, fatigue, lethargy, anorexia, weight loss, and growth problems. A history of a late-afternoon fever with temperature spiking up to 105°F (40.6°C) will also be part of the clinical manifestations. Hematuria is not specifically associated with JRA.

The nurse is caring for a client with an intracranial pressure (ICP) monitoring device. The nurse should become most concerned if the ICP readings drifted to and stayed in the vicinity of which finding? 1. 5 mm Hg 2. 8 mm Hg 3. 14 mm Hg 4. 22 mm Hg

22 mm Hg Normal ICP readings range from 5 to 15 mm Hg pressure. Pressures greater than 20 mm Hg are considered to represent increased ICP, which seriously impairs cerebral perfusion.

The nurse is monitoring a child with a brain tumor for complications associated with increased intracranial pressure. Which finding, if noted by the nurse, would indicate the presence of diabetes insipidus? 1. Weight gain 2. Hypertension 3. High urine output 4. Urine specific gravity greater than 1.030

3 Diabetes insipidus (DI) can occur in a child with increased intracranial pressure. Weight gain, hypertension, and a urine specific gravity greater than 1.030 are indications of the syndrome of inappropriate antidiuretic hormone secretion, not DI. A high urine output would be indicative of DI.

The nurse is assessing a client with fragile X syndrome. The nurse anticipates noting which physical assessment finding? 1. Low, straight palate 2. Short, narrow protruding ears 3. Long, narrow face with a prominent jaw 4. Short, rounded face with an indiscernible jaw

3 Fragile X syndrome is a genetic condition that causes developmental problems including learning disabilities and cognitive impairment. Physical assessment findings of fragile X syndrome include long, wide, and/or protruding ears; a long, narrow face with a prominent jaw; and large testes. Therefore, the descriptions in the remaining options are incorrect.

A nurse is reading the laboratory results for a client being treated with carbamazepine (Tegretol) for prophylaxis of complex partial seizures. When evaluating the client's laboratory data, the nurse determines that which value is consistent with an adverse effect of this medication? 1. Sodium level, 136 mEq/L 2. Platelet count, 350,000 cells/mm3 3. White blood cell count, 3200 cells/mm3 4. Blood urea nitrogen (BUN), 19 mg/dL

3 Rationale: Adverse effects of carbamazepine appear as blood dyscrasias, including aplastic anemia, agranulocytosis, thrombocytopenia, and leukopenia. Other adverse effects include cardiovascular disturbances, thrombophlebitis, dysrhythmias, and dermatological effects.

A client is taking the prescribed dose of phenytoin (Dilantin) to control seizures. Results of a phenytoin blood level study reveal a level of 35 mcg/mL. Which finding would be expected as a result of this laboratory result? 1. Hypotension 2. Tachycardia 3. Slurred speech 4. No abnormal finding

3 Rationale: The therapeutic phenytoin level is 10 to 20 mcg/mL. At a level higher than 20 mcg/mL, involuntary movements of the eyeballs (nystagmus) occur. At a level higher than 30 mcg/mL, ataxia and slurred speech occur.

The nurse is preparing for the admission to the unit of a client with a diagnosis of seizures and is preparing to institute full seizure precautions. Which item is contraindicated for use if a seizure occurs? 1.Oxygen source 2.Suction machine 3.Padded tongue blade 4.Padding for the side rails

3 Rationale: Full seizure precautions include bed rest with padded side rails in a raised position, a suction machine at the bedside, having diazepam (Valium) or lorazepam (Ativan) available, and providing an oxygen source. Objects such as tongue blades are contraindicated and should never be placed in the client's mouth during a seizure.

The nurse is monitoring a client who has returned to the nursing unit after a myelogram. Which client complaint would indicate the need to notify the health care provider (HCP)? 1.Backache 2.Headache 3.Neck stiffness 4. Feelings of fatigue

3 Rationale: Headache is relatively common after the procedure, but neck stiffness, especially on flexion, and pain should be reported because they signal meningeal irritation. The client also is monitored for evidence of allergic reactions to the dye such as confusion, dizziness, tremors, and hallucinations. Feelings of fatigue may be normal, and back discomfort may be owing to the positions required for the procedure.

A client is being hyperventilated by a mechanical ventilator to decrease the client's intracranial pressure (ICP). On monitoring arterial blood gas results, the nurse should expect values that are within which ranges? 1.Pao2 60 to 100 mm Hg, Paco2 25 to 30 mm Hg 2.Pao2 60 to 100 mm Hg, Paco2 30 to 35 mm Hg 3.Pao2 80 to 100 mm Hg, Paco2 25 to 30 mm Hg 4. Pao2 80 to 100 mm Hg, Paco2 35 to 40 mm Hg

3 Rationale: Hyperventilation with a Paco2 of 25 to 30 mm Hg causes cerebral vasoconstriction, which decreases intracranial blood volume and ICP. The Pao2 is not allowed to fall below 80 mm Hg, to prevent cerebral vasodilation from hypoxemia. Therefore, the remaining options are incorrect.

The nurse is assessing a client with a neurological deficit involving the hippocampus. Which finding is indicative of this deficit? 1. Disoriented to client, place, and time 2. Affect flat, with periods of emotional lability 3. Cannot recall what was eaten for breakfast today 4. Unable to add and subtract; does not know who is president

3 Rationale: Recall of recent events and the storage of memories are controlled by the hippocampus, which is a limbic system structure. The cerebral hemispheres, with specific regional functions, control orientation. The limbic system, overall, is responsible for feelings, affect, and emotions. Calculation ability and knowledge of current events are under the control of the frontal lobes of the cerebrum.

A nurse is monitoring an infant for signs of increased intracranial pressure (ICP). On assessment of the fontanelles, the nurse notes that the anterior fontanelle bulges when the infant is sleeping. Based on this finding, which is the priority nursing action? 1.Increase oral fluids. 2.Document the finding. 3.Notify the health care provider. 4.Place the infant supine in a side-lying position.

3 Rationale: The anterior fontanelle is diamond shaped and is located on the top of the head. It should be soft and flat in a normal infant, and it normally closes by 12 to 18 months of age. A larger-than-normal fontanelle may be a sign of increased ICP within the skull. Although the anterior fontanelle may bulge slightly when the infant cries, bulging at rest may indicate increased ICP. Options 1 and 4 are inaccurate interventions and will not be helpful. Although the nurse would document the finding, the priority action would be to report the finding to the health care provider.

A client with Guillain-Barré syndrome has ascending paralysis and is intubated and receiving mechanical ventilation. Which strategy should the nurse incorporate in the plan of care to help the client cope with this illness? 1.Giving client full control over care decisions and restricting visitors 2.Providing positive feedback and encouraging active range of motion 3.Providing information, giving positive feedback, and encouraging relaxation 4. Providing intravenously administered sedatives, reducing distractions, and limiting visitors

3 Rationale: The client with Guillain-Barré syndrome experiences fear and anxiety from the ascending paralysis and sudden onset of the disorder. The nurse can alleviate these fears by providing accurate information about the client's condition, giving expert care and positive feedback to the client, and encouraging relaxation and distraction. The family can become involved with selected care activities and provide diversion for the client as well.

The nurse has given instructions to a client with Parkinson's disease about maintaining mobility. Which action demonstrates that the client understands the directions? 1.Sits in soft, deep chairs to promote comfort. 2.Exercises in the evening to combat fatigue. 3.Rocks back and forth to start movement with bradykinesia. 4. Buys clothes with many buttons to maintain finger dexterity.

3 Rationale: The client with Parkinson's disease should exercise in the morning when energy levels are highest. The client should avoid sitting in soft deep chairs because they are difficult to get up from. The client can rock back and forth to initiate movement. The client should buy clothes with Velcro fasteners and slide-locking buckles to support the ability to dress self.

The nurse has provided instructions to a client with a diagnosis of myasthenia gravis about home care measures. Which client statement indicates the need for further instruction? 1. "I will rest each afternoon after my walk." 2. "I should cough and deep breathe many times during the day." 3. "I can change the time of my medication on the mornings when I feel strong." 4. "If I get abdominal cramps and diarrhea, I should call my health care provider."

3 Rationale: The client with myasthenia gravis and his or her family should be taught information about the disease and its treatment. They should be aware of the side and adverse effects of anticholinesterase medications and corticosteroids and should be taught that timing of anticholinesterase medication is critical. It is important to instruct the client to administer the medication on time to maintain a chemical balance at the neuromuscular junction. If it is not given on time, the client may become too weak to even swallow. Resting after a walk, coughing and deep breathing many times over the day, and calling the health care provider when experiencing abdominal cramps and diarrhea indicate correct understanding of home care instructions to maintain health with this neurological degenerative disease.

A client is receiving several adjunctive professional therapies while rehabilitating after a stroke. Which client statements indicate an understanding of the services? Select all that apply. 1. "Occupational therapy will help me learn how to properly use my walker." 2. "Physical therapy will help me learn how to dress myself again." 3. "Social services can help me find resources for affording my medications." 4. "Speech therapy will teach me how to eat my meals properly." 5. "Wound care will teach me how to properly dress this wound on my knee."

3,4,5

The nurse is preparing to care for a client after a lumbar puncture. The nurse should plan to place the client in which best position immediately after the procedure? Prone in semi-Fowler's position 2. Supine in semi-Fowler's position 3. Prone with a small pillow under the abdomen 4. Lateral with the head slightly lower than the rest of the body

3. Prone with a small pillow under the abdomen Rationale: After the procedure, the client assumes a flat position. If the client is able, a prone position with a pillow under the abdomen is the best position. This position helps reduce cerebrospinal fluid leakage and decreases the likelihood of post-lumbar puncture headache. Options 1, 2, and 4 are incorrect.

A client comes to the emergency department with diplopia and recent onset of nausea. Which statement by the client would indicate to the nurse that this is an emergency? 1. "I am very tired, and it's hard for me to keep my eyes open." 2. "I don't feel good, and I want to be seen." 3. "I have not taken my blood pressure medicine in over a week." 4. "I have the worst headache I've ever had in my life."

4

At 8 AM, medications are prescribed for assigned clients. Which medication should the nurse administer first? 1. Acetylsalicylic acid for a client with a history of coronary artery disease and ischemic stroke 2. Metformin for a client with serum glucose of 285 mg/dL (15.8 mmol/L) who is scheduled for a CT scan with contrast 3. Morphine sulfate for a client with terminal lung cancer who has chronic bone pain 4. Pyridostigmine for a client with myasthenia gravis exacerbation who reports difficulty swallowing

4

The nurse in the outpatient clinic is speaking with a client diagnosed with cerebral arteriovenous malformation. Which statement would be a priority for the nurse to report to the health care provider? 1. "I got short of breath this morning when I worked out." 2. "I have cut down on smoking to 1/2 pack per day." 3. "I haven't been feeling well, so I have been sleeping a lot." 4. "I took an acetaminophen in the waiting room for this bad headache."

4

The nurse is assisting a health care provider (HCP) during the examination of an infant with developmental hip dysplasia. The HCP performs the Ortolani maneuver. The nurse determines that the infant exhibits a positive response to this maneuver if which finding is noted? 1. A shrill cry from the infant 2. Asymmetry of the affected hip 3. Reduced range of motion in the right and left hip 4. A palpable click during abduction of the affected hip

4 In the Ortolani maneuver, the examiner abducts both hips. A positive finding is a palpable click on the affected side during abduction. Crying is expected. Asymmetry and reduced range of motion of the affected hip are not positive signs of this maneuver.

Carbamazepine (Tegretol) has been prescribed for a client. The nurse should tell the client that which blood test will be done periodically while the client is taking this medication? 1. Lipase level 2. Amylase level 3. Ammonia level 4. Complete blood cell (CBC) count

4 Rationale: Carbamazepine is classified as an iminostilbene derivative and is used as an anticonvulsant, antineuralgic, antimanic, and antipsychotic. The medication can cause blood dyscrasias as an adverse effect, and the client should have a CBC done before therapy and periodically during therapy. Additional laboratory tests that should be done include a serum iron determination, urinalysis, blood urea nitrogen determination, and a carbamazepine level. The tests identified in options 1, 2, and 3 are unnecessary.

A client who is taking phenytoin (Dilantin) for a seizure disorder is being admitted to the hospital because of an increase in seizure activity. The client reports severe vomiting for the last 24 hours and inability to take phenytoin during that time. The nurse anticipates that the health care provider will most likely prescribe which medication? 1. Phenobarbital (Luminal) 2. Clonazepam (Klonopin) 3. Valproic acid (Depakene) 4. Fosphenytoin sodium (Cerebyx)

4 Rationale: Fosphenytoin sodium is used for short-term parenteral (intravenous) infusion. A client who is not tolerating medications orally and has a seizure disorder would need an anticonvulsant administered by the parenteral route. Phenobarbital is an antiseizure medication that is given orally or parenterally. However, the medication of choice in this case would be fosphenytoin since its use if for short-term. Valproic acid and clonazepam usually are administered orally.

The parents of a child recently diagnosed with cerebral palsy ask the nurse about the disorder. The nurse bases the response on the understanding that cerebral palsy is which type of condition? 1. An infectious disease of the central nervous system 2. An inflammation of the brain as a result of a viral illness 3. A congenital condition that results in moderate to severe retardation 4. A chronic disability characterized by impaired muscle movement and posture

4 Rationale: Cerebral palsy is a chronic disability characterized by impaired movement and posture resulting from an abnormality in the extrapyramidal or pyramidal motor system. Meningitis is an infectious process of the central nervous system. Encephalitis is an inflammation of the brain that occurs as a result of viral illness or central nervous system infection. Down syndrome is an example of a congenital condition that results in moderate to severe retardation.

The nurse is planning care for the client with a neurogenic bladder caused by multiple sclerosis. The nurse plans for fluid administration of at least 2000 mL/day. Which plan would be most helpful to this client? 1.400 to 500 mL with each meal and 500 to 600 mL in the evening before bedtime 2.400 to 500 mL with each meal and additional fluids in the morning but not after midday 3.400 to 500 mL with each meal, with all extra fluid concentrated in the afternoon and evening 4.400 to 500 mL with each meal and 200 to 250 mL at midmorning, midafternoon, and late afternoon

4 Rationale: Spacing fluid intake over the day helps the client with a neurogenic bladder to establish regular times for successful voiding. Omitting intake after the evening meal minimizes incontinence or the need to empty the bladder during the night.

The nurse is admitting a client with Guillain-Barré syndrome to the nursing unit. The client has ascending paralysis to the level of the waist. Knowing the complications of the disorder, the nurse should bring which most essential items into the client's room? 1.Nebulizer and pulse oximeter 2.Blood pressure cuff and flashlight 3.Flashlight and incentive spirometer 4.Electrocardiographic monitoring electrodes and intubation tray

4 Rationale: The client with Guillain-Barré syndrome is at risk for respiratory failure because of ascending paralysis. An intubation tray should be available for use. Another complication of this syndrome is cardiac dysrhythmias, which necessitates the use of electrocardiographic monitoring. Because the client is immobilized, the nurse should assess for deep vein thrombosis and pulmonary embolism routinely. Although items in the incorrect options may be used in care, they are not the most essential items from the options provided.

The home care nurse is preparing to visit a client with a diagnosis of trigeminal neuralgia (tic douloureux). When performing the assessment, the nurse should plan to ask the client which question to elicit the most specific information regarding this disorder? 1."Do you have any visual problems?" 2."Are you having any problems hearing?" 3."Do you have any tingling in the face region?" 4."Is the pain experienced a stabbing type of pain?"

4. Trigeminal neuralgia is characterized by spasms of pain that start suddenly and last for seconds to minutes. The pain often is characterized as stabbing or as similar to an electric shock. It is accompanied by spasms of facial muscles that cause twitching of parts of the face or mouth, or closure of the eye. Options 1, 2, and 3 do not elicit data specifically related to this disorder. * Note the strategic word most. Focus on the subject, assessment of trigeminal neuralgia. Note the word neuralgia in the question and the relationship of this word to pain noted in the correct option.

A client with trigeminal neuralgia is being treated with carbamazepine (Tegretol), 400 mg orally daily. Which value indicates that the client is experiencing an adverse effect to the medication? 1. Uric acid level, 5 mg/dL 2. Sodium level, 140 mEq/L 3. Blood urea nitrogen level, 15 mg/dL 4. White blood cell count, 3000 cells/mm3

4Rationale: Adverse effects of carbamazepine appear as blood dyscrasias, including aplastic anemia, agranulocytosis, thrombocytopenia, and leukopenia; cardiovascular disturbances including thrombophlebitis and dysrhythmias; and dermatological effects. The low white blood cell count reflects agranulocytosis. The laboratory values in options 1, 2, and 3 are normal values.

A client with trigeminal neuralgia asks the nurse what causes the painful episodes associated with the condition. The nurse's response is based on an understanding that the symptoms can be triggered by which process? 1.A local reaction to nasal stuffiness 2.A hypoglycemic effect on the cranial nerve 3.Release of catecholamines with infection or stress 4. Stimulation of the affected nerve by pressure and temperature

4Rationale: The paroxysms of pain that accompany this neuralgia are triggered by stimulation of the terminal branches of the trigeminal nerve. Symptoms can be triggered by pressure from washing the face, brushing the teeth, shaving, eating, or drinking. Symptoms also can be triggered by thermal stimuli, such as a draft of cold air. Options 1, 2, and 3 are incorrect.

A client is scheduled for an EEG after having a seizure for the first time. Client preparation for this test should include which instruction? a) "Avoid stimulants and alcohol for 24 to 48 hours before the test." b) "Don't shampoo your hair for 24 hours before the test." c) "Don't eat anything for 12 hours before the test." d) "Avoid thinking about personal matters for 12 hours before the test."

A

A client recently experienced a stroke with accompanying left-sided paralysis. His family voices concerns about how to best interact with him. They report the client doesn't seem aware of their presence when they approach him on his left side. What advice should the nurse give the family? a) "The client is unaware of his left side. You should approach him on the right side." b) "The client is feeling an emotional loss. He'll eventually start acknowledging you on his left side." c) "This condition is temporary." d) "The client is unaware of his left side. You need to encourage him to interact from this side."

A

A client with a suspected brain tumor is scheduled for a computed tomography (CT) scan. What should the nurse do when preparing the client for this test? a) Determine whether the client is allergic to iodine, contrast dyes, or shellfish. b) Immobilize the neck before the client is moved onto a stretcher. c) Administer a sedative as ordered. d) Place a cap over the client's head.

A

Which of the following clients requires increased sensory stimulation to prevent sensory deprivation? a) A 65-year-old client who has employment-induced presbycusis and advanced glaucoma b) A 24-year-old client who has been admitted with an anxiety disorder and appears very agitated c) An 84-year-old client who has hemiparesis and ambulates with a walker d) A 60-year-old client who is blind, reads books through use of Braille, listens to the radio, and regularly takes walks around the unit

A

Which of the following nursing intervention can prevent a client from experiencing autonomic dysreflexia? a) Monitoring the patency of an indwelling urinary catheter b) Assessing laboratory test results as ordered c) Placing the client in the Trendelenburg's position d) Administering zolpidem tartrate

A

A nurse is assisting with a community screening for people at high risk for stroke. To which of the following clients would the nurse pay most attention?

A 60-year-old African-American man The 60-year-old African-American man has three risk factors: gender, age, and race. African Americans have almost twice the incidence of first stroke compared with Caucasians

The nurse is preparing a plan of care for a child with a head injury. On review of the records, the nurse notes that the health care provider has documented decorticate posturing. The nurse plans care knowing that this type of posturing indicates which finding? Damage to the pons 2. Damage to the midbrain 3. Damage to the diencephalon 4. A lesion in the cerebral hemisphere

A lesion in the cerebral hemisphere Decorticate posturing indicates a lesion in the cerebral hemisphere or disruption of the corticospinal tracts. Decerebrate posturing indicates damage in the diencephalon, midbrain, or pons.

The home care nurse is making extended follow-up visits to a client discharged from the hospital after a moderately severe head injury. The family states that the client is behaving differently than before the accident. The client is more fatigued and irritable and has some memory problems. The client, who was previously very even tempered, is prone to outbursts of temper now. The nurse determines that these behaviors are indicative of which problem? 1. Intracranial pressure 2. A long-term sequela of the 3. A worsening of the original 4. A short-term problem that will resolve in about 1 month

A long-term sequela of the injury Clients with moderate to severe head injury usually have residual physical and cognitive disabilities; these include personality changes, increased fatigue and irritability, mood alterations, and memory changes. The client also may require frequent to constant supervision. The nurse assesses the family's ability to cope and makes appropriate referrals to respite services, support groups, and state or local chapters of the National Head Injury Foundation.

The nurse is caring for a client with a diagnosis of cerebrovascular accident (CVA) with left-sided hemiparesis. What would be important nursing measures in the acute phase of care? Select all that apply. a) Turn and position every 2 hours. b) Perform passive range of motion on the affected side. c) Support the affected side with pillows. d) Perform active range of motion on both sides. e) Perform passive range of motion on both sides.

A,B,C

A client with a spinal cord injury becomes angry and belligerent whenever the nurse tries to administer care. The nurse should perform which action? 1. Ask the family to deliver the care. 2. Leave the client alone until ready to participate. 3. Advise the client that rehabilitation progresses more quickly with cooperation. 4. Acknowledge the client's anger and continue to encourage participation in care.

Acknowledge the client's anger and continue to encourage participation in care. Adjusting to paralysis is physically and psychosocially difficult for the client and family. The nurse recognizes that the client goes through the grieving process in adjusting to the loss and may move back and forth among the stages of grief. The nurse acknowledges the client's feelings while continuing to meet the client's physical needs and encouraging independence. The family also is in crisis and needs the nurse's support and should not be relied on to provide care. The nurse cannot simply neglect the client until the client is ready to participate. Option 3 represents a factual but noncaring approach to the client and is not therapeutic.

A nurse is teaching a community group about modifiable and nonmodifiable risk factors for ischemic strokes. Which of the following is a risk factor that cannot be modified?

Advanced age Modifiable risk factors for ischemic stroke include hypertension, atrial fibrillation, hyperlipidemia, diabetes mellitus, smoking, asymptomatic carotid stenosis, obesity, and excessive alcohol consumption. Nonmodifiable risk factors include advanced age, gender, and race.

The nurse is evaluating a function of the limbic system as a part of the neurological status of a client. What should the nurse assess? 1. Experience of pain 2. Affect or emotions 3. Response to verbal stimuli 4. Insight, judgment, and planning

Affect or emotions Affect and emotions are part of the role of the limbic system and involve both hemispheres of the brain. Pain is a complex experience involving several areas of the central nervous system. The response to verbal stimuli is part of the level of consciousness, which is under the control of the reticular activating system and both cerebral hemispheres. Insight, judgment, and planning are part of the functions of the frontal lobes of the brain in conjunction with association fibers connecting to other areas of the cerebrum.

Which term refers to the failure to recognize familiar objects perceived by the senses?

Agnosia Auditory agnosia is failure to recognize significance of sounds. Agraphia refers to disturbances in writing intelligible words. Apraxia refers to an inability to perform previously learned purposeful motor acts on a voluntary basis. Perseveration is the continued and automatic repetition of an activity, word, or phrase that is no longer appropriate.

The nurse is preparing a client who is scheduled to have cerebral angiography performed. Which should the nurse check before the procedure?

Allergy to iodine or shellfish The client undergoing cerebral angiography is assessed for possible allergy to the contrast dye, which can be determined by questioning the client about allergies to iodine or shellfish. Allergy to salmon is not associated with this procedure. Claustrophobia and excessive weight are areas of concern with magnetic resonance imaging.

A client with a history of atrial fibrillation has experienced a TIA. In an effort to reduce the risk of cerebrovascular accident (CVA), the nurse anticipates the priority medical treatment to include which of the following?

Anticoagulant therapy Anticoagulant or antiplatelet therapy can prevent clot formation associated with cardiac dysrhythmias such as atrial fibrillation. Cholesterol-lowering drugs can be ordered if indicated to manage atherosclerosis. Prothrombin and international normalized ratio (INR) levels may be ordered to monitor therapeutic effects of anticoagulant therapy. Carotid endarterectomy would be anticipated only when the carotids have narrowing from plaque.

After teaching a group of students about neural tube disorders, the instructor determines that additional teaching is needed when the students identify which of the following as a neural tube defect? a) Anencephaly b) Encephalocele c) Arnold-Chiari malformation d) Spina bifida occulta

Arnold-Chiari malformation Correct Explanation: Arnold-Chiari malformation is a deformity of the cerebellar tonsils being displaced into the upper cervical canal. Anencephaly is a neural tube defect. Encephalocele is a neural tube defect Spina bifida occulta is a neural tube defect.

A group of nursing students are reviewing cerebral vascular disorders and risk factors in children. The students demonstrate understanding of the material when they identify which of the following as a risk factor for hemorrhagic stroke? a) Meningitis b) Congenital heart defect c) Sickle cell disease d) Arteriovenous malformations (AVMs)

Arteriovenous malformations (AVMs) Correct Explanation: Vascular malformations such as intracranial AVMs are a risk factor for hemorrhagic stroke. Sickle cell disease is a risk factor for ischemic stroke. Congenital heart defects are risk factors for ischemic stroke. Meningitis or other infection is a risk factor for ischemic stroke.

The nurse is preparing a plan of care for a client with a brain attack (stroke) who has global aphasia. The nurse incorporates communication strategies in the plan of care, knowing that the client's speech should fit which characterization?

Associated with poor comprehension Global aphasia is a condition in which a person has few language skills as a result of extensive damage to the left hemisphere. The speech is nonfluent and is associated with poor comprehension and limited ability to name objects or repeat words. The client with conduction aphasia has difficulty repeating words spoken by another, and the speech is characterized by literal paraphasia with intact comprehension. The client with Wernicke's aphasia may exhibit a rambling type of speech.

The nurse is assessing the client's gait and notes it is unsteady and staggering. Which description should the nurse use when documenting the assessment finding? 1. Spastic 2. Ataxic 3. Festinating 4. Dystrophic or broad-based

Ataxic An ataxic gait is characterized by unsteadiness and staggering. A spastic gait is characterized by stiff, short steps with the legs held together, hip and knees flexed, and toes that catch and drag. A festinating gait is best described as walking on the toes with an accelerating pace. A dystrophic or broad-based gait is seen as waddling, with the weight shifting from side to side and the legs far apart.

A client who has a spinal cord injury that resulted in paraplegia experiences a sudden onset of severe headache and nausea. The client is diaphoretic with piloerection and has flushing of the skin. The client's systolic blood pressure (BP) is 210 mm Hg. What should the nurse immediately suspect? 1. Return of spinal shock 2. Malignant hypertension 3. Impending brain attack (stroke) 4. Autonomic dysreflexia (hyperreflexia)

Autonomic dysreflexia (hyperreflexia) Autonomic dysreflexia (hyperreflexia) results from sudden strong discharge of the sympathetic nervous system in response to a noxious stimulus. Signs and symptoms include pounding headache, nausea, nasal stuffiness, flushed skin, piloerection, and diaphoresis. Severe hypertension can occur, with a systolic BP rising potentially as high as 300 mm Hg. It often is triggered by thermal or mechanical events such as a kinking of catheter tubing, constipation, urinary tract infection, or any variety of cutaneous stimuli. The nurse must recognize this situation immediately and take corrective action to remove the stimulus. If untreated, this medical emergency could result in stroke, status epilepticus, or possibly death.

The nurse is providing instructions to the client with trigeminal neuralgia regarding measures to take to prevent the episodes of pain. Which should the nurse instruct the client to do? 1. Prevent stressful situations. 2. Avoid activities that may cause fatigue. 3. Avoid contact with people with an infection. 4. Avoid activities that may cause pressure near the face.

Avoid activities that may cause pressure near the face. The pain that accompanies trigeminal neuralgia is triggered by stimulation of the trigeminal nerve. Symptoms can be triggered by pressure such as from washing the face, brushing the teeth, shaving, eating, or drinking. Symptoms also can be triggered by stimulation by a draft or cold air. The remaining options are not associated with triggering episodes of pain.

Which nursing action should be included in the care plan to promote comfort in a 4-year-old child hospitalized with meningitis? a) Keep the lights on brightly so that he can see his mother b) Rock the child frequently c) Have the child's 2-year-old brother stay in the room d) Avoid making noise when in the child's room

Avoid making noise when in the child's room

An older adult has vertigo accompanied with tinnitus as the result of Ménière's disease. The nurse should instruct the client to restrict which dietary element? a) protein b) sodium c) fluids d) potassium

B

The unconscious client is to receive 200 mL of tube feeding every 4 hours. The nurse checks for the client's gastric residual before administering the next scheduled feeding and obtains 40 mL of gastric residual. The nurse should: a) dispose of the residual and continue with the feeding. b) readminister the residual to the client and continue with the feeding. c) withhold the tube feeding and notify the health care provider (HCP). d) delay feeding the client for 1 hour and then recheck the residual.

B

To encourage adequate nutritional intake for a client with Alzheimer's disease, a nurse should: a) fill out the menu for the client. b) stay with the client and encourage him to eat. c) give the client privacy during meals. d) help the client fill out his menu.

B

The nurse is monitoring a client with a blunt head injury sustained from a motor vehicle crash. Which would indicate a basal skull fracture as a result of the injury?

Bloody or clear drainage from the auditory canal Bloody or clear watery drainage from the auditory canal indicates a cerebrospinal fluid leak following trauma and suggests a basal skull fracture. This warrants immediate attention. Option 3 is indicative of an infectious process. Options 1 and 2 are not specifically associated with a basal skull fracture.

The nurse is preparing for the admission of a client with a suspected diagnosis of herpes simplex encephalitis. Which diagnostic test should be prescribed to confirm this diagnosis?

Brain biopsy The diagnosis of herpes simplex encephalitis can be made by brain biopsy and is rarely made from the culture of cerebrospinal fluid obtained from a lumbar puncture. The EEG is abnormal, in many cases, indicating temporal lobe abnormalities, but it will not confirm the diagnosis. The CT scan is normal up to the first 5 days, with low-density lesions in the temporal lobe noted later.

The nurse is instructing a client with Ménière's disease how to recognize vertigo. The nurse should tell the client to notice: a) light-headedness. b) an episode of blackout. c) a feeling that the environment is in motion. d) narrowed vision preceding fainting.

C

What assessment findings would indicate an emergency myasthenia crisis? a) Impairment of functioning of the autonomic and skeletal muscles b) Airway obstruction, profound muscle weakness, and inability to move c) Severe dyspnea, intensification of dysphagia, and dysarthria d) Paralysis of the muscles and hyperventilation

C

A 30-year-old was admitted to the progressive care unit with a C5 fracture from a motorcycle accident. Which of the following assessments would take priority? A Bladder distension B Neurological deficit C pulse ox readings D The client's feelings about the injury

C pulse ox readings After a spinal cord injury, ascending cord edema may cause a higher level of injury. The diaphragm is innervated at the level of C4, so assessment of adequate oxygenation and ventilation is necessary. Although the other options would be necessary at a later time, observation for respiratory failure is the priority.

The nurse is caring for a client with a neurological deficit involving the hippocampus. On assessment of the client, which signs and symptoms would most likely be noted? 1. Disoriented to client, place, and time 2. Affect flat, with periods of emotional lability 3. Cannot recall what was eaten for breakfast today 4. Unable to add and subtract; does not know who is president

Cannot recall what was eaten for breakfast today Recall of recent events and the storage of memories are controlled by the hippocampus, which is a limbic system structure. The cerebral hemispheres, with specific regional functions, control orientation. The limbic system, overall, is responsible for feelings, affect, and emotions. Calculation ability and knowledge of current events are under the control of the frontal lobes of the cerebrum.

22s A patient presents to the emergency room with complaints of having an "exploding headache" for the last 2 hours. The patient is immediately seen by a triage nurse who suspects the patient is experiencing a stroke. Which of the following is a possible cause based on the characteristic symptom?

Cerebral aneurysm A cerebral aneurysm is a type of hemorrhagic stroke that is characterized by an exploding headache.

A client with myasthenia gravis is experiencing prolonged periods of weakness. The health care provider prescribes a test dose of edrophonium (Enlon) and the client becomes weaker. The nurse interprets this outcome as indicative of which result?

Cholinergic crisis Edrophonium is administered to differentiate overdose of medication (cholinergic crisis) from the need for increased medication (myasthenic crisis). Worsening of the symptoms after edrophonium is administered indicates a cholinergic crisis (overdose of the medication), or a negative test.

The nurse knows that the heads of infants and toddlers are large in proportion to their bodies, placing them at risk for which of the following? a) Intracranial hemorrhaging b) Congenital hydrocephalus c) Positional plagiocephaly d) Closed head injury

Closed head injury Correct Explanation: A larger head size in relation to the rest of their body size gives young children a higher center of gravity, which causes them to hit their head more readily, thus placing them at risk for closed head injury. Fragile capillaries in the periventricular area of the brain put preterm infants at risk for intracranial hemorrhage. Congenital hydrocephalus may be caused by abnormal intrauterine development or infection. Positional plagiocephaly is caused by an infant's head remaining in the same position for too long.

A child is diagnosed with bacterial meningitis. The nurse would suspect which abnormality of cerebrospinal fluid (CSF)? a) Cloudy appearance b) Elevated sugar c) Decreased pressure d) Decreased leukocytes

Cloudy appearance

The nurse in the neurological unit is caring for a client who was in a motor vehicle crash and sustained a blunt head injury. On assessment of the client, the nurse notes the presence of bloody drainage from the nose. Which nursing action is most appropriate? 1. Insert nasal packing. 2. Document the findings. 3. Contact the health care provider (HCP). 4. Monitor the client's blood pressure and check for signs of increased intracranial pressure.

Contact the health care provider (HCP). Bloody or clear drainage from either the nasal or the auditory canal after head trauma could indicate a cerebrospinal fluid leak. The appropriate nursing action is to notify the HCP, because this finding requires immediate intervention. The remaining options are inappropriate nursing actions in this situation.

The nurse is discussing with a parent the difference between a breath-holding spell and a seizure. The nurse would be correct in telling the parent which of the following with regards to seizures? a) The EEG is normal. b) Cyanosis occurs at the onset of the seizure. c) The patient is bradycardiac. d) Convulsive activity occurs.

Convulsive activity occurs. Explanation: During seizures convulsive activity is typically noted. During a breath-holding spell, the child is bradycardiac, cyanosis occurs at the onset, and the EEG is normal.

When the nurse talks with a client with multiple sclerosis who has slurred speech, which nursing intervention is contraindicated? a) encouraging the client to speak slowly b) asking the client to repeat indistinguishable words c) encouraging the client to speak distinctly d) asking the client to speak louder when tired

D

The nurse is preparing for the admission of a client with a suspected diagnosis of Guillain-Barré syndrome. When the client arrives at the nursing unit, the nurse reviews the health care provider's documentation. The nurse expects to note documentation of which hallmark clinical manifestation of this syndrome? 1. Multifocal seizures 2. Altered level of consciousness 3. Abrupt onset of a fever and headache 4. Development of progressive muscle weakness

Development of progressive muscle weakness A hallmark clinical manifestation of Guillain-Barré syndrome is progressive muscle weakness that develops rapidly. Seizures are not normally associated with this disorder. The client does not have symptoms such as a fever or headache. Cerebral function, level of consciousness, and pupillary responses are normal.

The nurse is monitoring a 3-month-old infant for signs of increased intracranial pressure. On palpation of the fontanels, the nurse notes that the anterior fontanel is soft and flat. On the basis of this finding, which nursing action is most appropriate? Increase oral fluids. 2. Document the finding. 3. Notify the health care provider (HCP). 4. Elevate the head of the bed to 90 degrees.

Document the finding. Rationale The anterior fontanel is diamond-shaped and located on the top of the head. The fontanel should be soft and flat in a normal infant, and it normally closes by 12 to 18 months of age. The nurse would document the finding because it is normal. There is no useful reason to increase oral fluids, notify the HCP, or elevate the head of the bed to 90 degrees.

The nurse and an adolescent are reviewing the adolescent's record of her headaches and activities surrounding them. Which of the following would the nurse identify as a possible trigger? a) Swimming twice a week b) 11 p.m. bedtime; 6:30 a.m. wake-up c) Use of nonscented soap d) Drinking three cans of diet cola

Drinking three cans of diet cola Correct Explanation: Cola contains caffeine, which is an associated trigger. Intense activity, not regular exercise, may be a trigger. Odors, such as strong perfumes, may be a trigger. Changes in sleeping patterns may be a trigger.

The nurse is caring for a client who sustained a spinal cord injury. While administering morning care, the client developed signs and symptoms of autonomic dysreflexia. Which is the initial nursing action?

Elevate the head of the bed. Autonomic dysreflexia is a serious complication that can occur in the spinal cord of the injured client. Once the syndrome is identified, the nurse elevates the head of the client's bed and then examines the client for the source of noxious stimuli. The nurse also assesses the client's blood pressure, but the initial action is to elevate the head of the bed. The client should not be placed in the prone position.

A client with Parkinson's disease is embarrassed about the symptoms of the disorder and is bored and lonely. The nurse should plan which approach as therapeutic in assisting the client to cope with the disease?

Encourage and praise perseverance in exercising and performing ADL The client with Parkinson's disease tends to become withdrawn and depressed and therefore should become an active participant in his or her own care to prevent this. Activities should be planned throughout the day to inhibit daytime sleeping and boredom. The nurse gives the client encouragement and praises the client for perseverance. Activities such as exercise help prevent progression of the disease, and self-care improves self-esteem.

A client with right leg hemiplegia is experiencing difficulty with mobility. The nurse determines that the family needs reinforcement of teaching if the nurse observes which action by the family?

Encouraging the client to stand unassisted on the leg The question is worded to elicit an unsafe action on the part of the family. Depending on the client's functional ability, either passive or active ROM is indicated to keep the joint moving freely. Application of a premolded splint would also keep the limb aligned and in good position. The client should not attempt to stand unsupported on a weak or paralyzed limb. The inability to bear weight will cause the client to fall.

The nurse is caring for a patient with aphasia. Which of the following strategies will the nurse use to facilitate communication with the patient? a) Speaking loudly b) Avoiding the use of hand gestures c) Establishing eye contact d) Speaking in complete sentences

Establishing eye contact The following strategies should be used by the nurse to encourage communication with a patient with aphasia: face the patient and establish eye contact, speak in your usual manner and tone, use short phrases, and pause between phrases to allow the patient time to understand what is being said; limit conversation to practical and concrete matters; use gestures, pictures, objects, and writing; and as the patient uses and handles an object, say what the object is. It helps to match the words with the object or action, be consistent in using the same words and gestures each time you give instructions or ask a question, and keep extraneous noises and sounds to a minimum. Too much background noise can distract the patient or make it difficult to sort out the message being spoken.

The client recovering from a head injury is arousable and participating in care. The nurse determines that the client understands measures to prevent elevations in intracranial pressure (ICP) if the nurse observes the client doing which activity?

Exhaling during repositioning Activities that increase intrathoracic and intraabdominal pressures cause indirect elevation of the ICP. Some of these activities include isometric exercises, Valsalva maneuver, coughing, sneezing, and blowing the nose. Exhaling during activities such as repositioning or pulling up in bed opens the glottis, which prevents intrathoracic pressure from rising.

The nurse is performing an assessment on a client with a diagnosis of Bell's palsy. The nurse should expect to observe which finding in the client? 1. Facial drooping 2. Periorbital edema 3. Ptosis of the eyelid 4. Twitching on the affected side of the face

Facial drooping Bell's palsy is a one-sided facial paralysis caused by the compression of the facial nerve (cranial nerve VII). Assessment findings include facial droop from paralysis of the facial muscles; increased lacrimation; painful sensations in the eye, face, or behind the ear; and speech or chewing difficulty. The remaining options are not associated findings in Bell's palsy.

A doctor orders the placement of an ICP monitor in a patient with cerebral edema. The nurse is aware that this surgery will take place in the infratentorial region of the brain. a) True b) False

False

The nurse caring for a child with a cranial injury knows that broad-spectrum antibiotics are used to reduce cerebral edema. a) True b) False

False Glucocorticoids and diuretics are used to reduce cerebral edema.

The nurse has taught a client with a herniated lumbar disk about proper body mechanics and other items pertinent to low back care. The nurse determines that further teaching is needed if the client states the need to take which action? 1. Bend at the knees to pick up objects. 2. Increase fiber and fluid intake in the diet. 3. Strengthen the back muscles by swimming or walking. 4. Get out of bed by sitting straight up and swinging the legs over the side of the bed.

Get out of bed by sitting straight up and swinging the legs over the side of the bed. The client is taught to get out of bed by sliding near the edge of the mattress. The client then rolls onto 1 side and pushes up from the bed using 1 or both arms. The client keeps the back straight and swings the legs over the side. Proper body mechanics includes bending at the knees, not the waist, to lift objects. Increasing fluid intake and dietary fiber helps prevent straining at stool, thereby preventing increases in intraspinal pressure. Walking and swimming are excellent exercises for strengthening lower back muscles.

The nurse has just admitted to the nursing unit a client with a basilar skull fracture who is at risk for increased intracranial pressure. Pending specific health care provider prescriptions, the nurse should safely place the client in which positions? Select all that apply. 1. Head midline 2. Neck in neutral position 3. Head of bed elevated 30 to 45 degrees 4. Head turned to the side when flat in bed 5. Neck and jaw flexed forward when opening the mouth

Head midline 2. Neck in neutral position 3. Head of bed elevated 30 to 45 degrees Home History Help Calculator Study ModeQuestion 36 of 208 Previous 36 ▲ ▼ Go Next Stop Bookmark Rationale Strategy Reference(s) Submit The nurse has just admitted to the nursing unit a client with a basilar skull fracture who is at risk for increased intracranial pressure. Pending specific health care provider prescriptions, the nurse should safely place the client in which positions? Select all that apply.Rationale: Use of proper positions promotes venous drainage from the cranium to keep intracranial pressure from elevating.The head of the client at risk for or with increased intracranial pressure should be positioned so that the head is in a neutral, midline position. The head of the bed should be raised to 30 to 45 degrees. The nurse should avoid flexing or extending the neck or turning the head from side to side.

A physician orders several drugs for a client with hemorrhagic stroke. Which drug order should the nurse question?

Heparin sodium Administering heparin, an anticoagulant, could increase the bleeding associated with hemorrhagic stroke. Therefore, the nurse should question this order to prevent additional hemorrhage in the brain. In a client with hemorrhagic stroke, the physician may use dexamethasone to decrease cerebral edema and pressure; methyldopa, to reduce blood pressure; and phenytoin, to prevent seizures.

The nurse is monitoring a child with a brain tumor for complications associated with increased intracranial pressure (ICP). Which finding, if noted by the nurse, would indicate the presence of diabetes insipidus (DI)? Weight gain 2. Hypertension 3. High urine output 4. Urine specific gravity greater than 1.020

High urine output DI can occur in a child with increased ICP. Weight gain, hypertension and a urine specific gravity greater than 1.020 are indications of the syndrome of inappropriate antidiuretic hormone (SIADH) secretion, not DI. A high urine output would be indicative of DI.

A client brought to the emergency department had a seizure 1 hour ago. Family members were present during the episode and reported that the client's jaw was moving as though grinding food. In helping to determine the origin of this seizure, what should the nurse include in the client's assessment? 1. Loss of consciousness 2. Presence of diaphoresis 3. History of prior trauma 4. Rotating eye movements

History of prior trauma Seizures that originate with specific motor phenomena are considered focal and are indicative of a focal structural lesion in the brain, often caused by trauma, infection, or medication consumption. The remaining options address signs, rather than an origin of the seizure.

The nurse is caring for an unconscious client who is experiencing persistent hyperthermia with no signs and symptoms of infection. The nurse understands that there may be damage to the client's thermoregulatory center which is located in which part of the brain?

Hypothalamus Hypothalamic damage causes hyperthermia, which may also be called "central fever." It is characterized by a persistent high fever with no diurnal variation. There is also an absence of sweating. Options 1, 2, and 3 are not associated with temperature regulation.

In caring for the child with meningitis, the nurse recognizes that which of the following nursing diagnoses would be the most important to include in this child's plan of care? a) Risk for injury related to seizure activity b) Risk for acute pain related to surgical procedure c) Delayed growth and development related to physical restrictions d) Ineffective airway clearance related to history of seizures

In caring for the child with meningitis, the nurse recognizes that which of the following nursing diagnoses would be the most important to include in this child's plan of care? You selected: Risk for injury related to seizure activity Correct Explanation: Keeping the child free of injury would be an appropriate nursing diagnosis. Surgery is not indicated for the child with meningitis, and the history of seizures does not impact the airway clearance. Growth and development issues are a concern but not likely delayed due to this diagnosis.

The home care nurse is visiting a client with a diagnosis of Parkinson's disease. The client is taking benztropine mesylate orally daily. The nurse provides information to the spouse regarding the side effects of this medication and should tell the spouse to report which side effect if it occurs? 1. Shuffling gait 2. Inability to urinate 3. Decreased appetite 4. Irregular bowel movements

Inability to urinate Benztropine mesylate is an anticholinergic, which causes urinary retention as a side effect. The nurse would instruct the client or spouse about the need to monitor for difficulty with urinating, a distended abdomen, infrequent voiding in small amounts, and overflow incontinence. The remaining options are unrelated to the use of this medication.

The nurse is caring for a client with increased intracranial pressure (ICP). Which change in vital signs would occur if ICP is rising?

Increasing temperature, decreasing pulse, decreasing respirations, increasing BP A change in vital signs may be a late sign of increased ICP. Trends include increasing temperature and BP and decreasing pulse and respirations. Respiratory irregularities may also arise.

The nurse is caring for the client with increased intracranial pressure. The nurse would note which trend in vital signs if the intracranial pressure is rising? 1. Increasing temperature, increasing pulse, increasing respirations, decreasing blood pressure 2. Increasing temperature, decreasing pulse, decreasing respirations, increasing blood pressure 3. Decreasing temperature, decreasing pulse, increasing respirations, decreasing blood pressure 4. Decreasing temperature, increasing pulse, decreasing respirations, increasing blood pressure

Increasing temperature, decreasing pulse, decreasing respirations, increasing blood pressure A change in vital signs may be a late sign of increased intracranial pressure. Trends include increasing temperature and blood pressure and decreasing pulse and respirations. Respiratory irregularities also may occur.

A client is admitted with an exacerbation of multiple sclerosis. The nurse is assessing the client for possible precipitating risk factors. Which factor, if reported by the client, should the nurse identify as being unrelated to the exacerbation? 1. Annual influenza vaccination 2. Ingestion of increased fruits and vegetables 3. An established routine of walking 2 miles each evening 4. A recent period of extreme outside ambient temperatures

Ingestion of increased fruits and vegetables The onset or exacerbation of multiple sclerosis can be preceded by a number of different factors, including physical stress (e.g., vaccination, excessive exercise), emotional stress, fatigue, infection, physical injury, pregnancy, extremes in environmental temperature, and high humidity. No methods of primary prevention are known. Intake of fruits and vegetables is a healthy and an unrelated item.

The nurse is creating a plan of care for a client with a stroke (brain attack) who has right homonymous hemianopsia. Which should the nurse include in the plan of care for the client? 1. Place an eye patch on the left eye. 2. Place personal articles on the client's right side. 3. Approach the client from the right field of vision. 4. Instruct the client to turn the head to scan the right visual field.

Instruct the client to turn the head to scan the right visual field. Homonymous hemianopsia is a loss of half of the visual field. The nurse instructs the client to scan the environment and stands within the client's intact field of vision. The nurse should not patch the eye because the client does not have double vision. The client should have objects placed in the intact fields of vision, and the nurse should approach the client from the intact side.

The family of an unconscious client with increased intracranial pressure is talking at the client's bedside. They are discussing the gravity of the client's condition and wondering if the client will ever recover. How should the nurse interpret the client's situation?

It is possible the client can hear the family. Some clients who have awakened from an unconscious state report that they remember hearing specific voices and conversations. Family and staff should assume that the client's sense of hearing is still intact and act accordingly. Research has also demonstrated that positive outcomes are associated with coma stimulation, that is, speaking to and touching the client.

The home care nurse is making a visit to a client who requires use of a wheelchair after a spinal cord injury sustained 4 months earlier. Just before leaving the home, the nurse ensures that which intervention has been done to prevent an episode of autonomic dysreflexia (hyperreflexia)? 1. Updating the home safety sheet 2. Leaving the client in an unchilled area of the room 3. Noting a bowel movement on the client progress note 4. Recording the amount of urine obtained with catheterization

Leaving the client in an unchilled area of the room The most common cause of autonomic dysreflexia is visceral stimuli, such as with blockage of urinary drainage or with constipation. Barring these, other causes include noxious mechanical and thermal stimuli, particularly pressure and overchilling. For this reason, the nurse ensures that the client is positioned with no pinching or pressure on paralyzed body parts and that the client will be sufficiently warm.

The nurse in the neurological unit is caring for a client with a supratentorial lesion. The nurse assesses which measurement as the most critical index of central nervous system (CNS) dysfunction Temperature 2. Blood pressure 3. Ability to speak 4. Level of consciousness

Level of consciousness Level of consciousness is the most critical index of CNS dysfunction. Changes in level of consciousness can indicate clinical improvement or deterioration. Although blood pressure, temperature, and ability to speak may be components of the assessment, the client's level of consciousness is the most critical index of CNS dysfunction.

A client admitted to the nursing unit from the hospital emergency department has a C4 spinal cord injury. In conducting the admission assessment, what is the nurse's priority action? 1. Take the temperature. 2. Listen to breath sounds. 3. Observe for dyskinesias. 4. Assess extremity muscle strength.

Listen to breath sounds. Because compromise of respiration is a leading cause of death in cervical cord injury, respiratory assessment is the highest priority. Assessment of temperature and strength can be done after adequate oxygenation is ensured. Because dyskinesias occur in cerebellar disorders, this is not as important a concern as in cord-injured clients unless head injury is suspected.

The nurse is assisting in caring for a client with a suspected diagnosis of meningitis. The nurse reinforces to the client information regarding which diagnostic test that is commonly used to confirm this diagnosis?

Lumbar puncture Meningitis is an acute or chronic inflammation of the meningeal area and the cerebrospinal fluid. The key diagnostic test used in meningitis is the lumbar puncture. The remaining options also may be performed but will not confirm the diagnosis.

Which of the following is accurate regarding a hemorrhagic stroke?

Main presenting symptom is an "exploding headache." One of hemorrhagic stroke's main presenting symptom is an "exploding headache." In ischemic stroke, functional recovery usually plateaus at 6 months; it may be caused by a large artery thrombosis and may have a presenting symptoms of numbness or weakness of the face.

The nurse is suctioning an unconscious client who has a tracheostomy. The nurse should avoid which action during this procedure?

Making sure not to suction for longer than 30 seconds Suction equipment should be kept at the bedside of an unconscious client, regardless of whether an artificial airway is present. The nurse auscultates breath sounds every 2 to 4 hours, or more frequently if there is a need. The client should be hyperoxygenated before, during, and after suctioning to minimize cerebral hypoxia. The client should not be suctioned for longer than 10 seconds at one time to prevent cerebral hypoxia and an increase in intracranial pressure.

The nurse observes that a client with Parkinson's disease has very little facial expression. The nurse attributes this piece of data to which information?

Masklike facies is a component of Parkinson's disease. A masked facial expression is typical of the client with Parkinson's disease. There are no data to support the assumption provided in option 2. Option 3 is not a true statement. Option 4 places a false interpretation on the client's expression.

The nurse is collecting data from a child who may have a seizure disorder. Which is a description of an absence seizure? a) Brief, sudden contracture of a muscle or muscle group b) Minimal or no alteration in muscle tone, with a brief loss of consciousness c) Sudden, momentary loss of muscle tone, with a brief loss of consciousness d) Muscle tone maintained and child frozen in position

Minimal or no alteration in muscle tone, with a brief loss of consciousness

A thymectomy accomplished via a median sternotomy approach is performed in a client with a diagnosis of myasthenia gravis. The nurse creates a postoperative plan of care for the client that should include which intervention? 1. Monitor the chest tube drainage. 2. Restrict visitors for 24 hours postoperatively. 3. Maintain intravenous infusion of lactated Ringer's solution. 4. Avoid administering pain medication to prevent respiratory depression.

Monitor the chest tube drainage. The thymus has played a role in the development of myasthenia gravis. A thymectomy is the surgical removal of the thymus gland and may be used for management of clients with myasthenia gravis to improve weakness. The procedure is performed through a median sternotomy or a transcervical approach. Postoperatively the client will have a chest tube in the mediastinum. Lactated intravenous solutions usually are avoided because they can increase weakness. Pain medication is administered as needed, but the client is monitored closely for respiratory depression. There is no reason to restrict visitors.

The nurse is preparing a plan of care to monitor for complications in a client who will be returning from the operating room following transsphenoidal resection of a pituitary adenoma. Which intervention does the nurse document in the plan as the priority nursing intervention for this client?

Monitor urine output. The most common complication of surgery on the pituitary gland is temporary diabetes insipidus. This results from deficiency in antidiuretic hormone (ADH) secretion as a result of surgical trauma. The nurse measures the client's urine output to determine whether this complication is occurring. Options 1, 3, and 4 are also components of the plan, but option 2 clearly identifies the priority intervention for this type of surgery.

A client is admitted to the emergency department with a C4 spinal cord injury. The nurse performs which intervention first when collecting data on the client?

Monitoring the respiratory rate Because compromise of respiration is a leading cause of death in cervical spinal cord injury, respiratory assessment is the highest priority. Assessment of temperature and strength can be done after adequate oxygenation is assured. Dyskinesia occurs in cerebellar disorders, so it is not important in cord-injured clients, unless a head injury is suspected.

A client receives a dose of edrophonium (Enlon). The client shows improvement in muscle strength for a period of time following the injection. The nurse should interpret this finding as indicative of which disease process?

Myasthenia gravis Myasthenia gravis can often be diagnosed based on clinical signs and symptoms. The diagnosis can be confirmed by injecting the client with a dose of edrophonium. This medication inhibits the breakdown of an enzyme in the neuromuscular junction, so more acetylcholine binds to receptors. If the muscle is strengthened for 3 to 5 minutes after this injection, it confirms a diagnosis of myasthenia gravis. Another medication, neostigmine (Prostigmin), also may be used because its effect lasts for 1 to 2 hours, providing a better analysis. For either medication, atropine sulfate should be available as the antidote.

A child undergoes surgical removal of a brain tumor. During the postoperative period, the nurse is monitoring the child and notes that the child is restless, the pulse rate is elevated, and the blood pressure has decreased significantly from the baseline value. The nurse suspects that the child is in shock. Which is the most appropriate nursing action? Notify the health care provider (HCP). 2. Place the child in a supine position. 3. Place the child in Trendelenburg's position. 4. Increase the flow rate of the intravenous fluids.

Notify the health care provider (HCP). in the event of shock, the HCP is notified immediately before the nurse changes the child's position or increases intravenous fluids. After craniotomy, a child is never placed in the supine or Trendelenburg's position because it increases intracranial pressure (ICP) and the risk of bleeding. The head of the bed should be elevated. Increasing intravenous fluids can cause an increase in ICP.

The nurse is assessing the nasal dressing on a client who had a transsphenoidal resection of the pituitary gland. The nurse notes a small amount of serosanguineous drainage that is surrounded by clear fluid on the nasal dressing. Which nursing action is most appropriate? 1. Document the findings. 2. Reinforce the dressing. 3. Notify the health care provider (HCP). 4. Mark the area of drainage with a pen and monitor for further drainage.

Notify the health care provider (HCP). Cerebrospinal fluid (CSF) leakage after cranial surgery may be detected by noting drainage that is serosanguineous surrounded by an area of straw-colored or pale drainage. The physical appearance of CSF drainage is that of a halo. If the nurse notes the presence of this type of drainage, the HCP needs to be notified. The remaining options are inappropriate nursing actions.

A client with a history of myasthenia gravis presents at a clinic with bilateral ptosis and is drooling, and myasthenic crisis is suspected. The nurse assesses the client for which precipitating factor? 1. Getting too little exercise 2. Taking excess medication 3. Omitting doses of medication 4. Increasing intake of fatty foods

Omitting doses of medication Myasthenic crisis often is caused by undermedication and responds to the administration of cholinergic medications. Cholinergic crisis (the opposite problem) is caused by excess medication and responds to withholding of medications. Too little exercise and excessive fatty food intake are incorrect. Overexertion and overeating possibly could trigger myasthenic crisis.

A nurse demonstrates understanding of the various levels of consciousness as they progress from most alert to least alert. Place the following in the order that reflects this progression. Coma Disorientation Oriented to person, place, and time Obtundation Stupor

Oriented to person, place, and time Disorientation Obtundation Stupor Coma Explanation: Levels of consciousness in order from most alert to least alert are orientated to person, place, and time (full consciousness); confusion (disorientation); obtundation; stupor; and finally coma.

A nurse is caring for a newborn with anencephaly. Which of the following interventions will the nurse use? a) Monitor for increased intracranial pressure (ICP). b) Closely monitor neurologic status. c) Refer the family to an agency to assist with long-term care. d) Place a cap or similar covering on the infant's head.

Place a cap or similar covering on the infant's head. Correct Explanation: Using an infant cap can help parents deal with the malformed appearance of their child. Because the child was born with a small or missing brain, the baby will likely die within hours or days. Monitoring for increased intracranial pressure (ICP) or neurologic status are not necessary interventions.

The client with a spinal cord injury at the level of T4 is experiencing a severe throbbing headache with a blood pressure of 180/100 mm Hg. What is the priority nursing intervention? 1. Notify the health care provider (HCP). 2. Loosen tight clothing on the client. 3. Place the client in a sitting position. 4. Check the urinary catheter tubing for kinks or obstruction.

Place the client in a sitting position. The client is demonstrating clinical manifestations of autonomic dysreflexia, which is a neurological emergency. The first priority is to place the client in a sitting position to prevent hypertensive stroke. Loosening tight clothing and checking the urinary catheter can then be done, and the HCP can be notified once initial interventions are done.

The nurse is caring for a client with dysphagia. Which intervention would be contraindicated while caring for this client?

Placing food on the affected side of the mouth nterventions for dysphagia include placing food on the unaffected side of the mouth, allowing ample time to eat, assisting the client with meals, and testing the client's gag reflex before offering food or fluids.

A nurse is caring for a 3-year-old girl with microcephaly. Which of the following actions is appropriate for the nurse to take? a) Playfully ask the child to touch her nose. b) Administer antipyretics as ordered. c) Prepare the child for the experience of cranial surgery. d) Teach the parents about ventriculoperitoneal (VP) shunts.

Playfully ask the child to touch her nose

A nurse is assessing a 3-year-old child for possible bacterial meningitis. Which sign would indicate irritation of the meninges?

Positive Kernig sign A positive Kernig sign can indicate irritation of the meninges. A positive Brudzinski sign also is indicative of the condition. A positive Chadwick sign is a bluish discoloration of the cervix indicating pregnancy

A 10-year-old child is admitted to the hospital due to history of seizure activity. As his nurse, you are called into the room by his mother, who states he is having a seizure. What would be the priority nursing intervention? 1. Prevention of injury by removing the child from his bed 2. Prevention of injury by placing a tongue blade in the child's mouth 3. Prevention of injury by restraining the child 4. Prevention of injury by placing the child on his side and opening his airway

Prevention of injury by placing the child on his side and opening his airway

A client has sustained damage to Wernicke's area from a stroke (brain attack). On assessment of the client, which sign or symptom would be noted? 1. Difficulty speaking 2. Problem with understanding language 3. Difficulty controlling voluntary motor activity 4. Problem with articulating events from the remote past

Problem with understanding language Wernicke's area consists of a small group of cells in the temporal lobe whose function is the understanding of language. Damage to Broca's area is responsible for aphasia. The motor cortex in the precentral gyrus controls voluntary motor activity. The hippocampus is responsible for the storage of memory.

Question: Put the following events of a generalized epileptic seizure in correct order: 1 Prodromal period 2 Tonic stage 3 Postictal period 4 Clonic stage

Prodromal period Tonic stage Clonic stage Postictal period

The nurse is reviewing the laboratory results from a lumbar puncture performed in a client with a diagnosis of meningitis. Which laboratory findings are expected to be noted with bacterial meningitis? Select all that apply. Increased glucose level 2. Protein level of 20 mg/dL 3. Increased white blood cells 4. Clear appearance of the cerebrospinal fluid 5. Cerebrospinal fluid (CSF) pressure of 250 mm H2O

Protein level of 20 mg/dL 3. Increased white blood cells 5. Cerebrospinal fluid (CSF) pressure of 250 mm H2O If a bacterial infection of CSF is present, findings include reduced glucose level, a protein level greater than 15 mg/dL, increased white blood cells, a cloudy appearance of CSF, and CSF pressure greater than 200 mm H2O.

The client has an impairment of cranial nerve II. Specific to this impairment, what should the nurse plan to do to ensure client safety? 1. Speak loudly to the client. 2. Test the temperature of the shower water. 3. Check the temperature of the food on the dietary tray. 4. Provide a clear path for ambulation without obstacles.

Provide a clear path for ambulation without obstacles. Cranial nerve II is the optic nerve, which governs vision. The nurse can provide safety for the visually impaired client by clearing the path of obstacles when ambulating. Speaking loudly may help overcome a deficit of cranial nerve VIII (vestibulocochlear). Testing the shower water temperature would be useful if there was an impairment of peripheral nerves. Cranial nerves VII (facial) and IX (glossopharyngeal) control taste from the anterior two thirds and posterior third of the tongue, respectively.

The nurse is caring for a client following an aneurysm coiling procedure. The nurse documents that the client is experiencing Korsakoff syndrome. Which set of symptoms characterizes Korsakoff syndrome?

Psychosis, disorientation, delirium, insomnia, and hallucinations Advances in technology have led to the introduction of interventional neuroradiology for the treatment of aneurysms. Endovascular techniques may be used in selected clients to occlude the blood flow from the artery that feeds the aneurysm with coils or other techniques to occlude the aneurysm itself. Postoperative complications are rare but can occur. Potential complications include psychological symptoms such as disorientation, amnesia, and Korsakoff syndrome (disorder characterized by psychosis, disorientation, delirium, insomnia, hallucinations, and personality changes). Creutzfeldt-Jakob disease results in severe dementia and myoclonus. The three cardinal signs of Parkinson disease are tremor, rigidity, and bradykinesia. Huntington disease results in progressive involuntary choreiform (dancelike) movements and dementia.

A client has a difficulty with the ability to flex the hips. The nurse determines that the client is adapting successfully to this problem if the client demonstrates proper use of which item? 1. Walker 2. Slider board 3. Raised toilet seat 4. Adaptive eating utensils

Raised toilet seat A raised toilet seat is useful if the client does not have the mobility or ability to flex the hips. The cerebellum is responsible for balance and coordination. A walker would provide stability for the client during ambulation. A slider board is used in transferring a client from a bed to a stretcher or wheelchair. Adaptive eating utensils may be beneficial if the client has partial paralysis of the hand.

The nurse is collecting data on a client suspected of having Alzheimer's disease. The priority data should focus on which characteristic of this disease?

Recent memory loss Dementia is the hallmark of Alzheimer's disease. Recent memory loss is one characteristic. Others include problems with abstract thinking, problems with speech (not hearing), and difficulty in performing familiar tasks.

At the beginning of the work shift, the nurse assesses the status of the client wearing a halo device. The nurse determines that which assessment finding requires intervention? 1. Tightened screws 2. Red skin areas under the jacket 3. Clean and dry lamb's wool jacket lining 4. Finger-width space between the jacket and the skin

Red skin areas under the jacket Red skin areas under the jacket indicate that the jacket is too tight. The resulting pressure could lead to altered skin integrity and needs to be relieved by loosening the jacket. The screws all should be properly tightened. A clean, dry lamb's wool lining should be in place underneath the jacket, and there should be a finger-width space between the jacket and the skin. In addition, the client should wear a clean white cotton T-shirt next to the skin to help prevent itching.

The nurse is monitoring a client with a spinal cord injury who is experiencing spinal shock. Which assessment will provide the nurse with the best information about recovery from the spinal shock?

Reflexes Areflexia characterizes spinal shock; therefore, reflexes should provide the best information. Vital sign changes are not consistently affected by spinal shock.

The nurse is reinforcing instructions to the family of a stroke client who has homonymous hemianopsia about measures to help the client overcome the deficit. The nurse determines that the family understands the measures to use if they state that they will do which?

Remind the client to turn the head to scan the lost visual field Homonymous hemianopsia is loss of half of the visual field. The client with homonymous hemianopsia should have objects placed in the intact field of vision, and the nurse should approach the client from the intact side. The nurse instructs the client to scan the environment to overcome the visual deficit and performs client teaching from within the intact field of vision. The nurse encourages the use of personal eyeglasses if they are available.

The student nurse develops a plan of care for a client after a lumbar puncture. The nursing instructor corrects the student if the student documents which incorrect intervention in the plan? 1. Maintain the client in a flat position. 2. Restrict fluid intake for a period of 2 hours. 3. Assess the client's ability to void and move the extremities. 4. Inspect the puncture site for swelling, redness, and drainage.

Restrict fluid intake for a period of 2 hours. After the lumbar puncture the client remains flat in bed for at least 2 hours, depending on the health care provider's prescriptions. A liberal fluid intake is encouraged to replace the cerebrospinal fluid removed during the procedure, unless contraindicated by the client's condition. The nurse checks the puncture site for redness and drainage and assesses the client's ability to void and move the extremities.

A client on your unit is scheduled to have intracranial surgery in the morning. Which nursing intervention helps to avoid intraoperative complications, reduce cerebral edema, and prevent postoperative vomiting?

Restrict fluids before surgery. Before surgery, the nurse should restrict fluids to avoid intraoperative complications, reduce cerebral edema, and prevent postoperative vomiting. The nurse administers prescribed medications such as an anticonvulsant phenytoin, like Dilantin, to reduce the risk of seizures before and after surgery, an osmotic diuretic, and corticosteroids. Preoperative sedation is omitted.

In caring for the child with meningitis, the nurse recognizes that which of the following nursing diagnoses would be the most important to include in this child's plan of care? a) Delayed growth and development related to physical restrictions b) Risk for acute pain related to surgical procedure c) Ineffective airway clearance related to history of seizures d) Risk for injury related to seizure activity

Risk for injury related to seizure activity

The nurse is caring for the client who has suffered spinal cord injury. The nurse further monitors the client for signs of autonomic dysreflexia and suspects this complication if which sign/symptom is noted?

Severe, throbbing headache The client with spinal cord injury above the level of T7 is at risk for autonomic dysreflexia. It is characterized by a severe, throbbing headache, flushing of the face and neck, bradycardia, and sudden severe hypertension. Other signs include nasal stuffiness, blurred vision, nausea, and sweating. It is a life-threatening syndrome triggered by a noxious stimulus below the level of the injury

A client is about to undergo a lumbar puncture. The nurse describes to the client that which position will be used during the procedure? Side-lying with a pillow under the hip 2. Prone with a pillow under the abdomen 3. Prone in slight Trendelenburg's position 4. Side-lying with the legs pulled up and the head bent down onto the chest

Side-lying with the legs pulled up and the head bent down onto the chest

The nurse has applied a hypothermia blanket to a client with a fever. The nurse should inspect the skin frequently to detect which complication of hypothermia blanket use?

Skin breakdown When a hypothermia blanket is used, the skin is inspected frequently for pressure points that over time could lead to skin breakdown. Options 1, 3, and 4 are not complications of hypothermia blanket use.

The nurse is documenting nursing observations in the record of a client who experienced a tonic-clonic seizure. Which clinical manifestation did the nurse most likely note in the clonic phase of the seizure? 1. Body stiffening 2. Spasms of the entire body 3. Sudden loss of consciousness 4. Brief flexion of the extremities

Spasms of the entire body The clonic phase of a seizure is characterized by alternating spasms and momentary muscular relaxation of the entire body, accompanied by strenuous hyperventilation. The face is contorted and the eyes roll. Excessive salivation results in frothing from the mouth. The tongue may be bitten, the client sweats profusely, and the pulse is rapid. The clonic jerking subsides by slowing in frequency and losing strength of contractions over a period of 30 seconds. Body stiffening, sudden loss of consciousness, and brief flexion of the extremities are associated with the tonic phase of a seizure.

A nurse is examining a boy with cerebral palsy. He has hypertonic muscles and abnormal clonus in his legs and walks on his toes. Which of the following is the type of cerebral palsy that this boy is demonstrating? a) Spastic b) Ataxic c) Athetoid d) Dyskinetic

Spastic

Which data collection finding supports the possible diagnosis of Bell's palsy?

Speech or chewing difficulties accompanied by facial droop Bell's palsy is a one-sided facial paralysis from compression of cranial nerve VII (facial) VII. There is facial droop from paralysis of the facial muscles, increased lacrimation, and speech or chewing difficulty. The remaining options are not characteristics of Bell's palsy.

A nurse is performing a neurologic examination of a 5-year-old child. She asks the boy to close his eyes, and then she places a crayon in his hand and asks him to identify it. Which type of ability is the nurse testing for in this boy? a) Orientation b) Stereognosis c) Kinesthesia d) Graphesthesia

Stereognosis

After a difficult birth, the nurse observes that a newborn has swelling on part of his head. Which sign suggests cephalohematoma? a) Swelling crosses the midline of the infant's scalp. b) Infant had a low birth weight when born at 37 weeks. c) Infant has facial abnormalities. d) Swelling does not cross the suture lines.

Swelling does not cross the suture lines.

A 9-year-old boy is suffering from headaches but has no signs of physical or neurologic illness. Which intervention would be most appropriate? a) Have the child sleep without a pillow under his head. b) Review the signs of increased intracranial pressure with parents. c) Teach the child and his parents to keep a headache diary. d) Have the parents call the doctor if the child vomits more than twice.

Teach the child and his parents to keep a headache diary.

A 9-year-old boy is suffering from headaches but has no signs of physical or neurologic illness. Which of the following interventions would be most appropriate? a) Have the child sleep without a pillow under his head. b) Have the parents call the doctor if the child vomits more than twice. c) Teach the child and his parents to keep a headache diary. d) Review the signs of increased intracranial pressure with parents.

Teach the child and his parents to keep a headache diary. Correct Explanation: A headache diary can help identify any triggers so that the child can avoid them. Triggers can include foods eaten, amount of sleep the night before, or activities at home or school that might be causing stress. Reviewing signs of increased intracranial pressure would be inappropriate because increased intracranial pressure is not associated with headaches. Having the child sleep without a pillow is an intervention to reduce pain from meningitis. Vomiting more than twice is an indication that the parents should notify the physician or nurse practitioner when the child has a head injury.

A mother has brought her 5-month-old son to the clinic because he has been drowsy and unresponsive. The child has hydrocephalus and had a shunt placed about a month previously. Which symptom indicates that the shunt is infected? a) The child has a high-pitched cry. b) The child is not responding or eating well. c) The child's pupil reaction time is rapid and uneven. d) The fontanels are bulging or tense.

The child is not responding or eating well.

The nurse is developing a plan of care for an older client that addresses interventions to prevent cold discomfort and the development of accidental hypothermia. The nurse should document which desired outcome in the plan of care? 1. The client's fingers and toes are cool to touch. 2. The client's body temperature is 98°F (36.7°C). 3. The client remains in a fetal position when in bed. 4. The client complains of coolness in the hands and feet only.

The client's body temperature is 98°F (36.7°C). Desired outcomes for nursing interventions to prevent cold discomfort and the development of accidental hypothermia include the following: hands and limbs are warm; body is relaxed and not curled; body temperature is greater than 97°F (36.1°C); the client is not shivering; and the client has no complaints of feeling cold.

The client is admitted to the hospital for observation with a probable minor head injury after an automobile crash. The nurse expects the cervical collar will remain in place until which time?

The health care provider reviews the x-ray results. There is a significant association between cervical spine injury and head injury. For this reason, the nurse leaves any form of spinal immobilization in place until lateral cervical spine x-rays rule out fracture or other damage and the results have been reviewed by the health care provider.

An emergency department nurse is awaiting the arrival of a client with signs of an ischemic stroke that began 1 hour ago, as reported by emergency medical personnel. The treatment window for thrombolytic therapy is which of the following?

Three hours Rapid diagnosis of stroke and initiation of thrombolytic therapy (within 3 hours) in clients with ischemic stroke leads to a decrease in the size of the stroke and an overall improvement in functional outcome after 3 months

A nurse is teaching a community class that those experiencing symptoms of ischemic stroke need to enter the medical system early. The primary reason for this is which of the following? a) A ruptured arteriovenous malformation will cause deficits until it is stopped. b) Intracranial pressure is increased by a space-occupying bleed. c) A ruptured intracranial aneurysm must quickly be repaired. d) Thrombolytic therapy has a time window of only 3 hours.

Thrombolytic therapy has a time window of only 3 hours. Currently approved thrombolytic therapy for ischemic strokes has a treatment window of only 3 hours after the onset of symptoms. Urgency is needed on the part of the public for rapid entry into the medical system.

A client with hypertension comes to the outpatient department for a routine checkup. Because hypertension is a risk factor for cerebral hemorrhage, the nurse questions the client closely about warning signs and symptoms of hemorrhage. Which complaint is a possible indicator of cerebral hemorrhage in this client?

Tinnitus Tinnitus is commonly a warning sign of cerebral hemorrhage. Other warning signs include vomiting (without nausea), a change in level of consciousness, and localized seizures. Vertigo isn't a common indicator of cerebral hemorrhage.

The nurse is caring for an 8-year-old girl who was in a car accident. Which symptom suggests the child has a cerebral contusion?

Trouble focusing when reading Signs and symptoms for cerebral contusions include disturbances to vision, strength, and sensation. A child suffering a concussion will be distracted and unable to concentrate. Vomiting is a sign of a subdural hematoma. Bleeding from the ear is a sign of a basilar skull fracture.

The best way to evaluate a child's level of consciousness is through conversation. a) True b) False

True Correct Explanation: The best way to evaluate a child's level of consciousness is through conversation. Note any drowsiness or lethargy. Allow the child to answer questions without prompting, and listen carefully to be certain the answer is appropriate to the question.

The treatment for children with seizures disorders is most often which of the following? a) Use of anticonvulsant medications b) Surgical intervention c) Strict exercise regimen d) Restricted fat diet

Use of anticonvulsant medications Correct Explanation: Complete control of seizures can be achieved for most people through the use of anticonvulsant drug therapy. A few children may be candidates for surgical intervention but, in most cases, surgery is not the treatment. Ketogenic diets (high in fat and low in carbohydrates and protein) cause the child to have high levels of ketones, which help to reduce seizure activity. Exercise is not a treatment for seizure disorders.

The nurse is caring for a premature infant diagnosed with intraventricular hemorrhage (IVH). Which of the following interventions best serves the needs of this client? a) Using a squeak toy to attract the child's gaze b) Stroking the child's cheek with a finger c) Placing the crib in a room by itself d) Removing toys from the crib when not in use

Using a squeak toy to attract the child's gaze Explanation: Assessing neurological changes is part of a care plan for intraventricular hemorrhage (IVH). The squeak toy will check for normal reactions from the child. There is no need to remove toys (as a precaution for seizures), check sensory function, or isolate the child.

The nurse is reviewing the record of a client with a suspected diagnosis of Huntington's disease. Which documented early symptom supports this diagnosis?

Vertigo Early symptoms of Huntington's disease include restlessness, forgetfulness, clumsiness, falls, balance and coordination problems, vertigo, and altered speech and handwriting. Difficulty with swallowing occurs in the later stages. Aphasia and agnosia do not occur.

A client has a cerebellar lesion. The nurse determines that the client is adapting successfully to this problem if the client demonstrates proper use of which item?

Walker The cerebellum is responsible for balance and coordination. A walker would provide stability for the client during ambulation. Adaptive eating utensils may be beneficial when the client has partial paralysis of the hand. A raised toilet seat is useful when the client does not have the mobility or ability to flex the hips. A slider board is used in transferring a client from a bed to stretcher or wheelchair

An older gentleman is brought to the emergency department by a neighbor who heard him talking and wandering in the street at 3 am. The nurse should first determine which about the client?

Whether this is a change in his usual level of orientation The nurse should first determine whether this is a change in the client's neurological status. The next item to determine should include when the client last ate. Blood toxicology levels may be needed, but the health care provider would prescribe these. Insurance information must be obtained at some point, but it is not the priority from a clinical care viewpoint.

To promote optimal cerebral tissue perfusion in the postoperative phase following cranial surgery, the nurse should place the client with an incision in the anterior or middle fossa, in which position? 1. 15 degrees of Trendelenburg's 2. Side-lying with the head of the bed flat 3. With the head of the bed elevated at least 30 degrees 4. With the head of the bed elevated no more than 10 degrees

With the head of the bed elevated at least 30 degrees Correct positioning of the client following cranial surgery is important to avoid increased intracranial pressure and to promote optimal cerebral tissue perfusion. The surgeon's prescription for positioning is always followed. The client with an incision in the anterior or middle fossa should be positioned with the head of bed (HOB) elevated at least 30 degrees. If the incision is in the posterior fossa or burr holes have been made, the client is positioned flat, or with the HOB elevated no more than 10 to 15 degrees. If a craniectomy (bone flap) is performed, the client should not be positioned to the operative side. Trendelenburg's position is contraindicated in the postoperative phase following cranial surgery.

The nurse is preparing to discharge a client who is stable following a head injury. Which statement by the client indicates a need for further discharge instructions? a. "I have a leftover prescription at home I can use if I have pain" b. "I will cancel the wine tasting I have planned for this weekend" c. "I will have someone drive me home and will take a couple of days off work" d. "I will have someone stay with me and sure I am okay"

a. "I have a leftover prescription at home I can use if I have pain"

The home health nurse teaches an elderly client with dysphagia some strategies to help limit repeated hospitalizations for aspiration pneumonia. Which statement indicates that the client needs further teaching? a. "I have to remember to raise my chin slightly upward when I swallow" b. "I have to remember to swallow 2 times before taking another bite of food" c. "I should avoid taking over-the-counter cold medications when I'm sick" d. "I should sit upright for at least 30-40 minutes after I eat"

a. "I have to remember to raise my chin slightly upward when I swallow"

The nurse completes a neurological examination on a client who has suffered a stroke to determine if damage has occurred to any of the cranial nerves. The nurse understands that damage has occurred to cranial nerve IX based on which assessment finding? a. A tongue blade is used to touch the client's pharynx; gag reflux is absent b. Only one side of the mouth moves when the client is asked to smile and frown c. The absence of light tough and pain sensation on the left side of the client's face d. When the client shrugs against resistance, the left shoulder is weaker than the right

a. A tongue blade is used to touch the client's pharynx; gag reflux is absent

A hospitalized client develops acute hemorrhagic stroke and is transferred to the intensive care unit. Which nursing interventions should be included in the plan of care? Select all that apply. a. Administer PRN stool softeners daily b. Administer scheduled enoxaparin injection c. Implement seizure precautions d. Keep client NPO until swallow screen is performed e. Perform frequent neurological assessments

a. Administer PRN stool softeners daily c. Implement seizure precautions d. Keep client NPO until swallow screen is performed e. Perform frequent neurological assessments

Assessment of a client with a history of stroke reveals that the client understands and follows commands but answers questions with incorrect word choices. The nurse documents the presence of which communication deficit? a. Aphasia b. Apraxia c. Dysarthria d. Dysphagia

a. Aphasia

A client with massive trauma and possible spinal cord injury is admitted to the emergency department following a dirt bike accident. Which clinical manifestation does the nurse assess to help best confirm a diagnosis of neurogenic shock? a. Apical heart rate 48/min b. Blood pressure 186/92 mm Hg c. Cool, clammy skin d. Temperature 100 F (37.7 C) tympanic

a. Apical heart rate 48/min

The nurse is caring for a client who had a stroke 2 weeks ago and has moderate receptive aphasia. The nurse is trying to get the client to follow simple commands regarding activities of daily living (ADL). Which nursing interventions should be included in the plan of care? Select all that apply. a. Ask simple questions that require "yes" or "no" answers b. If the client becomes frustrated, seek a different care provider to complete ADL c. Perform ADL for the client until the goal of each activity is understood d. Show the client gestures or pictures of ADL (shower, toilet, and toothbrush) e. Speak slowly but loudly while looking directly at the client

a. Ask simple questions that require "yes" or "no" answers d. Show the client gestures or pictures of ADL (shower, toilet, and toothbrush)

The nurse is assessing a newly admitted client on a neurological inpatient unit. Which assessment findings are abnormal and require follow-up by the nurse? Select all that apply. a. Cannot touch chin to chest b. Eyes roll in opposite direction when turning head side to side c. Muscle strength of lower extremities is 3/5 d. Pupils are 8 mm in diameter e. Toes point downward when noxious stimuli are applied to the sole

a. Cannot touch chin to chest c. Muscle strength of lower extremities is 3/5 d. Pupils are 8 mm in diameter

A client sustained a concussion after falling off a ladder. What are essential instructions for the nurse to provide when the client is discharged from the hospital? Select all that apply. a. Client should abstain from alcohol b. Client should remain awake all night c. Client should return if having difficulty d. Responsible adult should be taught neurological examination e. Responsible adult should stay with the client

a. Client should abstain from alcohol c. Client should return if having difficulty e. Responsible adult should stay with the client

The nurse receives the change of shift report for assigned clients at 7 AM. Which client should the nurse assess first? a. Client with change in level of consciousness who fell in the nursing home b. Client with chronic headaches who is scheduled for an MRI at 9 AM c. Client with chronic obstructive pulmonary disease (COPD) and pulse oximeter reading of 90% d. Client with heart failure and 3+ pitting edema of the lower extremities

a. Client with change in level of consciousness who fell in the nursing home

what is not an appropriate intervention for patient with spinal cord tumor a. apply hot compress to relieve pain b. measure muscle strength c. check for bladder distention d. assess level of pain

a. apply hot compress to relieve pain

A highly intoxicated client was brought to the emergency department after found lying on the sidewalk. On admission, the client is awake with a pulse of 70/min and blood pressure of 160/80 mm Hg. An hour later, the client is lethargic, pulse is 48/min, and blood pressure is 200/80 mm Hg. Which action does the nurse anticipate taking next? a. Administer atropine for bradycardia b. Administer nifedipine for hypertension c. Have CT scan performed to rule out an intracranial bleed d. Perform hourly neurologic checks with Glasgow coma scale (GCS)

c. Have CT scan performed to rule out an intracranial bleed

A client is admitted to the hospital for severe headaches. The client has a history of increased intracranial pressure (ICP), which has required lumbar punctures to relieve the pressure by draining cerebrospinal fluid. The client suddenly vomits and states, "That's weird, I didn't even feel nauseated." Which action by the nurse is the most appropriate? a. Document the amount of emesis b. Lower the head of the bed c. Notify the health care provider (HCP) d. Offer anti-nausea medication

c. Notify the health care provider (HCP)

a client admitted to ED with TBI. what is the priority intervention? a. admin dopamin b. VS q 2 hrs c. positioning to avoid extreme flexion d. keep the temperature at normal levels using cooling blankets

c. positioning to avoid extreme flexion

on assessing a client with frontal lobe tumor, what type of deficit that nurse expects to see a. taste b. vision c. problem solving d. hearing

c. problem solving

which condition indicated that spinal shock is resolving in a client with C7 quadriplegia? a. absence of pain sensation b. spontaneous respiration c. return of reflexes below the injury d. urinary incontinence

c. return of reflexes below the injury

A nurse is reading a journal article about stroke and the underlying causes associated with this condition. The nurse demonstrates understanding of the information when identifying which subtype of stroke as being due to atrial fibrillation?

cardio embolic Ischemic strokes are further divided into five subtypes, according to a mechanism-based classification system: large-artery thrombotic strokes (representing 20% of ischemic strokes); small, penetrating artery thrombotic strokes (25%); cardio embolic strokes (20%); cryptogenic strokes (strokes that cannot be attributed to any specific cause) (30%); and "other" (5%). Large-artery thrombotic strokes are caused by atherosclerotic plaques in the large blood vessels of the brain. Thrombus formation and occlusion can occur at the site of the atherosclerosis and result in ischemia and infarction (tissue death). Small, penetrating artery thrombotic strokes that affect one or more vessels and cause reduced blood flow are the most common type of ischemic stroke, typically caused by longstanding hypertension, hyperlipidemia, or diabetes. Cardio embolic strokes are associated with cardiac dysrhythmias, such as atrial fibrillation, but can also be associated with valvular heart disease or left ventricular thrombus. The last two classifications of ischemic strokes are cryptogenic strokes, which have no identified cause, and strokes from other causes, such as illicit drug use (cocaine), coagulopathies, migraine, or spontaneous dissection of the carotid or vertebral arteries.

A 45-year-old client presents to the ED reporting trouble speaking and numbness of the right arm and leg. The nurse suspects an ischemic stroke. Which insult or abnormality can cause an ischemic stroke?

cocaine use Two classifications of ischemic strokes are cryptogenic strokes, which have no known cause, and strokes from other causes, such as illicit drug use, coagulopathies, migraine, and spontaneous dissection of the carotid or vertebral arteries. Cocaine is a potent vasoconstrictor and may result in a life-threatening reaction, even with the individual's first use of the drug. Arteriovenous malformations, trauma, and intracerebral aneurysm are associated with hemorrhagic strokes.

A client comes to the emergency department with diplopia and recent onset of nausea. Which statement by the client would indicate to the nurse that this is an emergency? a. "I am very tired, and it's hard to for me to keep my eyes open" b. "I don't feel good, and I want to be seen" c. "I have not taken my blood pressure medicine in over a week" d. "I have the worst headache I've ever had in my life"

d. "I have the worst headache I've ever had in my life"

The nurse is caring for a client with left-sided weakness from a stroke. When assisting the client to a chair, what should the nurse do? a. Bend at the wrist b. Keep the feet close together c. Pivot on the foot proximal to the chair d. Use a transfer belt

d. Use a transfer belt

a client with SCI T4-T5 will likely to develop respiratory complications d/t a. aspiration d/t absence of gag reflex b. inability to breath due to respiratory muscle paresthesia c. inability to cough d/t the paralysis of pharyngeal nerve d. inability to deep breath and cough due to intercostal muscles

d. inability to deep breath and cough due to intercostal muscles

a client admitted with c6 injury. the nurse anticipates the client most likely has a. aphasia b. paraplegia c. hemiparesis d. quadriplegia

d. quadriplegia

The nurse is assisting the health care provider in performing a lumbar puncture on a client. The nurse prepares the client for the procedure by placing the client in which position? 1. Fetal 2. Prone 3. Supine 4. Lateral

fetal The client is assisted into a fetal position at the edge of the bed with the knees drawn up to the chest. This position allows full flexion of the spine and wider spaces between the vertebrae. The nurse also would place a pillow between the client's legs to prevent the upper leg from rolling forward and a small pillow under the client's head to support the spine in a horizontal position.

A client's spouse relates how the client reported a severe headache and then was unable to talk or move their right arm and leg. After diagnostics are completed and the client is admitted to the hospital, when would basic rehabilitation begin?

immediately Beginning basic rehabilitation during the acute phase is an important nursing function. Measures such as position changes and prevention of skin breakdown and contractures are essential aspects of care during the early phase of rehabilitation. The nursing goal is to prevent complications that may interfere with the client's potential to recover function.

Which is a contraindication for the administration of tissue plasminogen activator (t-PA)?

intracranial hemorrhage Intracranial hemorrhage, neoplasm, and aneurysm are contraindications for t-PA. Clinical diagnosis of ischemic stroke, age 18 years or older, and a systolic blood pressure less than or equal to 185 mm Hg are eligibility criteria.

While observing a child, the nurse notes that the child's arms and legs are extended and pronated. The nurse interprets this as indicating damage to the: a) midbrain. b) cerebral cortex. c) meninges. d) cranial nerves.

midbrain

a client with t4 SCI is complaining about severe throbbing headache and he appears to be flushed and diaphoretic. which are the interventions that the nurse should take?

remove constrictive clothing measure the clients blood pressure determine if there is bladder distention

A client has a 12-year history of migraine headaches and is frustrated over how these headaches impact lifestyle. The nurse discusses the potential triggers of the client's migraines. Which is not a potential trigger to migraines?

seasonal changes Researchers believe the contributing cofactors for the cause of migraines are from changes in serotonin receptors that promote dilation of cerebral blood vessels and pain intensification from neurochemicals released from the trigeminal nerve. It has been suggested that fluctuations in reproductive hormones, chemicals in certain foods, and medications can trigger migraines.

nurse explain that the spinal cord extends from the brainstem to the level of which vertebra?

second lumbar

A client is taking the prescribed dose of phenytoin (Dilantin) to control seizures. Results of a phenytoin blood level study reveal a level of 35 mcg/mL. Which finding would be expected as a result of this laboratory result? 1. Hypotension 2. Tachycardia 3. Slurred speech 4. No abnormal finding

slurred speech The therapeutic phenytoin level is 10 to 20 mcg/mL. At a level higher than 20 mcg/mL, involuntary movements of the eyeballs (nystagmus) occur. At a level higher than 30 mcg/mL, ataxia and slurred speech occur.

A nurse in a rehabilitation facility is coordinating the discharge of a client who is tetraplegic. The client, who is married and has two children in high school, is being discharged to home and will require much assistance. Who would the discharge planner recognize as being the most important member of this client's care team?

spouse The client's spouse and family would need to be involved in the everyday care of the client; without their support, it is unlikely that the client would be able to manage at home.

When communicating with a client who has sensory (receptive) aphasia, the nurse should:

use short, simple sentences. Although sensory aphasia allows the client to hear words, it impairs the ability to comprehend their meaning. The nurse should use short, simple sentences to promote comprehension. Allowing time for the client to respond might be helpful but is less important than simplifying the communication. Because the client's hearing isn't affected, speaking loudly isn't necessary. A writing pad is helpful for clients with expressive, not receptive, aphasia.

The nurse determines that a child is experiencing late signs of increased intracranial pressure based on which assessment findings? Select all that apply. a) Sunset eyes b) Irregular respirations c) Increased blood pressure d) Fixed dilated pupils e) Bradycardia

• Bradycardia • Fixed dilated pupils • Irregular respirations

The nurse is using the pediatric Glasgow Coma Scale to assess a child's level of consciousness. What would the nurse assess?

• Eye opening • Verbal response • Motor response

The nurse is caring for a client with a diagnosis of multiple sclerosis who has been prescribed oxybutynin (Ditropan). The nurse evaluates the effectiveness of the medication by asking the client which question?

"Are you getting up at night to urinate?" Oxybutynin is an antispasmodic used to relieve symptoms of urinary urgency, frequency, nocturia, and incontinence in clients with uninhibited or reflex neurogenic bladder. Expected effects include improved urinary control and decreased urinary frequency, incontinence, and nocturia. Options 1, 2, and 4 are unrelated to the use of this medication.

A 6-year-old has had a viral infection for the past 5 days and is having severe vomiting, confusion, and irritability, although he is now afebrile. During the assessment, the nurse should ask the parent which question? a) "Did you use any medications like aspirin for the fever?" b) "Did you give your child any acetaminophen, such as Tylenol?" c) "What type of fluids did your child take when he had a fever?" d) "How high did his temperature rise when he was ill?"

"Did you use any medications like aspirin for the fever?"

The nurse has given medication instructions to the client receiving phenytoin (Dilantin). The nurse determines that the client understands the instructions if the client makes which comment?

"Good oral hygiene is needed, including brushing and flossing." Typical anticonvulsant medication instructions include taking the prescribed dose daily to keep the blood level of the drug constant, having a serum drug level drawn before taking the morning dose, avoiding abruptly stopping the medication, avoiding alcohol, checking with the health care provider before taking over-the-counter medications, avoiding activities in which alertness and coordination are required until medication effects are known, providing good oral hygiene and getting regular dental care, and wearing a Medic-Alert bracelet or tag.

The nurse has provided instructions to a client with a diagnosis of myasthenia gravis about home care measures. Which client statement indicates the need for further teaching? 1. "I will rest each afternoon after my walk." 2. "I should cough and deep breathe many times during the day." 3. "I can change the time of my medication on the mornings when I feel strong." 4. "If I get abdominal cramps and diarrhea, I should call my health care provider."

"I can change the time of my medication on the mornings when I feel strong." The client with myasthenia gravis and the family should be taught information about the disease and its treatment. They should be aware of the side and adverse effects of anticholinesterase medications and corticosteroids and should be taught that timing of anticholinesterase medication is critical. It is important to instruct the client to administer the medication on time to maintain a chemical balance at the neuromuscular junction. If it is not given on time, the client may become too weak to even swallow. Resting after a walk, coughing and deep-breathing many times during the day, and calling the health care provider when experiencing abdominal cramps and diarrhea indicate a correct understanding of home care instructions to maintain health with this neurological degenerative disease.

A client seeking treatment for an episode of hyperthermia is being discharged to home. The nurse determines that the client needs clarification of discharge instructions if the client makes which statement?

"I can resume a full activity level immediately." Discharge instructions for the client hospitalized for hyperthermia include prevention of heat-related disorders, increased fluid intake for 24 hours, self-monitoring of voiding, and the importance of staying in a cool environment and resting.

The nurse is providing diet instructions to a client with Ménière's disease who is being discharged from the hospital after admission for an acute attack. Which statement, if made by the client, indicates an understanding of the dietary measures to take to help prevent further attacks? 1. "I need to restrict my carbohydrate intake." 2. "I need to drink at least 3 L of fluid per day." 3. "I need to maintain a low-fat and low-cholesterol diet." 4. "I need to be sure to consume foods that are low in sodium."

"I need to be sure to consume foods that are low in sodium." Dietary changes, such as salt and fluid restrictions, that reduce the amount of endolymphatic fluid are sometimes prescribed for the client with Ménière's disease. The client should be instructed to consume a low-sodium diet and restrict fluids as prescribed. Although helpful to treat other disorders, low-fat, low-carbohydrate, and low-cholesterol diets are not specifically prescribed for the client with Ménière's disease.

A client requires a myelogram, and the ambulatory care nurse is providing instructions to the client regarding preparation for the procedure. Which statement by the client indicates a need for further instruction? "My jewelry will need to be removed." 2. "An informed consent form will need to be signed." 3. "My procedure will take approximately 45 minutes." 4. "I need to be sure to eat a full meal before the procedure."

"I need to be sure to eat a full meal before the procedure." Rationale: Client preparation for a myelogram includes instructing the client to withhold food and fluids for 4 to 8 hours before the procedure as prescribed. Some health care providers may allow fluids or a light diet (but not a full meal). The client is told that the procedure takes about 45 minutes. An informed consent is required, and the client will need to remove jewelry and any metal objects. The client also is told that pretest medications may be administered for relaxation.

The nurse has just admitted a 17-year-old diagnosed with bacterial meningitis. The parents of the adolescent tell the nurse, "We just don't understand how this could have happened. Our child has always been healthy and also just received a booster vaccine last year?" How should the nurse respond?

"I understand your frustration. Unfortunately immunizations are not 100% effective in preventing the infection." Showing empathy while letting the parents know that vaccines are not 100% effective is the best response. Questioning them about being sure would not be the best response unless there was reason to believe their information was not accurate. There is nothing to lead the nurse to believe that a different strain of bacteria caused the infection, or that the the child's immune system is compromised

The nurse has given suggestions to a client with trigeminal neuralgia about strategies to minimize episodes of pain. The nurse determines that the client needs further teaching if the client makes which statement? 1. "I will wash my face with cotton pads." 2. "I'll have to start chewing on my unaffected side." 3. "I should rinse my mouth if toothbrushing is painful." 4. "I'll try to eat my food either very warm or very cold."

"I'll try to eat my food either very warm or very cold." Facial pain can be minimized by using cotton pads to wash the face and using room temperature water. The client should chew on the unaffected side of the mouth, eat a soft diet, and take in foods and beverages at room temperature. If brushing the teeth triggers pain, an oral rinse after meals may be helpful instead.

A resident in a long-term care facility prepares to walk out into a rainstorm after saying, "My father is waiting to take me for a ride." An appropriate response by the nurse is which?

"I'm glad you told me that. Let's have a cup of coffee and you can tell me about your father." The correct response acknowledges the client's comment and feelings. Option 1 is inappropriate and is inconsistent with legal aspects of care based on the information given. Option 2 fails to protect the client from possible harm. Option 3 does not preserve the client's dignity.

The nurse is caring for a client diagnosed with trigeminal neuralgia. The client asks the nurse, "Why do I have so much pain?" Which is the appropriate response by the nurse? 1. "It's a local reaction to nasal stuffiness." 2. "It's due to a hypoglycemic effect on the cranial nerve." 3. "Release of catecholamines with infection or stress leads to the pain." 4. "Pain is due to stimulation of the affected nerve by pressure and temperature.

"Pain is due to stimulation of the affected nerve by pressure and temperature." The paroxysms of pain that accompany this neuralgia are triggered by stimulation of the terminal branches of the trigeminal nerve. Symptoms can be triggered by pressure from washing the face, brushing the teeth, shaving, eating, or drinking. Symptoms also can be triggered by thermal stimuli, such as a draft of cold air. The remaining options are incorrect.

The parents of a child with a history of seizures who has been taking phenytoin (Dilantin) ask the nurse why it's difficult to maintain therapeutic plasma levels of this medication. Which statement by the nurse would be most accurate? a) "A drop in the plasma drug level will lead to a toxic state." b) "Large increments in dosage lead to a more rapid stabilizing therapeutic effect." c) "The capacity to metabolize the drug becomes overwhelmed over time." d) "Small increments in dosage lead to sharp increases in plasma drug levels."

"Small increments in dosage lead to sharp increases in plasma drug levels." Correct Explanation: Within the therapeutic range for phenytoin, small increments in dosage produce sharp increases in plasma drug levels. The capacity of the liver to metabolize phenytoin is affected by slight changes in the dosage of the drug, not necessarily the length of time the client has been taking the drug. Large increments in dosage will greatly increase plasma levels, leading to drug toxicity.

The mother of a 12-year-old with Reye syndrome approaches the nurse wanting to know how this happened to her child, saying, "I never give my kids aspirin!" What could the nurse say to begin educating the woman? a) "Aspirin in combination with the virus will make the brain swell and the liver fail." b) "Sometimes it's hard to tell what products may contain aspirin." c) "Don't worry; you're in good hands. We have it under control now." d) "Do you think that maybe your child took aspirin on his own?"

"Sometimes it's hard to tell what products may contain aspirin."

The mother of a 12-year-old with Reye syndrome approaches the nurse wanting to know how this happened to her child, saying, "I never give my kids aspirin!" What could the nurse say to begin educating the woman? a) "Sometimes it's hard to tell what products may contain aspirin." b) "Don't worry; you're in good hands. We have it under control now." c) "Aspirin in combination with the virus will make the brain swell and the liver fail." d) "Do you think that maybe your child took aspirin on his own?"

"Sometimes it's hard to tell what products may contain aspirin." Correct Explanation: Salicylates are in a wide variety of products, so consumers must read the small print very carefully or they will miss the warning. The parent needs to be receptive to further education, and raising the possibility the child was responsible does not accomplish that goal. Don't state the obvious, but also don't minimize the situation. Encouraging the mother to ask for information and offering explanations in terms she will understand are important, but this response does not address the mother's assertion.

A child who has been having seizures is admitted to the hospital for diagnostic testing. The child has had laboratory testing and an EEG, and is scheduled for a lumbar puncture. The parents voice concern to the nurse stating, "I don't understand why our child had to have a lumbar puncture since the EEG was negative." What is the best response by the nurse?

"The lumbar puncture can help rule out any infection in fluid surrounding the brain and spinal cord as the cause of the seizures." Lumbar punctures are performed to analyze cerebrospinal fluid (CSF) to rule out meningitis or encephalitis as a cause of seizures. A normal EEG does not rule out epilepsy because seizure activity rarely occurs during the actual testing time. A 24-hour or longer EEG can help in diagnosing a seizure disorder. Just telling the parents that it needs to be done, to be patient, or it is a routine does not address the parents' concerns.

A 1-year-old has just undergone surgery to correct craniosynostosis. Which of the following comments is the best psychosocial intervention for the parents? a) "This only happens in 1 out of 2,000 births." b) "The surgery was successful. Do you have any questions?" c) "I'll be watching hemoglobin and hematocrit closely." d) "I told you yesterday there would be facial swelling."

"The surgery was successful. Do you have any questions?" Correct Explanation: Often what parents need most is someone to listen to their concerns. Although this is a good time for education, let the parents adjust to their baby's appearance and adapt your teaching to their questions, comments, and knowledge level.

The nurse is educating the family of a 7-year-old epilepsy patient about care and safety for this child. Which of the following comments will be most valuable in helping the parent and the child cope? a) "Use this information to teach family and friends." b) "If he is out of bed, the helmet's on the head." c) "You'll always need a monitor in his room." d) "Bike riding and swimming are just too dangerous."

"Use this information to teach family and friends." Explanation: Families need and want information they can share with relatives, childcare providers, and teachers. Wearing a helmet and having a monitor in the room are precautions that may need to be modified as the child matures. The boy may be able to bike ride and swim with proper precautions.

The emergency room nurse is taking a history of a 1-year-old child whose parent said that she had a "fit" at home. Which inquiry would be best to start with? a) "How did you treat the child afterwards?" b) "Were there any jerky movements?" c) "Was the child unconscious?" d) "What happened just before the seizures?"

"What happened just before the seizures?"

The nurse is planning care for a client being admitted with newly diagnosed quadriplegia (tetraplegia). Which intervention will the nurse prioritize? 1. Assess vital capacity and tidal volume once per shift and PRN 2. Perform passive range of motion exercises on affected joints every 4 hours 3. Provide time during each shift for the client to express feelings 4. Turn the client every 2 hours throughout the day and night

1

The nurse is preparing to discharge a client who is stable following a head injury. Which statement by the client indicates a need for further discharge instructions? 1. "I have a leftover prescription at home I can use if I have pain." 2. "I will cancel the wine tasting I have planned for this weekend." 3. "I will have someone drive me home and will take a couple of days off work." 4. "I will have someone stay with me and make sure I am okay."

1

The nurse is performing an assessment on a child with a head injury. The nurse notes an abnormal flexion of the upper extremities and an extension of the lower extremities. What should the nurse document that the child is experiencing? 1. Decorticate posturing 2. Decerebrate posturing 3. Flexion of the arms and legs 4. Normal expected positioning after head injury

1 Decorticate posturing is an abnormal flexion of the upper extremities and an extension of the lower extremities with possible plantar flexion of the feet. Decerebrate posturing is an abnormal extension of the upper extremities with internal rotation of the upper arms and wrists and an extension of the lower extremities with some internal rotation.

A client with a cervical spinal cord injury has been placed in fixed skeletal traction with a halo fixation device. When caring for this client, the nurse may assign which actions to the LPN/LVN? Select all that apply. 1. Checking the client's skin for pressure from the device 2. Assessing the client's neurologic status for changes 3. Observing the halo insertion sites for signs of infection 4. Cleaning the halo insertion sites with hydrogen peroxide 5. Developing the nursing plan of care for the client 6. Administering oral medications as ordered

1, 3, 4, 6 Checking and observing for signs of pressure or infection is within the scope of practice of the LPN/LVN. The LPN/LVN also has the appropriate skills for cleaning the halo insertion sites with hydrogen peroxide. Administering oral drugs is within the scope of practice for an LPN/LVN. Neurologic examination and care plan development require additional education and skill appropriate to the professional RN.

The nurse is assessing a newly admitted client on a neurological inpatient unit. Which assessment findings are abnormal and require follow-up by the nurse? Select all that apply. 1. Cannot touch chin to chest 2. Eyes roll in opposite direction when turning head side to side 3. Muscle strength of lower extremities is 3/5 4. Pupils are 8 mm in diameter 5. Toes point downward when noxious stimuli are applied to the sole

1,3,4

The nurse is caring for a client who had a stroke two weeks ago and has moderate receptive aphasia. Which interventions should the nurse include in the plan of care to help the client follow simple commands regarding activities of daily living (ADL)? Select all that apply. 1. Ask simple questions that require "yes" or "no" answers 2. If the client becomes frustrated, seek a different care provider to complete ADL 3. Remain calm and allow the client time to understand each instruction 4. Show the client pictures of ADL (eg, shower, toilet, and toothbrush) or use gestures 5. Speak slowly but loudly while looking directly at the client

1,3,4

The nurse is performing an assessment on a client with a diagnosis of Bell's palsy. The nurse should expect to observe which finding in the client? 1.Facial drooping 2.Periorbital edema 3.Ptosis of the eyelid 4.Twitching on the affected side of the face

1. Bell's palsy is a one-sided facial paralysis caused by the compression of the facial nerve (cranial nerve VII). Assessment findings include facial droop from paralysis of the facial muscles; increased lacrimation; painful sensations in the eye, face, or behind the ear; and speech or chewing difficulty. Options 2, 3, and 4 are not associated findings in Bell's palsy. * Focus on the subject, manifestations of Bell's palsy. Recalling that Bell's palsy is a type of paralysis will direct you to the correct option. Also noting the word palsy in the question and the word drooping in the correct option will assist in answering correctly.

The nurse is planning to perform an assessment of the client's level of consciousness using the Glasgow Coma Scale. Which assessments should the nurse include in order to calculate the score? Select all that apply. 1. Eye opening 2. Reflex response 3. Best verbal response 4. Best motor response 5. Pupil size and reaction

1. Eye opening 3. Best verbal response 4. Best motor response Assessment of pupil size and reaction and reflex response are not part of the Glasgow Coma Scale. The 3 categories included are eye opening, best verbal response, and best motor response. Pupil assessment and reflex response is a necessary part of a total assessment of the neurological status of a client but is not part of this particular scale.

A client is taking phenytoin (Dilantin) for seizure control. A blood sample is drawn to determine the serum drug level, and the nurse reviews the results. Which would indicate a therapeutic serum drug range? 1. 5 to 10 mcg/mL 2. 10 to 20 mcg/mL 3. 20 to 30 mcg/mL 4. 30 to 40 mcg/mL

2 Rationale: The therapeutic serum drug level range for phenytoin is 10 to 20 mcg/mL; therefore the remaining options are incorrect.

An 84-year-old client in an acute state of disorientation is brought to the hospital emergency department by his or her daughter. The daughter states that the client was "clear as a bell this morning." The nurse determines from this piece of information that which is an unlikely cause of the disorientation? 1.Hypoglycemia 2.Alzheimer's disease 3.Medication dosage error 4.Impaired circulation to the brain

2 Rationale: Alzheimer's disease is a chronic disease with progression of memory deficits over time. The situation presented in the question represents an acute problem. Evaluation is necessary to determine whether hypoglycemia, medication use, or impaired cerebral circulation has had a role in causing the client's current symptoms.

A client is brought to the emergency department by emergency medical services with a flaccid right arm and leg and lack of verbal response. The stroke alert team is initiated. The nurse takes which priority action? 1. Determine onset of symptoms 2. Ensure that the client has 2 large-bore intravenous (IV) lines 3. Maintain patent airway 4. Prepare for head CT scan

3

A client with a history of headaches is scheduled for a lumbar puncture to assess the cerebrospinal fluid pressure. The nurse is preparing the client for the procedure. Which statement by the client indicates a need for further teaching by the nurse? 1. "I may feel a sharp pain that shoots to my leg, but it should pass soon." 2. "I will go to the bathroom and try to urinate before the procedure." 3. "I will need to lie on my stomach during the procedure." 4. "The physician will insert a needle between the bones in my lower spine."

3

The nurse is evaluating the respiratory outcomes for a client with Guillain-Barré syndrome. The nurse determines that which is the least optimal outcome for the client? 1. Spontaneous breathing 2. Oxygen saturation of 98% 3. Adventitious breath sounds 4. Vital capacity within normal range

3 Rationale: Satisfactory respiratory outcomes for a client with Guillain-Barré syndrome include clear breath sounds on auscultation, spontaneous breathing, normal vital capacity, normal arterial blood gas levels, and normal pulse oximetry. Adventitious breath sounds are an abnormal finding.

The emergency department nurse is triaging clients. Which neurologic presentation is most concerning for a serious etiology and should be given priority for definitive treatment? 1. History of Bell's palsy with unilateral facial droop and drooling 2. History of multiple sclerosis and reporting recent blurred vision 3. Reports unilateral facial pain when consuming hot foods 4. Temple region hit by ball, loss of consciousness, but Glasgow Coma Scale score is now 14

4

The nurse is caring for a client who has homonymous hemianopsia following an acute stroke. Which nursing diagnosis is the most appropriate for this client? 1. Risk for ineffective airway maintenance 2. Risk for knowledge deficit 3. Risk for poor fluid intake 4. Risk for self-neglect

4

The nurse is reviewing the record of a client with a suspected diagnosis of Huntington's disease. The nurse should expect to note documentation of which early symptom of this disease? 1.Aphasia 2.Agnosia 3.Difficulty with swallowing 4.Balance and coordination problems

4. Early symptoms of Huntington's disease include restlessness, forgetfulness, clumsiness, falls, balance and coordination problems, altered speech, and altered handwriting. Difficulty with swallowing occurs in the later stages. Aphasia and agnosia do not occur. * Note the strategic word early. Recalling the pathophysiology of Huntington's disease and that alterations in balance and coordination occur will direct you to the correct option.

The nurse is developing a plan of care for a client with a diagnosis of brain attack (stroke) with anosognosia. To meet the needs of the client with this deficit, the nurse should include activities that will achieve which outcome? 1.Encourage communication. 2.Provide a consistent daily routine. 3.Promote adequate bowel elimination. 4.Increase the client's awareness of the affected side.

4. In anosognosia, the client exhibits neglect of the affected side of the body. The nurse will plan care activities that remind the client to perform actions that require looking at the affected arm or leg, as well as activities that will increase the client's awareness of the affected side. Options 1, 2, and 3 are not associated with this deficit. * Focus on the subject, care of the client following brain attack (stroke). Recall that in anosognosia the client demonstrates neglect of the affected side of the body. This will direct you to the correct option.

A home health nurse visits a client who's taking pilocarpine, a miotic agent, to treat glaucoma. The nurse notes that the client's pilocarpine solution is cloudy. What should the nurse do first? a) Advise the client to discard the drug because it may have undergone chemical changes or become contaminated. b) Advise the client to keep the container closed tightly and protected from light. c) Watch the client or a family member administer the drug to determine possible contamination sources. d) Advise the client to obtain a fresh container of pilocarpine solution to avoid omitting ordered doses.

A

After 1 month of therapy, the client in spinal shock begins to experience muscle spasms in the legs, and calls the nurse in excitement to report the leg movement. Which response by the nurse would be the most accurate? a) "The movements occur from muscle reflexes that cannot be initiated or controlled by the brain." b) "These movements indicate that the damaged nerves are healing." c) "This is a good sign. Keep trying to move all the affected muscles." d) "The return of movement means that eventually you should be able to walk again."

A

A client has dysfunction of the cochlear division of the vestibulocochlear nerve (cranial nerve VIII). The nurse should determine that the client is adequately adapting to this problem if he or she states a plan to obtain which item? 1. A walker 2. Eyeglasses 3. A hearing aid 4. A bath thermometer

A hearing aid The cochlear division of cranial nerve VIII is responsible for hearing. Clients with hearing difficulty may benefit from the use of a hearing aid. The vestibular portion of this nerve controls equilibrium; difficulty with balance caused by dysfunction of this division could be addressed with use of a walker. Eyeglasses would correct visual problems (cranial nerve II); a bath thermometer would be of use to clients with sensory deficits of peripheral nerves, such as with diabetic neuropathy.

A client in the emergency department is diagnosed with Bell's palsy. The nurse collecting data on this client expects to note which observation?

A lag in closing the bottom eyelid The facial drooping associated with Bell's palsy makes it difficult for the client to close the eyelid on the affected side. A widening of the palpebral fissure (the opening between the eyelids) and an asymmetrical smile are seen with Bell's palsy. Paroxysms of excruciating pain are seen with trigeminal neuralgia.

The nurse is conducting home visits with a head-injured client with residual cognitive deficits. The client has problems with memory, has a shortened attention span, is easily distracted, and processes information slowly. The nurse plans to talk with the primary health care provider about referring the client to which professional? 1. A psychologist 2. A social worker 3. A neuropsychologist 4. A vocational rehabilitation specialist

A neuropsychologist Clients with cognitive deficits after head injury may benefit from referral to a neuropsychologist, who specializes in evaluating and treating cognitive problems. The neuropsychologist plans an individual program of therapy and initiates counseling to help the client reach maximal potential. The neuropsychologist works in collaboration with other disciplines that are involved in the client's care and rehabilitation. The remaining options are incorrect because these health care workers do not specialize in evaluating and treating cognitive problems.

The nurse is evaluating the status of a client who had a craniotomy 3 days ago. Which assessment finding would indicate that the client is developing meningitis as a complication of surgery? 1. A negative Kernig sign 2. Absence of nuchal rigidity 3. A positive Brudzinski sign 4. A Glasgow Coma Scale score of 15

A positive Brudzinski sign Signs of meningeal irritation compatible with meningitis include nuchal rigidity, a positive Brudzinski sign, and positive Kernig sign. Nuchal rigidity is characterized by a stiff neck and soreness, which is especially noticeable when the neck is flexed. Kernig's sign is positive when the client feels pain and spasm of the hamstring muscles when the leg is fully flexed at the knee and hip. Brudzinski's sign is positive when the client flexes the hips and knees in response to the nurse gently flexing the head and neck onto the chest. A Glasgow Coma Scale score of 15 is a perfect score and indicates that the client is awake and alert, with no neurological deficits.

The nurse is evaluating the status of a client who had a craniotomy 3 days ago. Which assessment finding would indicate that the client is developing meningitis as a complication of surgery? 1. A negative Kernig's sign 2. Absence of nuchal rigidity 3. A positive Brudzinski's sign 4. A Glasgow Coma Scale score of 15

A positive Brudzinski's sign Signs of meningeal irritation compatible with meningitis include nuchal rigidity, a positive Brudzinski's sign, and positive Kernig's sign. Nuchal rigidity is characterized by a stiff neck and soreness, which is especially noticeable when the neck is flexed. Kernig's sign is positive when the client feels pain and spasm of the hamstring muscles when the leg is fully flexed at the knee and hip. Brudzinski's sign is positive when the client flexes the hips and knees in response to the nurse gently flexing the head and neck onto the chest. A Glasgow Coma Scale score of 15 is a perfect score and indicates that the client is awake and alert, with no neurological deficits.

The nurse is caring for a client diagnosed with a subarachnoid hemorrhage resulting from a leaking aneurysm. The client is awaiting surgery. Which nursing interventions would be appropriate for the nurse to implement? Select all that apply.

Administer docusate per order. Elevate the head of bed 30 degrees. Provide a dimly lit environment. Cerebral aneurysm precautions are implemented for the patient with a diagnosis of aneurysm to provide a nonstimulating environment, prevent increases in intracranial pressure (ICP), and prevent further bleeding. The patient is placed on immediate and absolute bed rest in a quiet, nonstressful environment, because activity, pain, and anxiety elevate blood pressure, which increases the risk for bleeding. Visitors, except for family, are restricted. Dim lighting is helpful because photophobia (visual intolerance of light) is common. The head of the bed is elevated 15 to 30 degrees to promote venous drainage and decrease ICP. No enemas are permitted, but stool softeners (Colace) and mild laxatives are prescribed. Both prevent constipation, which would cause an increase in ICP, as would enemas.

The nurse has obtained a personal and family history from a client with a neurological disorder. Which finding in the client's history is least likely associated with a risk for neurological problems?

Allergy to pollen Previous neurological problems such as headaches or back injuries place the client more at risk for development of a neurological disorder. Chronic diseases such as hypertension and diabetes mellitus also place the client at greater risk. Assessment of allergies is a routine part of the health history, regardless of the nature of the client's problem. In addition, an allergy to pollen would not place the client at risk for a neurological problem.

An older client in an acute state of disorientation is brought to the hospital emergency department by the client's daughter. The daughter states that the client was "clear as a bell this morning." The nurse determines from this piece of information that which is an unlikely cause of the disorientation? 1. Hypoglycemi 2. Alzheimer's disease 3. Medication dosage 4. Impaired circulation to the brain

Alzheimer's disease Alzheimer's disease is a chronic disease with progression of memory deficits over time. The situation presented in the question represents an acute problem. Evaluation is necessary to determine whether hypoglycemia, medication use, or impaired cerebral circulation has had a role in causing the client's current symptoms.

A nurse is performing a complete neurological examination of a 7-year-old boy. She will now test his cerebellar function. Which of the following tests would be appropriate for this purpose? a) Ask the boy to close his eyes and then touch his skin with a cotton wisp; ask him to point to where he was touched b) Measure the circumference of the calves and thighs with a tape measure c) Ask the boy who he is, where he is, and what day it is d) Ask the boy to touch each finger on one hand with the thumb that hand in rapid succession

Ask the boy to touch each finger on one hand with the thumb that hand in rapid succession

The eyes of a 9-year-old who suffered a head injury are crossed. Besides checking ICP, which of the following interventions would be most important for the nurse to perform? a) Pull up the side rails on the bed. b) Monitor core body temperature. c) Assess the child's level of consciousness. d) Help the child cope with an altered appearance.

Assess the child's level of consciousness. Correct Explanation: Decreased level of consciousness is frequently the first sign of major neurologic problems after head trauma. While body temperature is an important indicator of infection, it is not a priority here. Preventing harm by setting the side rails is more important for a seizure patient. The child's eyes will correct themselves when ICP is reduced.

Which nursing action should be included in the care plan to promote comfort in a 4-year-old child hospitalized with meningitis? a) Avoid making noise when in the child's room b) Keep the lights on brightly so that he can see his mother c) Rock the child frequently d) Have the child's 2-year-old brother stay in the room

Avoid making noise when in the child's room Correct Explanation: Meningeal irritation may cause seizures and heightens a child's sensitivity to all stimuli, including noise, lights, movement, and touch. Frequent rocking, presence of a younger sibling, and bright lights would increase stimulation.

The nurse enters the client's room as the client, who is sitting in a chair, begins to have a seizure. The nurse should first: a) restrain the client's body movements. b) ease the client to the floor. c) insert an airway into the client's mouth. d) lift the client onto his bed.

B

What should the nurse do first>/b> when a client with a head injury begins to have clear drainage from the nose? a) Administer an antihistamine for postnasal drip. b) Collect the drainage. c) Compress the nares. d) Tilt the head back.

B

When assessing the client with Parkinson's disease, the nurse should observe the client for: a) aphasia. b) a stiff, masklike facial expression. c) dry mouth. d) an exaggerated sense of euphoria.

B

Which is an initial sign of Parkinson's disease? a) akinesia b) tremor c) bradykinesia d) rigidity

B

Which of the following is consistent with increased ICP in the child? a) Increased appetite b) Bulging fontanel c) Emotional lability d) Narcolepsy

Bulging fontanel

What finding is consistent with increased ICP in the child?

Bulging fontanel Children with increased ICP exhibit bulging fontanels. They typically have a decreased appetite, are restless, and have trouble sleeping.

Which of the following is consistent with increased ICP in the child? a) Increased appetite b) Emotional lability c) Bulging fontanel d) Narcolepsy

Bulging fontanel Correct Explanation: Children with increased ICP exhibit bulging fontanels. They typically have a decreased appetite, are restless, and have trouble sleeping.

At 8:00 a.m., A client who has had a stroke (brain attack) was awake and alert with vital signs of temperature 98°F (37.2°C) orally, pulse 80 beats/min, respirations 18 breaths/min, and blood pressure 138/80 mm Hg. At noon, the client is confused and only responsive to tactile stimuli, and vital signs are temperature 99°F (36.7°C) orally, pulse 62 beats/min, respirations 20 breaths/min, and blood pressure 166/72 mm Hg. The nurse should take which action? 1. Reorient the client. 2. Retake the vital signs. 3. Call the health care provider (HCP). 4. Administer an antihypertensive PRN (as needed).

Call the health care provider (HCP). The important nursing action is to call the HCP. The deterioration in neurological status, decreasing pulse, and increasing blood pressure with a widening pulse pressure all indicate that the client is experiencing increased intracranial pressure, which requires immediate treatment to prevent further complications and possible death. The nurse should retake the vital signs and reorient the client to surroundings. If the client's blood pressure falls within parameters for PRN antihypertensive medication, the medication also should be administered. However, options 1, 2, and 4 are secondary nursing actions.

The nurse reviews the health care provider's treatment plan for a client with Guillain-Barré syndrome. Which prescription noted in the client's record should the nurse question?

Clear liquid diet Clients with Guillain-Barré syndrome have dysphagia. Clients with dysphagia are more likely to aspirate clear liquids than thick or semisolid foods. Clients with Guillain-Barré syndrome are at risk for hypotension or hypertension, bradycardia, and respiratory depression and require frequent monitoring of vital signs. Passive range-of-motion exercises can help prevent contractures, and checking calf measurements can help detect deep vein thrombosis, for which clients are at risk.

The nursing instructor asks a nursing student about the points to document if the client has had a seizure. The instructor determines that the student needs to research seizures and related documentation points if the student states which assessment is important?

Client's diet in the 2 hours preceding seizure activity Typically, seizure assessment includes the time the seizure began, part(s) of the body affected, the type of movements and progression of the seizure, changes in pupil size, eye deviation or nystagmus, client condition during the seizure, and postictal status.

The nurse is caring for a client after a craniotomy and monitors the client for signs of increased intracranial pressure (ICP). Which finding, if noted in the client, would indicate an early sign of increased ICP? 1. Confusion 2. Bradycardia 3. Sluggish pupils 4. A widened pulse pressure

Confusion Early manifestations of increased ICP are subtle and often may be transient, lasting for only a few minutes in some cases. These early clinical manifestations include episodes of confusion, drowsiness, and slight pupillary and breathing changes. Later manifestations include a further decrease in the level of consciousness, a widened pulse pressure, and bradycardia. Cheyne-Stokes respiratory pattern, or a hyperventilation respiratory pattern, and pupillary sluggishness and dilatation appear in the late stages.

The nurse in the neurological unit is caring for a client who was in a motor vehicle crash and sustained a blunt head injury. On assessment of the client, the nurse notes the presence of bloody drainage from the nose. Which nursing action is most appropriate? Insert nasal packing. 2. Document the findings. 3. Contact the health care provider (HCP). 4. Monitor the client's blood pressure and check for signs of increased intracranial pressure.

Contact the health care provider (HCP). Rationale: Bloody or clear drainage from either the nasal or the auditory canal after head trauma could indicate a cerebrospinal fluid leak. The appropriate nursing action is to notify the HCP, because this finding requires immediate intervention. Options 1, 2, and 4 are inappropriate nursing actions in this situation.

Complications associated with a tracheostomy tube include: a) pneumothorax. b) acute respiratory distress syndrome (ARDS). c) decreased cardiac output. d) damage to the laryngeal nerve.

D

Which statement indicates that a client understands the nurse's teaching about phenytoin for the diagnosis of seizures? a) "I will only be on this type of medication for a short while." b) "This medication can help reduce my anxiety." c) "This medication may keep me awake." d) "This medication will not cure my disease."

D

A client who is paraplegic after spinal cord injury has been taught muscle-strengthening exercises for the upper body. The nurse determines that the client will derive the least muscle-strengthening benefit from which activity?

Doing active range of motion to finger joints Range-of-motion exercises of the finger joints prevent contractures but do not actively strengthen muscle groups needed for self-mobilization with paraplegia. Other activities that are more effective include push-ups from a prone position, sit-ups from a sitting position, extending the arms while holding weights, and squeezing rubber balls or crumpling newspaper.

The nurse is teaching a client hospitalized with a seizure disorder and the client's spouse about safety precautions after discharge. The nurse determines that the client needs further teaching if the client states an intention to take which action? 1. Refrain from smoking alone. 2. Take all prescribed medications on time. 3. Have the spouse nearby when showering. 4. Drink alcohol in small amounts and only on weekends.

Drink alcohol in small amounts and only on weekends. The client should avoid the intake of alcohol. Alcohol could interact with the client's seizure medications, or it could precipitate seizure activity. The client should take all medications on time to avoid decreases in therapeutic medication levels, which could precipitate seizures. The client should not bathe in the shower or tub without someone nearby and should not smoke alone, to minimize the risk of injury if a seizure occurs.

The nurse and an adolescent are reviewing the adolescent's record of her headaches and activities surrounding them. What activity would the nurse identify as a possible trigger? a) Swimming twice a week b) 11 p.m. bedtime; 6:30 a.m. wake-up c) Drinking three cans of diet cola d) Use of nonscented soap

Drinking three cans of diet cola

A client is newly admitted to the hospital with a diagnosis of stroke (brain attack) manifested by complete hemiplegia. Which item in the medical history of the client should the nurse be most concerned about? 1. Glaucoma 2. Emphysema 3. Hypertension 4. Diabetes mellitus

Emphysema The nurse should be most concerned about emphysema. The respiratory system is the priority in the acute phase of a stroke. The client with a stroke is vulnerable to respiratory complications such as atelectasis and pneumonia. Because the client has complete hemiplegia (is unable to move) and has emphysema, these risks are very significant. Although the other conditions of glaucoma, hypertension, and diabetes mellitus are important, they are not as significant as emphysema.

The nurse is caring for a client who is on bed rest as part of aneurysm precautions. The nurse should avoid doing which action when giving respiratory care to this client? 1. Encouraging hourly coughing 2. Assisting with incentive spirometer 3. Encouraging hourly deep breathing 4. Repositioning gently side to side every 2 hours

Encouraging hourly coughing With aneurysm precautions, any activity that could raise the client's intracranial pressure (ICP) is avoided. For this reason, activities such as straining, coughing, blowing the nose, and even sneezing are avoided whenever possible. The other interventions (repositioning, deep breathing, and incentive spirometry) do not provide added risk of increasing ICP and are beneficial in reducing the respiratory complications of bed rest.

The nurse is caring for a client diagnosed with Bell's palsy 1 week ago. Which data would indicate a potential complication associated with Bell's palsy?

Excessive tearing Complications of Bell's palsy include abnormal return of nerve function; "crocodile tears" (autonomic fibers reconnect to the lacrimal duct instead of the salivary glands, so the client develops excessive tearing while eating); abnormal facial movements because of reinnervation of inappropriate muscles; and spasms, atrophy, and contractures caused by incomplete motor fiber reinnervation. Partial facial paralysis is a factor indicating recovery. Negative outcomes on the electromyography performed 1 week after symptom onset indicate that nerve function is present (a negative test indicates a positive prognostic outcome). Tasting food 1 week after symptom onset indicates a good prognosis for recovery.

A client recovering from a head injury is participating in care. The nurse determines that the client understands measures to prevent elevations in intracranial pressure if the nurse observes the client doing which activity? Blowing the nose 2. Isometric exercises 3. Coughing vigorously 4. Exhaling during repositioning

Exhaling during repositioning Activities that increase intrathoracic and intraabdominal pressures cause an indirect elevation of the intracranial pressure. Some of these activities include isometric exercises, Valsalva's maneuver, coughing, sneezing, and blowing the nose. Exhaling during activities such as repositioning or pulling up in bed, opens the glottis, which prevents intrathoracic pressure from rising.

The nurse is assigned to care for an adult client who had a stroke and is aphasic. Which interventions should the nurse use for communicating with the client? Select all that apply.

Face the client when talking. Speak slowly and maintain eye contact. Use gestures when talking to enhance words. Give the client directions using short phrases and simple terms. A client who is aphasic has difficulty expressing or understanding language. The nurse should face the client when talking, establish and maintain eye contact, and speak slowly and distinctly. The nurse should use gestures and pantomime when talking to enhance words and use body language to enhance the message. The nurse should give the client directions using short phrases and simple terms, and phrase questions so that they can be answered with a yes or no. If there is a need to repeat something, the nurse should use the same words a second time.

The nurse caring for a child with a cranial injury knows that broad-spectrum antibiotics are used to reduce cerebral edema.

False Antibiotics or antivirals are used to treat infectious disease processes. Glucocorticoids and diuretics are used to reduce cerebral edema.

The nurse is caring for a client with a spinal cord injury. High-top sneakers on the client's feet will prevent the occurrence of which?

Foot drop The most effective way to prevent foot drop is to use posterior splints or high-top sneakers. A foot board prevents plantar flexion but also places the client at greater risk for developing pressure ulcers of the feet. Pneumatic boots prevent deep vein thrombosis but not foot drop.

The nurse is preparing for the admission of a client with a diagnosis of early stage Alzheimer's disease. The nurse assists in developing a plan of care, knowing that which is a characteristic of early Alzheimer's disease?

Forgetfulness In early Alzheimer's disease, forgetfulness begins to interfere with daily routines. The client has difficulty concentrating and difficulty learning new material. Options 1, 2, and 4 are characteristics of dementia that occur late as the disease progresses.

The nurse is providing discharge education to a client diagnosed with trigeminal neuralgia. Which medication will likely be prescribed upon discharge for this condition? 1. Lorazepam 2. Gabapentin 3. Carisoprodol 4. Chlordiazepoxide

Gabapentin Trigeminal neuralgia is characterized by spasms of pain that start suddenly and last from seconds to minutes. The pain often is described as either stabbing or similar to an electric shock. It is accompanied by spasms of the facial muscles that cause twitching of parts of the face or mouth, or closure of the eye. It is treated by giving antiseizure medications, such as gabapentin, and sometimes tricyclic antidepressants. These medications work by stabilizing the neuronal membrane and blocking the nerve.

The nurse is caring for a client who has just been admitted to the hospital with a diagnosis of a hemorrhagic stroke. The nurse should place the client in which position? 1. Prone 2. Supine 3. Semi Fowler's with the hip and the neck flexed 4. Head of the bed elevated 30 degrees with the head in midline position

Head of the bed elevated 30 degrees with the head in midline position The health care provider's prescriptions are always followed with regard to positioning the client after stroke. Clients with hemorrhagic stroke usually have the head of the bed elevated to 30 degrees to reduce intracranial pressure that can occur from the hemorrhage. The head should be in a midline, neutral position to facilitate venous drainage from the brain. Extreme hip and neck flexion should be avoided to prevent an increase in intrathoracic pressure and to promote venous drainage from the brain. For clients with ischemic stroke, the head of the bed usually is kept flat to ensure adequate blood flow and thus oxygenation to the brain. Prone, supine, and hip and neck flexion are incorrect positions for clients with hemorrhagic stroke.

The nurse is performing a neurological assessment on a client who had a brain attack (stroke). The nurse checks for proprioception by which assessment technique? Tapping the Achilles tendon using the reflex hammer 2. Gently pricking the client's skin on the dorsum of the foot in two places 3. Firmly stroking the lateral sole of the foot and under the toes with a blunt instrument 4. Holding the sides of the client's great toe and, while moving it, asking what position it is in

Holding the sides of the client's great toe and, while moving it, asking what position it is in A method of testing for proprioception is to hold the sides of the client's great toe and, while moving it, asking the client what position it is in. Option 1 describes gastrocnemius muscle contraction. Option 2 describes two-point discrimination. Testing the plantar reflex is described in option 3.

Which term will the nurse use when referring to blindness in the right or left half of the visual field in both eyes?

Homonymous hemianopsia Homonymous hemianopsia occurs with occipital lobe tumors. Scotoma refers to a defect in vision in a specific area in one or both eyes. Diplopia refers to double vision or the awareness of two images of the same object occurring in one or both eyes. Nystagmus refers to rhythmic, involuntary movements or oscillations of the eyes.

The nurse is caring for an unconscious client who is experiencing persistent hyperthermia with no signs of infection. On the basis of these findings the nurse suspects dysfunction in which area of the brain? 1. Cerebrum 2. Cerebellum 3. Hippocampus 4. Hypothalamus

Hypothalamus Hypothalamic damage causes persistent hyperthermia, which also may be called central fever. It is characterized by a persistent high fever with no diurnal variation. Another characteristic feature is absence of sweating. Hyperthermia would not result from damage to the cerebrum, cerebellum, or hippocampus.

The nurse notes documentation that a postcraniotomy client is having difficulty with body image. The nurse determines that the client is still working on the postoperative outcome criteria when the client indicates which altered personal appearance?

Indicates that facial puffiness will be a permanent problem After craniotomy, the client may experience difficulty with altered personal appearance. The nurse can help by listening to the client's concerns and by clarifying any misconceptions about facial edema, periorbital bruising, and hair loss, which are temporary. The nurse can encourage the client to participate in self-grooming and use personal articles of clothing. Finally, the nurse can suggest the use of a turban, followed by a hairpiece, to help the client adapt to the temporary change in appearance.

The nurse is caring for a 6-year-old child who has a history of febrile seizures and is admitted with a temperature of 102.2° F(39° C). What is the nurse's highest priority? a) Institute safety precautions. b) Provide family teaching related to the child's history. c) Offer age-appropriate activities. d) Encourage the child to do his or her own self-care.

Institute safety precautions.

A client with multiple sclerosis is experiencing muscle weakness, spasticity, and an ataxic gait. On the basis of this information, the nurse should include which client problem in the plan of care? 1. Inability to care for self 2. Interruption in skin integrity 3. Interruption in physical mobility 4. Inability to perform daily activities

Interruption in physical mobility Multiple sclerosis is a chronic, nonprogressive, noncontagious degenerative disease of the central nervous system characterized by demyelination of the neurons. Interruption in physical mobility is most appropriate for the client with multiple sclerosis experiencing muscle weakness, spasticity, and ataxic gait. The remaining options are not related to the data in the question.

A nurse is working with a student nurse who is caring for a client with an acute bleeding cerebral aneurysm. Which action by the student nurse requires further intervention?

Keeping the client in one position to decrease bleeding The student nurse shouldn't keep the client in one position. She should carefully reposition the client often (at least every hour). The client needs to be positioned so that a patent airway can be maintained. Fluid administration must be closely monitored to prevent complications such as increased intracranial pressure. The client must be maintained in a quiet environment to decrease the risk of rebleeding.

The nurse is preparing a toddler for a lumbar puncture. For this procedure, the nurse should place the child in which position? a) Lying prone, with the neck flexed b) Sitting up, with the back straight c) Lying on one side, with the back curved d) Lying prone, with the feet higher than the head

Lying on one side, with the back curved Correct Explanation: Lumbar puncture involves placing a needle between the lumbar vertebrae into the subarachnoid space. For this procedure, the nurse should position the client on one side with the back curved because curving the back maximizes the space between the lumbar vertebrae, facilitating needle insertion. Prone and seated positions don't achieve maximum separation of the vertebrae.

A client with a spinal cord injury expresses little interest in food and is very particular about the choice of meals that are actually eaten. How should the nurse interpret this information? 1. Anorexia is a sign of clinical depression, and a referral to a psychologist is needed. 2. The client has compulsive habits that should be ignored as long as they are not harmful. 3. The client probably has a naturally slow metabolism, and the decreased nutritional intake will not matter. 4. Meal choices represent an area of client control and should be encouraged as much as is nutritionally reasonable.

Meal choices represent an area of client control and should be encouraged as much as is nutritionally reasonable. Depression frequently may be seen in the client with spinal cord injury and may be exhibited as a loss of appetite. However, the client should be allowed to choose the types of food eaten and when they are eaten as much as is feasible because it is one of the few areas of control that the client has left. There is no information in the question that would indicate that the client is anorexic or obsessive-compulsive or has a slow metabolism.

A client with a seizure disorder is being admitted to the hospital. Which should the nurse plan to implement for this client? Select all that apply.

Pad the bed's side rails. Place an airway at the bedside Place oxygen equipment at the bedside. Place suction equipment at the bedside. The nurse should plan seizure precautions for a client with a seizure disorder. The precautions include padded side rails and an airway, and oxygen and suction equipment at the bedside. Attempts to force a padded tongue blade between clenched teeth may result in injury to the teeth and mouth; therefore a padded tongue blade is not placed at the bedside.

The clinic nurse is reviewing the medical record of a client scheduled to be seen in the clinic. The nurse notes that the client is prescribed selegiline hydrochloride (Eldepryl). The nurse understands that this medication is prescribed for which diagnosis?

Parkinson's disease Selegiline hydrochloride is an antiparkinsonian medication. The medication increases dopaminergic action, assisting in the reduction of tremor, akinesia, and the rigidity of parkinsonism. This medication is not used to treat coronary artery disease, diabetes mellitus, or Alzheimer's disease.

Put the following events of a generalized epileptic seizure in correct order: Tonic stage Prodromal period Clonic stage Postictal period

Prodromal period Tonic stage Clonic stage Postictal period Correct Explanation: A tonic-clonic seizure is characterized by the following events: 1) prodromal period, 2) tonic stage, 3) clonic stage, and 4) postictal period.

The nurse has a prescription to begin aneurysm precautions for a client with a subarachnoid hemorrhage secondary to aneurysm rupture. The nurse would plan to incorporate which intervention in controlling the environment for this client? Keep the window blinds open. 2. Turn on a small spotlight above the client's head. 3. Make sure the door to the room is open at all times. 4. Prohibit or limit the use of a radio or television and reading.

Prohibit or limit the use of a radio or television and reading. Rationale: Environmental stimuli are kept to a minimum with subarachnoid precautions to prevent or minimize increases in intracranial pressure. For this reason, lighting is reduced by closing window blinds and keeping the door to the client's room shut. Overhead lighting also is avoided for the same reason. The nurse prohibits television, radio, and reading unless this is so stressful for the client that it would be counterproductive. In that instance, minimal amounts of stimuli by these means are allowed with approval of the health care provider.

A client admitted to the hospital with a neurological problem indicates to the nurse that magnetic resonance imaging (MRI) may be done. Which finding noted in the client history indicates that the client may be ineligible for this diagnostic procedure?

Prosthetic valve replacement The client having an MRI must have all metallic objects removed because of the magnetic field generated by the device. A careful history is done to determine if any metal objects are inside the client, such as orthopedic hardware, pacemakers, artificial heart valves, aneurysm clips, or intrauterine devices. These may heat up, become dislodged, or malfunction during this procedure. The client may be ineligible if there is significant risk.

A client has an impairment of cranial nerve II. Specific to this impairment, the nurse plans to do which to ensure client safety?

Provide a clear path for ambulation without obstacles. Cranial nerve II is the optic nerve, which governs vision. The nurse can provide safety for the visually impaired client by clearing the path of obstacles when ambulating. Testing the shower water temperature would be useful if there were impairment of peripheral nerves. Speaking loudly may help overcome a deficit of cranial nerve VIII (vestibulocochlear). Cranial nerves VII (facial) and IX (glossopharyngeal) control taste from the anterior two thirds and posterior one third of the tongue, respectively.

A client with Guillain-Barré syndrome has ascending paralysis and is intubated and receiving mechanical ventilation. Which strategy should the nurse incorporate in the plan of care to help the client cope with this illness? 1. Giving client full control over care decisions and restricting visitors 2. Providing positive feedback and encouraging active range of motion 3. Providing information, giving positive feedback, and encouraging relaxation 4. Providing intravenously administered sedatives, reducing distractions, and limiting visitors

Providing information, giving positive feedback, and encouraging relaxation The client with Guillain-Barré syndrome experiences fear and anxiety from the ascending paralysis and sudden onset of the disorder. The nurse can alleviate these fears by providing accurate information about the client's condition, giving expert care and positive feedback to the client, and encouraging relaxation and distraction. The family can become involved with selected care activities and provide diversion for the client as well.

The nurse is providing care to a client with increased intracranial pressure (ICP). Which approach is beneficial in controlling the client's ICP from an environmental viewpoint? 1. Reduce environmental noise. 2. Allow visitors as desired by the client and family. 3. Awaken the client every 2 to 3 hours to monitor mental status. 4. Cluster nursing activities to reduce the number of interruptions.

Reduce environmental noise. Nursing interventions to control ICP include maintaining a calm, quiet, and restful environment. Environmental noise should be kept at a minimum. Visiting should be monitored to avoid emotional stress and interruption of sleep. Interventions should be spaced out over the shift to minimize the risk of a sustained rise in ICP.

A client has tension headaches. The nurse recommends massage as a treatment for tension headaches. How does massage help clients with tension headaches? a) Relieves migraines b) Relaxes muscles c) Reduces hypotension d) Increases appetite

Relaxes muscles Massaging relaxes tense muscles, causes local dilation of blood vessels, and relieves headache. However, this approach is not likely to help a client with migraine or cluster headaches. Massage is not offered to clients with tension headaches to increase their appetite or reduce hypotension

At the end of the work shift, the nurse is reviewing the respiratory status of a client admitted with acute brain attack (stroke) earlier in the day. The nurse determines that the client's airway is patent if which data are identified? Respiratory rate 24 breaths/min, oxygen saturation 94%, breath sounds clear 2. Respiratory rate 18 breaths/min, oxygen saturation 98%, breath sounds clear 3. Respiratory rate 16 breaths/min, oxygen saturation 85%, wheezes bilaterally 4. Respiratory rate 20 breaths/min, oxygen saturation 92%, diminished breath sounds in lung bases

Respiratory rate 18 breaths/min, oxygen saturation 98%, breath sounds clear

The nurse is reviewing the record of a child with increased intracranial pressure and notes that the child has exhibited signs of decerebrate posturing. On assessment of the child, the nurse expects to note which characteristic of this type of posturing? Flaccid paralysis of all extremities 2. Adduction of the arms at the shoulders 3. Rigid extension and pronation of the arms and legs 4. Abnormal flexion of the upper extremities and extension and adduction of the lower extremities

Rigid extension and pronation of the arms and legs Decerebrate (extension) posturing is characterized by the rigid extension and pronation of the arms and legs. Option 1 is incorrect. Options 2 and 4 describe decorticate (flexion) posturing.

The nurse is preparing to care for a client who had a supratentorial craniotomy. The nurse should plan to place the client in which position? 1. Prone 2. Supine 3. Side-lying 4. Semi Fowler's

Semi Fowler's After supratentorial surgery (surgery above the tentorium of the brain), the head of the client's bed usually is elevated 30 degrees to promote venous outflow through the jugular veins. Prone, supine, and side-lying denote incorrect positions after this surgery, and these positions could result in edema at the surgical site and increased intracranial pressure. The health care provider's prescriptions are always followed with regard to positioning the client.

The nurse is caring for a client with Parkinson's disease. Which finding about gait should the nurse expect to note in the client? 1. Walking on the toes 2. Unsteady and staggering 3. Shuffling and propulsive 4. Broad-based and waddling

Shuffling and propulsive The parkinsonian gait is characterized by short, accelerating, shuffling steps. The client leans forward with the head, hips, and knees flexed and has difficulty starting and stopping. An ataxic gait is unsteady and staggering. A dystrophic gait is broad-based and waddling. Walking on the toes can occur from shortened Achilles tendons.

Which is indicative of a right hemisphere stroke?

Spatial-perceptual deficits Clients with right hemisphere stroke exhibit partial perceptual deficits, left visual field deficit, and paralysis with weakness on the left side of the body. Left hemisphere damage causes aphasia; slow, cautious behavior; and altered intellectual ability.

The nurse is preparing for the admission of a client with a prescription for seizure precautions. Which supplies will the nurse make available to this client? Select all that apply.

Suction machine Oxygen administration Padding for the side rails Prescribed diazepam (Valium) Full seizure precautions include bed rest with padded side rails in a raised position, a suction machine at the bedside, having diazepam (Valium) or lorazepam (Ativan) available, and oxygen. Objects such as tongue blades are not necessary and should never be placed in the client's mouth during a seizure.

A 9-year-old boy is suffering from headaches but has no signs of physical or neurologic illness. Which intervention would be most appropriate?

Teach the child and his parents to keep a headache diary. A headache diary can help identify any triggers so that the child can avoid them. Triggers can include foods eaten, amount of sleep the night before, or activities at home or school that might be causing stress. Reviewing signs of increased intracranial pressure would be inappropriate because increased intracranial pressure is not associated with headaches. Having the child sleep without a pillow is an intervention to reduce pain from meningitis. Vomiting more than twice is an indication that the parents should notify the physician or nurse practitioner when the child has a head injury.

A client who had a stroke (brain attack) has right-sided hemianopsia. What should the nurse plan to do to help the client adapt to this problem? 1. Teach the client to scan the environment. 2. Place all objects within the left visual field. 3. Place all objects within the right visual field. 4. Ensure that the family brings the client's eyeglasses to hospital.

Teach the client to scan the environment. Hemianopsia is blindness in half of the visual field. The client with hemianopsia is taught to scan the environment. This allows the client to take in the entirety of the visual field, which is necessary for proper functioning within the environment and helps to prevent injury to the client. Options 2 and 3 will not help the client adapt to this visual impairment. Eyeglasses are useful if the client already wears them, but they will not correct this visual field deficit.

The nurse is providing education to the parents of a female toddler with hydrocephalus who has just had a shunt placed. Which of the following statements is the best to use for a teaching session? a) Tell me your concerns about your child's shunt. b) Her autoregulation mechanism to absorb spinal fluid has failed. c) Call the doctor if she gets a persistent headache. d) Always keep her head raised 30º.

Tell me your concerns about your child's shunt.

The nurse is interviewing the caregivers of a child brought to the emergency unit. The caregiver states, "She has a history of seizures but this time it lasted more than 30 minutes and she just keeps having them." The most accurate description of this child's condition would be:

The child is in status epilepticus. Status epilepticus is the term used to describe a seizure that lasts longer than 30 minutes or a series of seizures in which the child does not return to his or her previous normal level of consciousness. The child likely is having generalized seizures, but the most accurate description of what is happening is status epilepticus. With infantile spasms, muscle contractions are sudden, brief, symmetrical, and accompanied by rolling eyes. With absence seizures the child loses awareness and stares straight ahead but does not fall.

The nurse is preparing to care for a client with a diagnosis of brain attack (stroke). The nurse notes in the client's record that the client has anosognosia. The nurse plans care, knowing which is a characteristic of anosognosia?

The client neglects the affected side. In anosognosia, the client neglects the affected side of the body. The client may neglect the affected side (often creating a safety hazard as a result of potential injuries) or state that the involved arm or leg belongs to someone else. Options 1, 3, and 4 are not associated with anosognosia.

The nurse is preparing a plan of care for a client with a diagnosis of brain attack (stroke). On reviewing the client's record, the nurse notes an assessment finding of anosognosia. The nursing care plan should address which manifestation related to this finding? The client will be easily fatigued. 2. The client will have difficulty speaking. 3. The client will have difficulty swallowing. 4. The client will exhibit neglect of the affected side.

The client will exhibit neglect of the affected side. In anosognosia, the client neglects the affected side of the body. The client either may ignore the presence of the affected side (often creating a safety hazard as a result of potential injuries) or may state that the involved arm or leg belongs to someone else. Options 1, 2, and 3 are not associated with anosognosia.

The nurse is creating a plan of care for a client with a diagnosis of stroke (brain attack). On reviewing the client's record, the nurse notes an assessment finding of anosognosia. The nursing care plan should address which manifestation related to this finding? 1. The client will be easily fatigued. 2. The client will have difficulty speaking. 3. The client will have difficulty swallowing. 4. The client will exhibit neglect of the affected side.

The client will exhibit neglect of the affected side. In anosognosia, the client neglects the affected side of the body. The client either may ignore the presence of the affected side (often creating a safety hazard as a result of potential injuries) or may state that the involved arm or leg belongs to someone else. The remaining options are not associated with anosognosia.

The nurse is developing a plan of care for a child who is at risk for seizures. Which interventions apply if the child has a seizure? Select all that apply. Time the seizure. 2. Restrain the child. 3. Stay with the child. 4. Place the child in a prone position. 5. Move furniture away from the child. 6. Insert a padded tongue blade in the child's mouth.

Time the seizure. Stay with the child. Move furniture away from the child. A seizure is a disorder that occurs as a result of excessive and unorganized neuronal discharges in the brain that activate associated motor and sensory organs. During a seizure, the child is placed on his or her side in a lateral position. Positioning on the side prevents aspiration because saliva drains out the corner of the child's mouth. The child is not restrained because this could cause injury to the child. The nurse would loosen clothing around the child's neck and ensure a patent airway. Nothing is placed into the child's mouth during a seizure because this action may cause injury to the child's mouth, gums, or teeth. The nurse would stay with the child to reduce the risk of injury and allow for observation and timing of the seizure.

Which nursing actions apply to the care of a child who is having a seizure? Select all that apply. Time the seizure. 2. Restrain the child. 3. Stay with the child. 4. Insert an oral airway. 5. Place the child in a lateral side-lying position. 6. Loosen clothing around the child's neck.

Time the seizure. Stay with the child. Place the child in a lateral side-lying position. Loosen clothing around the child's neck. During a seizure, the child is placed on his or her side in a lateral position. Positioning on the side prevents aspiration because saliva drains out the corner of the child's mouth. The child is not restrained because this could cause injury to the child. The nurse should loosen clothing around the child's neck and ensure a patent airway. Nothing is placed in the child's mouth during a seizure because this could injure the child's mouth, gums, or teeth. The nurse should stay with the child to reduce the risk of injury and allow for observation and timing of the seizure.

A client who is experiencing an inferior wall myocardial infarction has had a drop in heart rate into the range of 50 to 56 beats/minute. The client also is complaining of nausea. On the basis of these findings, the nurse determines that the client is experiencing parasympathetic stimulation of which cranial nerve? 1. Vagus (CN X) 2. Hypoglossal (CN XII) 3. Spinal accessory (CN XI) 4. Glossopharyngeal (CN IX)

Vagus (CN X) The vagus nerve is responsible for sensations in the thoracic and abdominal viscera. It also is responsible for the decrease in heart rate because approximately 75% of all parasympathetic stimulation is carried by the vagus nerve. CN XII is responsible for tongue movement. CN XI is responsible for neck and shoulder movement. CN IX is responsible for taste in the posterior two thirds of the tongue, pharyngeal sensation, and swallowing.

A nurse is performing a neurological examination of a preschool girl. She is testing her remote memory. Which of the following would be an appropriate type of memory to ask the girl to recall? a) What the girl had for dinner last night b) A string of three digits that the nurse has just spoken to her c) The name of an object that the nurse showed her 5 minutes ago d) Where the girl and her family went on vacation last year

What the girl had for dinner last night

A child with a seizure disorder is being admitted to the inpatient unit. When preparing the room for the child, what should be included?

• Oxygen gauge and tubing • Suction at bedside • Padding for side rails When planning the client's environment it is imperative that both safety items and those to manage the seizure are present. The side rails should be padded to prevent injury during seizure activity. Oxygen setup should be provided. Suction may be needed. Tongue blades and smelling salts are not employed.

An 11-year-old child was recently diagnosed with chickenpox. His parents gave him aspirin for a fever and the child is now hospitalized. Which nursing interventions are appropriate for this child?

• Request order for an antiemetic • Assess intake and output every shift • Request order for anticonvulsant This child likely has Reye syndrome and may require an anti-emetic for severe vomiting. The nurse should monitor the child's intake and output every shift for the development of fluid imbalance. The child may require an anticonvulsant due an increased intracranial pressure that may induce seizures. A distinctive rash is associated with the development of meningococcal meningitis. The nurse should monitor the Reye's syndrome child's laboratory values for indications that the liver is not functioning well

A child is home with the caregivers following a treatment for a head injury. If the child makes which of the following statements, the caregiver should contact the care provider. a) "I am glad that my headache is getting better." b) "My stomach is upset. I feel like I might throw up." c) "You look funny. Well, both of you do. I see two of you." d) "It will be nice when you will let me take a long nap. I am sleepy."

"You look funny. Well, both of you do. I see two of you."

A speeding driver sustained a closed-head injury in an acceleration/deceleration accident from striking a tree front end first. Based on the coup-contrecoup phenomenon, which assessments are most likely to be affected related to the involved areas of the brain? 1. Expressive speech, vision 2. Light touch, hearing 3. Sense of position, graphesthesia 4. Weber tuning fork test, cranial nerve I

1

The nurse in the same-day surgery unit admits a client who will receive general anesthesia. The client has never had surgery before. Which question is most critical for the nurse to ask the client during the preoperative assessment and health history? 1. "Has any family member ever had a bad reaction to general anesthesia?" 2. "Have you ever experienced low back pain?" 3. "Have you ever had an anaphylactic reaction to a bee sting?" 4. "Have you ever received opioid pain medications?"

1

The nurse is preparing to discharge a client with chronic low back pain. Which statement by the client indicates the need for additional teaching? 1. "I will avoid exercise because the pain gets worse." 2. "I will use heat or ice to help control the pain." 3. "I will not wear high-heeled shoes at home or work." 4. "I will purchase a firm mattress to replace my old one."

1 Exercises are used to strengthen the back, relieve pressure on compressed nerves, and protect the back from reinjury. Ice, heat, and firm mattresses are appropriate interventions for back pain. People with chronic back pain should avoid wearing high-heeled shoes at all times.

The nurse is assessing a client with advanced amyotrophic lateral sclerosis. Which assessment findings does the nurse expect? Select all that apply. 1. Difficulty breathing 2. Difficulty swallowing 3. Muscle weakness 4. Resting tremor 5. Significant memory loss

1,2,3

An adult client with altered mental status and fever has suspected bacterial meningitis with sepsis. Blood pressure is 80/60 mm Hg. Which prescribed intervention should the nurse implement first? 1. Administer IV antibiotics 2. Infuse bolus of IV normal saline 3. Prepare to assist with lumbar puncture 4. Transport client for head CT scan

2

The nurse is caring for a client after a lumbar puncture (spinal tap). Which client assessment is most concerning and requires a nursing response? 1. Consumes 600 mL liquid over 4 hours 2. Insertion site dressing saturated with clear fluid 3. Observed lying in the right-sided Sim's position 4. Reports a headache rated 6/10

2

A client who recently started taking phenytoin to control simple partial seizures is seen in the outpatient clinic. Which information obtained during the nurse's chart review and assessment will be of greatest concern? 1. The gums appear enlarged and inflamed. 2. The white blood cell count is 2300/mm3 (2.3 x 109/L). 3. The client sometimes forgets to take the phenytoin until the afternoon. 4. The client wants to renew her driver's license in the next month.

2 Leukopenia is a serious adverse effect of phenytoin therapy and would require discontinuation of the medication. The other data indicate a need for further assessment or client teaching but will not require a change in medical treatment for the seizures.

The nurse is floated from the emergency department to the neurologic floor. Which action should the nurse delegate to the unlicensed assistive personnel (UAP) when providing nursing care for a client with a spinal cord injury? 1. Assessing the client's respiratory status every 4 hours 2. Checking and recording the client's vital signs every 4 hours 3. Monitoring the client's nutritional status, including calorie counts 4. Instructing the client how to turn, cough, and breathe deeply every 2 hours

2 The UAP's training and education covers measuring and recording vital signs. The UAP may help with turning and repositioning the client and may remind the client to cough and deep breathe, but he or she does not teach the client how to perform these actions. Assessing and monitoring clients require additional education and are appropriate to the scope of practice of professional nurses

A nurse is evaluating an acutely ill client with suspected meningitis. The nurse should take what action first? 1. Check for Kernig's and Brudzinski's signs 2. Establish IV access 3. Place the client on droplet precautions 4. Prepare the client for lumbar puncture

3

The nurse has a prescription to administer diazepam (Valium) 5 mg by the intravenous (IV) route to a client. The nurse should administer the medication over a period of at least how long? 1. 15 seconds 2. 30 seconds 3. 1 minute 4. 5 minutes

3 Rationale: The recommended rate of infusion of diazepam is to give each 5 mg of the medication over at least 1 minute. This will prevent adverse effects including apnea, bradycardia, hypotension, and possibly cardiac arrest. Focus on the subject, administering diazepam by the IV route. A majority of medications administered by the IV route to a client must be given over at least 1 minute. Because of this requirement, you could eliminate options 1 and 2 first as excessively brief. Regarding the remaining options, it is much more likely that it is given over 1 minute than 5 minutes.

The nursing student is writing a plan of care for a child who presents with an acute head injury. The nursing instructor reviews the plan of care and praises the student for identifying which assessment as a priority? 1. Inspecting the scalp 2. Pupillary assessment 3. Airway and breathing 4. Palpating the child's head

3 The first step in the emergency treatment of child with head injury includes the ABCs-airway, breathing, and circulation-assessments. The other assessments are included when evaluating a head injury, but the priority is ABC.

A client with a neurological impairment experiences urinary incontinence. Which nursing action would be most helpful in assisting the client to adapt to this alteration? 1.Using adult diapers 2.Inserting a Foley catheter 3.Establishing a toileting schedule 4. Padding the bed with an absorbent cotton pad

3 Rationale: A bladder retraining program, such as use of a toileting schedule, may be helpful to clients experiencing urinary incontinence. A Foley catheter should be used only when necessary because of the associated risk of infection. Use of diapers or pads is the least acceptable alternative because of the risk of skin breakdown.

A client with a neurological problem is experiencing hyperthermia. Which measure would be least appropriate for the nurse to use in trying to lower the client's body temperature? 1.Giving tepid sponge baths 2.Applying a hypothermia blanket 3.Placing ice packs in the axilla and groin areas 4. Administering acetaminophen (Tylenol) per protocol

3 Rationale: Standard measures to lower body temperature include removing bed covers, providing cool sponge baths, using an electric fan in the room, administering acetaminophen, and placing a hypothermia blanket under the client. Ice packs are not used because they could cause shivering, which increases cellular oxygen demands, with the potential for increased intracranial pressure.

The nurse is preparing for the admission to the unit of a client with a diagnosis of seizures and is preparing to institute full seizure precautions. Which item is contraindicated for use if a seizure occurs? 1.Oxygen source 2.Suction machine 3.Padded tongue blade 4.Padding for the side rails

3. Full seizure precautions include bed rest with padded side rails in a raised position, a suction machine at the bedside, having diazepam (Valium) or lorazepam (Ativan) available, and providing an oxygen source. Objects such as tongue blades are contraindicated and should never be placed in the client's mouth during a seizure. * Note the word full and the closed-ended word contraindicated in the question. Recall that objects should not be placed in the client's mouth during a seizure. This principle will direct you to the correct option.

A nurse caring for a child who has sustained a head injury in an automobile crash is monitoring the child for signs of increased intracranial pressure (ICP). For which early sign of increased ICP should the nurse monitor? 1.Increased systolic blood pressure 2.Abnormal posturing of extremities 3.Significant widening pulse pressure 4.Changes in level of consciousness (LOC)

4 Rationale: An altered level of consciousness is an early sign of increased ICP. Late signs of increased ICP include tachycardia leading to bradycardia, apnea, systolic hypertension, widening pulse pressure, and posturing.

A client has been diagnosed with a basal skull fracture following a motor vehicle accident and now presents with increasing drowsiness and is febrile. The nurse knows that the client is most at risk for developing which of the following? a) Paralytic ileus b) Renal failure c) Meningitis d) Pneumonia

C

Which activity should the nurse encourage the client to avoid when there is a risk for increased intracranial pressure (ICP)? a) turning b) deep breathing c) passive range-of-motion (ROM) exercises d) coughing

D

The nurse is performing an assessment on a client with a head injury and notes that the client is assuming this posture. The nurse contacts the health care provider and reports that the client is exhibiting which posture?

Decorticate rigidity

The nurse and an adolescent are reviewing the adolescent's record of her headaches and activities surrounding them. What activity would the nurse identify as a possible trigger?

Drinking three cans of diet cola Cola contains caffeine, which is an associated trigger. Intense activity, not regular exercise, may be a trigger. Odors, such as strong perfumes, may be a trigger. Changes in sleeping patterns may be a trigger.

The nurse is caring for a client with an intracranial aneurysm who was previously alert. Which finding should be an early indication that the level of consciousness (LOC) is deteriorating?

Drowsiness Early changes in LOC relate to orientation, alertness, and verbal responsiveness. Less frequent speech, slight slurring of speech, and mild drowsiness are early signs of decreasing LOC. Ptosis of the eyelid is due to pressure on and dysfunction of cranial nerve III and does not relate to LOC.

The nurse is positioning a client who has increased intracranial pressure. Which position should the nurse avoid? 1. Head midline 2. Head turned to the side 3. Neck in neutral position 4. Head of bed elevated 30 to 45 degrees

Head turned to the side The head of a client with increased intracranial pressure should be kept in a neutral midline position. The nurse should avoid flexing or extending the client's neck or turning the head from side to side. The head of the bed should be raised to 30 to 45 degrees. Use of proper positions promotes venous drainage from the cranium to keep intracranial pressure down.

A nurse is completing discharge teaching for the client who has left-sided hemiparesis following a stroke. When investigating the client's home environment, the nurse should focus on which nursing diagnosis?

Risk for injury Because of decreased physical mobility, a client with recent left-sided hemiparesis is at risk for falls in the home setting. His ability to cope with the stroke is important, but investigating the home environment doesn't provide information about this nursing diagnosis. Diarrhea and Noncompliance aren't related to the client's home environment.

The nurse is caring for a client after hypophysectomy and notes clear nasal drainage from the client's nostril. The nurse should take which initial action? Lower the head of the bed. 2. Test the drainage for glucose. 3. Obtain a culture of the drainage. 4. Continue to observe the drainage.

Test the drainage for glucose. Rationale: After hypophysectomy, the client should be monitored for rhinorrhea, which could indicate a cerebrospinal fluid leak. If this occurs, the drainage should be collected and tested for the presence of cerebrospinal fluid. The head of the bed should not be lowered to prevent increased intracranial pressure. Clear nasal drainage would not indicate the need for a culture. Continuing to observe the drainage without taking action could result in a serious complication.

A 6-year-old has had a viral infection for the past 5 days and is having severe vomiting, confusion, and irritability, although he is now afebrile. During the assessment, the nurse should ask the parent which question? a) "Did you use any medications like aspirin for the fever?" b) "What type of fluids did your child take when he had a fever?" c) "How high did his temperature rise when he was ill?" d) "Did you give your child any acetaminophen, such as Tylenol?"

"Did you use any medications like aspirin for the fever?"

A 6-year-old has had a viral infection for the past 5 days and is having severe vomiting, confusion, and irritability, although he is now afebrile. During the assessment, the nurse should ask the parent which question?

"Did you use any medications like aspirin for the fever?" Severe and continual vomiting, changes in mental status, lethargy, and irritability are some of the signs and symptoms of Reye syndrome, which can occur as a result of ingesting aspirin or aspirin-containing products during a viral infection. Tylenol (acetaminophen) is allowed for viral infections in the school-age child. The type of fluids consumed during the illness has nothing to do with Reye syndrome. The temperature rise would be important for a much younger child because of the chance of febrile seizures, but not in this age child.

A 6-year-old has had a viral infection for the past 5 days and is now having severe vomiting, confusion, and irritability, although he is now afebrile. During the assessment, the nurse should ask the parent which of the following questions? a) "How high did his temperature rise when he was ill?" b) "Did you use any medications like aspirin for the fever?" c) "Did you give your child any acetaminophen, such as Tylenol?" d) "What type of fluids did your child take when he had a fever?"

"Did you use any medications like aspirin for the fever?" Correct Explanation: Severe and continual vomiting, changes in mental status, lethargy, and irritability are some of the signs and symptoms of Reye syndrome, which can occur as a result of ingesting aspirin or aspirin-containing products during a viral infection. Tylenol is allowed for viral infections in the school-age child. The type of fluids consumed during the illness has nothing to do with Reye syndrome. The temperature rise would be important for a much younger child because of the chance of febrile seizures, but not in this age child.

The nurse is performing an assessment of a 7-year-old child who is suspected of having episodes of absence seizures. Which assessment question to the mother will assist in providing information that will identify the symptoms associated with this type of seizure? 1. "Does twitching occur in the face and neck?" 2. "Does the muscle twitching occur on one side of the body?" 3. "Does the muscle twitching occur on both sides of the body?" 4. "Does the child have a blank expression during these episodes?"

"Does the child have a blank expression during these episodes?" Absence seizures are brief episodes of altered awareness or momentary loss of consciousness. No muscle activity occurs except eyelid fluttering or twitching. The child has a blank facial expression. These seizures last only 5 to 10 seconds, but they may occur one after another several times a day. Simple partial seizures consist of twitching of an extremity, face, or neck, or the sensation of twitching or numbness in an extremity or face or neck. Myoclonic seizures are brief random contractions of a muscle group that can occur on one or both sides of the body.

The mother of a newborn with a caput succedaneum asks the nurse how this happened to her baby. Which response by the nurse would be most appropriate?

"During delivery, your vaginal wall put pressure on the baby's head." Caput succedaneum results from pressure from the uterus or vaginal wall during a head-first delivery. The use of forceps is associated with a cephalohematoma. Caput succedaneum is not due to the baby's head becoming blocked inside the vagina. The cause of caput succedaneum is known; it is caused by pressure from the uterus or vaginal wall during a head-first delivery.

A pregnant client asks if there is any danger to the development of her fetus in the first few weeks of her pregnancy. How should the nurse respond? a) "During the first 3 to 4 weeks of pregnancy brain and spinal cord development occur and are affected by nutrition, drugs, infection, or trauma." b) "The respiratory system matures during this time so good prenatal care during the first weeks of pregnancy is very important." c) "Bones begin to harden in the first 5 to 6 weeks of pregnancy so vitamin D consumption is particularly important." d) "As long as you were taking good care of your health before becoming pregnant, your fetus should be fine during the first few weeks of pregnancy."

"During the first 3 to 4 weeks of pregnancy brain and spinal cord development occur and are affected by nutrition, drugs, infection, or trauma."

13s A nurse is instructing the spouse of a client who suffered a stroke about the use of eating devices the client will be using. During the teaching, the spouse starts to cry and states, "One minute he is laughing, and the next he's crying; I just don't understand what's wrong with him." Which statement is the best response by the nurse?

"Emotional lability is common after a stroke, and it usually improves with time." This is the most therapeutic and informative response. Often, most relatives of clients with stroke handle the physical changes better than the emotional aspects of care. The family should be prepared to expect occasional episodes of emotional lability. The client may laugh or cry easily and may be irritable and demanding or depressed and confused. The nurse can explain to the family that the client's laughter does not necessarily connote happiness, nor does crying reflect sadness, and that emotional lability usually improves with time. The remaining responses are nontherapeutic and do not address the spouse's concerns.

The nurse is reinforcing teaching with the caregivers of a child who has had a head injury and is being discharged. Which statement made by the caregiver indicates an accurate understanding of the follow-up care for their child? a) "Even if the flashlight bothers him, we will check his eyes." b) "If he vomits again, we will bring him back immediately." c) "We can give him Tylenol for a headache, but no aspirin." d) "If he falls asleep, we will wake him up every 15 minutes."

"Even if the flashlight bothers him, we will check his eyes."

The nurse is reinforcing teaching with the caregivers of a child who has had a head injury and is being discharged. Which of the following statements made by the caregiver indicate an accurate understanding of the follow-up care for their child? a) "We can give him Tylenol for a headache, but no aspirin." b) "Even if the flashlight bothers him, we will check his eyes." c) "If he vomits again, we will bring him back immediately." d) "If he falls asleep, we will wake him up every 15 minutes."

"Even if the flashlight bothers him, we will check his eyes." Explanation: The child's pupils are checked for reaction to light every four hours for 48 hours. If the child falls asleep, he or she should be awakened every one to two hours to determine that the level of consciousness has not changed. No analgesics or sedatives should be administered during this period of observation. The caregiver should observe the child for at least six hours for vomiting or a change in the child's level of consciousness. The caregiver should notify the health-care provider immediately if the child vomits more than three times, but if the child vomits once, returning to the care provider immediately is not needed.

A client is somewhat nervous about undergoing magnetic resonance imaging (MRI). Which statement by the nurse would provide the most reassurance to the client about the procedure? 1. "The MRI machine is a long, narrow, hollow tube and may make you feel somewhat claustrophobic." 2. "You will be able to eat before the procedure unless you get nauseated easily. If so, you should eat lightly." 3. "Even though you are alone in the scanner, you will be in voice communication with the technologist at all times during the procedure." 4. "It is necessary to remove any metal or metal-containing objects before having the MRI done to avoid the metal being drawn into the magnetic field."

"Even though you are alone in the scanner, you will be in voice communication with the technologist at all times during the procedure." The MRI scanner is a hollow tube that gives some clients a feeling of claustrophobia. Metal objects must be removed before the procedure so that they are not drawn into the magnetic field. The client may eat and may take all prescribed medications before the procedure. If a contrast medium is used, the client may wish to eat lightly if he or she has a tendency to become nauseated easily. The client lies supine on a padded table that moves into the imager. The client must lie still during the procedure. The imager makes tapping noises during the scanning. The client is alone in the imager, but the nurse can reassure the client that the technologist will be in voice communication with the client at all times during the procedure.

A client is somewhat nervous about having magnetic resonance imaging (MRI). Which statement by the nurse should provide reassurance to the client about the procedure?

"Even though you are alone in the scanner, you will be in voice communication with the technologist during the procedure." The MRI scanner is a hollow tube, which gives some clients a feeling of claustrophobia. Metal objects must be removed before the procedure so that they are not drawn into the magnetic field. The client may eat and take all prescribed medications before the procedure. If a contrast medium is used, the client may wish to eat lightly if there is a tendency to get nauseated easily. The client lies supine on a padded table, which moves into the imager. The client must lie still during the procedure. The imager makes tapping noises while scanning. The client is alone in the imager, but the nurse can reassure the client that the technician is in voice communication with the client at all times during the procedure.

The nurse has instructed a client with myasthenia gravis about strategies for self-management at home. The nurse determines a need for further teaching if the client makes which statement? 1. "Here's the MedicAlert bracelet I obtained." 2. "I should take my medications an hour before mealtime." 3. "Going to the beach will be a nice, relaxing form of activity." 4. "I've made arrangements to get a portable resuscitation bag and home suction equipment."

"Going to the beach will be a nice, relaxing form of activity." Most ongoing treatment for myasthenia gravis is done in outpatient settings, and the client must be aware of the lifestyle changes needed to maintain independence. The client should carry medical identification about the presence of the condition. Taking medications an hour before mealtime gives greater muscle strength for chewing and is indicated. The client should have portable suction equipment and a portable resuscitation bag available in case of respiratory distress. The client should avoid situations and other factors, including stress, infection, heat, surgery, and alcohol, that could worsen the symptoms.

The nurse has instructed the client with myasthenia gravis about ways to manage his or her own health at home. The nurse determines that the client needs further teaching if the client makes which statements?

"Going to the beach will be a nice, relaxing form of activity." Most ongoing treatment for myasthenia gravis is done in outpatient settings, and the client needs to be aware of the lifestyle changes needed to maintain independence. Taking medications 1 hour before mealtime gives greater muscle strength for chewing and is indicated. The client should have portable suction equipment and a portable resuscitation bag available in case of respiratory distress. The client should carry medical identification about the condition. The client should avoid activities that could worsen the symptoms, including stress, infection, heat (including staying out of the sun at the beach), surgery, or alcohol.

The nurse is performing an assessment on a client with a diagnosis of thrombotic stroke (brain attack). Which assessment question would elicit data specific to this type of stroke? 1. "Have you had any headaches in the past few days?" 2. "Have you recently been having difficulty with seeing at nighttime?" 3. "Have you had any sudden episodes of passing out in the past few days?" 4. "Have you had any numbness or tingling or paralysis-type feelings in any of your extremities recently?"

"Have you had any numbness or tingling or paralysis-type feelings in any of your extremities recently?" Cerebral thrombosis (thrombotic stroke) does not occur suddenly. In the few days or hours preceding the thrombotic stroke, the client may experience a transient loss of speech, hemiparesis, or paresthesias on 1 side of the body. Signs and symptoms of this type of stroke vary but may also include dizziness, cognitive changes, or seizures. Headache is rare, but some clients with stroke experience signs and symptoms similar to those of cerebral embolism or intracranial hemorrhage. The client does not complain of difficulty with night vision as part of this clinical problem. In addition, most clients do not have repeated episodes of loss of consciousness.

The nurse is performing an assessment on a client with a diagnosis of thrombotic brain attack (stroke). Which assessment question would elicit data specific to this type of stroke? "Have you had any headaches in the past few days?" 2. "Have you recently been having difficulty with seeing at nighttime?" 3. "Have you had any sudden episodes of passing out in the past few days?" 4. "Have you had any numbness or tingling or paralysis-type feelings in any of your extremities recently?"

"Have you had any numbness or tingling or paralysis-type feelings in any of your extremities recently?" Cerebral thrombosis (thrombotic stroke) does not occur suddenly. In the few days or hours preceding the thrombotic stroke, the client may experience a transient loss of speech, hemiparesis, or paresthesias on one side of the body. Signs and symptoms of this type of stroke vary but may also include dizziness, cognitive changes, or seizures. Headache is rare, but some clients with stroke experience signs and symptoms similar to those of cerebral embolism or intracranial hemorrhage. The client does not complain of difficulty with night vision as part of this clinical problem. In addition, most clients do not have repeated episodes of loss of consciousness.

The nurse is collecting data from the caregivers of a child admitted with seizures. Which of the following statements indicates the child most likely had an absence seizure? a) "He kept smacking his lips and rubbing his hands." b) "He was just staring into space and was totally unaware." c) "He usually is very coordinated, but he couldn't even walk without falling." d) "His arms had jerking movements in his legs and face."

"He was just staring into space and was totally unaware."

The nurse is collecting data from the caregivers of a child admitted with seizures. Which of the following statements indicates the child most likely had an absence seizure? a) "He kept smacking his lips and rubbing his hands." b) "His arms had jerking movements in his legs and face." c) "He was just staring into space and was totally unaware." d) "He usually is very coordinated, but he couldn't even walk without falling."

"He was just staring into space and was totally unaware." Correct Explanation: Absence seizures rarely last longer than 20 seconds. The child loses awareness and stares straight ahead but does not fall. Absence seizures rarely last longer than 20 seconds. The child loses awareness and stares straight ahead but does not fall. Myoclonic seizures are characterized by a sudden jerking of a muscle or group of muscles, often in the arms or legs, without loss of consciousness. Complex partial seizures cause nonpurposeful movements, such as hand rubbing and lip smacking. During the prodromal period of the tonic-clonic seizure, the child might have a lack of coordination.

An otherwise healthy 18-month-old child with a history of febrile seizures is in the wellchild clinic. Which statement by the father would indicate to the nurse that additional teaching should be done? a) "I have ibuprofen available in case it's needed." b) "The most likely time for a seizure is when the fever is rising." c) "My child will likely outgrow these seizures by age 5." d) "I always keep phenobarbital with me in case of a fever."

"I always keep phenobarbital with me in case of a fever." Correct Explanation: Antiepileptics, such as phenobarbital (Luminal), are administered to children with prolonged seizures or neurologic abnormalities. Ibuprofen, not phenobarbital, is given for fever. Febrile seizures usually occur after age 6 months and are unusual after age 5. Treatment is to decrease the temperature because seizures occur as the temperature rises.

The nurse is collecting data on a client with myasthenia gravis. The nurse determines that the client may be developing myasthenic crisis if the client makes which statement?

"I can't swallow very well today." Because dysphagia is a classic sign of myasthenia gravis exacerbation, observing how a client is able to ingest food is an important assessment. Timing of this medication is of paramount concern. Although options 1, 2, and 4 may require further assessment, option 4 reflects the potential of developing myasthenic crisis.

The nurse is instructing a client with Parkinson's disease about preventing falls. Which client statement reflects a need for further teaching? 1. "I can sit down to put on my pants and shoes." 2. "I try to exercise every day and rest when I'm tired." 3. "My son removed all loose rugs from my bedroom." 4. "I don't need to use my walker to get to the bathroom."

"I don't need to use my walker to get to the bathroom." The client with Parkinson's disease should be instructed regarding safety measures in the home. The client should use his or her walker as support to get to the bathroom because of bradykinesia. The client should sit down to put on pants and shoes to prevent falling. The client should exercise every day in the morning when energy levels are highest. The client should have all loose rugs in the home removed to prevent falling.

The nurse is providing instructions to a client who will be taking phenytoin. Which statement, if made by the client, would indicate an understanding of the information about this medication? 1. "I need to perform good oral hygiene, including flossing and brushing my teeth." 2. "I should try to avoid alcohol, but if I'm not able to, I can drink alcohol in moderation." 3. "I should take my medication before coming to the laboratory to have a blood level drawn." 4. "I should monitor for side effects and adjust my medication dose depending on how severe the side effects are."

"I need to perform good oral hygiene, including flossing and brushing my teeth." Phenytoin is an anticonvulsant used to treat seizure disorders. The client also should see a dentist at regularly scheduled times because gingival hyperplasia is a side effect of this medication. The client should perform good oral hygiene, including flossing and brushing the teeth. The client should avoid alcohol while taking this medication. The client should also be instructed that follow-up serum blood levels are important and that, on the day of the scheduled laboratory test, the client should avoid taking the medication before the specimen is drawn. The client should not adjust medication dosages.

The nurse has provided discharge instructions to a client with an application of a halo device. The nurse determines that the client needs further teaching if which statement is made?

"I will drive only during the daytime." The client should not drive because the device impairs the range of vision. The halo device alters balance and can cause fatigue because of its weight. The client should cleanse the skin daily under the vest or the device to protect the skin from ulceration and should use powder or lotions sparingly or not at all. The wool liner should be changed if odor becomes a problem. The client should have food cut into small pieces to facilitate chewing and use a straw for drinking. Pin care is done as instructed.

The nurse has completed discharge instructions for a client with application of a halo device. Which statement indicates that the client needs further clarification of the instructions? 1. "I will use a straw for drinking." 2. "I will drive only during the daytime." 3. "I will be careful because the device alters balance." 4. "I will wash the skin daily under the lamb's wool liner of the vest."

"I will drive only during the daytime." The halo device alters balance and can cause fatigue because of its weight. The client should cleanse the skin daily under the vest to protect the skin from ulceration and should avoid the use of powder or lotions. The liner should be changed if odor becomes a problem. The client should have food cut into small pieces to facilitate chewing and use a straw for drinking. Pin care is done as instructed. The client cannot drive at all because the device impairs the range of vision.

The nurse has given the client with Bell's palsy instructions on preserving muscle tone in the face and preventing denervation. The nurse determines that the client needs further teaching if the client makes which statements? 1. "I will perform facial exercises." 2. "I will expose my face to cold to decrease the pain." 3. "I will massage my face with a gentle upward motion." 4. "I will wrinkle my forehead, blow out my cheeks, and whistle frequently."

"I will expose my face to cold to decrease the pain." Exposure to cold or drafts is avoided in Bell's palsy because it can cause discomfort. Prevention of muscle atrophy with Bell's palsy is accomplished with facial massage, facial exercises, and electrical nerve stimulation. Local application of heat to the face may improve blood flow and provide comfort.

The nurse reinforces home care instructions to the postcraniotomy client. Which statement by the client indicates the need for further teaching?

"I will not hear sounds clearly unless they are loud." Seizures are a complication that can occur for up to 1 year after surgery. For this reason, the client must diligently take anticonvulsant medications. The client and family are encouraged to keep track of doses administered. The family should learn seizure precautions and accompany the client while ambulating if dizziness or seizures tend to occur. The suture line is kept dry until sutures are removed to prevent infection. The postcraniotomy client can hear sounds, is typically sensitive to loud noises, and can find them irritating (e.g., loud television). Awareness control of environmental noise by others is helpful to this client.

The home care nurse is preparing to visit a client with a diagnosis of trigeminal neuralgia (tic douloureux). When performing the assessment, the nurse should plan to ask the client which question to elicit the most specific information regarding this disorder? 1. "Do you have any visual problems?" 2. "Are you having any problems hearing?" 3. "Do you have any tingling in the face region?" 4. "Is the pain experienced a stabbing type of pain?"

"Is the pain experienced a stabbing type of pain?" Trigeminal neuralgia is characterized by spasms of pain that start suddenly and last for seconds to minutes. The pain often is characterized as stabbing or as similar to an electric shock. It is accompanied by spasms of facial muscles that cause twitching of parts of the face or mouth, or closure of the eye. The remaining options do not elicit data specifically related to this disorder.

The nurse is educating parents of a male infant with Chiari type II malformation. Which statement about their child's condition is most accurate?

"Take your time feeding your baby." One of the problems associated with Chiari type II malformation is poor gag and swallowing reflexes, so the infant must be fed slowly. There is a great risk of aspiration, requiring that the child be placed in an upright position after feeding. The goal of surgery is to prevent further symptoms, rather than to relieve existing ones. Infrequent urination is a problem associated with type I malformations

The nurse is educating parents of a male infant with Chiari type II malformation. Which of the following statements about their child's condition is most accurate? a) "You won't need to change diapers often." b) "Lay him down after feeding." c) "Take your time feeding your baby." d) "You'll see a big difference after the surgery."

"Take your time feeding your baby." Explanation: One of the problems associated with Chiari type II malformation is poor gag and swallowing reflexes, so the infant must be fed slowly. There is a great risk of aspiration, requiring that the child be placed in an upright position after feeding. The goal of surgery is to prevent further symptoms, rather than to relieve existing ones. Infrequent urination is a problem associated with type I malformations.

A client with recent-onset Bell's palsy is upset and crying about the change in facial appearance. The nurse plans to support the client emotionally by making which statement to the client? 1. "This is not a stroke, and many clients recover in 3 to 5 weeks." 2. "This is caused by a small tumor, which can be removed easily." 3. "This is similar to a stroke, but all symptoms will reverse without treatment." 4. "This is a temporary problem, with treatment similar to that for migraine headaches."

"This is not a stroke, and many clients recover in 3 to 5 weeks." Clients with Bell's palsy should be reassured that they have not experienced stroke a (brain attack) and that symptoms often disappear spontaneously in 3 to 5 weeks. The client is given supportive treatment for symptoms. Bell's palsy usually is not caused by a tumor, and the treatment is not similar to that for migraine headaches.

Seven-year old Isabelle has been complaining of headache, coughing, and an aching chest. The care provider makes a diagnosis of a viral infection. The child's mother tells the nurse that when Isabelle first complained of a headache, the child's father gave her half of an adult aspirin. The mother has heard of Reye syndrome and asks the nurse if her child could get this. Which of the following statements would be best for the nurse to say to this mother? a) "This is a serious problem. Aspirin is likely to cause Reye syndrome, and Isabelle should be admitted to the hospital for observation as a precaution." b) "This might or might not be a problem. Watch Isabelle for signs of lethargy, unusual irritability, confusion, or vomiting. If you notice any of these, bring her to the emergency room immediately so she can be checked for Reye syndrome." c) "This is unlikely to be a problem. Half an aspirin is not enough to cause harm. Reye syndrome generally only develops from prolonged use of aspirin in connection with a virus." d) "This might or might not be a problem. Watch Isabelle for signs of nasal discharge, sneezing, itching of the nose, or dark circles under the eyes. If you notice any of these, bring her to the emergency room immediately so she can be checked for Reye syndrome."

"This might or might not be a problem. Watch Isabelle for signs of lethargy, unusual irritability, confusion, or vomiting. If you notice any of these, bring her to the emergency room immediately so she can be checked for Reye syndrome."

Seven-year-old Isabelle has been complaining of headache, coughing, and an aching chest. The care provider makes a diagnosis of a viral infection. The child's mother tells the nurse that when Isabelle first said she had a headache, the child's father gave her half of an adult aspirin. The mother has heard of Reye syndrome and asks the nurse if her child could get this. Which statement would be best for the nurse to say to this mother?

"This might or might not be a problem. Watch Isabelle for signs of lethargy, unusual irritability, confusion, or vomiting. If you notice any of these, bring her to the emergency room immediately so she can be checked for Reye syndrome." Reye syndrome usually occurs after a viral illness, particularly after an upper respiratory infection or varicella (chickenpox). Administration of aspirin during the viral illness has been implicated as a contributing factor. As a result, the American Academy of Pediatrics recommends that aspirin or aspirin compounds not be given to children with viral infections. The symptoms appear within three to five days after the initial illness: The child is recuperating unremarkably when symptoms of severe vomiting, irritability, lethargy, and confusion occur. Immediate intervention is needed to prevent serious insult to the brain including respiratory arrest.

Seven-year old Isabelle has been complaining of headache, coughing, and an aching chest. The care provider makes a diagnosis of a viral infection. The child's mother tells the nurse that when Isabelle first complained of a headache, the child's father gave her half of an adult aspirin. The mother has heard of Reye syndrome and asks the nurse if her child could get this. Which of the following statements would be best for the nurse to say to this mother? a) "This might or might not be a problem. Watch Isabelle for signs of nasal discharge, sneezing, itching of the nose, or dark circles under the eyes. If you notice any of these, bring her to the emergency room immediately so she can be checked for Reye syndrome." b) "This might or might not be a problem. Watch Isabelle for signs of lethargy, unusual irritability, confusion, or vomiting. If you notice any of these, bring her to the emergency room immediately so she can be checked for Reye syndrome." c) "This is a serious problem. Aspirin is likely to cause Reye syndrome, and Isabelle should be admitted to the hospital for observation as a precaution." d) "This is unlikely to be a problem. Half an aspirin is not enough to cause harm. Reye syndrome generally only develops from prolonged use of aspirin in connection with a virus."

"This might or might not be a problem. Watch Isabelle for signs of lethargy, unusual irritability, confusion, or vomiting. If you notice any of these, bring her to the emergency room immediately so she can be checked for Reye syndrome." Correct Explanation: Reye syndrome usually occurs after a viral illness, particularly after an upper respiratory infection or varicella (chickenpox). Administration of aspirin during the viral illness has been implicated as a contributing factor. As a result, the American Academy of Pediatrics recommends that aspirin or aspirin compounds not be given to children with viral infections. The symptoms appear within three to five days after the initial illness: The child is recuperating unremarkably when symptoms of severe vomiting, irritability, lethargy, and confusion occur. Immediate intervention is needed to prevent serious insult to the brain including respiratory arrest.

The nurse is educating the family of a 7-year-old epilepsy patient about care and safety for this child. Which of the following comments will be most valuable in helping the parent and the child cope? a) "You'll always need a monitor in his room." b) "Use this information to teach family and friends." c) "If he is out of bed, the helmet's on the head." d) "Bike riding and swimming are just too dangerous."

"Use this information to teach family and friends."

The nurse is educating the family of a 7-year-old with epilepsy about care and safety for this child. What comment will be most valuable in helping the parent and the child cope?

"Use this information to teach family and friends." Families need and want information they can share with relatives, childcare providers, and teachers. Wearing a helmet and having a monitor in the room are precautions that may need to be modified as the child matures. The boy may be able to bike ride and swim with proper precautions.

The nurse is admitting a client to the hospital emergency department from a nursing home. The client is unconscious with an apparent frontal head injury. A medical diagnosis of epidural hematoma is suspected. Which question is of the highest priority for the emergency department nurse to ask of the transferring nurse at the nursing home? 1. "When did the injury occur?" 2. "Was the client awake and talking right after the injury?" 3. "What medications has the client received since the fall?" 4. "What was the client's level of consciousness before the injury?"

"Was the client awake and talking right after the injury?" Epidural hematomas frequently are characterized by a "lucid interval" that lasts for minutes to hours, during which the client is awake and talking. After this lucid interval, signs and symptoms progress rapidly, with potentially catastrophic intracranial pressure increase. Epidural hematomas are medical emergencies. It is important for the nurse to assist in the differentiation between epidural hematoma and other types of head injuries.

The nurse has instructed the family of a client with stroke (brain attack) who has homonymous hemianopsia about measures to help the client overcome the deficit. Which statement suggests that the family understands the measures to use when caring for the client? 1. "We need to discourage him from wearing eyeglasses." 2. "We need to place objects in his impaired field of vision." 3. "We need to approach him from the impaired field of vision." 4. "We need to remind him to turn his head to scan the lost visual field."

"We need to remind him to turn his head to scan the lost visual field." Homonymous hemianopsia is loss of half of the visual field. The client with homonymous hemianopsia should have objects placed in the intact field of vision, and the nurse also should approach the client from the intact side. The nurse instructs the client to scan the environment to overcome the visual deficit and does client teaching from within the intact field of vision. The nurse encourages the use of personal eyeglasses, if they are available.

The emergency room nurse is taking a history of a 1-year-old child whose parent said that she had a "fit" at home. Which of the following inquiries would suggest what has happened? a) "Was the child unconscious?" b) "Were there any jerky movements?" c) "What happened just before the seizures?" d) "How did you treat the child afterwards?"

"What happened just before the seizures?" Explanation: Asking what happened just before the seizure will suggest whether the episode was a seizure or a breath-holding event, which is frequently precipitated by an expression of anger or frustration. Cyanotic breath holding can be accompanied by clinic movements, as can seizures. Both types of events render the child unconscious. One would expect concerned, caring treatment from the parents regardless of the cause.

The nurse is providing education to the parents of a 3-year-old girl with hydrocephalus who has just had an external ventricular drainage system placed. Which question is best to begin the teaching session? a) "Do you understand why you clamp the drain before she sits up?" b) "What questions or concerns do you have about this device?" c) "What do you know about her autoregulation mechanism failing?" d) "Why do you always keep her head raised 30 degrees?"

"What questions or concerns do you have about this device?"

A child is home with the caregivers following a treatment for a head injury. If the child makes which of the following statements, the caregiver should contact the care provider. a) "It will be nice when you will let me take a long nap. I am sleepy." b) "I am glad that my headache is getting better." c) "My stomach is upset. I feel like I might throw up." d) "You look funny. Well, both of you do. I see two of you."

"You look funny. Well, both of you do. I see two of you." Correct Explanation: The caregiver should notify the health-care provider immediately if the child vomits more than three times, has pupillary changes, has double or blurred vision, has a change in level of consciousness, acts strange or confused, has trouble walking, or has a headache that becomes more severe or wakes him or her from sleep. These instructions should be provided in written form to the caregiver. Just feeling naueauted is not a reason to notify the provider.

A client with blunt head injury is admitted for observation, including hourly neurologic checks. At 01:00 AM, the client reports a headache; the nurse obtains a normal neurologic assessment and administers the PRN acetaminophen. At 02:00 AM, the client appears to be sleeping. Which action should the nurse take? 1. Arouse the client and ask what the current month is 2. Document "relief apparently obtained" and recheck at 03:00 AM 3. Let the client sleep but verify respiratory rate 4. Wake the client up and check for paresthesia

1

An 81-year-old client is admitted to a rehabilitation facility 3 days after total hip replacement. The next morning, the unlicensed assistive personnel (UAP) takes the client's vital signs, but when the UAP goes back to assist the client with a shower, the client curses at and tries to hit the UAP. Which of the following is the most appropriate response by the registered nurse? Click on the exhibit button for additional information. 1. "I need to assess the client." 2. "It sounds like the client is not satisfied with the care provided. I'll see if we can make the client more comfortable." 3. "Just leave the client alone now and try again later." 4. "The client probably has dementia and is under a lot of stress with the change of environment."

1

Assessment of a client with a history of stroke reveals that the client understands and follows commands but answers questions with incorrect word choices. The nurse documents the presence of which communication deficit? 1. Aphasia 2. Apraxia 3. Dysarthria 4. Dysphagia

1

The client has increased intracranial pressure with cerebral edema, and mannitol is administered. Which assessment should the nurse make to evaluate if a complication from the mannitol is occurring? 1. Auscultate breath sounds to assess for crackles 2. Monitor for >50 mL/hr urine output 3. Monitor Glasgow Coma Scale increasing from 8/15 to 9/15 4. Press over the tibia to assess for pitting edema

1

The emergency department triage nurse is assessing 4 pediatric clients. Which client is a priority for further diagnostic workup and definitive care? 1. 1-year-old with ventriculoperitoneal shunt who has "lethargy" and pulse of 78/min 2. 3-year-old with history of meningocele who has unilateral ear pain and urinary incontinence 3. 6-year-old with muscular dystrophy who has "flu-like" symptoms and temperature of 100.4 F (38 C) 4. 8-year-old with history of cerebral palsy who has foot injury and spastic clonus

1

The following 4 clients are brought to the emergency department triage nurse. The client with which of these signs should be a priority to be seen for immediate care? 1. A 2-year-old has sclera visible above the iris (sunset eyes) 2. A 3-year-old has a single transverse crease across the entire palm of the hand 3. A 6-month-old breastfed client had 8 wet diapers in the last 24 hours 4. A 9-month-old client's toes fan out and the big toe dorsiflexes when the foot sole is stroked

1

The home health nurse prepares to give benztropine to a 70-year-old client with Parkinson disease. Which client statement is most concerning and would warrant health care provider notification? 1. "I am going for repeat testing to confirm glaucoma." 2. "I am not able to exercise as much as I used to." 3. "I started taking esomeprazole for heartburn." 4. "My bowel movements are not regular."

1

The home health nurse reviews the serum laboratory test results for a client with seizures. The phenytoin level is 27 mcg/mL. The client makes which statement that may indicate the presence of dose-related drug toxicity and prompt the nurse to notify the health care provider? 1. "I am feeling unsteady when I walk." 2. "I am getting up to urinate about 4 times during the night." 3. "I have a metallic taste in my mouth when I eat." 4. "My gums are getting so puffy and red."

1

The home health nurse teaches an elderly client with dysphagia some strategies to help limit repeated hospitalizations for aspiration pneumonia. Which statement indicates that the client needs further teaching? 1. "I have to remember to raise my chin slightly upward when I swallow." 2. "I have to remember to swallow 2 times before taking another bite of food." 3. "I should avoid taking over-the-counter cold medications when I'm sick." 4. "I should sit upright for at least 30-40 minutes after I eat."

1

The nurse completes a neurological examination on a client who has suffered a stroke to determine if damage has occurred to any of the cranial nerves. The nurse understands that damage has occurred to cranial nerve IX based on which assessment finding? 1. A tongue blade is used to touch the client's pharynx; gag reflex is absent 2. Only one side of the mouth moves when the client is asked to smile and frown 3. The absence of light touch and pain sensation on the left side of the client's face 4. When the client shrugs against resistance, the left shoulder is weaker than the right

1

The nurse is caring for a client with absence seizures. The unlicensed assistive personnel (UAP) asks if the client will "shake and jerk" when having a seizure. Which response from the nurse is the most helpful? 1. "No, absence seizures can look like daydreaming or staring off into space." 2. "No, you are wrong. Don't worry about that." 3. "Yes, so please let me know if you see the client do that." 4. "You don't have to monitor the client for seizures."

1

The nurse receives the change of shift report for assigned clients at 7 AM. Which client should the nurse assess first? 1. Client with change in level of consciousness who fell in the nursing home 2. Client with chronic headaches who is scheduled for an MRI at 9 AM 3. Client with chronic obstructive pulmonary disease (COPD) and pulse oximeter reading of 90% 4. Client with heart failure and 3+ pitting edema of the lower extremities

1

The nurse is caring for a newborn infant with spina bifida (myelomeningocele) who is scheduled for surgical closure of the sac. In the preoperative period, which is the priority problem? 1. Infection 2. Choking 3. Inability to tolerate stimulation 4. Delayed growth and development

1 A myelomeningocele is a type of spina bifida that results from failure of the neural tube to close during embryonic development. With a myelomeningocele, protrusion of the meninges, cerebrospinal fluid, nerve roots, and a portion of the spinal cord occurs. The newborn with spina bifida is at risk for infection before the closure of the sac, which is done soon after birth. Initial care of the newborn with myelomeningocele involves prevention of infection. A sterile normal saline dressing is placed over the sac to maintain moisture of the sac and its contents and to prevent tearing or breakdown of the skin integrity at the site. Any opening in the sac greatly increases the risk of infection of the central nervous system. Choking and inability to tolerate stimulation are not priority problems with this defect. Delayed growth and development is a problem for the infant with myelomeningocele, but preventing infection has priority in the preoperative period.

The nurse is caring for a child who sustained a head injury after falling from a tree. On assessment of the child, the nurse notes the presence of a watery discharge from the child's nose. The nurse should immediately test the discharge for the presence of which substance? 1. Protein 2. Glucose 3. Neutrophils 4. White blood cells

1 After a head injury, bleeding from the nose or ears necessitates further evaluation. A watery discharge from the nose (rhinorrhea) that tests positive for glucose is likely to be cerebrospinal fluid (CSF) leaking from a skull fracture. On noting watery discharge from the child's nose, the nurse should test the drainage for glucose using an agency approved reagent strip. If the results are positive, the nurse will contact the health care provider. The items in options 1, 3, and 4 are not normally found in mucus.

The nurse has reinforced teaching for a school-age child who was given a brace to wear for the treatment of scoliosis. The nurse determines that the child needs further teaching if the child makes which statement? 1. "This brace will correct my curve." 2. "I will wear my brace under my clothes." 3. "I may not need surgery if I wear my brace." 4. "I will do back exercises at least five times a week."

1 Bracing can halt the progression of most curvatures, but it is not curative for scoliosis. The statements in the remaining options represent correct understanding on the part of the child.

A school-age child with Down syndrome is brought to the ambulatory care center by the mother. The child has bruising all over the body. To work most effectively with this child, the nurse first addresses which complication associated with Down syndrome? 1. Children with Down syndrome are more likely to develop acute leukemia than the average child. 2. Children with Down syndrome fall down easily as a result of hyperflexibility and muscle weakness. 3. Children with Down syndrome are at risk for physical abuse because of their low intellectual functioning. 4. Children with Down syndrome scratch themselves a lot because of dry, cracked, and frequently fissuring skin.

1 Children with Down syndrome have an increased risk for developing leukemia compared with the average child. The other statements also could be true, but the nurse should first gather baseline data to determine the cause of the bruising before making other assumptions.

The RN is supervising a senior nursing student who is caring for a client with a right hemisphere stroke. Which action by the student nurse requires that the RN intervene? 1. Instructing the client to sit up straight and the client responds with a puzzled expression 2. Moving the client's food tray to the right side of his over-bed table 3. Assisting the client with passive range-of-motion (ROM) exercises 4. Combing the hair on the left side of the client's head when the client always combs his hair on the right side

1 Clients with right cerebral hemisphere stroke often manifest neglect syndrome. They lean to the left and, when asked, respond that they believe they are sitting up straight. They often neglect the left side of their bodies and ignore food on the left side of their food trays. The nurse needs to remind the student of this phenomenon and discuss the appropriate interventions.

A child who sustained a fractured ankle has a short leg cast applied, and the nurse provides home care instructions to the mother. The mother returns to the emergency department 16 hours later because the child is complaining of severe pain. The nurse notes that the child's toes are cool, pale, and puffy and that the child is agitated and crying loudly. The mother states, "I gave her the pain medication you sent with us just like you told us, and I have kept her foot up on two pillows since we left, except when she gets up to go to the bathroom. I don't understand why she hurts so much. Do something!" What is the most likely clinical situation that occurred? 1. Compartment syndrome 2. Inadequate pain medication 3. Skin breakdown around the cast edges 4. Noncompliance with home care instructions

1 Compartment syndrome occurs as a result of pressure buildup within a tissue compartment bound by anatomical structures such as fascia. With a fracture, this pressure increase may occur as a result of the intense inflammatory response or severe bleeding caused by the bone injury, even when diligent nursing care has been provided. Pain disproportionate to the injury despite analgesic administration is the classic sign of compartment syndrome. The nurse should constantly assess for this complication and should instruct the caregiver about the manifestations associated with this complication.

A 1-month-old infant is seen in a clinic and is diagnosed with developmental dysplasia of the hip. On assessment, the nurse understands that which finding should be noted in this condition? 1. Limited range of motion in the affected hip 2. An apparent lengthened femur on the affected side 3. Asymmetrical adduction of the affected hip when the infant is placed supine with the knees and hips flexed 4. Symmetry of the gluteal skinfolds when the infant is placed prone and the legs are extended against the examining table

1 In developmental dysplasia of the hip, the head of the femur is seated improperly in the acetabulum or hip socket of the pelvis. Asymmetrical and restricted abduction of the affected hip, when the child is placed supine with the knees and hips flexed, would be an assessment finding in developmental dysplasia of the hip in infants beyond the newborn period. Other findings include an apparent short femur on the affected side, asymmetry of the gluteal skinfolds, and limited range of motion in the affected extremity.

The nurse creates a plan of care for a child with Reye's syndrome. Which priority intervention should the nurse include in the plan of care? 1. Monitor for signs of increased intracranial pressure. 2. Immediately check the presence of protein in the urine. 3. Reassure the parents hyperglycemia is a common symptom. 4. Teach the parents signs and symptoms of a bacterial infection

1 Intracranial pressure and encephalopathy are major symptoms of Reye's syndrome. Protein is not present in the urine. Reye's syndrome is related to a history of viral infections, and hypoglycemia is a symptom of this disease.

The nurse is in charge of developing a standard plan of care for an Alzheimer disease care facility and is responsible for assigning and supervising resident care given by LPNs/LVNs and delegating and supervising care given by unlicensed assistive personnel (UAP). Which activity is best to assign to the LPN/LVN team leaders? 1. Checking for improvement in resident memory after medication therapy is initiated 2. Using the Mini-Mental State Examination to assess residents every 6 months 3. Assisting residents in using the toilet every 2 hours to decrease risk for urinary incontinence 4. Developing individualized activity plans after consulting with residents and family

1 LPN/LVN education and team leader responsibilities include checking for the therapeutic and adverse effects of medications. Changes in the residents' memory would be communicated to the RN supervisor, who is responsible for overseeing the plan of care for each resident. Assessing for changes in score on the Mini-Mental State Examination and developing the plan of care are RN responsibilities. Assisting residents with personal care and hygiene would be delegated to UAPs working at the long-term care facility.

A child is admitted to the hospital with a diagnosis of acute bacterial meningitis. In reviewing the health care provider's prescriptions, which would the nurse question as appropriate for a child with this diagnosis? 1. Administer an oral antibiotic. 2. Maintain strict intake and output. 3. Draw blood for a culture and sensitivity. 4. Place the child on droplet precautions in a private room.

1 Medication to treat acute bacterial meningitis is administered intravenously, not orally. A culture and sensitivity is done to determine if the diagnosis is bacterial or viral. Until meningitis is ruled out, the child is placed in isolation on droplet precautions because the disease is spread by airborne means. Strict intake and output should be maintained.

The nurse is mentoring a student nurse in the intensive care unit while caring for a client with meningococcal meningitis. Which action by the student requires that the nurse intervene mostrapidly? 1. Entering the room without putting on a protective mask and gown 2. Instructing the family that visits are restricted to 10 minutes 3. Giving the client a warm blanket when he says he feels cold 4. Checking the client's pupil response to light every 30 minutes

1 Meningococcal meningitis is spread through contact with respiratory secretions, so use of a mask and gown is required to prevent transmission of the infection to staff members or other clients. The other actions may or may not be appropriate. The presence of a family member at the bedside may decrease client confusion and agitation. Clients with hyperthermia frequently report feeling chilled, but warming the client is not an appropriate intervention. Checking the pupils' response to light is appropriate but is not needed every 30 minutes and is uncomfortable for a client with photophobia.

The nurse is monitoring a 7-year-old child who sustained a head injury in a motor vehicle crash for signs of increased intracranial pressure (ICP). The nurse should assess the child frequently for which early sign of increased ICP? 1. Nausea 2. Papilledema 3. Decerebrate posturing 4. Alterations in pupil size

1 Nausea is an early sign of increased ICP. Late signs of increased ICP include a significant decrease in level of consciousness, Cushing's triad (increased systolic blood pressure and widened pulse pressure, bradycardia, and irregular respirations), and fixed and dilated pupils. Other late signs include decreased motor response to command, decreased sensory response to painful stimuli, posturing, Cheyne-Stokes respirations, and papilledema.

The nurse is reviewing a chart for a child with a head injury. The nurse notes that the level of consciousness has been documented as obtunded. Which finding should the nurse expect to note on assessment of the child? 1. Not easily arousable and limited interaction 2. Loss of the ability to think clearly and rapidly 3. Loss of the ability to recognize place or person 4. Awake, alert, interacting with the environment

1 Obtunded indicates that the child sleeps unless aroused and once aroused has limited interaction with the environment. Confusion indicates that the ability to think clearly and rapidly is lost. Disorientation indicates that the ability to recognize place or person is lost. Full consciousness indicates that the child is alert, awake, oriented, and interacts with the environment.

The nurse is performing an admission assessment on a child with a seizure disorder. The nurse is interviewing the child's parents to determine their adjustment to caring for their child, who has a chronic illness. Which statement, if made by the parents, would indicate a need for further teaching? 1. "Our child sleeps in our bedroom at night." 2. "We worry about injuries when our child has a seizure." 3. "Our child is involved in a swim program with neighbors and friends." 4. "Our babysitter just completed cardiopulmonary resuscitation training."

1 Parents are especially concerned about seizures that might go undetected during the night. The nurse needs to decrease parental overprotection and should suggest the use of a baby monitor at night. Involvement in a swim program and knowing CPR identify parental understanding of the disorder. Worrying about injuries when a child has a seizure is a common concern. The parents need to be reminded that as the child grows, they cannot always observe their child, but their knowledge of seizure activity and care is appropriate to minimize complications.

A client who had a stroke needs to be fed. What instruction should the nurse give to the unlicensed assistive personnel (UAP) who will feed the client? 1. Position the client sitting up in bed before he or she is fed. 2. Check the client's gag and swallowing reflexes. 3. Feed the client quickly because there are three more clients to feed. 4. Suction the client's secretions between bites of food.

1 Positioning the client in a sitting position decreases the risk of aspiration. The UAP is not trained to assess gag or swallowing reflexes. The client should not be rushed during feeding. A client who needs suctioning performed between bites of food is not handling secretions and is at risk for aspiration. Such a client should be assessed further before feeding.

A client with a diagnosis of trigeminal neuralgia is started on a regimen of carbamazepine (Tegretol). The nurse provides instructions to the client about the side/adverse effects of the medication. Which client statement indicates an understanding of the side/adverse effects of the medication? 1. "I will report a fever or sore throat to my health care provider." 2. "I must brush my teeth frequently to avoid damage to my gums." 3. "If I notice ringing in my ears that doesn't stop, I'll seek medical attention." 4. "If I notice a pink color to my urine, I will stop the medication and call my health care provider (HCP)."

1 Rationale: Agranulocytosis is adverse effect of carbamazepine and places the client at risk for infection. If a fever or a sore throat develops, the HCP should be notified. Gum damage, ringing in the ears, and pink-colored urine are not effects associated with this medication.

Carbamazepine (Tegretol) has been prescribed for a client, and the client asks the nurse about the action of the medication. The nurse's response should incorporate which information as a primaryeffect of this medication? 1. Anticonvulsant effect 2. Prevention of cellular division 3. Decrease in intraocular pressure 4. Interference with DNA production

1 Rationale: Carbamazepine is classified as an iminostilbene derivative and is used as an anticonvulsant, antineuralgic, antimanic, and antipsychotic. Options 2, 3, and 4 are not actions or effects of this medication.

The nurse has given instructions for taking codeine sulfate to a client with a severe headache. Which statement by the client indicates the teaching has been effective? 1. "I should increase fluid intake." 2. "I should maintain a low-fiber diet." 3. "I should avoid all exercise to prevent lightheadedness." 4. "I should avoid the use of stool softeners to prevent diarrhea."

1 Rationale: Codeine sulfate can cause constipation, so the client is instructed to increase fluid intake to prevent this occurrence. A high-fiber diet and stool softeners may be prescribed to prevent constipation. All exercise is not avoided.

A client experiencing spasticity as a result of spinal cord injury has a new prescription for dantrolene (Dantrium). Before administering the first dose, the nurse checks to see if which baseline study has been done? 1. Liver function studies 2. Renal function studies 3. Otoscopic examination 4. Blood glucose measurements

1 Rationale: Dantrolene is a skeletal muscle relaxant and can cause liver damage; therefore the nurse should monitor the results of liver function studies. They should be done before therapy starts and periodically throughout therapy. Dantrolene is discontinued if no relief of spasticity is achieved in 6 weeks. The incorrect options are not specifically related to the administration of this medication.

The nurse has the following prescription for a postcraniotomy client, "dexamethasone (Decadron) 4 mg by the intravenous (IV) route now." How does the nurse administer the medication? 1. IV push over 1 minute 2. IV push over 4 minutes 3. IV piggyback in 50 mL of normal saline over 10 minutes 4. IV piggyback in 50 mL of normal saline over 30 minutes

1 Rationale: Dexamethasone (Decadron) is an adrenocorticosteroid administered after craniotomy to control cerebral edema. It is given by IV push, and single doses are administered over 1 minute. Dexamethasone IV doses are changed to the oral route after 24 to 72 hours and are tapered until discontinued. Additionally, IV fluids are administered cautiously after craniotomy to prevent increased cerebral edema.

A client is scheduled to begin medication therapy with valproic acid (Depakene). The nurse looks for the results of which laboratory test(s) before administering the first dose? 1. Liver function tests 2. Renal function tests 3. Pulmonary function test 4. Pancreatic enzyme studies

1 Rationale: Gastrointestinal effects from valproic acid (Depakene) are common and typically mild, but hepatotoxicity, although rare, is serious. To minimize the risk of fatal liver injury, liver function is evaluated before initiation of treatment and periodically thereafter. The other options are unrelated to the use of this medication.

A client with status epilepticus has been prescribed phenytoin (Dilantin) to be given by the intravenous (IV) route. The nurse administering the medication is careful not to exceed which recommended infusion rate? 1. 50 mg/min 2. 60 mg/min 3. 100 mg/min 4. 750 mg/min

1 Rationale: IV administration of phenytoin is performed slowly (no faster than 50 mg/min) because rapid administration can cause cardiovascular collapse. It should not be added to any existing IV infusion because this is likely to produce a precipitate in the solution. Solutions are highly alkaline and can cause local venous irritation. Test-Taking Strategy: Focus on the subject, infusion rate of phenytoin. Noting that the medication is to be administered by the IV route, select the choice that indicates the slowest route of administration. This will direct you to the correct option.

A client has a medication prescription for phenytoin (Dilantin) to be administered by the intravenous route. After drawing up the medication, the nurse notes the presence of precipitate in the syringe. Which action should the nurse take? 1. Discard the syringe and begin again. 2. Add sterile water to dissolve the precipitate. 3. Draw up an additional 1 mL of normal saline into the syringe. 4. Chart the medication as "not given," and write a note in the medical record.

1 Rationale: If the injectable solution is not clear or if precipitate is present, the medication should not be used and should be discarded. The nurse may have to call the pharmacy department to obtain another vial of the medication. Options 2, 3, and 4 are inaccurate actions.

Phenytoin (Dilantin), 100 mg to be given orally three times daily, has been prescribed to a client for seizure control. The home health nurse visits the client and provides instructions regarding the medication. Which statement, if made by the client, would indicate an understanding of the instructions? 1. "I will use a soft toothbrush to brush my teeth." 2. "It's okay to break the capsules to make it easier for me to swallow them." 3. "If I forget to take my medication, I can wait until the next dose and eliminate that dose." 4. "If my throat becomes sore, it's a normal effect of the medication, and it's nothing to be concerned about."

1 Rationale: Phenytoin is an anticonvulsant. Gingival hyperplasia, bleeding, swelling, and tenderness of the gums can occur with the use of this medication. The client needs to be taught good oral hygiene, gum massage, and the need for regular dentist visits. Capsules should not be chewed or broken. The client should not skip medication doses because inadequate blood levels could precipitate a seizure. The client needs to be instructed to report a sore throat, fever, glandular swelling, or any skin reaction because such findings may indicate hematological toxicity.

The nurse has given medication instructions to a client beginning anticonvulsant therapy with carbamazepine (Tegretol). The nurse determines that the client understands the use of the medication if he or she makes which statement? 1. Use sunscreen when outdoors. 2. Drive a car as long as it is not at night. 3. Keep tissues handy because of excess salivation. 4. Discontinue the medication if fever or sore throat occurs.

1 Rationale: The client should use protective clothing and sunscreen to avoid photosensitivity reactions. Carbamazepine acts by depressing synaptic transmission in the central nervous system (CNS). Because of this, the client should avoid driving at any time or doing other activities that require mental alertness until the effect of the medication on the client is known. The medication may cause dry mouth, and he or she should be instructed to provide good oral hygiene and use sugarless candy or gum as needed. The medication should not be abruptly discontinued because this could lead to return of seizures or status epilepticus. Fever and sore throat (leukopenia) should be reported to the health care provider.

A client has a prescription for valproic acid (Depakene) 250 mg once daily. To maximize the client's safety, the nurse should plan to schedule the medication at what time? 1. At bedtime 2. With lunch 3. After breakfast 4. Before breakfast

1 Rationale: Valproic acid is an anticonvulsant that causes central nervous system (CNS) depression. For this reason, the side effects of the medication include sedation, dizziness, ataxia, and confusion. When the client is taking this medication as a single daily dose, administering it at bedtime negates the risk of injury from sedation and enhances client safety. The medication should be taken at the same time each day.

A nursing student is assisting a school nurse in performing scoliosis screening on the children in the school. The nurse assesses the student's preparation for conducting the screening. The nurse determines that the student demonstrates understanding of the disorder when the student states that scoliosis is characterized by which finding? 1. Abnormal lateral curvature of the spine 2. Abnormal anterior curvature of the lumbar spine 3. Excessive posterior curvature of the thoracic spine 4. Abnormal curvature of the spine caused by inflammation

1 Scoliosis is defined as an abnormal lateral curvature in any area of the spine. The region of the spine most commonly affected is the right thoracic area, where it results in rib prominence. Option 2 describes lordosis, which usually is exaggerated during pregnancy, in obesity, or in persons with large tumors. Option 3 describes kyphosis, which also is known as humpback. Scoliosis does not occur as a sequela of inflammation.

A child with cerebral palsy is in a management program to achieve maximum potential for locomotion, self-care, and socialization in school. The nurse works with the child to meet these goals by performing which action? 1. Placing the child on a wheeled scooter board 2. Removing ankle-foot orthoses and braces once the child arrives at school 3. Keeping the child in a special education classroom with other children with similar disabilities 4. Placing the child in the supine position with a 30-degree elevation of the head of the bed to facilitate feeding

1 The correct option provides the child with maximum potential in locomotion, self-care, and socialization. While lying on the abdomen, the child can move around independently anywhere the child wants to go and can interact with others as desired. Orthoses need to be used all the time to aid locomotion. Children with cerebral palsy (CP) need to be mainstreamed as much as cognitive ability permits to provide for maximum socialization and normalization. Not all children with CP are intellectually challenged. Just as children without CP sit up and use assistive devices when eating, so should children with CP.

The nurse is caring for a client with a glioblastoma who is receiving dexamethasone 4 mg IV push every 6 hours to relieve symptoms of right arm weakness and headache. Which assessment information concerns the nurse the most? 1. The client no longer recognizes family members. 2. The blood glucose level is 234 mg/dL (13 mmol/L). 3. The client reports a continuing headache. 4. The daily weight has increased 2.2 lb (1 kg).

1 The inability to recognize family members is a new neurologic deficit for this client and indicates a possible increase in intracranial pressure (ICP). This change should be communicated to the health care provider immediately so that treatment can be initiated. The continuing headache also indicates that the ICP may be elevated but is not a new problem. The glucose elevation and weight gain are common adverse effects of dexamethasone that may require treatment but are not emergencies.

The nurse is reinforcing instructions to the mother of a child who has a plaster cast applied to the left arm. Which statement by the mother indicates a need for further teaching? 1. "I will have to use a heat lamp to help the cast dry." 2. "I need to cover the cast with plastic during baths or showers." 3. "I should call the health care provider if the cast feels warm or hot or has an unusual smell or odor." 4. "I will keep small toys and sharp objects away from the cast and be sure that my child does not put anything inside the cast."

1 The mother needs to be instructed not to use a heat lamp to help the cast dry because of the risk associated with a burn injury from the heat lamp. The statements in the remaining options indicate understanding of instructions.

A neighborhood nurse is attending a soccer game at a local middle school. One of the students falls off the bleachers and sustains an injury to the left arm. The nurse quickly attends to the child and suspects that the child's arm may be broken. Which nursing action would be the priority before transferring the child to the hospital emergency department? 1. Immobilize the arm. 2. Ask for the name of the child's pediatrician or family health care provider so that he or she can be contacted. 3. Have someone call the radiology department of the local hospital to let staff know that the child will be arriving. 4. Tell the child that the arm probably is fractured but not to worry because permanent damage to the arm will not occur.

1 When a fracture is suspected, it is imperative that the area be splinted and immobilized before the injured person is transferred or moved. The nurse should remain with the child and provide realistic reassurance. Although it may be necessary to contact the child's pediatrician, this is not the priority. It is not necessary to notify the radiology department because this would be the responsibility of the emergency department staff when the child arrives if it is determined that the child needs a radiograph. The child should not be told that permanent damage will not occur.

e nurse is caring for a newborn infant with spina bifida (myelomeningocele) who is scheduled for surgical closure of the sac. In the preoperative period, which is the priority problem? 1.Infection 2.Choking 3.Inability to tolerate stimulation 4.Delayed growth and development

1 Rationale: A myelomeningocele is a type of spina bifida that results from failure of the neural tube to close during embryonic development. With a myelomeningocele, protrusion of the meninges, cerebrospinal fluid, nerve roots, and a portion of the spinal cord occurs. The newborn with spina bifida is at risk for infection before the closure of the sac, which is done soon after birth. Initial care of the newborn with myelomeningocele involves prevention of infection. A sterile normal saline dressing is placed over the sac to maintain moisture of the sac and its contents and to prevent tearing or breakdown of the skin integrity at the site. Any opening in the sac greatly increases the risk of infection of the central nervous system. Choking and inability to tolerate stimulation are not priority problems with this defect. Delayed growth and development is a problem for the infant with myelomeningocele, but preventing infection has priority in the preoperative period.

A thymectomy accomplished via a median sternotomy approach is performed in a client with a diagnosis of myasthenia gravis. The nurse develops a postoperative plan of care for the client that should include which intervention? 1. Monitor the chest tube drainage. 2. Restrict visitors for 24 hours postoperatively. 3. Maintain intravenous infusion of lactated Ringer's solution. 4. Avoid administering pain medication to prevent respiratory depression.

1 Rationale: A thymectomy may be used for management of clients with myasthenia gravis. The procedure is performed through a median sternotomy or a transcervical approach. Postoperatively the client will have a chest tube in the mediastinum. Lactated intravenous solutions usually are avoided because they can increase weakness. Pain medication is administered as needed, but the client is monitored closely for respiratory depression. There is no reason to restrict visitors.

The nurse reviews the health care provider's (HCP) prescriptions for a client with Guillain-Barré syndrome. Which prescription written by the HCP should the nurse question? 1.Clear liquid diet 2.Bilateral calf measure 3.Monitor vital signs frequently 4.Passive range-of-motion (ROM) exercises

1 Rationale: Clients with Guillain-Barré syndrome have dysphagia. Clients with dysphagia are more likely to aspirate clear liquids than thick or semisolid foods. Passive ROM exercises can help prevent contractures, and assessing calf measurements can help detect deep vein thrombosis, for which these clients are at risk. Because clients with Guillain-Barré syndrome are at risk for hypotension or hypertension, bradycardia, and respiratory depression, frequent monitoring of vital signs is required.

A client with myasthenia gravis arrives at the hospital emergency department in suspected crisis. The health care provider plans to administer edrophonium to differentiate between myasthenic and cholinergic crises. The nurse ensures that which medication is available in the event that the client is in cholinergic crisis? 1.Atropine sulfate 2.Morphine sulfate 3.Protamine sulfate 4.Pyridostigmine bromide

1 Rationale: Clients with cholinergic crisis have experienced overdosage of medication. Edrophonium will exacerbate symptoms in cholinergic crisis to the point at which the client may need intubation and mechanical ventilation. Intravenous atropine sulfate is used to reverse the effects of these anticholinesterase medications. Morphine sulfate and pyridostigmine bromide would worsen the symptoms of cholinergic crisis. Protamine sulfate is the antidote for heparin.

The nurse in the neurological unit is monitoring a client for signs of increased intracranial pressure (ICP). The nurse reviews the assessment findings for the client and notes documentation of the presence of Cushing's reflex. The nurse determines that the presence of this reflex is obtained by assessing which item? 1.Blood pressure 2.Motor response 3.Pupillary response 4. Level of consciousness

1 Rationale: Cushing's reflex is a late sign of increased ICP and consists of a widening pulse pressure (systolic pressure rises faster than diastolic pressure) and bradycardia. Options 2, 3, and 4 are unrelated to monitoring for Cushing's reflex.

The nurse is caring for a client after a craniotomy and monitors the client for signs of increased intracranial pressure (ICP). Which finding, if noted in the client, would indicate an early sign of increased ICP? 1.Confusion 2.Bradycardia 3.Sluggish pupils 4.A widened pulse pressure

1 Rationale: Early manifestations of increased ICP are subtle and often may be transient, lasting for only a few minutes in some cases. These early clinical manifestations include episodes of confusion, drowsiness, and slight pupillary and breathing changes. Later manifestations include a further decrease in the level of consciousness, a widened pulse pressure, and bradycardia. Cheyne-Stokes respiratory pattern, or a hyperventilation respiratory pattern; pupillary sluggishness and dilatation appear in the late stages.

The nurse is performing a neurological assessment on a client and is assessing the function of cranial nerves III, IV, and VI. Assessment of which aspect of function by the nurse will yield the best information about these cranial nerves? 1.Eye movements 2.Response to verbal stimuli 3. Affect, feelings, or emotions 4. Insight, judgment, and planning

1 Rationale: Eye movements are under the control of cranial nerves III, IV, and VI. Level of consciousness (response to verbal stimuli) is controlled by the reticular activating system and both cerebral hemispheres. Feelings are part of the role of the limbic system and involve both hemispheres. Insight, judgment, and planning are part of the function of the frontal lobe in conjunction with association fibers that connect to other areas of the cerebrum.

A nurse reviews the plan of care for a child with Reye's syndrome. Which priority intervention should the nurse include in the plan of care? 1.Monitor for signs of increased intracranial pressure. 2.Immediately check the presence of protein in the urine. 3.Reassure the parents hyperglycemia is a common symptom. 4.Teach the parents signs and symptoms of a bacterial infection.

1 Rationale: Intracranial pressure and encephalopathy are major symptoms of Reye's syndrome. Protein is not present in the urine. Reye's syndrome is related to a history of viral infections, and hypoglycemia is a symptom of this disease.

A child is admitted to the hospital with a diagnosis of acute bacterial meningitis. In reviewing the health care provider's prescriptions, which would the nurse question as appropriate for a child with this diagnosis? 1.Administer an oral antibiotic. 2.Maintain strict intake and output. 3.Draw blood for a culture and sensitivity. 4. Place the child on droplet precautions in a private room.

1 Rationale: Medication to treat acute bacterial meningitis is administered intravenously, not orally. A culture and sensitivity is done to determine if the diagnosis is bacterial or viral. Until meningitis is ruled out, the child is placed in isolation on droplet precautions because the disease is spread by airborne means. Strict intake and output should be maintained.

A nurse is monitoring a 7-year-old child who sustained a head injury in a motor vehicle crash for signs of increased intracranial pressure (ICP). The nurse should assess the child frequently for which early sign of increased ICP? 1.Nausea 2.Papilledema 3.Decerebrate posturing 4.Alterations in pupil size

1 Rationale: Nausea is an early sign of increased ICP. Late signs of increased ICP include a significant decrease in level of consciousness, Cushing's triad (increased systolic blood pressure and widened pulse pressure, bradycardia, and irregular respirations), and fixed and dilated pupils. Other late signs include decreased motor response to command, decreased sensory response to painful stimuli, posturing, Cheyne-Stokes respirations, and papilledema.

A client with multiple sclerosis tells a home health care nurse that she is having increasing difficulty in transferring from the bed to a chair. What is the initial nursing action? 1. Observe the client demonstrating the transfer technique. 2. Start a restorative nursing program before an injury occurs. 3. Seize the opportunity to discuss potential nursing home placement. 4. Determine the number of falls that the client has had in recent weeks.

1 Rationale: Observation of the client's transfer technique is the initial intervention. Starting a restorative program is important but not unless an assessment has been completed first. Discussing nursing home placement would be inappropriate in view of the information provided in the question. Determining the number of falls is another important intervention, but observing the transfer technique should be done first.

A nurse is performing an admission assessment on a child with a seizure disorder. The nurse is interviewing the child's parents to determine their adjustment to caring for their child, who has a chronic illness. Which statement, if made by the parents, would indicate a need for further teaching? 1."Our child sleeps in our bedroom at night." 2."We worry about injuries when our child has a seizure." 3."Our child is involved in a swim program with neighbors and friends." 4."Our babysitter just completed cardiopulmonary resuscitation [CPR] training."

1 Rationale: Parents are especially concerned about seizures that might go undetected during the night. The nurse needs to decrease parental overprotection and should suggest the use of a baby monitor at night. Options 3 and 4, involvement in a swim program and knowing CPR identify parental understanding of the disorder. Worrying about injuries when a child has a seizure is a common concern. The parents need to be reminded that as the child grows, they cannot always observe their child, but their knowledge of seizure activity and care is appropriate to minimize complications.

The nurse is providing instructions to a client with a seizure disorder who will be taking phenytoin (Dilantin). Which statement, if made by the client, would indicate an understanding of the information about this medication? 1."I need to perform good oral hygiene, including flossing and brushing my teeth." 2."I should try to avoid alcohol, but if I'm not able to, I can drink alcohol in moderation." 3. "I should take my medication before coming to the laboratory to have a blood level drawn." 4. "I should monitor for side effects and adjust my medication dose depending on how severe the side effects are."

1 Rationale: The client should perform good oral hygiene, including flossing and brushing the teeth. The client also should see a dentist at regularly scheduled times because gingival hyperplasia is a side effect of this medication. The client should avoid alcohol while taking this medication. The client should also be instructed that follow-up serum blood levels are important and that, on the day of the scheduled laboratory test, the client should avoid taking the medication before the specimen is drawn. The client should not adjust medication dosages.

An adult client has a diagnosis of hydrocephalus. The nurse plans care, knowing that this condition leads to more serious neurological consequences in adults as a result of closure of which structures? 1.Cranial sutures 2.Arachnoid villi 3.Foramen of Monro 4.Aqueduct of Sylvius

1 Rationale: The closure of cranial sutures during childhood prevents expansion of the cranial vault when hydrocephalus occurs in the adult. This leads to increased neurological changes with lesser degrees of hydrocephalus compared with hydrocephalus during early childhood. The other structures identified are associated with cerebrospinal fluid formation and circulation, but these do not close off.

The nurse is teaching a client with paraplegia measures to maintain skin integrity. Which instruction will be most helpful to the client? 1.Shift weight every 2 hours while in a wheelchair. 2.Change bed sheets every other week to maintain cleanliness. 3.Place a pillow on the seat of the wheelchair to provide extra comfort. 4. Use a mirror to inspect for redness and skin breakdown twice a week.

1 Rationale: To maintain skin integrity, the client should shift weight in the wheelchair every 2 hours and use a pressure relief pad. A pillow is not sufficient to relieve the pressure. While the client is in bed, the bottom sheet should be free of wrinkles and wetness. Sheets should be changed as needed and more frequently than every other week. The client should use a mirror to inspect the skin twice daily (morning and evening) to assess for redness, edema, and breakdown. General additional measures include a nutritious diet and meticulous skin care.

The nurse knows that symptoms associated with a TIA, usually a precursor of a future stroke, usually subside in what period of time?

1 hour A transient ischemic attack (TIA) is a neurologic deficit typically lasting less than 1 hour. A TIA is manifested by a sudden loss of motor, sensory, or visual function. The symptoms result from temporary ischemia (impairment of blood flow) to a specific region of the brain; however, when brain imaging is performed, there is no evidence of ischemia.

The nurse is creating a teaching plan for a client with newly diagnosed migraine headaches. Which key items will be included in the teaching plan? Select all that apply. 1. Foods that contain tyramine, such as alcohol and aged cheese, should be avoided. 2. Drugs such as nitroglycerin and nifedipine should be avoided. 3. Abortive therapy is aimed at eliminating the pain during the aura. 4. A potential side effect of medications is rebound headache. 5. Complementary therapies such as biofeedback and relaxation may be helpful. 6. Estrogen therapy should be continued as prescribed by the client's health care provider.

1, 2, 3, 4, 5 Medications such as estrogen supplements may actually trigger a migraine headache attack. All of the other statements are accurate and should be included in the teaching plan.

Which actions should the nurse delegate to an experienced unlicensed assistive personnel (UAP) when caring for a client with a thrombotic stroke who has residual left-sided weakness? Select all that apply. 1. Assisting the client to reposition every 2 hours 2. Reapplying pneumatic compression boots 3. Reminding the client to perform active range-of-motion (ROM) exercises 4. Assessing the extremities for redness and edema 5. Setting up meal trays and assisting with feeding 6. Using a lift to assist the client up to a bedside chair

1, 2, 3, 5, 6 An experienced UAP would know how to reposition the client, reapply compression boots, and feed a client, and would remind the client to perform activities the client has been taught to perform. UAPs are also trained to use a client lift to get clients into or out of bed. Assessing for redness and swelling (signs of deep vein thrombosis) requires additional education and skill, appropriate to the professional nurse.

The nurse is helping a client with a spinal cord injury to establish a bladder retraining program. Which strategies may stimulate the client to void? Select all that apply. 1. Stroking the client's inner thigh 2. Pulling on the client's pubic hair 3. Initiating intermittent straight catheterization 4. Pouring warm water over the client's perineum 5. Tapping the bladder to stimulate the detrusor muscle 6. Reminding the client to void in a urinal every hour while awake

1, 2, 4, 5 All of the strategies except straight catheterization may stimulate voiding in clients with a spinal cord injury (SCI). Intermittent bladder catheterization can be used to empty the client's bladder, but it will not stimulate voiding. To use a urinal, the client must have bladder control, which is often absent after SCI. In addition, every hour while awake would be too often and ignore the bladder filling at night.

A 23-year-old client with a recent history of encephalitis is admitted to the medical unit with new-onset generalized tonic-clonic seizures. Which nursing activities included in the client's care will be best to assign to an LPN/LVN whom the nurse is supervising? Select all that apply. 1. Observing and documenting the onset and duration of any seizure activity 2. Administering phenytoin 200 mg PO three times a day 3. Teaching the client about the need for frequent tooth brushing and flossing 4. Developing a discharge plan that includes referral to the Epilepsy Foundation 5. Assessing for adverse effects caused by new antiseizure medications 6. Turning the client to his or her side to avoid aspiration

1, 2, 6 Any nursing staff member who is involved in caring for the client should observe for the onset and duration of seizures (although a more detailed assessment of seizure activity should be done by the RN). Administration of medications is included in LPN/LVN education and scope of practice. Turning the client on his or her side to avoid aspiration is certainly within the scope of practice for an LPN/LVN. Teaching, discharge planning, and assessment for adverse effects of new medications are complex activities that require RN-level education and scope of practice.

The nurse on the neurologic acute care unit is assessing the orientation of a client with severe headaches. Which questions would the nurse use to determine orientation? Select all that apply. 1. When did you first experience the headache symptoms? 2. Who is the Mayor of Cleveland? 3. What is your health care provider's name? 4. What year and month is this? 5. What is your parents' address? 6. What is the name of this health care facility?

1, 3, 4, 6 After determining alertness in a client, the next step is to evaluate orientation. When the client's attention is engaged, ask him or her questions to determine orientation. Varying the sequence of questioning on repeated assessments prevents the client from memorizing the answers. Responses that indicate orientation include the ability to answer questions about person, place, and time by asking for information such as the client's ability to relate the onset of symptoms, the name of his or her health care provider or nurse, the year and month, his or her address, and the name of the referring physician or health care agency. Asking about mayors' names or parents' address may be inappropriate to assess orientation.

The nurse is preparing a plan of care for a client with a diagnosis of amyotrophic lateral sclerosis (ALS). On assessment, the nurse notes that the client is severely dysphagic. Which intervention should be included in the care plan for this client? Select all that apply. 1.Provide oral hygiene after each meal. 2.Assess swallowing ability frequently. 3.Allow the client sufficient time to eat. 4.Maintain a suction machine at the bedside. 5.Provide a full liquid diet for ease in swallowing.

1,2,3,4. A client who is severely dysphagic is at risk for aspiration. Swallowing is assessed frequently. The client should be given a sufficient amount of time to eat. Semisoft foods are easiest to swallow and require less chewing. Oral hygiene is necessary after each meal. Suctioning should be available for clients who experience dysphagia and are at risk for aspiration. * Focus on the subject, care of the client with ALS, and note the words severely dysphagic. Think about the complications associated with this disorder. Recall that liquids are most difficult to swallow in the client with dysphagia.

The nurse is caring for a female young adult newly diagnosed with epilepsy and treated with phenytoin. Which of the following should the nurse include in client teaching? Select all that apply. 1. Avoid excess caffeine 2. Do not stop antiepileptic medicine abruptly 3. Do not use oral contraceptives for birth control 4. Go to an emergency department if a seizure occurs 5. Wear MedicAlert® identification

1,2,3,5

The nurse is developing a plan of care for a client with dysphagia following a stroke (brain attack). Which should the nurse include in the plan? Select all that apply. 1.Thicken liquids. 2.Assist the client with eating. 3.Assess for the presence of a swallow reflex. 4.Place the food on the affected side of the mouth. 5.Provide ample time for the client to chew and swallow.

1,2,3,5. Liquids are thickened to prevent aspiration. The nurse should assist the client with eating and place food on the unaffected side of the mouth. The nurse should assess for gag and swallowing reflexes before the client with dysphagia is started on a diet. The client should be allowed ample time to chew and swallow to prevent choking. * Focus on the subject, the client with dysphagia, and recall that the client with dysphagia has difficulty swallowing. Recalling that the client is therefore at risk for aspiration will assist in directing you to the correct options.

The nurse has determined that a client with a neurological disorder also has difficulty breathing. Which activities would be appropriate components of the care plan for this client? Select all that apply. 1.Keep suction equipment at the bedside. 2.Elevate the head of the bed 30 degrees. 3.Keep the client lying in a supine position. 4.Keep the head and neck in good alignment. 5. Administer prescribed respiratory treatments as needed.

1,2,4,5 Rationale: The nurse maintains a patent airway for the client with difficulty breathing by keeping the head and neck in good alignment and elevating the head of bed 30 degrees unless contraindicated. Suction equipment is kept at the bedside if secretions need to be cleared. The client should be kept in a side-lying position whenever possible to minimize the risk of aspiration.

The emergency department nurse is assessing a client brought in after a car accident in which the client's head hit the steering column. Which assessment findings would indicate that the triage nurse should apply spinal immobilization? Select all that apply. 1. Breath smells of alcohol 2. Client disoriented to place 3. Client reports eyes burning 4. History of multiple sclerosis 5. Point tenderness over spine

1,2,5

The nurse is planning to institute seizure precautions for a client who is being admitted from the emergency department. Which measures should the nurse include in planning for the client's safety? Select all that apply. 1.Padding the side rails of the bed 2.Placing an airway at the bedside 3.Placing the bed in the high position 4.Putting a padded tongue blade at the head of the bed 5.Placing oxygen and suction equipment at the bedside 6.Having intravenous equipment ready for insertion of an intravenous catheter

1,2,5,6 Rationale: Seizure precautions may vary from agency to agency, but they generally have some common features. Usually, an airway, oxygen, and suctioning equipment are kept available at the bedside. The side rails of the bed are padded, and the bed is kept in the lowest position. The client has an intravenous access in place to have a readily accessible route if anticonvulsant medications must be administered. The use of padded tongue blades is highly controversial, and they should not be kept at the bedside. Forcing a tongue blade into the mouth during a seizure more likely will harm the client who bites down during seizure activity. Risks include blocking the airway from improper placement, chipping the client's teeth, and subsequent risk of aspirating tooth fragments. If the client has an aura before the seizure, it may give the nurse enough time to place an oral airway before seizure activity begins.

The nurse is caring for a client with Bell's palsy. The nurse most likely expects which finding(s) on assessment? Select all that apply. 1. Change in lacrimation on the affected side 2. Electric shock-like pain in the lips and gums 3. Flattening of the nasolabial fold 4. Inability to smile symmetrically 5. Severe facial pain along the cheekbone

1,3,4

A hospitalized client develops acute hemorrhagic stroke and is transferred to the intensive care unit. Which nursing interventions should be included in the plan of care? Select all that apply. 1. Administer PRN stool softeners daily 2. Administer scheduled enoxaparin injection 3. Implement seizure precautions 4. Keep client NPO until swallow screen is performed 5. Perform frequent neurological assessments

1,3,4,5

A client sustained a concussion after falling off a ladder. What are essential instructions for the nurse to provide when the client is discharged from the hospital? Select all that apply. 1. Client should abstain from alcohol 2. Client should remain awake all night 3. Client should return if having difficulty walking 4. Responsible adult should be taught neurological examination 5. Responsible adult should stay with the client

1,3,5

A client is admitted to the hospital with an exacerbation of myasthenia gravis. What are the appropriate nursing actions? Select all that apply. 1. Administer an anticholinesterase drug AC 2. Anticipate a need for an anticholinergic drug 3. Develop a bladder training schedule 4. Encourage semi-solid food consumption 5. Teach the necessity for annual flu vaccination

1,4,5

The nurse is caring for a client admitted for a seizure disorder. The nurse witnesses the client having a tonic-clonic seizure with increasing salivation. Which actions should the nurse take? Select all that apply. 1. Call for help 2. Hold down the client's arms 3. Insert a tongue depressor to move the tongue 4. Prepare for suctioning 5. Turn the client on the side

1,4,5

A thymectomy accomplished via a median sternotomy approach is performed in a client with a diagnosis of myasthenia gravis. The nurse develops a postoperative plan of care for the client that should include which intervention? 1.Monitor the chest tube drainage. 2.Restrict visitors for 24 hours postoperatively. 3.Maintain intravenous infusion of lactated Ringer's solution. 4.Avoid administering pain medication to prevent respiratory depression.

1. A thymectomy may be used for management of clients with myasthenia gravis. The procedure is performed through a median sternotomy or a transcervical approach. Postoperatively the client will have a chest tube in the mediastinum. Lactated intravenous solutions usually are avoided because they can increase weakness. Pain medication is administered as needed, but the client is monitored closely for respiratory depression. There is no reason to restrict visitors. * Focus on the subject, general postoperative measures. Note the words median sternotomy approach in the question. Think about this anatomical location; this should provide you with a clue that the client will have a chest tube in place after this procedure.

The nurse is caring for a client who sustained a spinal cord injury. During administration of morning care, the client begins to exhibit signs and symptoms of autonomic dysreflexia. Which initial nursing action should the nurse take? 1.Elevate the head of the bed. 2.Examine the rectum digitally. 3.Assess the client's blood pressure. 4.Place the client in the prone position.

1. Autonomic dysreflexia is a serious complication that can occur in the spinal cord-injured client. Once the syndrome is identified, the nurse elevates the head of the client's bed and then examines the client for the source of noxious stimuli. The nurse also assesses the client's blood pressure, but the initial action would be to elevate the head of the bed. The client would not be placed in the prone position. * Note the strategic word initial in the question. Noting that both options 1 and 4 identify a client position will assist in eliminating options 2 and 3 because positioning is a priority. Recalling the pathophysiology of autonomic dysreflexia will direct you to the correct option.

he home health nurse is visiting a client with myasthenia gravis and is discussing methods to minimize the risk of aspiration during meals related to decreased muscle strength. Which suggestions should the nurse give to the client? Select all that apply. 1. Chew food thoroughly. 2. Cut food into very small pieces. 3. Sit straight up in the chair while eating. 4. Lift the head while swallowing liquids. 5. Swallow when the chin is tipped slightly downward to the chest.

1. Chew food thoroughly. 2. Cut food into very small pieces. 3. Sit straight up in the chair while eating. 5. Swallow when the chin is tipped slightly downward to the chest. The client avoids swallowing any type of food or drink with the head lifted upward, which could actually cause aspiration by opening the glottis. The client should be advised to sit upright while eating, not to talk with food in the mouth (talking requires opening the glottis), cut food into very small pieces, chew thoroughly, and tip the chin downward to swallow.

The nurse in the neurological unit is monitoring a client for signs of increased intracranial pressure (ICP). The nurse reviews the assessment findings for the client and notes documentation of the presence of Cushing's reflex. The nurse determines that the presence of this reflex is obtained by assessing which item? 1.Blood pressure 2.Motor response 3.Pupillary response 4.Level of consciousness

1. Cushing's reflex is a late sign of increased ICP and consists of a widening pulse pressure (systolic pressure rises faster than diastolic pressure) and bradycardia. Options 2, 3, and 4 are unrelated to monitoring for Cushing's reflex. * Use knowledge regarding Cushing's reflex and the ABCs-airway, breathing, and circulation-to assist in directing you to the correct option.

The nurse is developing a plan of care for an older client that addresses interventions to prevent cold discomfort and the development of accidental hypothermia. The nurse should document which desired outcome in the plan of care? 1.The client's body temperature is 98° F. 2.The client's fingers and toes are cool to touch. 3.The client remains in a fetal position when in bed. 4.The client complains of coolness in the hands and feet only.

1. Desired outcomes for nursing interventions to prevent cold discomfort and the development of accidental hypothermia include the following: hands and limbs are warm; body is relaxed and not curled; body temperature is greater than 97° F; the client is not shivering; and the client has no complaints of feeling cold. * Eliminate option 4 first because of the closed-ended word only. Regarding the remaining options, focusing on the subject, a desired outcome to prevent cold discomfort and hypothermia, will direct you to the correct option.

The nurse is caring for a client after a craniotomy and monitors the client for signs of increased intracranial pressure (ICP). Which finding, if noted in the client, would indicate an early sign of increased ICP? 1.Confusion 2.Bradycardia 3.Sluggish pupils 4.A widened pulse pressure

1. Early manifestations of increased ICP are subtle and often may be transient, lasting for only a few minutes in some cases. These early clinical manifestations include episodes of confusion, drowsiness, and slight pupillary and breathing changes. Later manifestations include a further decrease in the level of consciousness, a widened pulse pressure, and bradycardia. Cheyne-Stokes respiratory pattern, or a hyperventilation respiratory pattern; pupillary sluggishness and dilatation appear in the late stages. * Note the strategic word early in the question. Recalling that the earliest indicator of increased ICP is changes in level of consciousness will direct you to the correct option.

The nurse is performing a neurological assessment on a client and is assessing the function of cranial nerves III, IV, and VI. Assessment of which aspect of function by the nurse will yield the best information about these cranial nerves? 1.Eye movements 2.Response to verbal stimuli 3.Affect, feelings, or emotions 4.Insight, judgment, and planning

1. Eye movements are under the control of cranial nerves III, IV, and VI. Level of consciousness (response to verbal stimuli) is controlled by the reticular activating system and both cerebral hemispheres. Feelings are part of the role of the limbic system and involve both hemispheres. Insight, judgment, and planning are part of the function of the frontal lobe in conjunction with association fibers that connect to other areas of the cerebrum. * Focus on the subject, cranial nerves III, IV, and VI. Recalling that these nerves control eye movement will direct you to the correct option.

A client with a neurological problem is experiencing hyperthermia. Which measures would be appropriate for the nurse to use in trying to lower the client's body temperature? Select all that apply. 1. Giving tepid sponge baths 2. Applying a hypothermia blanket 3. Covering the client with blankets 4. Administering acetaminophen per protocol 5. Placing ice packs over the client's abdomen and in the axilla and groin

1. Giving tepid sponge baths 2. Applying a hypothermia blanket 4. Administering acetaminophen per protocol Standard measures to lower body temperature include removing bed covers, providing cool sponge baths, using an electric fan in the room, administering acetaminophen, and placing a hypothermia blanket under the client. Ice packs are not used because they could cause shivering, which increases cellular oxygen demands, with the potential for increased intracranial pressure.

The nurse has just admitted to the nursing unit a client with a basilar skull fracture who is at risk for increased intracranial pressure. Pending specific health care provider prescriptions, the nurse should safely place the client in which positions? Select all that apply. 1. Head midline 2. Neck in neutral position 3. Head of bed elevated 30 to 45 degrees 4. Head turned to the side when flat in bed 5. Neck and jaw flexed forward when opening the mouth

1. Head midline 2. Neck in neutral position 3. Head of bed elevated 30 to 45 degrees Use of proper positions promotes venous drainage from the cranium to keep intracranial pressure from elevating. The head of the client at risk for or with increased intracranial pressure should be positioned so that it is in a neutral, midline position. The head of the bed should be raised to 30 to 45 degrees. The nurse should avoid flexing or extending the client's neck or turning the client's head from side to side.

A client who had a brain attack (stroke) has right-sided hemianopsia. What should the nurse plan to do to help the client adapt to this problem? 1.Teach the client to scan the environment. 2.Place all objects within the left visual field. 3.Place all objects within the right visual field. 4.Ensure that the family brings the client's eyeglasses to hospital.

1. Hemianopsia is blindness in half the visual field. The client with hemianopsia is taught to scan the environment. This allows the client to take in the entirety of the visual field, which is necessary for proper functioning within the environment and helps to prevent injury to the client. Options 2 and 3 will not help the client adapt to this visual impairment. Eyeglasses are useful if the client already wears them, but they will not correct this visual-field deficit. * Note the subject, measures to assist the client experiencing hemianopsia. The correct option teaches the client a response to overcome this visual deficit. Recalling that hemianopsia is blindness in one half of the field of vision will direct you to the correct option.

The nurse has determined that a client with a neurological disorder also has difficulty breathing. Which activities would be appropriate components of the care plan for this client? Select all that apply. 1. Keep suction equipment at the bedside. 2. Elevate the head of the bed 30 degrees. 3. Keep the client lying in a supine position. 4. Keep the head and neck in good alignment. 5. Administer prescribed respiratory treatments as needed.

1. Keep suction equipment at the bedside. 2. Elevate the head of the bed 30 degrees. 4. Keep the head and neck in good alignment. 5. Administer prescribed respiratory treatments as needed. The nurse maintains a patent airway for the client with difficulty breathing by keeping the head and neck in good alignment and elevating the head of the bed 30 degrees unless contraindicated. Suction equipment is kept at the bedside if secretions need to be cleared. The client should be kept in a side-lying position whenever possible to minimize the risk of aspiration.

A client with a spinal cord injury is prone to experiencing autonomic dysreflexia. The nurse should include which measures in the plan of care to minimize the risk of occurrence? Select all that apply. 1. Keeping the linens wrinkle-free under the client 2. Preventing unnecessary pressure on the lower limbs 3. Limiting bladder catheterization to once every 12 hours 4. Turning and repositioning the client at least every 2 hours 5. Ensuring that the client has a bowel movement at least once a week

1. Keeping the linens wrinkle-free under the client 2. Preventing unnecessary pressure on the lower limbs 4. Turning and repositioning the client at least every 2 hours The most frequent cause of autonomic dysreflexia is a distended bladder. Straight catheterization should be done every 4 to 6 hours (catheterization every 12 hours is too infrequent), and urinary catheters should be checked frequently to prevent kinks in the tubing. Constipation and fecal impaction are other causes, so maintaining bowel regularity is important. Ensuring a bowel movement once a week is much too infrequent. Other causes include stimulation of the skin from tactile, thermal, or painful stimuli. The nurse administers care to minimize risk in these areas.

The nurse is caring for a client who begins to experience seizure activity while in bed. Which actions should the nurse take? Select all that apply. 1. Loosening restrictive clothing 2. Restraining the client's limbs 3. Removing the pillow and raising padded side rails 4. Positioning the client to the side, if possible, with the head flexed forward 5. Keeping the curtain around the client and the room door open so when help arrives they can quickly enter to assist

1. Loosening restrictive clothing 3. Removing the pillow and raising padded side rails 4. Positioning the client to the side, if possible, with the head flexed forward Nursing actions during a seizure include providing for privacy, loosening restrictive clothing, removing the pillow and raising padded side rails in the bed, and placing the client on one side with the head flexed forward, if possible, to allow the tongue to fall forward and facilitate drainage. The limbs are never restrained because the strong muscle contractions could cause the client harm. If the client is not in bed when seizure activity begins, the nurse lowers the client to the floor, if possible; protects the head from injury; and moves furniture that may injure the client.

The nurse is caring for a client with an intracranial aneurysm who has been alert. Which signs and symptoms are an early indication that the level of consciousness (LOC) is deteriorating? Select all that apply. 1. Mild drowsiness 2. Drooping eyelids 3. Ptosis of the left eyelid 4. Slight slurring of speech 5. Less frequent spontaneous speech

1. Mild drowsiness 4. Slight slurring of speech 5. Less frequent spontaneous speech Early changes in LOC relate to orientation, alertness, and verbal responsiveness. Mild drowsiness, slight slurring of speech, and less frequent spontaneous speech are early signs of decreasing LOC. Ptosis (drooping) of the eyelid is caused by pressure on and dysfunction of cranial nerve III. Once ptosis occurs, it is ongoing; it does not relate to LOC.

A client arrives in the hospital emergency department with a closed head injury to the right side of the head caused by an assault with a baseball bat. The nurse assesses the client neurologically, looking primarily for motor response deficits that involve which area? 1.The left side of the body 2.The right side of the body 3.Both sides of the body equally 4.Cranial nerves only, such as speech and pupillary response

1. Motor responses such as weakness and decreased movement will be seen on the side of the body that is opposite an area of head injury. Contralateral deficits result from compression of the cortex of the brain or the pyramidal tracts. Depending on the severity of the injury, the client may have a variety of neurological deficits. * Note the strategic word primarily. Eliminate option 4 because of the closed-ended word only. Noting that the injury was on the right side of the head and using knowledge of anatomy and physiology will direct you to the correct option.

The nurse is planning to institute seizure precautions for a client who is being admitted from the emergency department. Which measures should the nurse include in planning for the client's safety? Select all that apply. 1. Padding the side rails of the bed 2. Placing an airway at the bedside 3. Placing the bed in the high position 4. Putting a padded tongue blade at the head of the bed 5. Placing oxygen and suction equipment at the bedside 6. Having intravenous equipment ready for insertion of an intravenous catheter

1. Padding the side rails of the bed 2. Placing an airway at the bedside 5. Placing oxygen and suction equipment at the bedside 6. Having intravenous equipment ready for insertion of an intravenous catheter Seizure precautions may vary from agency to agency, but they generally have some common features. Usually, an airway, oxygen, and suctioning equipment are kept available at the bedside. The side rails of the bed are padded, and the bed is kept in the lowest position. The client has an intravenous access in place to have a readily accessible route if anticonvulsant medications must be administered. The use of padded tongue blades is highly controversial, and they should not be kept at the bedside. Forcing a tongue blade into the mouth during a seizure more likely will harm the client who bites down during seizure activity. Risks include blocking the airway from improper placement, chipping the client's teeth, and subsequent risk of aspirating tooth fragments. If the client has an aura before the seizure, it may give the nurse enough time to place an oral airway before seizure activity begins.

The nurse is instituting seizure precautions for a client who is being admitted from the emergency department. Which measures should the nurse include in planning for the client's safety? Select all that apply. 1. Padding the side rails of the bed 2. Placing an airway at the bedside 3. Placing the bed in the high position 4. Putting a padded tongue blade at the head of the bed 5. Placing oxygen and suction equipment at the bedside 6. Flushing the intravenous catheter to ensure that the site is patent

1. Padding the side rails of the bed 2. Placing an airway at the bedside 5. Placing oxygen and suction equipment at the bedside 6. Flushing the intravenous catheter to ensure that the site is patent Seizure precautions may vary from agency to agency, but they generally have some common features. Usually, an airway, oxygen, and suctioning equipment are kept available at the bedside. The side rails of the bed are padded, and the bed is kept in the lowest position. The client has an intravenous access in place to have a readily accessible route if antiseizure medications must be administered, and as part of the routine assessment the nurse should be checking patency of the catheter. The use of padded tongue blades is highly controversial, and they should not be kept at the bedside. Forcing a tongue blade into the mouth during a seizure more likely will harm the client who bites down during seizure activity. Risks include blocking the airway from improper placement, chipping the client's teeth, and subsequent risk of aspirating tooth fragments. If the client has an aura before the seizure, it may give the nurse enough time to place an oral airway before seizure activity begins.

The nurse is assessing a client who is experiencing seizure activity. The nurse understands that it is necessary to determine information about which items as part of routine assessment of seizures? Select all that apply. 1. Postictal status 2. Duration of the seizure 3. Changes in pupil size or eye deviation 4. Seizure progression and type of movements 5. What the client ate in the 2 hours preceding seizure activity

1. Postictal status 2. Duration of the seizure 3. Changes in pupil size or eye deviation 4. Seizure progression and type of movements Typically seizure assessment includes the time the seizure began, parts of the body affected, type of movements and progression of the seizure, change in pupil size or eye deviation or nystagmus, client condition during the seizure, and postictal status. Determining what the client ate 2 hours prior to the seizure is not a component of seizure assessment.

The nurse is preparing a plan of care for a client with a diagnosis of amyotrophic lateral sclerosis (ALS). On assessment, the nurse notes that the client is severely dysphagic. Which intervention should be included in the care plan for this client? Select all that apply. 1. Provide oral hygiene after each meal. 2. Assess swallowing ability frequently. 3. Allow the client sufficient time to eat. 4. Maintain a suction machine at the bedside. 5. Provide a full liquid diet for ease in swallowing.

1. Provide oral hygiene after each meal. 2. Assess swallowing ability frequently. 3. Allow the client sufficient time to eat. 4. Maintain a suction machine at the bedside. A client who is severely dysphagic is at risk for aspiration. Swallowing is assessed frequently. The client should be given a sufficient amount of time to eat. Semisoft foods are easiest to swallow and require less chewing. Oral hygiene is necessary after each meal. Suctioning should be available for clients who experience dysphagia and are at risk for aspiration.

The nurse is planning to put aneurysm precautions in place for a client with a cerebral aneurysm. Which nursing measures would be implemented? Select all that apply. 1. Provide physical aspects of care. 2. Prevent pushing or straining activities. 3. Limit caffeinated coffee to 1 cup per day. 4. Keeping the lights on in the client's room. 5. Maintain the head of the bed at 15 degrees.

1. Provide physical aspects of care. 2. Prevent pushing or straining activities. 5. Maintain the head of the bed at 15 degrees. Aneurysm precautions include placing the client on bed rest (as prescribed) in a quiet setting. Stimulants such as caffeine and nicotine are prohibited; decaffeinated coffee or tea may be used. Lights are kept dim to minimize environmental stimulation. Any activity that increases the blood pressure or impedes venous return from the brain is prohibited, such as pushing, pulling, sneezing, coughing, or straining. The nurse provides physical care to minimize increases in blood pressure. For the same reason, visitors, radio, television, and reading materials are prohibited or limited.

The nurse is planning care for a client who displays confusion secondary to a neurological problem. Which approaches by the nurse would be helpful in assisting this client? Select all that apply. 1. Providing sensory cues 2. Giving simple, clear directions 3. Providing a stable environment 4. Keeping family pictures at the bedside 5. Encouraging family members to visit at the same time

1. Providing sensory cues 2. Giving simple, clear directions 3. Providing a stable environment 4. Keeping family pictures at the bedside Clients with cognitive impairment from neurological dysfunction respond best to a stable environment that is limited in amount and type of sensory input. The nurse can provide sensory cues and give clear, simple directions in a positive manner. Confusion can be minimized by reducing environmental stimuli (such as television or multiple visitors) and by keeping familiar personal articles (such as family pictures) at the bedside.

The nurse is trying to communicate with a client who had a stroke and has aphasia. Which actions by the nurse would be most helpful to the client? Select all that apply. 1. Speaking to the client at a slower rate 2. Allowing plenty of time for the client to respond 3. Completing the sentences that the client cannot finish 4. Looking directly at the client during attempts at speech 5. Shouting words if it seems as though the client has difficulty understanding

1. Speaking to the client at a slower rate 2. Allowing plenty of time for the client to respond 4. Looking directly at the client during attempts at speech Clients with aphasia after brain attack often fatigue easily and have a short attention span. General guidelines when trying to communicate with the aphasic client include speaking more slowly and allowing adequate response time, listening to and watching attempts to communicate, and trying to put the client at ease with a caring and understanding manner. The nurse would avoid shouting (because the client is not deaf), appearing rushed for a response, and letting family members provide all responses for the client.

The nurse is evaluating the respiratory outcomes for a client with Guillain-Barré syndrome. The nurse determines that which are acceptable outcomes for the client? Select all that apply. 1. Spontaneous breathing 2. Oxygen saturation of 98% 3. Adventitious breath sounds 4. Normal arterial blood gas levels 5. Vital capacity within normal range

1. Spontaneous breathing 2. Oxygen saturation of 98% 4. Normal arterial blood gas levels 5. Vital capacity within normal range Satisfactory respiratory outcomes for a client with Guillain-Barré syndrome include clear breath sounds on auscultation, spontaneous breathing, normal vital capacity, normal arterial blood gas levels, and normal pulse oximetry. Adventitious breath sounds are an abnormal finding.

The nurse is assigned to care for a client with complete right-sided hemiparesis from a stroke (brain attack). Which characteristics are associated with this condition? Select all that apply. 1. The client is aphasic. 2. The client has weakness on the right side of the body. 3. The client has complete bilateral paralysis of the arms and legs. 4. The client has weakness on the right side of the face and tongue. 5. The client has lost the ability to move the right arm but is able to walk independently. 6. The client has lost the ability to ambulate independently but is able to feed and bathe himself or herself without assistance.

1. The client is aphasic. 2. The client has weakness on the right side of the body. 4. The client has weakness on the right side of the face and tongue. Hemiparesis is a weakness of one side of the body that may occur after a stroke. It involves weakness of the face and tongue, arm, and leg on one side. These clients are also aphasic: unable to discriminate words and letters. They are generally very cautious and get anxious when attempting a new task. Complete bilateral paralysis does not occur in hemiparesis. The client with right-sided hemiparesis has weakness of the right arm and leg and needs assistance with feeding, bathing, and ambulating.

The nurse is providing instructions to a client with a seizure disorder who will be taking phenytoin (Dilantin). Which statement, if made by the client, would indicate an understanding of the information about this medication? 1."I need to perform good oral hygiene, including flossing and brushing my teeth." 2."I should try to avoid alcohol, but if I'm not able to, I can drink alcohol in moderation." 3."I should take my medication before coming to the laboratory to have a blood level drawn." 4."I should monitor for side effects and adjust my medication dose depending on how severe the side effects are."

1. The client should perform good oral hygiene, including flossing and brushing the teeth. The client also should see a dentist at regularly scheduled times because gingival hyperplasia is a side effect of this medication. The client should avoid alcohol while taking this medication. The client should also be instructed that follow-up serum blood levels are important and that, on the day of the scheduled laboratory test, the client should avoid taking the medication before the specimen is drawn. The client should not adjust medication dosages. * Focus on the subject, client understanding about phenytoin. Knowledge that gingival hyperplasia is a side effect of this medication will assist in directing you to the correct option.

A client who had cranial surgery 5 days earlier to remove a brain tumor has a few cognitive deficits and does not seem to be progressing as quickly as the client or family hoped. The nurse plans to implement which approach as most helpful to the client and family at this time? 1.Emphasize progress in a realistic manner. 2.Set high goals to give the client something to "aim for." 3.Tell the family to be extremely optimistic with the client. 4.Inform the client and family of standardized goals of care.

1. The most helpful approach by the nurse is to emphasize progress that is being made in a realistic manner. The nurse does not offer false hope but does provide factual information in a clear and positive manner. The nurse encourages the family to be realistic in their expectations and attitudes. The plan of care should be individualized for each client. * Note the strategic word most. Use therapeutic communication techniques to answer this question. Also, noting the word realistic in the correct option will direct you to this one.

The client with a cervical spine injury has cervical tongs applied in the emergency department. What should the nurse include when planning care for this client? Select all that apply. 1. Using a RotoRest bed 2. Ensuring that weights hang freely 3. Removing the weights to reposition the client 4. Assessing the integrity of the weights and pulleys 5. Comparing the amount of prescribed traction with the amount in use

1. Using a RotoRest bed 2. Ensuring that weights hang freely 4. Assessing the integrity of the weights and pulleys 5. Comparing the amount of prescribed traction with the amount in use

The nurse is caring for a client who is on bed rest as part of aneurysm precautions. The nurse should avoid doing which action when giving respiratory care to this client? 1.Encourage hourly coughing. 2.Assist with incentive spirometer. 3.Encourage hourly deep breathing. 4.Reposition gently side to side every 2 hours.

1. With aneurysm precautions, any activity that could raise the client's intracranial pressure (ICP) is avoided. For this reason, activities such as straining, coughing, blowing the nose, and even sneezing are avoided whenever possible. The other interventions (repositioning, deep breathing, and incentive spirometry) do not provide added risk of increasing ICP and are beneficial in reducing the respiratory complications of bed rest. * Focus on the subject, the action that should be avoided with a client on aneurysm precautions. Using principles related to increased ICP will direct you to the correct option.

A girl who is playing in the playroom experiences a tonic-clonic seizure. During the seizure, the nurse should take which actions? Select all that apply. 1. Remain calm. 2. Time the seizure. 3. Ease the child to the floor. 4. Loosen restrictive clothing. 5. Keep the child on her back.

1.2.3.4 When a child is having a seizure, the nurse should remain calm, time the seizure, ease the child to the floor if the child is standing or seated, keep the child on the side (to prevent aspiration), and loosen restrictive clothing.

A child is brought to the emergency department, and diagnostic x-rays of the child reveal that a fracture is present. The mother states that the child was rollerblading and attempted to break a fall with an outstretched arm. A plaster of Paris cast is applied to the arm. Which instructions should the nurse provide the mother? Select all that apply. 1. The cast will mold to the body part. 2. The cast should be dry in about 6 hours. 3. Keep the cast elevated on pillows for the first day. 4. Make sure that the child can frequently wiggle the fingers. 5. The cast is water-resistant, so the child is able to take a bath or a shower. 6. The cast needs to be kept dry because it will begin to disintegrate when wet.

1.3.4.6 Plaster of Paris is a heavier material than that used in a synthetic cast. It molds easily to the extremity and is less expensive than a synthetic cast. It takes about 24 hours to dry, but drying time could be longer, depending on the size of the cast. Plaster of Paris is not water resistant and will begin to disintegrate when wet. The cast should be elevated on a pillow for the first day to decrease swelling as the cast begins to mold to the arm. As the cast molds, it is imperative that the child can wiggle the fingers because the extremity continues to swell. If the child can wiggle the fingers, adequate motion is present. Color and sensation of the fingers should also be assessed. All of these are important components of a teaching plan for a parent.

The nurse is creating a plan of care for a child who is at risk for seizures. Which interventions apply if the child has a seizure? Select all that apply. 1. Time the seizure. 2. Restrain the child. 3. Stay with the child. 4. Place the child in a prone position. 5. Move furniture away from the child. 6. Insert a padded tongue blade in the child's mouth.

1.3.5 A seizure is a disorder that occurs as a result of excessive and unorganized neuronal discharges in the brain that activate associated motor and sensory organs. During a seizure, the child is placed on his or her side in a lateral position. Positioning on the side prevents aspiration because saliva drains out the corner of the child's mouth. The child is not restrained because this could cause injury to the child. The nurse would loosen clothing around the child's neck and ensure a patent airway. Nothing is placed into the child's mouth during a seizure because this action may cause injury to the child's mouth, gums, or teeth. The nurse would stay with the child to reduce the risk of injury and allow for observation and timing of the seizure.

The nurse is assigned to care for a child with a brain injury who has a temporal lobe herniation and increasing intracranial pressure. Which signs should the nurse identify as indicative of this type of injury? Select all that apply. 1. Flaccid paralysis 2. Pupil response to light 3. Ipsilateral pupil dilation 4. Compression of the sixth cranial nerve 5. Shifting of the temporal lobe laterally across the tentorial notch

1.3.5 Temporal lobe herniation or uncal herniation refers to a shifting of the temporal lobe laterally across the tentorial notch. This produces compression of the third cranial nerve and ipsilateral pupil dilation. If pressure continues to rise, flaccid paralysis, pupil fixation, and death will result.

Which nursing actions apply to the care of a child who is having a seizure? Select all that apply. 1. Time the seizure. 2. Restrain the child. 3. Stay with the child. 4. Insert an oral airway. 5. Loosen clothing around the child's neck. 6. Place the child in a lateral side-lying position.

1.3.5.6 During a seizure, the nurse should stay with the child to reduce the risk of injury and allow for observation and timing of the seizure. The child is not restrained because this could cause injury to the child. The child is placed on his or her side in a lateral position. Nothing is placed in the child's mouth during a seizure because this could injure the child's mouth, gums, or teeth. Positioning on the side prevents aspiration because saliva drains out of the corner of the child's mouth. The nurse should loosen clothing around the child's neck and ensure a patent airway.

The nurse is assessing a client who is experiencing seizure activity. The nurse understands that it is necessary to determine information about which items as part of routine assessment of seizures? Select all that apply. 1.Postictal status 2.Duration of the seizure 3.Changes in pupil size or eye deviation 4.Seizure progression and type of movements 5.What the client ate in the 2 hours preceding seizure activity

1234Rationale: Typically seizure assessment includes the time the seizure began, parts of the body affected, type of movements and progression of the seizure, change in pupil size or eye deviation or nystagmus, client condition during the seizure, and postictal status. Option 5 is not a component of seizure assessment

The home health nurse is visiting a client with myasthenia gravis and is discussing methods to minimize the risk of aspiration during meals related to decreased muscle strength. Which suggestions should the nurse give to the client? Select all that apply. 1.Chew food thoroughly. 2.Cut food into very small pieces. 3. Sit straight up in the chair while eating. 4. Lift the head while swallowing liquids. 5. Swallow when the chin is tipped slightly downward to the chest.

1235Rationale: The client avoids swallowing any type of food or drink with the head lifted upward, which could actually cause aspiration by opening the glottis. The client should be advised to sit upright while eating, not to talk with food in the mouth (talking requires opening the glottis), cut food into very small pieces, chew thoroughly, and tip the chin downward to swallow.

Cerebral palsy (CP) is a term applied to a disorder that impairs movement and posture. The effects on perception, language, and intellect are determined by the type that is diagnosed. What are the potential warning signs of CP? Select all that apply. 1.The infant's arms or legs are stiff or rigid. 2.A high risk factor for CP is very low birth weight. 3.By 8 months of age, the infant can sit without support. 4.The infant has strong head control but a limp body posture. 5.The infant has feeding difficulties, such as poor sucking and swallowing. 6.If the infant is able to crawl, only one side is used to propel himself or herself.

1256 Rationale: Options 1, 2, 5, and 6 are potential warning signs of CP. By 8 months of age, if the infant cannot sit up without support, this would be considered a potential warning sign, because this developmental task should be completed by this time. The infant with a potential diagnosis of CP has poor head control by 3 months of age, when head control should be strong.

a client's level of consciousness using the Glasgow Coma Scale. The student understands that which categories of client functioning are included in this assessment? Select all that apply. 1.Eye opening 2.Reflex response 3.Best verbal response 4.Best motor response 5.Pupil size and reaction

134 Rationale: Assessment of pupil size and reaction and reflex response are not part of the Glasgow Coma Scale. The three categories included are eye opening, best verbal response, and best motor response. Pupil assessment and reflex response is a necessary part of a total assessment of the neurological status of a client but is not part of this particular scale.

The nurse is developing a plan of care for a child who is at risk for seizures. Which interventions apply if the child has a seizure? Select all that apply. 1.Time the seizure. 2.Restrain the child. 3.Stay with the child. 4.Place the child in a prone position. 5.Move furniture away from the child. 6. Insert a padded tongue blade in the child's mouth.

135 Rationale: A seizure is a disorder that occurs as a result of excessive and unorganized neuronal discharges in the brain that activate associated motor and sensory organs. During a seizure, the child is placed on his or her side in a lateral position. Positioning on the side prevents aspiration because saliva drains out the corner of the child's mouth. The child is not restrained because this could cause injury to the child. The nurse would loosen clothing around the child's neck and ensure a patent airway. Nothing is placed into the child's mouth during a seizure because this action may cause injury to the child's mouth, gums, or teeth. The nurse would stay with the child to reduce the risk of injury and allow for observation and timing of the seizure.

Which nursing actions apply to the care of a child who is having a seizure? Select all that apply. 1. Time the seizure. 2. Restrain the child. 3. Stay with the child. 4. Insert an oral airway. 5. Place the child in a lateral side-lying position. 6. Loosen clothing around the child's neck.

1356 Rationale: During a seizure, the child is placed on his or her side in a lateral position. Positioning on the side prevents aspiration because saliva drains out the corner of the child's mouth. The child is not restrained because this could cause injury to the child. The nurse should loosen clothing around the child's neck and ensure a patent airway. Nothing is placed in the child's mouth during a seizure because this could injure the child's mouth, gums, or teeth. The nurse should stay with the child to reduce the risk of injury and allow for observation and timing of the seizure.

A client is admitted to the ambulatory care unit for an endoscopic procedure. The gastroenterologist administers midazolam 1 mg intravenously for sedation and titrates the dosage upward to 3.5 mg. The client becomes hypotensive (86/60 mm Hg), develops severe respiratory depression (SpO2 86%), and has periods of apnea. The nurse anticipates the administration of which antidote drug? 1. Benztropine 2. Flumazenil 3. Naloxone 4. Phentolamine

2

A client is brought to the emergency department with stroke symptoms that began 7 hours ago. A CT scan confirms the presence of an ischemic stroke. The client's current blood pressure is 202/108 mm Hg. Which nursing action is most appropriate? 1. Anticipate IV labetalol to keep blood pressure <140/90 mm Hg 2. Document the current findings in the client's chart 3. Prepare to administer thrombolytic therapy 4. Request a prescription for IV antiseizure medication

2

A client is receiving scheduled doses of carbidopa-levodopa. The nurse evaluates the medication as having the intended effect if which finding is noted? 1. Improvement in short-term memory 2. Improvement in spontaneous activity 3. Reduction in number of visual hallucinations 4. Reduction of dizziness with standing

2

A client was prescribed phenytoin 100 mg orally 3 times a day a month ago. The serum phenytoin level is 32 mcg/mL and the nurse notifies the health care provider (HCP). Which action is anticipated from the HCP? 1. Administer phenytoin as prescribed 2. Decrease phenytoin daily dose 3. Increase phenytoin daily dose 4. Repeat serum phenytoin level in 2 hours

2

A client was struck on the head by a baseball bat during a robbery attempt. The nurse gives this report to the oncoming nurse at shift change and conveys that the client's current Glasgow Coma Scale (GCS) score is a "10." Which client assessment is most important for the reporting nurse to include? 1. Belief that the current surroundings are a racetrack 2. GCS score was "11" one hour ago 3. Recent vital signs show blood pressure of 120/80 mm Hg and pulse of 82/min 4. Reported allergy to penicillin and vancomycin

2

A client with a brain tumor is admitted for surgery. The health care provider prescribes levetiracetam. The client asks why. What is the nurse's response? 1. "It destroys tumor cells and helps shrink the tumor." 2. "It prevents seizure development." 3. "It prevents blood clots in legs." 4. "It reduces swelling around the tumor."

2

A client with seizure disorder is prescribed a moderately high dose of phenytoin. Which teaching topic should the nurse discuss with this client? 1. Diet high in iron 2. Good oral care and dental follow-up 3. Shaving with an electric razor 4. Use of sunglasses for eye protection

2

A newborn has a large myelomeningocele. What nursing intervention is priority? 1. Assess the anus for muscle tone 2. Cover the area with a sterile, moist dressing 3. Measure the occipital frontal circumference 4. Place the newborn supine with the head of the bed elevated

2

The nurse is caring for a client with a history of headaches. The client has talked to the nurse, smiled at guests, and maintained stable vital signs. The nurse notes the following changes in the client's status. Which assessment finding is critical to report to the health care provider (HCP)?

2

The nurse is caring for a client with an acute ischemic stroke who has a blood pressure of 178/95 mm Hg. The health care provider prescribes as-needed antihypertensives to be given if the systolic pressure is >200 mm Hg. Which action by the nurse is most appropriate? 1. Give the antihypertensive medication 2. Monitor the blood pressure 3. Notify the health care provider 4. Question the prescription

2

The nurse is preparing an intravenous infusion of phenytoin (Dilantin) as prescribed by the health care provider for the client with seizures. Which solution should the nurse plan to use to dilute this medication? 1. Dextrose 5% 2. Normal saline solution 3. Lactated Ringer's solution 4. Dextrose 5% and half-normal saline (0.45%)

2

The nurse plans care for a client who has a positive Romberg test. The nurse will prioritize which intervention? 1. Monitor gag and swallowing reflexes closely 2. Provide for client assistance with ambulation 3. Provide sensory stimulation 4. Speak at a normal volume while facing the client directly

2

The nurse is caring for a child who fractured the ulna bone and had a cast applied 24 hours ago. The child tells the nurse that the arm feels like it is falling asleep. Which nursing action is appropriate? 1. Encourage the child to keep the arm elevated. 2. Report the findings to the health care provider. 3. Document the findings and reassess the arm in 4 hours. 4. Tell the child that this is normal while the cast is drying.

2 A child's complaint of pins and needles or of the extremity falling asleep needs to be reported to the health care provider. These complaints indicate the possibility of circulatory impairment and paresthesia. Paresthesia is a serious concern because paralysis can result if the problem is not corrected. The five Ps of vascular impairment are pain, pallor, pulselessness, paresthesia, and paralysis. Prompt intervention is critical if neurovascular impairment is to be prevented.

The nurse is providing home care instructions to the parents of a child with a seizure disorder. Which statement indicates to the nurse that the teaching regarding seizure disorders has been effective? 1. "We're glad we only have to give our child the medication for 30 days." 2. "We will make appointments for follow-up blood work and care as directed." 3. "We're glad there are no side effects from taking the antiseizure medications." 4. "After our child has been seizure free for 1 month, we can discontinue the medication."

2 Antiseizure medications are continued for a prolonged time even if seizures are controlled. Periodic reevaluation of the child is important to assess the continued effectiveness of the medication, to check serum medication levels, and to determine the need to alter the dosage if indicated. Antiseizure medications have potential side effects, and parents should be informed of such effects specific to the medication the child will be taking. Withdrawal of medication follows a predesigned protocol, usually begun when the child has been seizure free for at least 2 years. When a medication is discontinued, the dosage should be reduced gradually over 1 to 2 weeks.

A child must wear a brace for correction of scoliosis. The nurse creates a plan of care knowing the child is at risk for which problem? 1. Inability to ambulate 2. Breaks in skin integrity 3. Decreased oxygenation 4. Delayed growth and development

2 Braces for treatment of scoliosis usually are worn 16 to 23 hours a day. The skin should be kept clean and dry and inspected for signs of redness or breakdown. Therefore, breaks in skin integrity are the client problem that should be included in this child's plan of care. The brace assists with posture, so mobility is not an issue. The brace does not compromise the respiratory status, so oxygenation is not decreased. The child will not have a risk for delayed growth and development because normal developmental milestones can be met while wearing a brace.

A child has just returned from surgery and has a hip spica cast. What is the nurse's priority action for this client? 1. Elevate the head of the bed. 2. Assess the circulatory status. 3. Abduct the hips using pillows. 4. Turn the child onto the right side.

2 During the first few hours after a cast is applied, the chief concern is swelling, which may cause the cast to act as a tourniquet and obstruct circulation. Therefore, circulatory assessment is a high priority. Elevating the head of the bed of a child in a hip spica cast would cause discomfort. Using pillows to abduct the hips is not necessary because a hip spica cast immobilizes the hip and knee. Turning the child from one side to the other at least every 2 hours is important because it allows the body cast to dry evenly and prevents complications related to immobility; however, it is not a higher priority than checking circulation.

An infant with a diagnosis of hydrocephalus is scheduled for surgery. Which is the priority nursing intervention in the preoperative period? 1. Test the urine for protein. 2. Reposition the infant frequently. 3. Provide a stimulating environment. 4. Assess blood pressure every 15 minutes.

2 Hydrocephalus occurs as a result of an imbalance of cerebrospinal fluid absorption or production that is caused by malformations, tumors, hemorrhage, infections, or trauma. It results in head enlargement and increased intracranial pressure (ICP). In infants with hydrocephalus, the head grows at an abnormal rate, and if the infant is not repositioned frequently, pressure ulcers can occur on the back and side of the head. An egg crate mattress under the head is also a nursing intervention that can help to prevent skin breakdown. Proteinuria is not specific to hydrocephalus. Stimulus should be kept at a minimum because of the increase in ICP. It is not necessary to check the blood pressure every 15 minutes.

The nurse should plan to place a child who had a medulloblastoma brain tumor (infratentorial) removed in which position postoperatively? 1. Trendelenburg's 2. Flat, on either side 3. With the head of the bed elevated above heart level 4. With the head of the bed elevated in low Fowler's position

2 If an infratentorial tumor has been removed, the child is positioned flat on either side. The pillow is placed behind the child's back for comfort and for maintaining the position. The pillow is not placed behind the head because when the pillow is behind the head, proper alignment is not maintained, and this misalignment can impair circulation. The child should never be placed in a Trendelenburg's position (head down) because this position increases intracranial pressure. The head is elevated when the tumor is a supratentorial one. Remember though that the surgeon's prescription for positioning is always followed.

The nurse is caring for a client receiving mannitol (Osmitrol) via intravenous (IV) infusion. A vial is sent from the pharmacy, and in preparing the medication the nurse notes that the vial contains crystals. What is the most appropriate nursing action? 1. Discard the vial. 2. Place the vial in warm water. 3. Send the vial back to the pharmacy. 4. Shake the vial to dissolve the crystals.

2 Rationale: Crystals form in a mannitol solution if the solution is cooled, but they will quickly dissolve if the container is placed in warm water and then cooled to body temperature before administration. However, if crystals remain after the warming procedure, the medication should not be used and should be returned to the pharmacy. The nurse would not discard the medication. The medication is not initially returned to the pharmacy because it is not defective. Shaking the vial should not be done and will not dissolve the crystals. Test-Taking Strategy: Note the strategic words most appropriate.Knowledge regarding the administration of mannitol is required to answer this question. It is necessary to know that crystals may form in the mannitol solution and that the solution is placed in warm water to dissolve the crystals. Remember that not all medications should be placed in warm water if crystals are noted; however, with this medication, it is acceptable.

SCI not TBI Dexamethasone (Decadron) intravenously is prescribed for the client with cerebral edema. The nurse prepares the medication for administration and plans to perform which action? 1. Mix the medication in 1000 mL of 5% dextrose. 2. Prepare an undiluted direct injection of the medication. 3. Mix the medication in 100 mL of lactated Ringer's solution. 4. Dilute the medication in lactated Ringer's solution and administer as a direct injection.

2 Rationale: Dexamethasone may be given by direct intravenous injection or intravenous infusion. Dexamethasone may be mixed with 0.9% sodium chloride or 5% dextrose. If administered as an infusion, a minimum amount of diluting solution is needed.

The nurse has given medication instructions to a client receiving phenytoin (Dilantin). Which statement indicates that the client has an adequate understanding of the instructions? 1. "Alcohol is not contraindicated while taking this medication." 2. "Good oral hygiene is needed, including brushing and flossing." 3. "The medication dose may be self-adjusted, depending on side effects." 4. "The morning dose of the medication should be taken before a serum drug level is drawn."

2 Rationale: Typical anticonvulsant medication instructions include taking the prescribed daily dosage to keep the blood level of the drug constant and having a sample drawn for serum drug level determination before taking the morning dose. The client is taught not to stop the medication abruptly, to avoid alcohol, to check with a health care provider before taking over-the-counter medications, to avoid activities in which alertness and coordination are required until medication effects are known, to provide good oral hygiene, and to obtain regular dental care. The client should also wear a Medic-Alert bracelet.

Russell's traction is prescribed for a child with a lower leg fracture. The mother of the child asks the nurse about the purpose of the traction. The nurse explains to the mother that which is the primary action of this type of traction? 1. Relieves the child's pain 2. Reduces or realigns a fracture site 3. Provides a form of restraint for the child 4. Keeps the child from moving around in bed

2 Russell's traction uses skin traction to realign a fracture in the lower extremity and to immobilize the hip and knee in a flexed position. It is important to keep the hip flexion at the prescribed angle to prevent fracture malalignment. The traction may also relieve pain by reducing muscle spasms, but this is not the primary reason for this traction. The child can still move in bed with some restriction as a result of the traction. Traction is never used to restrain a child.

A 4-year-old child sustains a fall at home. After an x-ray examination, the child is determined to have a fractured arm and a plaster cast is applied. The nurse provides instructions to the parents regarding care for the child's cast. Which statement by the parents indicates a need for further instruction? 1. "The cast may feel warm as the cast dries." 2. "I can use lotion or powder around the cast edges to relieve itching." 3. "A small amount of white shoe polish can touch up a soiled white cast." 4. "If the cast becomes wet, a blow drier set on the cool setting may be used to dry the cast."

2 Teaching about cast care is essential to prevent complications from the cast. The parents need to be instructed not to use lotion or powders on the skin around the cast edges or inside the cast. Lotions or powders can become sticky or caked and cause skin irritation. Options 1, 3, and 4 are appropriate statements.

The critical care nurse is assessing a client whose baseline Glasgow Coma Scale (GCS) score in the emergency department was 5. The current GCS score is 3. What is the nurse's best interpretation of this finding? 1. The client's condition is improving. 2. The client's condition is deteriorating. 3. The client will need intubation and mechanical ventilation. 4. The client's medication regime will need adjustments.

2 The GCS is used in many acute care settings to establish baseline data in these areas: eye opening, motor response, and verbal response. The client is assigned a numeric score for each of these areas. The lower the score, the lower the client's neurologic function. A decrease of 2 or more points in the Glasgow Coma Scale score total is clinically significant and should be communicated to the health care provider immediately.

The nurse is preparing to admit a client with a seizure disorder. Which action can be assigned to an LPN/LVN? 1. Completing the admission assessment 2. Setting up oxygen and suction equipment 3. Placing a padded tongue blade at the bedside 4. Padding the side rails before the client arrives

2 The LPN/LVN scope of practice includes setting up the equipment for oxygen and suctioning. The RN should perform the complete initial assessment. Controversy exists as to whether padded side rails actually provide safety, and their use may embarrass the client and family. Tongue blades should not be at the bedside and should never be inserted into the client's mouth after a seizure begins.

The RN notes that a client with myasthenia gravis has an elevated temperature (102.2°F [39°C]), an increased heart rate (120 beats/min), and a rise in blood pressure (158/94 mm Hg) and is incontinent of urine and stool. What is the nurse's best action at this time? 1. Administer an acetaminophen suppository. 2. Notify the health care provider immediately. 3. Recheck vital signs in 1 hour. 4. Reschedule the client's physical therapy.

2 The changes that the RN notes are characteristic of myasthenic crisis, which often follows some type of infection. The client is at risk for inadequate respiratory function. In addition to notifying the health care provider or Rapid Response Team, the nurse should carefully monitor the client's respiratory status. The client may need intubation and mechanical ventilation.

A 70-year-old client with alcoholism who has become lethargic, confused, and incontinent during the last week is admitted to the emergency department. His wife tells the nurse that he fell down the stairs about a month ago but that "he didn't have a scratch afterward." Which collaborative interventions will the nurse implement first? 1. Place the client on the hospital alcohol withdrawal protocol. 2. Transport the client to the radiology department for a computed tomography (CT) scan. 3. Make a referral to the social services department. 4. Give the client phenytoin 100 mg PO.

2 The client's history and assessment data indicate that he may have a chronic subdural hematoma. The priority goal is to obtain a rapid diagnosis and send the client to surgery to have the hematoma evacuated. The other interventions also should be implemented as soon as possible, but the initial nursing activities should be directed toward diagnosis and treatment of any intracranial lesion.

A client with a spinal cord injury (SCI) reports sudden severe throbbing headache that started a short time ago. Assessment of the client reveals increased blood pressure (168/94 mm Hg) and decreased heart rate (48 beats/min), diaphoresis, and flushing of the face and neck. What action should the nurse take first? 1. Administer the ordered acetaminophen. 2. Check the Foley tubing for kinks or obstruction. 3. Adjust the temperature in the client's room. 4. Notify the health care provider about the change in status.

2 The client's signs and symptoms are characteristic of autonomic dysreflexia, a neurologic emergency that must be promptly treated to prevent a hypertensive stroke. The cause of this syndrome is noxious stimuli, most often a distended bladder or constipation, so checking for poor catheter drainage, bladder distention, and fecal impaction is the first action that should be taken. Adjusting the room temperature may be helpful because too cool a temperature in the room may contribute to the problem. Acetaminophen will not decrease the autonomic dysreflexia that is causing the client's headache. Notifying the health care provider may be necessary if nursing actions do not resolve symptoms.

A client who has Alzheimer disease is hospitalized with new-onset angina. Her spouse tells the nurse that he does not sleep well because he needs to be sure the client does not wander during the night. He insists on checking each of the medications the nurse gives the client to be sure they are "the same pills she takes at home." Based on this information, which nursing problem is mostappropriate for this client? 1. Acute client confusion 2. Care provider role stress 3. Increased risk for falls 4. Noncompliance with therapeutic plan

2 The husband's statement about lack of sleep and concern about whether his wife is receiving the correct medications are behaviors that support the problem of care provider role stress. The husband's statements about how he monitors the client and his concern with medication administration do not indicate difficulty complying with the therapeutic plan. The client may be confused, but the nurse would need to gather more data, and this is not the main focus of the husband's concerns. Falls are not an immediate concern at this time.

The nurse is caring for a child with a fracture who is placed in skeletal traction. The nurse should monitor for which sign of a serious complication associated with this type of traction? 1. Lack of appetite 2. Elevated temperature 3. Decrease in the urinary output 4. Increase in the blood pressure

2 The most serious complication associated with skeletal traction is osteomyelitis, an infection involving the bone. Organisms gain access to the bone systemically or through the opening created by the metal pins or wires used with the traction. Osteomyelitis may occur with any open fracture. Clinical manifestations include complaints of localized pain, swelling, warmth, tenderness, an unusual odor from the fracture site, and an elevated temperature. The remaining options are not specifically associated with osteomyelitis.

A child sustains a fall at home and is brought to the hospital emergency department by the child's mother. After a radiographic examination, the child is determined to have a fractured arm, and a plaster cast is applied. The nurse provides instructions to the mother regarding neurocirculatory assessment and function. Which statement by the mother indicates a need for further instruction? 1. "I'll need to check her skin twice a day at the cast edges." 2. "If her hand gets real cool and pale, I can apply the heating pad to it." 3. "For the first couple of days, I should try to keep her hand higher than her heart most of the time using pillows." 4. "If she seems way too fussy and her arm is painful even after I've given her the pain medication, it might be a problem, and I should call you for help to decide on what is happening."

2 The mother needs to understand that compartment syndrome is a complication of fracture and casting and can result in permanent limb damage as a result of pressure-related tissue necrosis. The extremity is elevated to prevent swelling, and the health care provider is notified immediately if any signs of neurovascular impairment develop. Cold fingers could indicate neurovascular impairment and should be reported. A heating pad is not applied to the cast or fingers. Skin edges are checked to monitor for irritation and skin breakdown.

The nurse is caring for a child after surgical removal of a brain tumor. The nurse should assess the child for which sign that would indicate that brainstem involvement occurred during the surgical procedure? 1. Inability to swallow 2. Elevated temperature 3. Altered hearing ability 4. Orthostatic hypotension

2 Vital signs and neurological status are assessed frequently after surgical removal of a brain tumor. Special attention is given to the child's temperature, which may be elevated because of hypothalamic or brainstem involvement during surgery. A cooling blanket should be in place on the bed or readily available if the child becomes hyperthermic. Inability to swallow and altered hearing ability are related to functional deficits after surgery. Orthostatic hypotension is not a common clinical manifestation after brain surgery. An elevated blood pressure and widened pulse pressure may be associated with increased intracranial pressure, which is a complication after brain surgery, but they are not related to brainstem involvement.

he nurse is caring for the client with increased intracranial pressure. The nurse would note which trend in vital signs if the intracranial pressure is rising? 1.Increasing temperature, increasing pulse, increasing respirations, decreasing blood pressure 2.Increasing temperature, decreasing pulse, decreasing respirations, increasing blood pressure 3.Decreasing temperature, decreasing pulse, increasing respirations, decreasing blood pressure 4.Decreasing temperature, increasing pulse, decreasing respirations, increasing blood pressure

2 Rationale: A change in vital signs may be a late sign of increased intracranial pressure. Trends include increasing temperature and blood pressure and decreasing pulse and respirations. Respiratory irregularities also may occur.

A nurse is evaluating the neurological status of a client. To assess the function of the limbic system, the nurse should gather data about which item? 1.Experience of pain 2.Affect or emotions 3.Response to verbal stimuli 4.Insight, judgment, and planning

2 Rationale: Affect and emotions are part of the role of the limbic system and involve both hemispheres of the brain. Pain is a complex experience involving several areas of the central nervous system. The response to verbal stimuli is part of the level of consciousness, which is under the control of the reticular activating system and both cerebral hemispheres. Insight, judgment, and planning are part of the functions of the frontal lobes of the brain in conjunction with association fibers connecting to other areas of the cerebrum.

The student nurse develops a plan of care for a client after a lumbar puncture. The nursing instructor corrects the student if the student documents which incorrect intervention in the plan? 1.Maintain the client in a flat position. 2.Restrict fluid intake for a period of 2 hours. 3.Assess the client's ability to void and move the extremities. 4.Inspect the puncture site for swelling, redness, and drainage.

2 Rationale: After the lumbar puncture the client remains flat in bed for at least 2 hours, depending on the health care provider's prescriptions. A liberal fluid intake is encouraged to replace the cerebrospinal fluid removed during the procedure, unless contraindicated by the client's condition. The nurse checks the puncture site for redness and drainage and assesses the client's ability to void and move the extremities.

The nurse is assessing a client's gait, which is characterized by unsteadiness and staggering steps. The nurse determines the presence of which type of gait? 1.Spastic 2.Ataxic 3.Festinating 4. Dystrophic or broad-based

2 Rationale: An ataxic gait is characterized by unsteadiness and staggering. A spastic gait is characterized by stiff, short steps with the legs held together, hip and knees flexed, and toes that catch and drag. A festinating gait is best described as walking on the toes with an accelerating pace. A dystrophic or broad-based gait is seen as waddling, with the weight shifting from side to side and the legs far apart.

The nurse is providing home care instructions to the parents of a child with a seizure disorder. Which statement indicates to the nurse that the teaching regarding seizure disorders has been effective? 1. "We're glad we only have to give our child the medication for 30 days." 2. "We will make appointments for follow-up blood work and care as directed." 3. "We're glad there are no side effects from taking the antiseizure medications." 4. "After our child has been seizure free for 1 month, we can discontinue the medication."

2 Rationale: Antiseizure medications are continued for a prolonged time even if seizures are controlled. Periodic reevaluation of the child is important to assess the continued effectiveness of the medication, check serum medication levels, and determine the need to alter the dosage if indicated. Antiseizure medications have potential side effects, and parents should be informed of such effects specific to the medication the child will be taking. Withdrawal of medication follows a predesigned protocol, usually begun when the child has been seizure free for at least 2 years. When a medication is discontinued, the dosage should be reduced gradually over 1 to 2 weeks.

The home care nurse is visiting a client with a diagnosis of Parkinson's disease. The client is taking benztropine mesylate (Cogentin) orally daily. The nurse provides information to the spouse regarding the side effects of this medication and should tell the spouse to report which side effect if it occurs? 1. Shuffling gait 2. Inability to urinate 3. Decreased appetite 4. Irregular bowel movements

2 Rationale: Benztropine mesylate is an anticholinergic, which causes urinary retention as a side effect. The nurse would instruct the client or spouse about the need to monitor for difficulty with urinating, a distended abdomen, infrequent voiding in small amounts, and overflow incontinence. Options 1, 3, and 4 are unrelated to the use of this medication.

A client has a high level of carbon dioxide (CO2) in the bloodstream, as measured by arterial blood gases. A nurse reviewing the client's record plans care, knowing that a high CO2 level will have which effect on circulation to the brain? 1.It will cause arteriovenous shunting. 2.It will cause vasodilation of blood vessels in the brain. 3. It will cause blood vessels in the circle of Willis to collapse. 4. It will cause hyperresponsiveness of blood vessels in the brain.

2 Rationale: CO2 is one of the metabolic end products that can alter the tone of the blood vessels in the brain. High CO2 levels cause vasodilation, which may cause headache, whereas low CO2 levels cause vasoconstriction, which may cause lightheadedness. The statements included in the other options are incorrect effects.

A client with recent-onset Bell's palsy is upset and crying about the change in facial appearance. The nurse plans to support the client emotionally by making which statement to the client? 1. This is caused by a small tumor, which can be removed easily. 2.This is not a brain attack (stroke), and many clients recover in 3 to 5 weeks. 3. This is a temporary problem, with treatment similar to that for migraine headaches. 4. This is similar to a brain attack (stroke), but all symptoms will reverse without treatment.

2 Rationale: Clients with Bell's palsy should be reassured that they have not experienced a brain attack (stroke) and that symptoms often disappear spontaneously in 3 to 5 weeks. The client is given supportive treatment for symptoms. Bell's palsy usually is not caused by a tumor, and the treatment is not similar to that for migraine headaches.

The nurse in the health care clinic is providing medication instructions to a client with a seizure disorder who will be taking divalproex sodium (Depakote). The nurse should instruct the client about the importance of returning to the clinic for monitoring of which laboratory study? 1.Electrolyte panel 2. Liver function studies 3.Renal function studies 4. Blood glucose level determination

2 Rationale: Divalproex sodium, an anticonvulsant, can cause fatal hepatotoxicity. The nurse should instruct the client about the importance of monitoring the results of liver function studies and ammonia level determinations. Options 1, 3, and 4 are not studies that are required with the use of this medication

An infant with a diagnosis of hydrocephalus is scheduled for surgery. Which is the priority nursing intervention in the preoperative period? 1.Test the urine for protein. 2.Reposition the infant frequently. 3.Provide a stimulating environment. 4.Assess blood pressure every 15 minutes.

2 Rationale: Hydrocephalus occurs as a result of an imbalance of cerebrospinal fluid absorption or production that is caused by malformations, tumors, hemorrhage, infections, or trauma. It results in head enlargement and increased intracranial pressure. In infants with hydrocephalus, the head grows at an abnormal rate, and if the infant is not repositioned frequently, pressure ulcers can occur on the back and side of the head. An egg crate mattress under the head is also a nursing intervention that can help prevent skin breakdown. Proteinuria is not specific to hydrocephalus. Stimulus should be kept at a minimum because of the increase in intracranial pressure. It is not necessary to check the blood pressure every 15 minutes.

The nurse is assessing the function of cranial nerve XII in a client who sustained a stroke. To assess function of this nerve, which action should the nurse ask the client to perform? 1.Extend the arms. 2.Extend the tongue. 3.Turn the head toward the nurse's arm. 4.Focus the eyes on the object held by the nurse.

2 Rationale: Impairment of cranial nerve XII can occur with a stroke. To assess the function of cranial nerve XII (the hypoglossal nerve), the nurse would assess the client's ability to extend the tongue. Options 1, 3, and 4 do not test the function of cranial nerve XII.

The nurse assigned to the care of an unconscious client is making initial daily rounds. On entering the client's room, the nurse observes that the client is lying supine in bed, with the head of the bed elevated approximately 5 degrees. The nasogastric tube feeding is running at 70 mL/hr, as prescribed. The nurse assesses the client and auscultates adventitious breath sounds. Which judgment should the nurse formulate for the client? 1.Impaired nutritional intake 2.Increased risk for aspiration 3.Increased likelihood for injury 4. Susceptibility to fluid volume deficit

2 Rationale: Increased risk for aspiration is a condition in which an individual is at risk for entry of gastrointestinal (GI) secretions, oropharyngeal secretions, or solids or fluids into tracheobronchial passages. Conditions that place the client at risk for aspiration include reduced level of consciousness, depressed cough and gag reflexes, and feeding via a GI tube. There is no information in the question indicating that option 1, 3, or 4 is a concern.

The nurse is assessing the motor function of an unconscious client. The nurse should plan to use which technique to test the client's peripheral response to pain? 1.Sternal rub 2.Nail bed pressure 3.Pressure on the orbital rim 4.Squeezing of the sternocleidomastoid muscle

2 Rationale: Motor testing in the unconscious client can be done only by testing response to painful stimuli. Nail bed pressure tests a basic peripheral response. Cerebral responses to pain are tested using a sternal rub, placing upward pressure on the orbital rim, or squeezing the clavicle or sternocleidomastoid muscle.

The nurse is caring for a client who begins to experience seizure activity while in bed. Which action by the nurse is contraindicated? 1.Loosening restrictive clothing 2.Restraining the client's limbs 3.Removing the pillow and raising padded side rails 4. Positioning the client to the side, if possible, with the head flexed forward

2 Rationale: Nursing actions during a seizure include providing for privacy, loosening restrictive clothing, removing the pillow and raising padded side rails in the bed, and placing the client on one side with the head flexed forward, if possible, to allow the tongue to fall forward and facilitate drainage. The limbs are never restrained because the strong muscle contractions could cause the client harm. If the client is not in bed when seizure activity begins, the nurse lowers the client to the floor, if possible, protects the head from injury, and moves furniture that may injure the client.

A nurse is assessing a client's muscle strength and notes that when asked, the client cannot maintain his or her hands in a supinated position with the arms extended and eyes closed. How should the nurse correctly document this finding on the medical record? 1. Client is demonstrating ataxia. 2. Client is exhibiting pronator drift. 3. Client appears to have nystagmus. 4. Client examination reveals hyperreflexia.

2 Rationale: Pronator drift occurs when a client cannot maintain his or her hands in a supinated position with the arms extended and eyes closed. This assessment may be done to detect small changes in muscle strength that might not otherwise be noted. Ataxia is a disturbance in gait. Hyperreflexia is an excessive reflex action. Nystagmus is characterized by fine, involuntary eye movements. It can occur with neurological disease or as a side effect of selected medications.

The nurse is performing an assessment on a client with Guillain-Barré syndrome. The nurse determines that which finding would be of most concern? 1.Difficulty articulating words 2.Lung vital capacity of 10 mL/kg 3.Paralysis progressing from the toes to the waist 4.A blood pressure (BP) decrease from 110/78 to 102/70 mm Hg

2 Rationale: Respiratory compromise is a major concern in clients with Guillain-Barré syndrome. Clients often are intubated and mechanically ventilated when the vital capacity is less than 15 mL/kg. Options 1 and 3 are expected, depending on the degree of paralysis that occurs. Although orthostatic hypotension is a problem with these clients, the BP drop in option 4 is less than 10 mm Hg and is not significant.

The clinic nurse is reviewing the record of a client scheduled to be seen in the clinic. The nurse notes that the client is taking selegiline hydrochloride (Eldepryl). The nurse suspects that the client has which disorder? 1. Diabetes mellitus 2. Parkinson's disease 3. Alzheimer's disease 4. Coronary artery disease

2 Rationale: Selegiline hydrochloride is an antiparkinsonian medication. The medication increases dopaminergic action, assisting in the reduction of tremor, akinesia, and the rigidity of parkinsonism. This medication is not used to treat diabetes mellitus, Alzheimer's disease, or coronary artery disease.

A client with a neurological deficit is able, with eyes closed, to identify a set of keys placed in his or her hands. A nurse observing the client interprets this to mean that which area of the brain is intact? 1. Frontal lobe 2. Parietal lobe 3. Temporal lobe 4. Occipital lobe

2 Rationale: The ability to distinguish an object by touch is called stereognosis, which is a function of the right parietal area. The parietal lobe of the brain is responsible for spatial orientation and awareness of sizes and shapes. The left parietal area is responsible for mathematics and right-left orientation. The other lobes of the brain are not responsible for this function.

A client with myasthenia gravis is having difficulty with airway clearance and difficulty with maintaining an effective breathing pattern. The nurse should keep which items available at the client's bedside? 1.Oxygen and metered-dose inhaler 2.Ambu bag and suction equipment 3.Pulse oximeter and cardiac monitor 4.Incentive spirometer and cough pillow

2 Rationale: The client with myasthenia gravis may experience episodes of respiratory distress if excessively fatigued or with development of myasthenic or cholinergic crisis. For this reason, an Ambu bag, intubation tray, and suction equipment should be available at the bedside.

The nurse is documenting nursing observations in the record of a client who experienced a tonic-clonic seizure. Which clinical manifestation did the nurse most likely note in the clonic phase of the seizure? 1.Body stiffening 2.Spasms of the entire body 3.Sudden loss of consciousness 4.Brief flexion of the extremities

2 Rationale: The clonic phase of a seizure is characterized by alternating spasms and momentary muscular relaxation of the entire body, accompanied by strenuous hyperventilation. The face is contorted and the eyes roll. Excessive salivation results in frothing from the mouth. The tongue may be bitten, the client sweats profusely, and the pulse is rapid. The clonic jerking subsides by slowing in frequency and losing strength of contractions over a period of 30 seconds. Options 1, 3, and 4 identify the tonic phase of a seizure.

A client is diagnosed with Bell's palsy. The nurse assessing the client expects to note which symptom? 1.A symmetrical smile 2.Difficulty closing the eyelid on the affected side 3.Narrowing of the palpebral fissure on the affected side 4.Paroxysms of excruciating pain in the lips and cheek on the affected side

2 Rationale: The facial drooping associated with Bell's palsy makes it difficult for the client to close the eyelid on the affected side. A widening of the palpebral fissure (the opening between the eyelids) and an asymmetrical smile are seen with Bell's palsy. Paroxysms of excruciating pain are characteristic of trigeminal neuralgia.

A nurse notes that a client who has suffered a brain injury has an adequate heart rate, blood pressure, fluid balance, and body temperature. The nurse concludes that which area of the client's brain is functioning adequately? 1. Thalamus 2. Hypothalamus 3. Limbic system 4. Reticular activating system

2 Rationale: The hypothalamus is responsible for autonomic nervous system functions, such as heart rate, blood pressure, temperature, and fluid and electrolyte balance (among others). The thalamus acts as a relay station for sensory and motor information. The limbic system is responsible for emotions. The reticular activating system is responsible for the sleep-wake cycle.

The nurse is reviewing the record for a client seen in the health care clinic and notes that the health care provider has documented a diagnosis of amyotrophic lateral sclerosis (ALS). Which initial clinical manifestation of this disorder should the nurse expect to see documented in the record? 1. Muscle wasting 2. Mild clumsiness 3. Altered mentation 4. Diminished gag reflex

2 Rationale: The initial symptom of ALS is a mild clumsiness, usually noted in the distal portion of one extremity. The client may complain of tripping and drag one leg when the lower extremities are involved. Mentation and intellectual function usually are normal. Diminished gag reflex and muscle wasting are not initial clinical manifestations.

A client has a neurological deficit involving the limbic system. Which assessment finding is specific to this type of deficit? 1.Is disoriented to person, place, and time 2.Affect is flat, with periods of emotional lability 3.Cannot recall what was eaten for breakfast today 4. Demonstrates inability to add and subtract; does not know who is the president of the United States

2 Rationale: The limbic system is responsible for feelings (affect) and emotions. Calculation ability and knowledge of current events relate to function of the frontal lobe. The cerebral hemispheres, with specific regional functions, control orientation. Recall of recent events is controlled by the hippocampus.

A client is admitted with an exacerbation of multiple sclerosis. The nurse is assessing the client for possible precipitating risk factors. Which factor, if reported by the client, should the nurse identify as being unrelated to the exacerbation? 1.Annual influenza vaccination 2.Ingestion of increased fruits and vegetables 3.An established routine of walking 2 miles each evening 4.A recent period of extreme outside ambient temperatures

2 Rationale: The onset or exacerbation of multiple sclerosis can be preceded by a number of different factors, including physical stress (e.g., vaccination, excessive exercise), emotional stress, fatigue, infection, physical injury, pregnancy, extremes in environmental temperature, and high humidity. No methods of primary prevention are known. Intake of fruits and vegetables is an unrelated item.

A client with Parkinson's disease is at risk for falls because of an abnormal gait. The nurse assesses the client, expecting to observe which type of gait? 1. Unsteady and staggering 2. Shuffling and propulsive 3. Broad-based and waddling 4.Accelerating with walking on the toes

2 Rationale: The parkinsonian gait is characterized by short, accelerating, shuffling steps. The client leans forward with the head, hips, and knees flexed and has difficulty starting and stopping. An ataxic gait is unsteady and staggering. A dystrophic gait is broad-based and waddling. A festinating gait is accelerating with walking on the toes.

A nurse is testing the spinal reflexes of a client during neurological assessment. Which reflex will assist in determining that the client has an adequate spinal reflex? 1.Cough reflex 2.Withdrawal reflex 3.Accommodation reflex 4.Munroe-Kellie reflex

2 Rationale: The withdrawal reflex is one of the spinal reflexes. It is an abrupt withdrawal of a body part in response to painful or injurious stimuli. The cough reflex is a brainstem-associated reflex. Accommodation reflex is associated with cranial nerve III and is part of the ocular motor system. Munroe-Kellie is not a reflex; it is a doctrine or a hypothesis addressing the cerebral volume relationships among the brain, the cerebrospinal fluid, and intracranial blood and their cumulative impact on intracranial pressure.

The nursing student is caring for a child with increased intracranial pressure. On review of the chart, the student nurse notes that a transtentorial herniation has occurred. The nursing instructor asks the student about this type of herniation. Which statement by the student indicates a need for further research about this condition? 1."The herniation can be unilateral or bilateral." 2."It involves only anterior portions of the brain." 3."It can cause death if large amounts of tissue are involved." 4."The brain herniates downward and around the tentorium cerebelli."

2 Rationale: Transtentorial herniation occurs when part of the brain herniates downward and around the tentorium cerebelli. It can be unilateral or bilateral and may involve anterior or posterior portions of the brain. If a large amount of tissue is involved, the risk of death is increased because vital brain structures are compressed and become unable to perform their function.

A client has sustained damage to Wernicke's area in the temporal lobe from a stroke (brain attack). The nurse anticipates that the client will have difficulty with which function? 1.Articulating words 2. Understanding language 3. Moving one side of the body 4. Recalling events in the remote past

2 Rationale: Wernicke's area consists of a small group of cells in the temporal lobe whose function is the understanding of language. Damage to Broca's area is responsible for aphasia. The motor cortex in the precentral gyrus controls voluntary motor activity. The hippocampus is responsible for the storage of memory.

The nurse has given the client with Bell's palsy instructions on preserving muscle tone in the face and preventing denervation. The nurse determines that the client needs additional teaching if the client makes which statements? 1."I will perform facial exercises." 2."I will expose my face to cold to decrease the pain." 3."I will massage my face with a gentle upward motion." 4."I will wrinkle my forehead, blow out my cheeks, and whistle frequently."

2 ationale: Exposure to cold or drafts is avoided in Bell's palsy because it can cause discomfort. Prevention of muscle atrophy with Bell's palsy is accomplished with facial massage, facial exercises, and electrical nerves stimulation. Local application of heat to the face may improve blood flow and provide comfort.

A client with a T4 spinal cord injury has a severe throbbing headache and appears flushed and diaphoretic. Which priority interventions should the nurse perform? Select all that apply. 1. Administer an analgesic as needed 2. Determine if there is bladder distention 3. Measure the client's blood pressure 4. Place the client in the Sims' position 5. Remove constrictive clothing

2,3,5

The nurse is caring for a client after a motor vehicle accident. The client's injuries include 2 fractured ribs and a concussion. The nurse notes which of the following as expected neurological changes for the client with a concussion? Select all that apply. 1. Asymmetrical pupillary constriction 2. Brief loss of consciousness 3. Headache 4. Loss of vision 5. Retrograde amnesia

2,3,5

The nurse is preparing for the admission of a 9-year-old client with new-onset tonic-clonic seizures. It is important for the nurse to ensure that what is in the room? Select all that apply. 1. Oral bite prevention device 2. Oxygen delivery system 3. Padding on the bed siderails 4. Soft arm and leg restraints 5. Suction equipment

2,3,5

The nurse moves a finger in a horizontal and vertical motion in front of the client's face while directing the client to follow the finger with the eyes. Which cranial nerves is the nurse assessing? Select all that apply. 1. II 2. III 3. IV 4. V 5. VI

2,3,5

The nurse develops a plan of care for a client with a brain aneurysm who will be placed on aneurysm precautions. Which interventions should be included in the plan? Select all that apply. 1.Leave the lights on in the client's room at night. 2.Place a blood pressure cuff at the client's bedside. 3.Close the shades in the client's room during the day. 4.Allow the client to drink one cup of caffeinated coffee a day. 5.Allow the client to ambulate four times a day with assistance.

2,3. Aneurysm precautions include placing the client on bed rest in a quiet setting. The use of lights is kept to a minimum to prevent environmental stimulation. The nurse should monitor the blood pressure and note any changes that could indicate rupture. Any activity, such as pushing, pulling, sneezing, or straining, that increases the blood pressure or impedes venous return from the brain is prohibited. The nurse provides physical care to minimize increases in blood pressure. Visitors, radio, television, and reading materials are restricted or limited. Stimulants, such as nicotine and coffee and other caffeine-containing products, are prohibited. Decaffeinated coffee or tea may be used. * Focus on the client's diagnosis and the subject, measures to prevent aneurysm rupture. Read each option in terms of whether it would cause stimulation or increased intracranial pressure, which can lead to rupture. This will direct you to the correct options.

A nurse is assisting in the care of a client being discharged on phenytoin (Dilantin), 100 mg three times daily, for seizure control. When providing client teaching about this medication, the nurse should be sure to include which points? Select all that apply. 1. Break the capsules so they are easier to swallow. 2. Use a soft toothbrush while taking this medication. 3. If a dose is missed, just wait until the next one is due. 4. Alcohol should be avoided while taking this medication. 5. The medication may turn the client's urine pink, red, or brown. 6. Sore throat is a common side effect of the medication and is nothing to worry about.

2,4,5 Rationale: Phenytoin is an anticonvulsant that can cause gingival hyperplasia, as well as bleeding, swelling, and tenderness of the gums. The client should use good oral hygiene, gum massage, and have regular dental checkups. Alcohol interferes with the absorption of phenytoin, so it should be avoided. Change in the color of the urine is a normal reaction. A sore throat, fever, glandular swelling, or any skin reaction indicates hematological toxicity and needs to be reported.

The home care nurse is visiting a client with a diagnosis of Parkinson's disease. The client is taking benztropine mesylate (Cogentin) orally daily. The nurse provides information to the spouse regarding the side effects of this medication and should tell the spouse to report which side effect if it occurs? 1.Shuffling gait 2.Inability to urinate 3.Decreased appetite 4.Irregular bowel movements

2. Benztropine mesylate is an anticholinergic, which causes urinary retention as a side effect. The nurse would instruct the client or spouse about the need to monitor for difficulty with urinating, a distended abdomen, infrequent voiding in small amounts, and overflow incontinence. Options 1, 3, and 4 are unrelated to the use of this medication. * Focus on the subject and recall that benztropine mesylate is an anticholinergic and that these types of medications cause urinary retention. This will direct you to the correct option.

The nurse in the health care clinic is providing medication instructions to a client with a seizure disorder who will be taking divalproex sodium (Depakote). The nurse should instruct the client about the importance of returning to the clinic for monitoring of which laboratory study? 1.Electrolyte panel 2.Liver function studies 3.Renal function studies 4.Blood glucose level determination

2. Divalproex sodium, an anticonvulsant, can cause fatal hepatotoxicity. The nurse should instruct the client about the importance of monitoring the results of liver function studies and ammonia level determinations. Options 1, 3, and 4 are not studies that are required with the use of this medication. * Focus on the subject, the toxic effects of divalproex sodium. Recalling that divalproex sodium can lead to hepatotoxicity will direct you to the correct option.

The nurse should place a child who had a medulloblastoma brain tumor (infratentorial) removed in which position postoperatively? Trendelenburg's 2. Flat, on either side 3. With the head of the bed elevated above heart level 4. With the head of the bed elevated in low Fowler's position

2. Flat, on either side If an infratentorial tumor has been removed, the child is positioned flat on either side. The pillow is placed behind the child's back for comfort and to maintain the position. The pillow is not placed behind the head because when the pillow is behind the head, proper alignment is not maintained, and this misalignment can impair circulation. The child should never be placed in a Trendelenburg's position (head down) because this position increases intracranial pressure. The head is elevated when the tumor is a supratentorial one.

The nurse is caring for a client who is at risk for increased intracranial pressure (ICP) after a stroke. Which activities performed by the nurse will assist with preventing increases in ICP? Select all that apply. 1. Clustering nursing activities 2. Hyperoxygenating before suctioning 3. Maintaining 20 degree flexion of the knees 4. Maintaining the head and neck in midline position 5. Maintaining the head of the bed (HOB) at 30 degrees elevation

2. Hyperoxygenating before suctioning 4. Maintaining the head and neck in midline position 5. Maintaining the head of the bed (HOB) at 30 degrees elevation Measures aimed at preventing increased ICP in the poststroke client include hyperoxgenating before suctioning to avoid transient hypoxemia and resultant ICP elevation from dilation of cerebral arteries; maintaining the head in a midline, neutral position to help promote venous drainage from the brain; and keeping the HOB elevated to between 25 and 30 degrees to prevent a decreased blood flow to the brain. Clustering activities can be stressful for the client and increase ICP. Maintaining 20 degree flexion of the knees increases intraabdominal pressure and consequently ICP.

The nurse is performing the oculocephalic response (doll's-eyes maneuver) test on an unconscious client. The nurse turns the client's head and notes movement of the eyes in the same direction as for the head. How should the nurse document these findings? 1.Normal 2.Abnormal 3.Insignificant 4.Inconclusive

2. In an unconscious client, eye movements are an indication of brainstem activity and are tested by the oculocephalic response. When the doll's-eyes maneuver is intact, the eyes move in the opposite direction when the head is turned. Abnormal responses include movement of the eyes in the same direction as for the head and maintenance of a midline position of the eyes when the head is turned. An abnormal response indicates a disruption in the processing of information through the brainstem. * Eliminate options 3 and 4 first because they are comparable or alike findings. Regarding the remaining choices, it is necessary to know that the assessment finding noted in the question is an abnormal response.

The nurse develops a plan of care for a client with a brain aneurysm who will be placed on aneurysm precautions. Which interventions should be included in the plan? Select all that apply. 1. Leave the lights on in the client's room at night. 2. Place a blood pressure cuff at the client's bedside. 3. Close the shades in the client's room during the day. 4. Allow the client to drink 1 cup of caffeinated coffee a day. 5. Allow the client to ambulate 4 times a day with assistance.

2. Place a blood pressure cuff at the client's bedside. 3. Close the shades in the client's room during the day. Aneurysm precautions include placing the client on bed rest in a quiet setting. The use of lights is kept to a minimum to prevent environmental stimulation. The nurse should monitor the blood pressure and note any changes that could indicate rupture. Any activity, such as pushing, pulling, sneezing, or straining, that increases the blood pressure or impedes venous return from the brain is prohibited. The nurse provides physical care to minimize increases in blood pressure. Visitors, radio, television, and reading materials are restricted or limited. Stimulants, such as nicotine and coffee and other caffeine-containing products, are prohibited. Decaffeinated coffee or tea may be used.

The nurse is administering mouth care to an unconscious client. The nurse should perform which actions in the care of this person? Select all that apply. 1. Use products that contain alcohol. 2. Position the client on his or her side. 3. Brush the teeth with a small, soft toothbrush. 4. Cleanse the mucous membranes with soft sponges. 5. Use lemon glycerin swabs when performing mouth care.

2. Position the client on his or her side. 3. Brush the teeth with a small, soft toothbrush. 4. Cleanse the mucous membranes with soft sponges. The unconscious client is positioned on the side during mouth care to prevent aspiration. The teeth are brushed at least twice daily with a small toothbrush. The gums, tongue, roof of the mouth, and oral mucous membranes are cleansed with soft sponges to avoid encrustation and infection. The lips are coated with water-soluble lubricant to prevent drying, cracking, and encrustation. The use of products with alcohol and lemon glycerin swabs should be avoided because they have a drying effect.

At the beginning of the work shift, the nurse assesses the status of the client wearing a halo device. The nurse determines that which assessment finding requires intervention? 1.Tightened screws 2.Red skin areas under the jacket 3.Clean and dry lamb's wool jacket lining 4.Finger-width space between the jacket and the skin

2. Red skin areas under the jacket indicate that the jacket is too tight. The resulting pressure could lead to altered skin integrity and needs to be relieved by loosening the jacket. The screws all should be properly tightened. A clean, dry lamb's wool lining should be in place underneath the jacket, and there should be a finger-width space between the jacket and the skin. In addition, the client should wear a clean white cotton T-shirt next to the skin to help prevent itching. * Focus on the subject, care of a halo device. Note the words requires intervention. This phrasing tells you that the correct answer is an option that represents a problem. Noting the word red in the correct option will direct you to this one.

The clinic nurse is reviewing the record of a client scheduled to be seen in the clinic. The nurse notes that the client is taking selegiline hydrochloride (Eldepryl). The nurse suspects that the client has which disorder? 1.Diabetes mellitus 2.Parkinson's disease 3.Alzheimer's disease 4.Coronary artery disease

2. Selegiline hydrochloride is an antiparkinsonian medication. The medication increases dopaminergic action, assisting in the reduction of tremor, akinesia, and the rigidity of parkinsonism. This medication is not used to treat diabetes mellitus, Alzheimer's disease, or coronary artery disease. * Focus on the subject, uses for this medication. Knowledge regarding the action and use of selegiline hydrochloride is required to answer this question. Recalling that this medication increases dopaminergic action and thinking about the pathophysiology associated with Parkinson's disease will direct you to the correct option. Remember that this medication is used to treat Parkinson's disease.

The nurse is documenting nursing observations in the record of a client who experienced a tonic-clonic seizure. Which clinical manifestation did the nurse most likely note in the clonic phase of the seizure? 1.Body stiffening 2.Spasms of the entire body 3.Sudden loss of consciousness 4.Brief flexion of the extremities

2. The clonic phase of a seizure is characterized by alternating spasms and momentary muscular relaxation of the entire body, accompanied by strenuous hyperventilation. The face is contorted and the eyes roll. Excessive salivation results in frothing from the mouth. The tongue may be bitten, the client sweats profusely, and the pulse is rapid. The clonic jerking subsides by slowing in frequency and losing strength of contractions over a period of 30 seconds. Options 1, 3, and 4 identify the tonic phase of a seizure. * Note the strategic words most likely. Focus on the subject of the question, clonic phase of a seizure. Use knowledge regarding the characteristics of this phase of the seizure, recalling that spasm of the body occurs in this phase. This will direct you to the correct option.

The nurse is reviewing the record for a client seen in the health care clinic and notes that the health care provider has documented a diagnosis of amyotrophic lateral sclerosis (ALS). Which initial clinical manifestation of this disorder should the nurse expect to see documented in the record? 1.Muscle wasting 2.Mild clumsiness 3.Altered mentation 4.Diminished gag reflex

2. The initial symptom of ALS is a mild clumsiness, usually noted in the distal portion of one extremity. The client may complain of tripping and drag one leg when the lower extremities are involved. Mentation and intellectual function usually are normal. Diminished gag reflex and muscle wasting are not initial clinical manifestations. * Use knowledge regarding the early manifestations of ALS to assist in answering the question. Focusing on the strategic word initial in the question and noting the word mild in the correct option will direct you to this one.

The home care nurse is making a visit to a client who is wheelchair bound after a spinal cord injury sustained 4 months earlier. Just before leaving the home, the nurse ensures that which intervention has been done to prevent an episode of autonomic dysreflexia (hyperreflexia)? 1.Updating the home safety sheet 2.Leaving the client in an unchilled area of the room 3.Noting a bowel movement on the client progress note 4.Recording the amount of urine obtained with catheterization

2. The most common cause of autonomic dysreflexia is visceral stimuli, such as with blockage of urinary drainage or with constipation. Barring these, other causes include noxious mechanical and thermal stimuli, particularly pressure and overchilling. For this reason, the nurse ensures that the client is positioned with no pinching or pressure on paralyzed body parts and that the client will be sufficiently warm. * Note the subject, autonomic dysreflexia. Note the word prevent. This implies an action orientation on the part of the nurse. Each of the incorrect options contains an item related to documentation rather than an action to be taken by the nurse just before leaving.

A client who has had a stroke (brain attack) has residual dysphagia. When a diet prescription is initiated, the nurse should take which actions? Select all that apply. 1. Giving the client thin liquids 2. Thickening liquids to the consistency of oatmeal 3. Placing food on the unaffected side of the mouth 4. Allowing plenty of time for chewing and swallowing 5. Leave the client alone so that the client will gain independence by feeding self

2. Thickening liquids to the consistency of oatmeal 3. Placing food on the unaffected side of the mouth 4. Allowing plenty of time for chewing and swallowing The client with dysphagia is started on a diet only after the gag and swallow reflexes have returned. The client is assisted with meals as needed and is given ample time to chew and swallow. Food is placed on the unaffected side of the mouth. Liquids are thickened to avoid aspiration. The client is not left alone because of the risk of aspiration.

The nurse prepares a list of home care instructions for the parents of a child who has a plaster cast applied to the left forearm. Which instructions should be included on the list? Select all that apply. 1. Use the fingertips to lift the cast while it is drying. 2. Keep small toys and sharp objects away from the cast. 3. Use a padded ruler or another padded object to scratch the skin under the cast if it itches. 4. Place a heating pad on the lower end of the cast and over the fingers if the fingers feel cold. 5. Elevate the extremity on pillows for the first 24 to 48 hours after casting to prevent swelling. 6. Contact the health care provider (HCP) if the child complains of numbness or tingling in the extremity.

2.5.6 While the cast is drying, the palms of the hands are used to lift the cast. If the fingertips are used, indentations in the cast could occur and cause constant pressure on the underlying skin. Small toys and sharp objects are kept away from the cast, and no objects (including padded objects) are placed inside the cast because of the risk of altered skin integrity. The extremity is elevated to prevent swelling, and the HCP is notified immediately if any signs of neurovascular impairment develop. A heating pad is not applied to the cast or fingers. Cold fingers could indicate neurovascular impairment, and the HCP should be notified.

A client is admitted to the hospital for severe headaches. The client has a history of increased intracranial pressure (ICP), which has required lumbar punctures to relieve the pressure by draining cerebrospinal fluid. The client suddenly vomits and states, "That's weird, I didn't even feel nauseated." Which action by the nurse is the most appropriate? 1. Document the amount of emesis 2. Lower the head of the bed 3. Notify the health care provider (HCP) 4. Offer anti-nausea medication

3

A client with seizure activity is receiving a continuous tube feeding via a small-bore enteral tube. The nurse prepares to administer phenytoin oral suspension via the enteral route. What is the nurse's priority action before administering this medication? 1. Check renal function laboratory results 2. Flush tube with normal saline, not water 3. Stop the feeding for 1 to 2 hours 4. Take the blood pressure (BP)

3

A client with stroke symptoms has a blood pressure of 240/124 mm Hg. The nurse prepares the prescribed nicardipine intravenous (IV) infusion solution correctly to yield 0.1 mg/mL. The nurse then administers the initial prescription to infuse at 5 mg/hr by setting the infusion pump at 50 mL/hr. What is the nurse's priority action at this time? 1. Assess hourly urinary output 2. Increase pump setting to correct administration rate to 100 mL/hr 3. Keep systolic blood pressure above 170 mm Hg 4. Monitor for a widening QT interval

3

A highly intoxicated client was brought to the emergency department after found lying on the sidewalk. On admission, the client is awake with a pulse of 70/min and blood pressure of 160/80 mm Hg. An hour later, the client is lethargic, pulse is 48/min, and blood pressure is 200/80 mm Hg. Which action does the nurse anticipate taking next? 1. Administer atropine for bradycardia 2. Administer nifedipine for hypertension 3. Have CT scan performed to rule out an intracranial bleed 4. Perform hourly neurologic checks with Glasgow coma scale (GCS)

3

A nurse is teaching the parent of a child who has a new diagnosis of absence seizures. Which statement by the parent indicates understanding of the teaching? 1. "My child may experience incontinence." 2. "My child may seem confused afterwards." 3. "My child may stare and seem inattentive." 4. "My child will notice unusual odors prior to the event."

3

Four children are brought to the emergency department. Which child should be assessed first? 1. A 13-month-old who ingested an unknown quantity of children's multivitamins 2. A 15-month-old with a fever of 100.5 F (38.1 C) after being vaccinated 3. A 3-year-old with a forehead laceration and colorless nasal drainage 4. A 4-year-old with enlarged tonsillar lymph nodes who is crying in pain

3

The health care provider (HCP) has prescribed amitriptyline 25 mg orally every morning for an elderly client with recent herpes zoster infection (shingles) and severe postherpetic neuralgia. What is the priority nursing action? 1. Encourage increased fluid intake 2. Provide frequent rest periods 3. Teach the client to get up slowly from the bed or a sitting position 4. Tell the client to wear sunglasses when outdoors

3

The health care provider prescribes a multivitamin regimen that includes thiamine for a client with a history of chronic alcohol abuse. The nurse is aware that thiamine is given to this client population for which purpose? 1. To lower the blood alcohol level 2. To prevent gross tremors 3. To prevent Wernicke encephalopathy 4. To treat seizures related to acute alcohol withdrawal

3

The nurse is caring for a client diagnosed with Guillain-Barré syndrome (GBS) after a recent gastrointestinal (GI) illness. Monitoring for which of the following is a nursing care priority for this client? 1. Diaphoresis with facial flushing 2. Hypoactive or absent bowel sounds 3. Inability to cough or lift the head 4. Warm, tender, and swollen leg

3

The nurse is caring for a client with a history of tonic-clonic seizures. After a seizure lasting 25 seconds, the nurse notes that the client is confused for 20 minutes. The client does not know the current location, does not know the current season, and has a headache. The nurse documents the confusion and headache as which phase of the client's seizure activity? 1. Aural phase 2. Ictal phase 3. Postictal phase 4. Prodromal phase

3

The nurse is caring for an infant with suspected meningitis and preparing to assist with a spinal tap. What is the appropriate nursing intervention? 1. Administer oxygen via nasal cannula for client comfort and safety 2. Clean area with povidone iodine in a circular motion moving outward 3. Hold the child with the head and knees tucked in and the back rounded out 4. Monitor and record vital signs every 15 minutes throughout the procedure

3

The nurse receives the assigned clients for today on a neurology unit. The nurse should check on which client first? 1. Client with history of head injury whose Glasgow Coma Scale (GCS) changes from 13 to 14 2. Client with history of myasthenia gravis who had ptosis in the evening 3. Client with history of T2 spinal injury who has diaphoresis, pulse 54/min, and hypertension 4. Client with history of transverse myelitis with 2+ bilateral lower extremity muscle strength

3

The nurse taught the caregiver of a child with a ventriculoperitoneal (VP) shunt about when to contact the health care provider (HCP). The caregiver shows understanding of the instructions by contacting the HCP about which symptom? 1. A temperature of 99 F (37 C) that occurs during the evening 2. The child cannot recall items eaten for lunch the previous day 3. The child vomits after awakening from a nap and 1 hour later 4. The VP shunt is palpated along the posterior-lateral portion of the skull

3

The nurse is providing instructions to the parents of a child with scoliosis regarding the use of a brace. Which statement by the parents indicates a need for further instruction? 1. "I will encourage my child to perform prescribed exercises." 2. "I will have my child wear soft fabric clothing under the brace." 3. "I should apply lotion under the brace to prevent skin breakdown." 4. "I should avoid the use of powder because it will cake under the brace."

3 A brace may be prescribed to treat scoliosis. Braces are not curative, but may slow the progression of the curvature to allow skeletal growth and maturity. The use of lotions or powders under a brace should be avoided because they can become sticky and cake under the brace, causing irritation. Options 1, 2, and 4 are appropriate interventions in the care of a child with a brace.

The nurse is creating a plan of care for a newborn infant with spina bifida (myelomeningocele type). The nurse includes assessment measures in the plan to monitor for increased intracranial pressure. Which assessment technique should be performed that will best detect the presence of an increase in intracranial pressure? 1. Check urine for specific gravity. 2. Monitor for signs of dehydration. 3. Assess anterior fontanel for bulging. 4. Assess blood pressure for signs of hypotension.

3 A bulging or taut anterior fontanel would indicate the presence of increased intracranial pressure. Urine concentrating ability is not well developed at the newborn stage of development. Monitoring for signs of dehydration will not provide data related to increased intracranial pressure. Blood pressure is difficult to assess during the newborn period and is not the best indicator of intracranial pressure.

The nurse is assessing a client with a neurologic health problem and discovers a change in level of consciousness from alert to lethargic. What is the nurse's best action? 1. Perform a complete neurologic assessment. 2. Assess the cranial nerve functions. 3. Contact the Rapid Response Team. 4. Reassess the client in 30 minutes.

3 A change in level of consciousness and orientation is the earliest and most reliable indication that central neurologic function has declined. If a decline occurs, contact the Rapid Response Team or health care provider immediately. The nurse should also perform a focused assessment to determine if there are any other changes

The nurse assists a health care provider in performing a lumbar puncture on a 3-year-old child with leukemia in whom central nervous system disease is suspected. In which position will the nurse place the child during this procedure? 1. Lithotomy position 2. Modified Sims' position 3. Lateral recumbent position with the knees flexed and chin resting on the chest 4. Prone with knees flexed to the abdomen and head bent with chin resting on the chest

3 A lateral recumbent position with the knees flexed to the abdomen and the head bent with the chin resting on the chest is assumed for a lumbar puncture. This position separates the spinal processes and facilitates needle insertion into the subarachnoid space. The positions in the remaining options are incorrect.

The nurse receives a telephone call from the admissions office and is told that a child with acute bacterial meningitis will be admitted to the pediatric unit. The nurse prepares for the child's arrival and plans to implement which type of precautions? 1. Enteric 2. Contact 3. Droplet 4. Neutropenic

3 A major priority in nursing care for a child with suspected meningitis is to administer the appropriate antibiotic as soon as it is prescribed. The child will be placed in a private room, with droplet transmission precautions, for at least 24 hours after antibiotics are given. Enteric, contact, and neutropenic precautions are not associated with the mode of transmission of meningitis. Enteric precautions are instituted when the mode of transmission is through the gastrointestinal tract. Contact precautions are instituted when contact with infectious items or materials is likely. Neutropenic precautions are instituted when the client has a low neutrophil count.

An alert child, who is crying loudly, is brought to the hospital emergency department for a simple fracture to the lower right arm that occurred after a fall off a bicycle. What is the nurse's priority assessment? 1. Mobility 2. Skin integrity 3. Neurovascular 4. Level of consciousness

3 A simple fracture is a fracture of the bone across its entire shaft with some possible displacement but without breaking the skin. The priority assessment is the neurovascular status in the affected arm. The affected arm should be immobilized. Skin integrity is a higher priority in a compound fracture since there is an open wound. The level of consciousness is already established, as the child is alert and crying.

The nurse is providing care for a client with an acute hemorrhagic stroke. The client's spouse tells the nurse that he has been reading a lot about strokes and asks why his wife has not received alteplase. What is the nurse's best response? 1. "Your wife was not admitted within the time frame that alteplase is usually given." 2. "This drug is used primarily for clients who experience an acute heart attack." 3. "Alteplase dissolves clots and may cause more bleeding into your wife's brain." 4. "Your wife had gallbladder surgery just 6 months ago, so we can't use alteplase."

3 Alteplase is a clot buster. In a client who has experienced hemorrhagic stroke, there is already bleeding into the brain. A drug, such as alteplase, dissolves the clot and can cause more bleeding in the brain. The other statements about the use of alteplase are accurate but are not pertinent to this client's diagnosis.

A client with Parkinson disease has a problem with decreased mobility related to neuromuscular impairment. The nurse observes the unlicensed assistive personnel (UAP) performing all of these actions. For which action must the nurse intervene? 1. Helping the client ambulate to the bathroom and back to bed 2. Reminding the client not to look at his feet when he is walking 3. Performing the client's complete bathing and oral care 4. Setting up the client's tray and encouraging the client to feed himself

3 Although all of these actions fall within the scope of practice for a UAP, the UAP should help the client with morning care as needed, but the goal is to keep this client as independent and mobile as possible. The client should be encouraged to perform as much morning care as possible. Assisting the client in ambulating, reminding the client not to look at his feet (to prevent falls), and encouraging the client to feed himself are all appropriate to the goal of maintaining independence.

The parents of a child recently diagnosed with cerebral palsy ask the nurse about the limitations of the disorder. The nurse responds by explaining that the limitations occur as a result of which pathophysiological process? 1. An infectious disease of the central nervous system 2. An inflammation of the brain as a result of a viral illness 3. A chronic disability characterized by impaired muscle movement and posture 4. A congenital condition that results in moderate to severe intellectual disabilities

3 Cerebral palsy is a chronic disability characterized by impaired movement and posture resulting from an abnormality in the extrapyramidal or pyramidal motor system. Meningitis is an infectious process of the central nervous system. Encephalitis is an inflammation of the brain that occurs as a result of viral illness or central nervous system infection. Down syndrome is an example of a congenital condition that results in moderate to severe intellectual disabilities.

Parents bring their 2-week-old infant to a clinic for treatment after a diagnosis of clubfoot made at birth. Which statement by the parents indicates a need for further teaching regarding this disorder? 1. "Treatment needs to be started as soon as possible." 2. "I realize my infant will require follow-up care until fully grown." 3. "I need to bring my infant back to the clinic in 1 month for a new cast." 4. "I need to come to the clinic every week with my infant for the casting."

3 Clubfoot is a complex deformity of the ankle and foot that includes forefoot adduction, midfoot supination, hindfoot varus, and ankle equinus; the defect may be unilateral or bilateral. Treatment for clubfoot is started as soon as possible after birth. Serial manipulation and casting are performed at least weekly. If sufficient correction is not achieved in 3 to 6 months, surgery usually is indicated. Because clubfoot can recur, all children with clubfoot require long-term interval follow-up until they reach skeletal maturity to ensure an optimal outcome.

The nurse is reviewing the record of a child with increased intracranial pressure and notes that the child has exhibited signs of decerebrate posturing. Which assessment finding should the nurse expect if this type of posturing is present? 1. Flexion of the upper extremities and extension of the lower extremities. 2. Unilateral or bilateral postural change in which the extremities are rigid. 3. Abnormal extension of the upper and lower extremities with some internal rotation. 4. Arms are adducted with fists clenched, and the legs are flaccid with external rotation.

3 Decerebrate (extension) posturing is an abnormal extension of the upper extremities, with internal rotation of the upper arm and wrist and extension of the lower extremities with some internal rotation. Decorticate posturing involves flexion of the upper extremities and extension of the lower extremities. The remaining two options are incorrect and not characteristics of decerebrate posturing.

The parents of a child with juvenile idiopathic arthritis call the clinic nurse because the child is experiencing a painful exacerbation of the disease. The parents ask the nurse if the child can perform range-of-motion exercises at this time. The nurse should make which response? 1. "Avoid all exercise during painful periods." 2. "Range-of-motion exercises must be performed every day." 3. "Have the child perform simple isometric exercises during this time." 4. "Administer additional pain medication before performing range-of-motion exercises."

3 Juvenile idiopathic arthritis is an autoimmune inflammatory disease affecting the joints and other tissues, such as articular cartilage. During painful episodes of juvenile idiopathic arthritis, hot or cold packs and splinting and positioning the affected joint in a neutral position help reduce the pain. Although resting the extremity is appropriate, beginning simple isometric or tensing exercises as soon as the child is able is important. These exercises do not involve joint movement.

A lumbar puncture is performed on a child suspected to have bacterial meningitis, and cerebrospinal fluid (CSF) is obtained for analysis. The nurse reviews the results of the CSF analysis and determines that which results would verify the diagnosis? 1. Clear CSF, decreased pressure, and elevated protein level 2. Clear CSF, elevated protein, and decreased glucose levels 3. Cloudy CSF, elevated protein, and decreased glucose levels 4. Cloudy CSF, decreased protein, and decreased glucose levels

3 Meningitis is an infectious process of the central nervous system caused by bacteria and viruses; it may be acquired as a primary disease or as a result of complications of neurosurgery, trauma, infection of the sinus or ears, or systemic infections. Meningitis is diagnosed by testing CSF obtained by lumbar puncture. In the case of bacterial meningitis, findings usually include an elevated pressure; turbid or cloudy CSF; and elevated leukocyte, elevated protein, and decreased glucose levels.

A client taking carbamazepine (Tegretol) asks the nurse what to do if a dose is inadvertently missed. The nurse responds that which action should be taken? 1. Withhold until the next scheduled dose. 2. Withhold and call the health care provider (HCP). 3. Take the dose as long as it is not close to the time for the next dose. 4. Withhold until the next scheduled dose, which should then be doubled.

3 Rationale: Carbamazepine is an anticonvulsant that should be taken around the clock, precisely as directed. If a dose is omitted, the client should take the dose as soon as it is remembered, as long as it is not close to the time that the next dose is due. The medication should not be double-dosed. If more than one dose is omitted, the client should call the HCP.

A client who was started on anticonvulsant therapy with clonazepam (Klonopin) tells the nurse of increasing clumsiness and unsteadiness since starting the medication. The client is visibly upset by these manifestations and asks the nurse what to do. The nurse's response is based on which understanding? 1. These symptoms probably result from interaction with another medication. 2. These symptoms usually occur when the client takes the medication with food. 3. These symptoms are most severe during initial therapy and decrease or disappear with long-term use. 4. These symptoms indicate that the client is experiencing a severe adverse reaction to the medication.

3 Rationale: Clonazepam is classified as a benzodiazepine and is used as an anticonvulsant and antianxiety agent. Drowsiness, unsteadiness, and clumsiness are expected effects of the medication during early therapy. They are dose related and usually diminish or disappear altogether with continued use of the medication. They are unrelated to an interaction with another medication. The client is encouraged to take this medication with food to minimize gastrointestinal upset. These symptoms do not indicate that an adverse effect is occurring.

The nurse is preparing to give a postcraniotomy client medication for incisional pain. The family asks the nurse why the client is receiving codeine sulfate and not "something stronger." In formulating a response, the nurse incorporates which information about codeine sulfate? 1. Is one of the strongest opioid analgesics available 2. Cannot lead to physical or psychological dependence 3. Does not alter respirations or mask neurological signs as do other opioids 4. Does not cause gastrointestinal (GI) upset or constipation as do other opioids

3 Rationale: Codeine sulfate is an opioid analgesic used for clients after craniotomy. It often is combined with a nonopioid analgesic such as acetaminophen (Tylenol) for added effect. It does not alter the respiratory rate or mask neurological signs as do other opioids. Side effects of codeine sulfate include GI upset and constipation. Chronic use of the medication can lead to physical and psychological dependence.

Dantrolene sodium (Dantrium) has been administered to a client with a spinal cord injury. The nurse determines that the client is experiencing an adverse effect of the medication if which is noted? 1. Dizziness 2. Drowsiness 3. Abdominal pain 4. Lightheadedness

3 Rationale: Dantrium is hepatotoxic. The nurse observes for indications of liver dysfunction, which include jaundice, abdominal pain, and malaise. The nurse notifies the health care provider if these occur. The items in options 1, 2, and 4 are expected side effects due to the central nervous system (CNS)-depressant effects of the medication.

The nurse is observing a new nursing graduate who is preparing an intermittent intravenous (IV) infusion of phenytoin (Dilantin) for a client with a diagnosis of seizures. Which solution used by the nursing graduate should indicate to the nurse an understanding of proper preparation of this medication? 1. 5% dextrose in water 2. Lactated Ringer's solution 3. 0.9% sodium chloride (normal saline) 4. 5% dextrose and 0.45% sodium chloride

3 Rationale: Intermittent IV infusion of phenytoin is administered by injection into a large vein, using normal saline solution. Dextrose solutions are avoided because the medication will precipitate in these solutions. Options 1, 2, and 4 identify incorrect solutions for IV administration with this medication.

The home health nurse visits a client who is taking phenytoin (Dilantin) for control of seizures. During the assessment, the nurse notes that the client is taking birth control pills. Which information should the nurse include in the teaching plan? 1. Pregnancy should be avoided while taking phenytoin. 2. The client may stop the medication if it is causing severe gastrointestinal effects. 3. There is the potential of decreased effectiveness of birth control pills while taking phenytoin. 4. There is the increased risk of thrombophlebitis while taking phenytoin and birth control pills together.

3 Rationale: Phenytoin enhances the rate of estrogen metabolism, which can decrease the effectiveness of some birth control pills. Options 1, 2, and 4 are inappropriate instructions.

The nurse is providing instructions to an adolescent prescribed phenytoin (Dilantin) for the control of seizures. Which statement by the adolescent indicates a need for further teaching regarding the medication? 1. "The medication may cause acne or oily skin." 2. "Drinking alcohol may affect the medication." 3. "If my gums become sore and swollen I need to stop the medication." 4. "Birth control pills may not be effective when I take this medication."

3 Rationale: The adolescent should not stop taking antiseizure medications suddenly or without discussing it with a health care provider (HCP) or nurse. Acne or oily skin may be a problem for the adolescent, and the adolescent is advised to call a health care provider (HCP) for skin problems. Alcohol will lower the seizure threshold, and it is best to avoid the use of alcohol. Birth control pills may be less effective when the client is taking antiseizure medication.

After review of the client's laboratory values, the nurse notes that a phenytoin (Dilantin) level for a client receiving phenytoin (Dilantin) is 7 mcg/mL. The nurse makes which interpretation regarding this laboratory result? 1. The level is within the expected therapeutic range. 2. The level indicates the medication should be stopped. 3. The level is lower than the expected therapeutic range. 4. The level is higher than the expected therapeutic range.

3 Rationale: The target range for a therapeutic serum level of phenytoin is between 10 and 20 mcg/mL. Levels below 10 mcg/mL are too low to control seizures. At levels above 20 mcg/mL, signs of toxicity begin to appear. This client has a low serum level, and the dosage is likely to be increased.

The nurse in the health care provider's office is reviewing the results of a client's phenytoin (Dilantin) level determination performed that morning. The nurse identifies that a therapeutic drug level has been achieved if which result is noted? 1. 3 mcg/mL 2. 8 mcg/mL 3. 15 mcg/mL 4. 24 mcg/mL

3 Rationale: The therapeutic range for serum phenytoin levels is 10 to 20 mcg/mL in clients with normal serum albumin levels and renal function. A level below this range indicates that the client is not receiving sufficient medication and is at risk for seizure activity. In this case, the medication dose should be adjusted upward. A level above this range indicates that the client is entering the toxic range and is at risk for toxic effects of the medication. In this case, the dose should be adjusted downward.

The nurse is monitoring an infant for signs of increased intracranial pressure. On assessment of the fontanelles, the nurse notes that the anterior fontanelle bulges when the infant is sleeping. Based on this finding, which is the priority nursing action? 1. Increase oral fluids. 2. Document the finding. 3. Notify the health care provider. 4. Place the infant supine in a side-lying position.

3 The anterior fontanelle is diamond shaped and is located on the top of the head. It should be soft and flat in a normal infant, and it normally closes by 12 to 18 months of age. A larger-than-normal fontanelle may be a sign of increased intracranial pressure (ICP) within the skull. Although the anterior fontanelle may bulge slightly when the infant cries, bulging at rest may indicate ICP. Increasing oral fluids and placing the infant in a side-lying position are inaccurate interventions and will not be helpful. Although the nurse would document the finding, the priority action would be to report the finding to the health care provider.

The nurse is providing instructions to the parents of a child with scoliosis regarding the use of a brace. Which statement by a parent indicates a need for further teaching? 1. "I cannot place powder under the brace." 2. "I need to place a soft shirt on my child under the brace." 3. "I need to be sure to apply lotion on the skin under the brace." 4. "I need to encourage my child to perform prescribed exercises."

3 The use of lotions or powders should be avoided with a brace because they can become sticky or cake under the brace, causing irritation. The actions in the remaining options are appropriate interventions for the use of a brace on a child.

The nurse is developing a plan of care for a newborn infant with spina bifida (myelomeningocele type). The nurse includes assessment measures in the plan to monitor for increased intracranial pressure. Which assessment technique should be performed that will best detect the presence of an increase in intracranial pressure? 1.Check urine for specific gravity. 2.Monitor for signs of dehydration. 3.Assess anterior fontanel for bulging. 4.Assess blood pressure for signs of hypotension.

3 Rationale: A bulging or taut anterior fontanel would indicate the presence of increased intracranial pressure. Urine concentrating ability is not well developed at the newborn stage of development. Monitoring for signs of dehydration will not provide data related to increased intracranial pressure. Blood pressure is difficult to assess during the newborn period and is not the best indicator of intracranial pressure.

The nurse receives a telephone call from the admissions office and is told that a child with acute bacterial meningitis will be admitted to the pediatric unit. The nurse prepares for the child's arrival and plans to implement which type of precautions? 1.Enteric 2.Contact 3.Droplet 4.Neutropenic

3 Rationale: A major priority in nursing care for a child with suspected meningitis is to administer the appropriate antibiotic as soon as it is prescribed. The child will be placed in a private room, with droplet transmission precautions, for at least 24 hours after antibiotics are given. Enteric, contact, and neutropenic precautions are not associated with the mode of transmission of meningitis. Enteric precautions are instituted when the mode of transmission is through the gastrointestinal tract. Contact precautions are instituted when contact with infectious items or materials is likely. Neutropenic precautions are instituted when the client has a low neutrophil count.

The nurse is preparing to care for a client after a lumbar puncture. The nurse should plan to place the client in which best position immediately after the procedure? 1.Prone in semi-Fowler's position 2.Supine in semi-Fowler's position 3.Prone with a small pillow under the abdomen 4. Lateral with the head slightly lower than the rest of the body

3 Rationale: After the procedure, the client assumes a flat position. If the client is able, a prone position with a pillow under the abdomen is the best position. This position helps reduce cerebrospinal fluid leakage and decreases the likelihood of post-lumbar puncture headache. Options 1, 2, and 4 are incorrect.

The home care nurse is performing an assessment on a client with a diagnosis of Bell's palsy. Which assessment question will elicit the most specific information regarding this client's disorder? 1."Do your eyes feel dry?" 2."Do you have any spasms in your throat?" 3."Are you having any difficulty chewing food?" 4."Do you have any tingling sensations around your mouth?"

3 Rationale: Bell's palsy is a one-sided facial paralysis caused by compression of the facial nerve. Manifestations include facial droop from paralysis of the facial muscles; increased lacrimation; painful sensations in the eye, face, or behind the ear; and speech or chewing difficulties.

A nurse is reviewing the record of a child with increased intracranial pressure and notes that the child has exhibited signs of decerebrate posturing. Which assessment finding should the nurse expect if this type of posturing were present? 1.Flexion of the upper extremities and extension of the lower extremities 2.Unilateral or bilateral postural change in which the extremities are rigid 3.Abnormal extension of the upper and lower extremities with some internal rotation 4.Arms are adducted with fists clenched and the legs are flaccid with external rotation

3 Rationale: Decerebrate (extension) posturing is an abnormal extension of the upper extremities, with internal rotation of the upper arm and wrist and extension of the lower extremities with some internal rotation. Option 1 describes decorticate posturing. Options 2 and 4 are incorrect and not characteristics of decerebrate posturing.

The nurse has given suggestions to a client with trigeminal neuralgia about strategies to minimize episodes of pain. The nurse determines that the client needs further education if the client makes which statement? 1."I will wash my face with cotton pads." 2."I'll have to start chewing on my unaffected side." 3."I'll try to eat my food either very warm or very cold." 4."I should rinse my mouth if toothbrushing is painful."

3 Rationale: Facial pain can be minimized by using cotton pads to wash the face and using room temperature water. The client should chew on the unaffected side of the mouth, eat a soft diet, and take in foods and beverages at room temperature. If toothbrushing triggers pain, an oral rinse after meals may be helpful instead.

he nurse is providing instructions to the parents of an infant with a ventriculoperitoneal shunt. The nurse should include which instruction? 1.Expect an increased urine output from the shunt. 2.Notify the health care provider if the infant is fussy. 3.Call the health care provider if the infant has a high-pitched cry. 4.Position the infant on the side of the shunt when the infant is put to bed.

3 Rationale: If the shunt is malfunctioning, the fluid from the ventricle part of the brain will not be diverted to the peritoneal cavity. The cerebrospinal fluid will build up in the cranial area. The result is increased intracranial pressure, which then causes a high-pitched cry in the infant. The infant should not be positioned on the side of the shunt because this will cause pressure on the shunt and skin breakdown. This type of shunt affects the gastrointestinal system, not the genitourinary system, and an increased urinary output is not expected. Being fussy is a concern only if other signs indicative of a complication are occurring.

A lumbar puncture is performed on a child suspected to have bacterial meningitis, and cerebrospinal fluid (CSF) is obtained for analysis. The nurse reviews the results of the CSF analysis and determines that which results would verify the diagnosis? 1.Clear CSF, decreased pressure, and elevated protein level 2.Clear CSF, elevated protein, and decreased glucose levels 3.Cloudy CSF, elevated protein, and decreased glucose levels 4.Cloudy CSF, decreased protein, and decreased glucose levels

3 Rationale: Meningitis is an infectious process of the central nervous system caused by bacteria and viruses; it may be acquired as a primary disease or as a result of complications of neurosurgery, trauma, infection of the sinus or ears, or systemic infections. Meningitis is diagnosed by testing cerebrospinal fluid obtained by lumbar puncture. In the case of bacterial meningitis, findings usually include an elevated pressure; turbid or cloudy cerebrospinal fluid; and elevated leukocyte, elevated protein, and decreased glucose levels

The nurse has instructed a client with myasthenia gravis about strategies for self-management at home. The nurse determines a need for more information if the client makes which statement? 1."Here's the Medic-Alert bracelet I obtained." 2."I should take my medications an hour before mealtime." 3."Going to the beach will be a nice, relaxing form of activity." 4."I've made arrangements to get a portable resuscitation bag and home suction equipment."

3 Rationale: Most ongoing treatment for myasthenia gravis is done in outpatient settings, and the client must be aware of the lifestyle changes needed to maintain independence. The client should carry medical identification about the presence of the condition. Taking medications an hour before mealtime gives greater muscle strength for chewing and is indicated. The client should have portable suction equipment and a portable resuscitation bag available in case of respiratory distress. The client should avoid situations and other factors, including stress, infection, heat, surgery, and alcohol, that could worsen the symptoms.

A client with a history of myasthenic gravis presents at a clinic with bilateral ptosis and is drooling, and myasthenia crisis is suspected. The nurse assesses the client for which precipitating factor? 1. Getting too little exercise 2. Taking excess medication 3. Omitting doses of medication 4. Increasing intake of fatty foods

3 Rationale: Myasthenic crisis often is caused by undermedication and responds to the administration of cholinergic medications. Cholinergic crisis (the opposite problem) is caused by excess medication and responds to withholding of medications. Too little exercise and excessive fatty food intake are incorrect. Overexertion and overeating possibly could trigger myasthenic crisis.

The home health nurse is visiting a client with a diagnosis of multiple sclerosis. The client has been taking oxybutynin (Ditropan XL). The nurse evaluates the effectiveness of the medication by asking the client which assessment question? 1."Are you consistently fatigued?" 2. "Are you having muscle spasms?" 3. "Are you getting up at night to urinate?" 4. "Are you having normal bowel movements?"

3 Rationale: Oxybutynin is an antispasmodic used to relieve symptoms of urinary urgency, frequency, nocturia, and incontinence in clients with uninhibited or reflex neurogenic bladder. Expected effects include improved urinary control and decreased urinary frequency, incontinence, and nocturia. Options 1, 2, and 4 are unrelated to the use of this medication.

A client had a seizure 1 hour ago. Family members were present during the episode and reported that the client's jaw was moving as though grinding food. In helping to determine the origin of this seizure, what should the nurse include in the client's assessment? 1.Presence of diaphoresis 2.Loss of consciousness 3.History of prior trauma 4. Rotating eye movements

3 Rationale: Seizures that originate with specific motor phenomena are considered focal and are indicative of a focal structural lesion in the brain, often caused by trauma, infection, or drug consumption. Options 1, 2, and 4 address signs, rather than an origin of the seizure.

A client is somewhat nervous about undergoing magnetic resonance imaging (MRI). Which statement by the nurse would provide the most reassurance to the client about the procedure? 1."The MRI machine is a long, narrow, hollow tube and may make you feel somewhat claustrophobic." 2."You will be able to eat before the procedure unless you get nauseated easily. If so, you should eat lightly." 3."Even though you are alone in the scanner, you will be in voice communication with the technologist at all times during the procedure." 4. "It is necessary to remove any metal or metal-containing objects before having the MRI done to avoid the metal being drawn into the magnetic field."

3 Rationale: The MRI scanner is a hollow tube that gives some clients a feeling of claustrophobia. Metal objects must be removed before the procedure so that they are not drawn into the magnetic field. The client may eat and may take all prescribed medications before the procedure. If a contrast medium is used, the client may wish to eat lightly if he or she has a tendency to become nauseated easily. The client lies supine on a padded table that moves into the imager. The client must lie still during the procedure. The imager makes tapping noises during the scanning. The client is alone in the imager, but the nurse can reassure the client that the technologist will be in voice communication with the client at all times during the procedure.

A client has dysfunction of the cochlear division of the vestibulocochlear nerve (cranial nerve VIII). The nurse should determine that the client is adequately adapting to this problem if he or she states a plan to obtain which item? 1.A walker 2.Eyeglasses 3.A hearing aid 4. A bath thermometer

3 Rationale: The cochlear division of cranial nerve VIII is responsible for hearing. Clients with hearing difficulty may benefit from the use of a hearing aid. The vestibular portion of this nerve controls equilibrium; difficulty with balance caused by dysfunction of this division could be addressed with use of a walker. Eyeglasses would correct visual problems (cranial nerve II); a bath thermometer would be of use to clients with sensory deficits of peripheral nerves, such as with diabetic neuropathy.

A client is anxious about an upcoming diagnostic procedure. The client's pupils are dilated, and the respiratory rate, heart rate, and blood pressure are increased from baseline. The nurse plans care, knowing that these changes are the result of which response? 1.Vagal response 2.Peripheral nervous system response 3.Sympathetic nervous system response 4. Parasympathetic nervous system response

3 Rationale: The sympathetic nervous system is responsible for the so-called fight or flight response, which is characterized by dilated pupils, increases in heart rate and cardiac output, and increases in respiratory rate and blood pressure. The sympathetic nervous system response affects some type of change in most systems of the body. The responses stated in the other options do not produce these effects.

Acetazolamide (Diamox) is prescribed for a client hospitalized with a diagnosis of a supratentorial lesion. The nurse understands that which is the primary action of the medication? 1.Prevention of hypertension 2.Prevention of hyperthermia 3.Decrease in cerebrospinal fluid production 4.Maintenance of blood pressure adequate for cerebral perfusion

3. Acetazolamide (Diamox) is a carbonic anhydrase inhibitor. It is used in the client with or at risk for increased intracranial pressure to decrease cerebrospinal fluid production. Options 1, 2, and 4 are not actions of this medication. * Focus on the diagnosis presented in the question and the strategic word primary in the question. Note the relationship between supratentorial in the question and cerebrospinal in the correct option.

The home care nurse is performing an assessment on a client with a diagnosis of Bell's palsy. Which assessment question will elicit the most specific information regarding this client's disorder? 1."Do your eyes feel dry?" 2."Do you have any spasms in your throat?" 3."Are you having any difficulty chewing food?" 4."Do you have any tingling sensations around your mouth?"

3. Bell's palsy is a one-sided facial paralysis caused by compression of the facial nerve. Manifestations include facial droop from paralysis of the facial muscles; increased lacrimation; painful sensations in the eye, face, or behind the ear; and speech or chewing difficulties. * Note the strategic word most. Focus on the subject, manifestations of Bell's palsy. Recalling that palsy is a type of paralysis will direct you to the correct option.

The nurse is reviewing the medical records of a client admitted to the nursing unit with a diagnosis of a thrombotic brain attack (stroke). The nurse would expect to note that which is documented in the assessment data section of the record? 1.Sudden loss of consciousness occurred. 2.Signs and symptoms occurred suddenly. 3.The client experienced paresthesias a few days before admission to the hospital. 4.The client complained of a severe headache, which was followed by sudden onset of paralysis.

3. Cerebral thrombosis does not occur suddenly. In the few hours or days preceding a thrombotic brain attack (stroke), the client may experience a transient loss of speech, hemiplegia, or paresthesias on one side of the body. Signs and symptoms of thrombotic brain attack (stroke) vary but may include dizziness, cognitive changes, or seizures. Headache is rare, but some clients with brain attack (stroke) experience signs and symptoms similar to those of cerebral embolism or intracranial hemorrhage. * Focus on the client's diagnosis, thrombotic brain attack (stroke). Note that options 1, 2, and 4 are comparable or alike and indicate a sudden occurrence. Recalling that a cerebral thrombosis does not occur suddenly will direct you to the correct option.

The nurse is monitoring a client who has returned to the nursing unit after a myelogram. Which client complaint would indicate the need to notify the health care provider (HCP)? 1.Backache 2.Headache 3.Neck stiffness 4.Feelings of fatigue

3. Headache is relatively common after the procedure, but neck stiffness, especially on flexion, and pain should be reported because they signal meningeal irritation. The client also is monitored for evidence of allergic reactions to the dye such as confusion, dizziness, tremors, and hallucinations. Feelings of fatigue may be normal, and back discomfort may be owing to the positions required for the procedure. *Focus on the subject, the finding indicating the need to notify the HCP. Recalling that meningeal irritation is a complication and that neck stiffness is a characteristic sign will direct you to the correct option.

A client with a traumatic closed head injury shows signs of secondary brain injury. What are some manifestations of secondary brain injury? Select all that apply. 1. Fever 2. Seizures 3. Hypoxia 4. Ischemia 5. Hypotension 6. Increased intracranial pressure (ICP)

3. Hypoxia 4. Ischemia 5. Hypotension 6. Increased intracranial pressure (ICP) Secondary brain injury can occur several hours to days after the initial brain injury and is a major concern when managing brain trauma. Nursing management of the client with an acute intracranial problem must include management of secondary injury. Manifestations of secondary injury includes hypoxia, ischemia, hypotension, and increased ICP that follows primary injury. It does not include fever or seizures.

The nurse is assisting the neurologist in performing an assessment on a client who is unconscious after sustaining a head injury. The nurse understands that the neurologist would avoid performing the oculocephalic response (doll's-eyes maneuver) if which condition is present in the client? 1.Dilated pupils 2.Lumbar trauma 3.A cervical cord injury 4.Altered level of consciousness

3. In an unconscious client, eye movements are an indication of brainstem activity and are tested by the oculocephalic response. When the doll's-eyes maneuver is intact, the eyes move in the opposite direction when the head is turned. Abnormal responses include movement of the eyes in the same direction as that for the head and maintenance of a midline position of the eyes when the head is turned. An abnormal response indicates a disruption in the processing of information through the brainstem. Contraindications to performing this test include cervical-level spinal cord injuries and severely increased intracranial pressure. * Focus on the subject, contraindication to performing the oculocephalic response. Visualize this maneuver and recall that with a cervical injury, the head is not turned but maintained in a midline position.

The home health nurse is visiting a client with a diagnosis of multiple sclerosis. The client has been taking oxybutynin (Ditropan XL). The nurse evaluates the effectiveness of the medication by asking the client which assessment question? 1."Are you consistently fatigued?" 2."Are you having muscle spasms?" 3."Are you getting up at night to urinate?" 4."Are you having normal bowel movements?"

3. Oxybutynin is an antispasmodic used to relieve symptoms of urinary urgency, frequency, nocturia, and incontinence in clients with uninhibited or reflex neurogenic bladder. Expected effects include improved urinary control and decreased urinary frequency, incontinence, and nocturia. Options 1, 2, and 4 are unrelated to the use of this medication. * Focus on the subject, the effectiveness of oxybutynin, and think about its classification. Recalling that this medication is used to treat bladder dysfunction will direct you to the correct option.

A client who has had a brain attack (stroke) is being managed on the medical nursing unit. At 0800, the client was awake and alert with vital signs of temperature 98° F orally, pulse 80 beats/min, respirations 18 breaths/min, and blood pressure 138/80 mm Hg. At noon, the client is confused and only responsive to tactile stimuli, and vital signs are temperature 99° F orally, pulse 62 beats/min, respirations 20 breaths/min, and blood pressure 166/72 mm Hg. The nurse should take which action? 1.Reorient the client. 2.Retake the vital signs. 3.Call the health care provider (HCP). 4.Administer an antihypertensive PRN.

3. The important nursing action is to call the HCP. The deterioration in neurological status, decreasing pulse, and increasing blood pressure with a widening pulse pressure all indicate that the client is experiencing increased intracranial pressure, which requires immediate treatment to prevent further complications and possible death. The nurse should retake the vital signs and reorient the client to surroundings. If the client's blood pressure falls within parameters for PRN antihypertensive medication, the medication also should be administered. However, options 1, 2, and 4 are secondary nursing actions. * Note the subject, care of the client who experiences brain attack (stroke). Noting the changes in neurological signs and vital signs and that these changes indicate a deterioration in the client's condition will direct you to the correct option.

The post-head injury client opens eyes to sound, has no verbal response, and localizes to painful stimuli when applied to each extremity. How should the nurse document the Glasgow Coma Scale (GCS) score? 1.GCS = 3 2. GCS = 6 3. GCS = 9 4. GCS = 11

3Rationale: The GCS is a method is assessing neurological status. The highest possible score in the GCS is 15. A score lower than 8 indicates that coma is present. Motor response points are as follows: Obeys a simple response = 6; Localizes painful stimuli = 5; Normal flexion (withdrawal) = 4; Abnormal flexion (decorticate posturing) = 3; Extensor response (decerebrate posturing) = 2; No motor response to pain = 1. Verbal response points are as follows: Oriented = 5; Confused conversation = 4; Inappropriate words = 3; Responds with incomprehensible sounds = 2; No verbal response = 1. Eye opening points are as follows: Spontaneous = 4; In response to sound = 3; In response to pain = 2; No response, even to painful stimuli = 1. Using the GCS, a score of 3 is given when the client opens the eyes to sound. Localization to pain is scored as 5. When there is no verbal response the score is a 1. The total score is then equal to 9.

The clinic nurse is assessing a previously healthy 60-year-old client when the client says, "My hand has been shaking when I try to cut food. I did some research online. Could I have Parkinson's disease?" Which response from the nurse is the most helpful? 1. "It can't be Parkinson's disease because you aren't old enough." 2. "Make sure you tell the physician about your concerns." 3. "Parkinson's disease does not cause that kind of hand shaking." 4. "Tell me more about your symptoms. When did they start?"

4

The health care provider (HCP) has prescribed amitriptyline 25 mg orally every morning for an elderly client with recent herpes zoster infection (shingles) and severe postherpetic neuralgia. What is the priority nursing action? 1. Encourage increased fluid intake 2. Provide frequent rest periods 3. Teach the client to get up slowly from the bed or a sitting position 4. Tell the client to wear sunglasses when outdoors

4

The nurse is caring for a client with increased intracranial pressure (ICP). Which statement by the unlicensed assistive personnel would require immediate intervention by the nurse? 1. "I will raise the head of the bed so it is easier to see the television." 2. "I will turn down the lights when I leave." 3. "Let me move your belongings closer so you can reach them." 4. "You should do deep breathing and coughing exercises."

4

The nurse is caring for a client with left-sided weakness from a stroke. When assisting the client to a chair, what should the nurse do? 1. Bend at the waist 2. Keep the feet close together 3. Pivot on the foot proximal to the chair 4. Use a transfer belt

4

The nurse is providing discharge education for a postoperative client who had a partial laryngectomy for laryngeal cancer. The client is concerned because the health care provider said there was damage to the ninth cranial nerve. Which statement made by the nurse is most appropriate? 1. "I will ask the health care provider to explain the consequences of your procedure." 2. "This is a common complication that will require you to have a hearing test every year." 3. "This is a common complication; your health care provider will order a consult for the speech pathologist." 4. "This is the reason you are using a special swallowing technique when you eat and drink."

4

The nurse develops a plan of care for a child at risk for tonic-clonic seizures. In the plan of care, the nurse identifies seizure precautions and documents that which item(s) need to be placed at the child's bedside? 1. Emergency cart 2. Tracheotomy set 3. Padded tongue blade 4. Suctioning equipment and oxygen

4 A seizure results from the excessive and unorganized neuronal discharges in the brain that activate associated motor and sensory organs. A type of generalized seizure is a tonic-clonic seizure. This type of seizure causes rigidity of all body muscles, followed by intense jerking movements. Because increased oral secretions and apnea can occur during and after the seizure, oxygen and suctioning equipment are placed at the bedside. A tracheotomy is not performed during a seizure. No object, including a padded tongue blade, is placed into the child's mouth during a seizure. An emergency cart would not be left at the bedside, but would be available in the treatment room or nearby on the nursing unit.

The nurse is performing an assessment of a 7-year-old child who is suspected of having episodes of absence seizures. Which assessment question to the mother will assist in providing information that will identify the symptoms associated with this type of seizure? 1. "Does twitching occur in the face and neck?" 2. "Does the muscle twitching occur on one side of the body?" 3. "Does the muscle twitching occur on both sides of the body?" 4. "Does the child have a blank expression during these episodes?"

4 Absence seizures are brief episodes of altered awareness or momentary loss of consciousness. No muscle activity occurs except eyelid fluttering or twitching. The child has a blank facial expression. These seizures last only 5 to 10 seconds, but they may occur one after another several times a day. Simple partial seizures consist of twitching of an extremity, face, or neck, or the sensation of twitching or numbness in an extremity or the face or neck. Myoclonic seizures are brief, random contractions of a muscle group that can occur on one or both sides of the body.

The nurse caring for a child with suspected absence seizures is collecting data from the parents on how to manage the disorder. Which statement, if made by the parents, indicates the presence of signs congruent with this disorder? 1. "My child does well with group activities." 2. "My child leads the other children during group play." 3. "My child is doing really well in school and has high grades." 4. "My child's teacher mentioned that he seems to daydream a lot."

4 Absence seizures are very brief episodes of altered awareness. There is no muscle activity except eyelid fluttering or twitching. The child has a blank facial expression. These seizures last only 5 to 10 seconds but may occur one after another several times a day. The child experiencing absence seizures may appear to be daydreaming. If the child is participating in group activities, they sometimes need help catching up with the group, especially if a seizure occurs. Decreasing grades is a sign of absence seizures, as well as lowered intellectual processes.

The nurse caring for an infant with a diagnosis of hydrocephalus should monitor the infant for which sign of increased intracranial pressure? 1. Proteinuria 2. Bradycardia 3. A drop in blood pressure 4. A bulging anterior fontanel

4 An elevated or bulging anterior fontanel indicates an increase in cerebrospinal fluid collection in the cerebral ventricle. Proteinuria, bradycardia, and a drop in blood pressure are not specific signs of increased intracranial pressure (ICP). Changes in the level of consciousness and a widened pulse pressure are additional signs of increased ICP.

The nurse is reviewing the health care record of an infant suspected of having unilateral hip dysplasia. Which assessment finding should the nurse expect to note documented in the infant's record regarding this condition? 1. Full range of motion in the affected hip 2. An apparent short femur on the unaffected side 3. Asymmetrical adduction of the affected hip when placed supine, with the knees and hips flexed 4. Asymmetry of the gluteal skin folds when the infant is placed prone and the legs are extended against the examining table

4 Asymmetry of the gluteal skin folds when the infant is placed prone and the legs are extended against the examining table is noted in hip dysplasia. Asymmetrical abduction of the affected hip when an infant is placed supine with the knees and hips flexed would also be an assessment finding in hip dysplasia in infants beyond the newborn period. An apparent short femur on the affected side is noted, as well as limited range of motion.

The nurse is providing home care instructions to the mother of a child who is recovering from Reye's syndrome. Which instruction should the nurse provide to the mother? 1. Increase stimuli in the home environment. 2. Avoid daytime naps so that the child will sleep at night. 3. Give the child frequent small meals, if vomiting occurs. 4. Check the skin and eyes every day for a yellow discoloration.

4 Checking for jaundice will assist in identifying the presence of liver complications, which are characteristic of Reye's syndrome. Decreasing stimuli and providing rest decrease stress on the brain tissue. If vomiting occurs in Reye's syndrome, it is caused by cerebral edema and is a sign of intracranial pressure.

The nurse is caring for a child diagnosed with Down syndrome. Which explanation of this syndrome should the nurse provide the parents? 1. Subaverage intellectual functioning with a congenial nature 2. Above-average intellectual functioning with deficits in adaptive behavior 3. Average intellectual functioning and the absence of deficits in adaptive behavior 4. Moderate to severe intellectual disability and linkage to an extra chromosome 21, group G

4 Down syndrome is a form of mental retardation and is a congenital condition that results in moderate to severe intellectual disability. Most cases are attributable to an extra chromosome (group G)—hence the name trisomy 21. The characteristics in the remaining options are incorrect characteristics of this syndrome.

The nurse is caring for an infant with spina bifida (myelomeningocele type) who had the sac on the back containing cerebrospinal fluid, the meninges, and the nerves (gibbus) surgically removed. The nursing plan of care for the postoperative period should include which action to maintain the infant's safety? 1. Covering the back dressing with a binder 2. Placing the infant in a head-down position 3. Strapping the infant in a baby seat sitting up 4. Elevating the head with the infant in the prone position

4 Elevating the head will decrease the chance that cerebrospinal fluid will accumulate in the cranial cavity. The infant needs to be prone or side-lying to decrease the pressure on the surgical site on the back. Binders and a baby seat should not be used because of the pressure they would exert on the surgical site.

A mother arrives at the emergency department with her 5-year-old child and states that the child fell off a bunk bed. A head injury is suspected. The nurse checks the child's airway status and assesses the child for early and late signs of increased intracranial pressure (ICP). Which is a late sign of increased ICP? 1. Nausea 2. Irritability 3. Headache 4. Bradycardia

4 Head injury is the pathological result of any mechanical force to the skull, scalp, meninges, or brain. A head injury can cause bleeding in the brain and result in increased ICP. In a child, early signs include a slight change in level of consciousness, headache, nausea, vomiting, visual disturbances (diplopia), and seizures. Late signs of increased ICP include a significant decrease in level of consciousness, bradycardia, decreased motor and sensory responses, alterations in pupil size and reactivity, posturing, Cheyne-Stokes respirations, and coma.

The nurse notes that an infant with the diagnosis of hydrocephalus has a head that is heavier than that of the average infant. The nurse should determine that special safety precautions are needed when moving the infant with hydrocephalus. Which statement should the nurse plan to include in the discharge teaching with the parents to reflect this safety need? 1. "Feed your infant in a side-lying position." 2. "Place a helmet on your infant when in bed." 3. "Hyperextend your infant's head with a rolled blanket under the neck area." 4. "When picking up your infant, support the infant's neck and head with the open palm of your hand."

4 Hydrocephalus is a condition characterized by an enlargement of the cranium because of an abnormal accumulation of cerebrospinal fluid in the cerebral ventricular system. This characteristic causes the increase in the weight of the infant's head. The infant may experience significant head enlargement. Care must be exercised so that the head is well supported when the infant is fed or moved to prevent extra strain on the infant's neck, and measures must be taken to prevent the development of pressure areas. Supporting the infant's head and neck when picking up the infant will prevent the hyperextension of the neck area and prevent the infant from falling backward. The infant should be fed with the head elevated for proper motility of food processing. A helmet could suffocate an unattended infant during rest and sleep times, and hyperextension of the infant's head could put pressure on the neck vertebrae, causing injury.

The nurse is implementing a teaching plan for a 4-month-old child who has been diagnosed with developmental dysplasia of the hip. The child will be placed in the Pavlik harness. Which statement by the family indicates that they understand the care of their child while placed in the Pavlik harness? 1. "I know that the harness must be worn continuously." 2. "I will bring my child back to the orthopedic office in a month or two so the straps can be checked." 3. "I realize that I will also need to put two diapers on my child so that the harness will stay dry and does not get soiled." 4. "I will watch for any redness or skin irritation where the straps are applied and call the health care provider for red areas."

4 If stabilization of the hip is required, a cast is initially applied. This is kept in place for 3 to 6 months until the hip is stabilized. After this is completed, and if further treatment is required, a Pavlik harness is the treatment of choice next. A Pavlik harness is a removable abduction brace. The brace must be checked every 1 to 2 weeks for adjustment of the straps. The use of double diapering is not recommended for developmental dysplasia of the hip (DDH) because of the possibility of hip extension. Because there are straps applied to the child's skin, it is important to check the skin of the child frequently.

The nurse is planning care for a child with acute bacterial meningitis. Based on the mode of transmission of this infection, which precautionary intervention should be included in the plan of care? 1. Maintain enteric precautions. 2. Maintain neutropenic precautions. 3. No precautions are required as long as antibiotics have been started. 4. Maintain respiratory isolation precautions for at least 24 hours after the initiation of antibiotics.

4 Meningitis is an infectious process of the central nervous system caused by bacteria and viruses; it may be acquired as a primary disease or as a result of complications of neurosurgery, trauma, infection of the sinus or ears, or systemic infections. A major priority of nursing care for a child suspected to have meningitis is to administer the antibiotic as soon as it is prescribed. The child also is placed on respiratory isolation precautions for at least 24 hours while culture results are obtained and the antibiotic is having an effect. Enteric precautions and neutropenic precautions are not associated with the mode of transmission of meningitis. Enteric precautions are instituted when the mode of transmission is through the gastrointestinal tract. Neutropenic precautions are instituted when a child has a low neutrophil count.

A client is scheduled to begin therapy with carbamazepine (Tegretol). The nurse should assess the results of which test(s) before administering the first dose of this medication to the client? 1. Liver function tests 2. Renal function tests 3. Pancreatic enzyme studies 4. Complete blood cell count

4 Rationale: Carbamazepine (Tegretol) can cause leukopenia, anemia, thrombocytopenia, and, very rarely, fatal aplastic anemia. To reduce the risk of serious hematological effects, a complete blood cell count should be done before treatment and periodically thereafter. This medication should be avoided in clients with preexisting hematological abnormalities. The client also is told to report the occurrence of fever, sore throat, pallor, weakness, infection, easy bruising, and petechiae. Options 1, 2, and 3 are not associated with the use of this medication. Test-Taking Strategy: Focus on the subject, laboratory studies related to this medication. Recalling that carbamazepine causes hematological effects will direct you to the correct option. To help answer questions similar to this one, remember thatcarbamazepine and complete blood count begin with C.

A nurse is collecting data from a client and notes that the client is taking carbamazepine (Tegretol). The nurse determines that this medication has been prescribed to treat which condition? 1. Glaucoma 2. Diabetes mellitus 3. Parkinson's disease 4. Trigeminal neuralgia

4 Rationale: Carbamazepine is classified as an iminostilbene derivative and is used as an anticonvulsant, antineuralgic, antimanic, and antipsychotic. It is not used to treat any of the conditions noted in options 1, 2, or 3.

The health care provider (HCP) writes a prescription for carbamazepine (Tegretol) for a client who was admitted to the hospital. The nurse contacts the HCP to verify the prescription if which condition is noted in the assessment data? 1. Hypertension 2. Tonic-clonic seizures 3. Trigeminal neuralgia 4. Bone marrow depression

4 Rationale: Carbamazepine is classified as an iminostilbene derivative and is used as an anticonvulsant, antineuralgic, antimanic, and antipsychotic. It is used to treat seizure disorders, trigeminal neuralgia, and diabetic neuropathy. The medication can cause blood dyscrasias as an adverse effect and is contraindicated if the client has a history of bone marrow depression or hypersensitivity to tricyclic antidepressants or concurrent use of monoamine oxidase inhibitors.

A nurse is speaking with a client taking phenytoin (Dilantin) for seizure control. The client states that she has started using birth control pills to prevent pregnancy. Which would be an important point for the nurse to emphasize to the client? 1. Oral contraceptives decrease the effectiveness of phenytoin. 2. Severe gastrointestinal side effects can occur when phenytoin and oral contraceptives are taken together. 3. There is an increased risk of thrombophlebitis when phenytoin and oral contraceptives are taken at the same time. 4. Phenytoin may decrease effectiveness of birth control pills, and additional measures should be taken to avoid pregnancy.

4 Rationale: Phenytoin enhances the rate of estrogen metabolism, which can decrease the effectiveness of some birth control pills. The nurse should tell the client to alert the health care provider about the use of birth control pills so that counseling may be provided about alternative birth control methods. The other options are incorrect.

A nurse is providing instructions to a client beginning medication therapy with divalproex sodium (Depakote) for treatment of absence seizures. The nurse instructs the client that which represents the most frequent side effect of this medication? 1. Tinnitus 2. Irritability 3. Blue vision 4. Nausea and vomiting

4 Rationale: The most frequent side effects of medication therapy with divalproex sodium (Depakote) are gastrointestinal (GI) disturbances, such as nausea, vomiting, and indigestion. The items in the other options are incorrect.

A client is receiving anticonvulsant therapy with phenytoin (Dilantin). To monitor for adverse effects of this medication, the nurse assesses the results of which laboratory test? 1. Serum sodium 2. Serum potassium 3. Blood urea nitrogen 4. Complete blood count (CBC)

4 Rationale: The nurse monitors the CBC because hematological effects of this therapy include aplastic anemia, agranulocytosis, leukopenia, and thrombocytopenia. Other test results that warrant monitoring include serum calcium levels, urinalysis, and hepatic and thyroid function tests. Electrolyte results and renal function tests are not a concern with this medication

A nurse is reviewing the results of a test on a sample drawn from a child who is receiving carbamazepine (Tegretol) for the control of seizures. The results indicate a serum carbamazepine level of 10 mcg/mL. The nurse analyzes the results and anticipates that the health care provider (HCP) will note which prescription? 1. Discontinuation of the medication 2. An increased dose of the medication 3. A decreased dose of the medication 4. Continuation of the presently prescribed dosage

4 Rationale: When carbamazepine (Tegretol) is administered, blood levels need to be tested periodically to check for the child's absorption of the medication. The amount of the medication prescribed is based on the blood level achieved. Carbamazepine's therapeutic serum range is 3 to 14 mcg/mL. Therefore, the nurse anticipates that the HCP will continue the presently prescribed dosage.

A child is diagnosed with Reye's syndrome. The nurse creates a nursing care plan for the child and should include which intervention in the plan? 1. Assessing hearing loss 2. Monitoring urine output 3. Changing body position every 2 hours 4. Providing a quiet atmosphere with dimmed lighting

4 Reye's syndrome is an acute encephalopathy that follows a viral illness and is characterized pathologically by cerebral edema and fatty changes in the liver. In Reye's syndrome, supportive care is directed toward monitoring and managing cerebral edema. Decreasing stimuli in the environment by providing a quiet environment with dimmed lighting would decrease the stress on the cerebral tissue and neuron responses. Hearing loss and urine output are not affected. Changing the body position every 2 hours would not affect the cerebral edema directly. The child should be positioned with the head elevated to decrease the progression of the cerebral edema and promote drainage of cerebrospinal fluid.

A child who has undergone spinal fusion for scoliosis complains of abdominal discomfort and begins to have episodes of vomiting. On further assessment, the nurse notes abdominal distention. On the basis of these findings, the nurse should take which action? 1. Administer an antiemetic. 2. Increase the intravenous fluids. 3. Place the child in a Sims' position. 4. Notify the health care provider (HCP).

4 Scoliosis is a three-dimensional spinal deformity that usually involves lateral curvature, spinal rotation resulting in rib asymmetry, and hypokyphosis of the thorax. A complication after surgical treatment of scoliosis is superior mesenteric artery syndrome. This disorder is caused by mechanical changes in the position of the child's abdominal contents, resulting from lengthening of the child's body. The disorder results in a syndrome of emesis and abdominal distention similar to that which occurs with intestinal obstruction or paralytic ileus. Postoperative vomiting in children with body casts or children who have undergone spinal fusion warrants attention because of the possibility of superior mesenteric artery syndrome. Options 1, 2, and 3 are incorrect.

The community health nurse is providing information to parents of children in a local school regarding the signs of meningitis. The nurse informs the parents that the classic signs/symptoms of meningitis include which findings? 1. Nausea, delirium, and fever 2. Severe headache and back pain 3. Photophobia, fever, and confusion 4. Severe headache, fever, and a change in the level of consciousness

4 The classic signs/symptoms of meningitis include severe headache, fever, stiff neck, and a change in the level of consciousness. Photophobia also may be a prominent early manifestation and is thought to be related to meningeal irritation. Although nausea, confusion, delirium, and back pain may occur in meningitis, these are not the classic signs/symptoms.

A 9-year-old child fractures the left tibia along an epiphyseal line while using a skateboard. What is the nurse's priority concern during future growth? 1. Infection 2. Paralysis 3. Pressure ulcer 4. Uneven leg growth

4 The epiphyseal line is the area that is responsible for longitudinal bone growth. A fracture affecting this area places the child at risk for uneven future growth if proper healing does not occur. The epiphyses are located at the proximal and distal ends of a bone and are the insertion sites for muscles. The diaphysis is the shaft or main longitudinal portion of a long bone. The metaphysis is an area of flaring of bone, located between the epiphysis and the diaphysis. Paralysis, pressure ulcer, and infection are not priority concerns for future growth. Paralysis and neurovascular status are priority concerns during the immediate period postinjury, but not during future growth.

A client with a spinal cord injury at level C3 to C4 is being cared for by the nurse in the emergency department (ED). What is the priority nursing assessment? 1. Determine the level at which the client has intact sensation. 2. Assess the level at which the client has retained mobility. 3. Check blood pressure and pulse for signs of spinal shock. 4. Monitor respiratory effort and oxygen saturation level.

4 The first priority for the client with a spinal cord injury is assessing respiratory patterns and ensuring an adequate airway. A client with a high cervical injury is at risk for respiratory compromise because spinal nerves C3 through C5 innervate the phrenic nerve, which controls the diaphragm. The other assessments are also necessary but are not as high a priority.

An adolescent is seen in the emergency department for a suspected sprain of the ankle. X-rays have been obtained, and a fracture has been ruled out. Which instruction should the nurse provide to the adolescent regarding home care for treatment of the sprain? 1. Elevate the extremity, and maintain strict bed rest for a period of 7 days. 2. Immobilize the extremity, and maintain the extremity in a dependent position. 3. Apply heat to the injured area every 4 hours for the first 48 hours, and then begin to apply ice. 4. Apply ice to the injured area for a period of 30 minutes every 4 to 6 hours for the first 24 to 48 hours.

4 The injured area should be wrapped immediately to support the joint and control the swelling. Ice is applied to reduce the swelling and should be applied for not longer than 30 minutes every 4 to 6 hours for the first 24 to 48 hours. The joint should be immobilized and elevated, but strict bed rest for a period of 7 days is not required. A dependent position will cause swelling in the affected area.

The pediatric nurse educator provides a teaching session to the nursing staff regarding juvenile idiopathic arthritis (JIA). Which action by a nursing staff member in the care of a child with JIA indicates a need for further education? 1. Assesses for joint stiffness in the child 2. Encourages performance of isometric exercises 3. Administers nonsteroidal antiinflammatory medication 4. Emphasizes the importance of rising quickly in the mornings

4 The nursing plan of care for juvenile idiopathic arthritis (JIA) focuses on the status of affected joints. Isometric exercises and passive range of motion exercises will prevent contractures and deformities. Nonsteroidal antiinflammatory medications are used to treat joint stiffness and pain. The child may need more time than average to begin morning activities.

An LPN/LVN, under the RN's supervision, is assigned to provide nursing care for a client with Guillain-Barré syndrome (GBS). What observation should the LPN/LVN be instructed to report immediately? 1. Reports of numbness and tingling 2. Facial weakness and difficulty speaking 3. Rapid heart rate of 102 beats/min 4. Shallow respirations and decreased breath sounds

4 The priority intervention for a client with GBS is maintaining adequate respiratory function. Clients with GBS are at risk for respiratory failure, which requires urgent intervention. The other findings are important and should be reported to the nurse, but they are not life threatening.

The nurse is assessing a child with increased intracranial pressure. On assessment, the nurse notes that the child is now exhibiting decerebrate posturing. The nurse should modify the client's plan of care based on which interpretation of the client's change? 1. An insignificant finding 2. An improvement in condition 3. Decreasing intracranial pressure 4. Deteriorating neurological function

4 The progression from decorticate to decerebrate posturing usually indicates deteriorating neurological function and warrants health care provider notification. The remaining options are inaccurate interpretations.

The nurse is assessing for Kernig's sign in a child with a suspected diagnosis of meningitis. Which action should the nurse perform for this test? 1. Tap the child's facial nerve and assess for spasm. 2. Compress the child's upper arm and assess for tetany. 3. Bend the child's head toward the knees and hips and assess for pain. 4. Raise the child's leg with the knee flexed and then extend the leg at the knee and assess for pain.

4 To test for Kernig's sign, the client's leg is raised with the knee flexed and then extended at the knee. If any resistance is noted or pain is felt, the result is a positive Kernig's sign. This is a common finding in meningitis. Chvostek's sign, seen in tetany, is a spasm of the facial muscles elicited by tapping the facial nerve in the region of the parotid gland. Trousseau's sign is a sign for tetany in which carpal spasm can be elicited by compressing the upper arm and causing ischemia to the nerves distally. Brudzinski's sign occurs when flexion of the head causes flexion of the hips and knees.

After the nurse receives the change-of-shift report at 7:00 am, which client must the nurse assess first? 1. A 23-year-old client with a migraine headache who reports severe nausea associated with retching 2. A 45-year-old client who is scheduled for a craniotomy in 30 minutes and needs preoperative teaching 3. A 59-year-old client with Parkinson disease who will need a swallowing assessment before breakfast 4. A 63-year-old client with multiple sclerosis (MS) who has an oral temperature of 101.8°F (38.8°C) and flank pain

4 Urinary tract infections (UTIs) are a frequent complication in clients with MS because of the effect of the disease on bladder function, and UTIs may lead to sepsis in these clients. The elevated temperature and flank pain suggest that this client may have pyelonephritis. The health care provider should be notified immediately so that IV antibiotic therapy can be started quickly. The other clients should be assessed as soon as possible, but their needs are not as urgent as those of this client.

A child is placed in skeletal traction for treatment of a fractured femur. The nurse creates a plan of care and should include which intervention? 1. Ensure that all ropes are outside the pulleys. 2. Ensure that the weights are resting lightly on the floor. 3. Restrict diversional and play activities until the child is out of traction. 4. Check the health care provider's (HCP's) prescriptions for the amount of weight to be applied.

4 When a child is in traction, the nurse would check the HCP's prescription to verify the prescribed amount of traction weight. The nurse would maintain the correct amount of weight as prescribed, ensure that the weights hang freely, check the ropes for fraying and ensure that they are on the pulleys appropriately, monitor the neurovascular status of the involved extremity, and monitor for signs and symptoms of immobilization. The nurse would provide therapeutic and diversional play activities for the child.

The nurse is preparing for the admission of a client with a suspected diagnosis of Guillain-Barré syndrome. When the client arrives at the nursing unit, the nurse reviews the health care provider's documentation. The nurse expects to note documentation of which hallmark clinical manifestation of this syndrome? 1. Multifocal seizures 2. Altered level of consciousness 3. Abrupt onset of a fever and headache 4. Development of progressive muscle weakness

4 Rationale: A hallmark clinical manifestation of Guillain-Barré syndrome is progressive muscle weakness that develops rapidly. Seizures are not normally associated with this disorder. The client does not have symptoms such as a fever or headache. Cerebral function, level of consciousness, and pupillary responses are normal.

The nurse assists a health care provider in performing a lumbar puncture on a 3-year-old child with leukemia in whom central nervous system disease is suspected. In which position will the nurse place the child during this procedure? 1.Lithotomy position 2.Modified Sims position 3.Prone with knees flexed to the abdomen and head bent with chin resting on the chest 4.Lateral recumbent position with the knees flexed to the abdomen and head bent with the chin resting on chest

4 Rationale: A lateral recumbent position with the knees flexed to the abdomen and the head bent with the chin resting on the chest is assumed for a lumbar puncture. This position separates the spinal processes and facilitates needle insertion into the subarachnoid space. Options 1, 2, and 3 are incorrect positions.

A nurse is preparing to administer a prescribed antibiotic to a client with bacterial meningitis. The nurse understands that the selection of an antibiotic to treat meningitis is based on which fact? 1. It has a long half-life. 2. It acts within minutes to hours. 3. It can be easily excreted in the urine. 4. It is able to cross the blood-brain barrier.

4 Rationale: A primary consideration regarding medications to treat bacterial meningitis is the ability of the medication to cross the blood-brain barrier. If the medication cannot cross, it will not be effective. The duration, onset, and excretion of the medication(options 1, 2, and 3) are also of general concern but apply to all medications and not specifically to those that are used to treat meningitis.

The nurse develops a plan of care for a child at risk for tonic-clonic seizures. In the plan of care, the nurse identifies seizure precautions and documents that which item(s) need to be placed at the child's bedside? 1.Emergency cart 2.Tracheotomy set 3.Padded tongue blade 4.Suctioning equipment and oxygen

4 Rationale: A seizure results from the excessive and unorganized neuronal discharges in the brain that activate associated motor and sensory organs. A type of generalized seizure is a tonic-clonic seizure. This type of seizure causes rigidity of all body muscles, followed by intense jerking movements. Because increased oral secretions and apnea can occur during and after the seizure, oxygen and suctioning equipment are placed at the bedside. A tracheotomy is not performed during a seizure. No object, including a padded tongue blade, is placed into the child's mouth during a seizure. An emergency cart would not be left at the bedside, but would be available in the treatment room or nearby on the nursing unit.

The nurse is performing an assessment of a 7-year-old child who is suspected of having episodes of absence seizures. Which assessment question to the mother will assist in providing information that will identify the symptoms associated with this type of seizure? 1. "Does twitching occur in the face and neck?" 2. "Does the muscle twitching occur on one side of the body?" 3. "Does the muscle twitching occur on both sides of the body?" 4. "Does the child have a blank expression during these episodes?"

4 Rationale: Absence seizures are brief episodes of altered awareness or momentary loss of consciousness. No muscle activity occurs except eyelid fluttering or twitching. The child has a blank facial expression. These seizures last only 5 to 10 seconds, but they may occur one after another several times a day. Simple partial seizures consist of twitching of an extremity, face, or neck, or the sensation of twitching or numbness in an extremity or face or neck. Myoclonic seizures are brief random contractions of a muscle group that can occur on one or both sides of the body.

A nurse caring for an infant with a diagnosis of hydrocephalus should monitor the infant for which sign of increased intracranial pressure (ICP)? 1.Proteinuria 2.Bradycardia 3.A drop in blood pressure 4.A bulging anterior fontanel

4 Rationale: An elevated or bulging anterior fontanel indicates an increase in cerebrospinal fluid collection in the cerebral ventricle. Proteinuria, bradycardia, and a drop in blood pressure are not specific signs of increased ICP. Changes in the level of consciousness and a widened pulse pressure are additional signs of increased ICP.

The nurse is caring for a client with trigeminal neuralgia (tic douloureux). The client asks for a snack and something to drink. The nurse should offer which best snack to the client? 1.Cocoa with honey and toast 2.Hot herbal tea with graham crackers 3.Iced coffee and peanut butter and crackers 4.Vanilla wafers and room-temperature water

4 Rationale: Because mild tactile stimulation of the face can trigger pain in trigeminal neuralgia, the client needs to eat or drink lukewarm, nutritious foods that are soft and easy to chew. Extremes of temperature will cause trigeminal nerve pain. Therefore, options 1, 2, and 3 are incorrect.

The nurse is assisting with caloric testing of the oculovestibular reflex in an unconscious client. Cold water is injected into the left auditory canal. The client exhibits eye conjugate movements toward the left, followed by eye movement back to midline. The nurse understands that this finding indicates which situation? 1.Brain death 2.A cerebral lesion 3.A temporal lesion 4.An intact brainstem

4 Rationale: Caloric testing provides information about differentiating between cerebellar and brainstem lesions. After determining patency of the ear canal, cold or warm water is injected into the auditory canal. A normal response that indicates intact function of cranial nerves III, VI, and VIII is conjugate eye movements toward the side being irrigated, followed by eye movement back to midline. Absent or dysconjugate eye movements indicate brainstem damage.

The nurse is planning care for a client who displays confusion secondary to a neurological problem. Which approach by the nurse would be least helpful in assisting this client? 1.Providing sensory cues 2.Giving simple, clear directions 3.Providing a stable environment 4. Encouraging multiple visitors at one time

4 Rationale: Clients with cognitive impairment from neurological dysfunction respond best to a stable environment that is limited in amount and type of sensory input. The nurse can provide sensory cues and give clear, simple directions in a positive manner. Confusion can be minimized by reducing environmental stimuli (such as television or multiple visitors) and by keeping familiar personal articles (such as family pictures) at the bedside.

The nurse is teaching a client with myasthenia gravis about the prevention of myasthenic and cholinergic crises. Which client activity suggests that teaching is most effective? 1.Eating large, well-balanced meals 2.Doing muscle-strengthening exercises 3.Doing all chores early in the day while less fatigued 4. Taking medications on time to maintain therapeutic blood levels

4 Rationale: Clients with myasthenia gravis are taught to space out activities over the day to conserve energy and restore muscle strength. Taking medications correctly to maintain blood levels that are not too low or too high is important. Muscle-strengthening exercises are not helpful and can fatigue the client. Overeating is a cause of exacerbation of symptoms, as is exposure to heat, crowds, erratic sleep habits, and emotional stress.

The client has an impairment of cranial nerve II. Specific to this impairment, what should the nurse should plan to do to ensure client safety? 1. Speak loudly to the client. 2.Test the temperature of the shower water. 3.Check the temperature of the food on the dietary tray. 4. Provide a clear path for ambulation without obstacles.

4 Rationale: Cranial nerve II is the optic nerve, which governs vision. The nurse can provide safety for the visually impaired client by clearing the path of obstacles when ambulating. Speaking loudly may help overcome a deficit of cranial nerve VIII (vestibulocochlear). Testing the shower water temperature would be useful if there were an impairment of peripheral nerves. Cranial nerve VII (facial) and IX (glossopharyngeal) control taste from the anterior two thirds and posterior third of the tongue, respectively.

The nurse is providing diet instructions to a client with Ménière's disease who is being discharged from the hospital after admission for an acute attack. Which statement, if made by the client, indicates an understanding of the dietary measures to take to help prevent further attacks? 1."I need to restrict my carbohydrate intake." 2."I need to drink at least 3 L of fluid per day." 3."I need to maintain a low-fat and low-cholesterol diet." 4. "I need to be sure to consume foods that are low in sodium."

4 Rationale: Dietary changes, such as salt and fluid restrictions, that reduce the amount of endolymphatic fluid are sometimes prescribed for the client with Ménière's disease. The client should be instructed to consume a low-sodium diet and restrict fluids as prescribed. Although helpful to treat other disorders, low-fat, low-carbohydrate, and low-cholesterol diets are not specifically prescribed for the client with Ménière's disease.

The nurse is caring for an infant with spina bifida (myelomeningocele type) who had the sac on the back containing cerebrospinal fluid, the meninges, and the nerves (gibbus) surgically removed. The nursing plan of care for the postoperative period should include which nursing action to maintain the infant's safety? 1.Covering the back dressing with a binder 2. Placing the infant in a head-down position 3. Strapping the infant in a baby seat sitting up 4. Elevating the head with the infant in the prone position

4 Rationale: Elevating the head will decrease the chance that cerebrospinal fluid will accumulate in the cranial cavity. The infant needs to be prone or side-lying to decrease the pressure on the surgical site on the back. Binders and a baby seat should not be used because of the pressure they would exert on the surgical site.

The client is admitted to the hospital with a diagnosis of Guillain-Barré syndrome. Which past medical history finding makes the client most at risk for this disease? 1.Meningitis or encephalitis during the last 5 years 2.Seizures or trauma to the brain within the last year 3. Back injury or trauma to the spinal cord during the last 2 years 4.Respiratory or gastrointestinal infection during the previous month

4 Rationale: Guillain-Barré syndrome is a clinical syndrome of unknown origin that involves cranial and peripheral nerves. Many clients report a history of respiratory or gastrointestinal infection in the 1 to 4 weeks before the onset of neurological deficits. On occasion, the syndrome can be triggered by vaccination or surgery.

A nurse notes that an infant with the diagnosis of hydrocephalus has a head that is heavier than that of the average infant. The nurse should determine that special safety precautions are needed when moving the infant with hydrocephalus. Which statement should the nurse plan to include in the discharge teaching with the parents to reflect this safety need? 1."Feed your infant in a side-lying position." 2. "Place a helmet on your infant when in bed." 3. "Hyperextend your infant's head with a rolled blanket under the neck area." 4. "When picking up your infant, support the infant's neck and head with the open palm of your hand."

4 Rationale: Hydrocephalus is a condition characterized by an enlargement of the cranium because of an abnormal accumulation of cerebrospinal fluid in the cerebral ventricular system. This characteristic causes the increase in the weight of the infant's head. The infant may experience significant head enlargement. Care must be exercised so that the head is well supported when the infant is fed or moved to prevent extra strain on the infant's neck, and measures must be taken to prevent the development of pressure areas. Supporting the infant's head and neck when picking up the infant will prevent the hyperextension of the neck area and the infant from falling backward. The infant should be fed with the head elevated for proper motility of food processing. A helmet could suffocate an unattended infant during rest and sleep times, and hyperextension of the infant's head could put pressure on the neck vertebrae, causing injury.

The nurse is caring for an unconscious client who is experiencing persistent hyperthermia with no signs of infection. The nurse interprets that the hyperthermia may be related to damage to the client's thermoregulatory center in which structure? 1.Cerebrum 2.Cerebellum 3.Hippocampus 4.Hypothalamus

4 Rationale: Hypothalamic damage causes persistent hyperthermia, which also may be called central fever. It is characterized by a persistent high fever with no diurnal variation. Another characteristic feature is absence of sweating. Hyperthermia would not result from damage to the cerebrum, cerebellum, or hippocampus.

The nurse in the neurological unit is caring for a client with a supratentorial lesion. The nurse assesses which measurement as the most critical index of central nervous system (CNS) dysfunction? 1.Temperature 2.Blood pressure 3.Ability to speak 4.Level of consciousness

4 Rationale: Level of consciousness is the most critical index of CNS dysfunction. Changes in level of consciousness can indicate clinical improvement or deterioration. Although blood pressure, temperature, and ability to speak may be components of the assessment, the client's level of consciousness is the most critical index of CNS dysfunction.

The nurse notes documentation that a child with meningitis is exhibiting a positive Kernig's sign. Which observation is characteristic of this sign? 1. The child complains of muscle and joint pain. 2. Petechial and purpuric rashes are noted on the child's trunk. 3. Neck flexion causes adduction and flexion movements of the lower extremities. 4. The child is not able to extend the leg when the thigh is flexed anteriorly at the hip.

4 Rationale: Meningitis is an infectious process of the central nervous system caused by bacteria and viruses. The inability to extend the leg when the thigh is flexed anteriorly at the hip is a positive Kernig's sign, noted in meningitis. Muscle and joint pain is characteristic of meningococcal infection and H. influenzae infection. A petechial or purpuric rash is characteristic of meningococcal infection. A positive Brudzinski's sign is noted when neck flexion causes adduction and flexion movements of the lower extremities in children and adolescents. This is also a characteristic of meningitis.

A child is diagnosed with Reye's syndrome. The nurse develops a nursing care plan for the child and should include which intervention in the plan? 1.Assessing hearing loss 2.Monitoring urine output 3.Changing body position every 2 hours 4.Providing a quiet atmosphere with dimmed lighting

4 Rationale: Reye's syndrome is an acute encephalopathy that follows a viral illness and is characterized pathologically by cerebral edema and fatty changes in the liver. A definitive diagnosis is made by liver biopsy. In Reye's syndrome, supportive care is directed toward monitoring and managing cerebral edema. Decreasing stimuli in the environment by providing a quiet environment with dimmed lighting would decrease the stress on the cerebral tissue and neuron responses. Hearing loss and urine output are not affected. Changing the body position every 2 hours would not affect the cerebral edema directly. The child should be positioned with the head elevated to decrease the progression of the cerebral edema and promote drainage of cerebrospinal fluid.

The community health nurse is providing information to parents of children in a local school regarding the signs of meningitis. The nurse informs the parents that the classic signs/symptoms of meningitis include which findings? 1.Nausea, delirium, and fever 2.Severe headache and back pain 3.Photophobia, fever, and confusion 4.Severe headache, fever, and a change in the level of consciousness

4 Rationale: The classic signs/symptoms of meningitis include severe headache, fever, stiff neck, and a change in the level of consciousness. Photophobia also may be a prominent early manifestation and is thought to be related to meningeal irritation. Although nausea, confusion, delirium, and back pain may occur in meningitis, these are not the classic signs/symptoms.

The nurse is teaching a client hospitalized with a seizure disorder and the client's spouse about safety precautions after discharge. The nurse determines that the client needs more information if he or she states an intention to take which action? 1.Refrain from smoking alone. 2.Take all prescribed medications on time. 3.Have the spouse nearby when showering. 4.Drink alcohol in small amounts and only on weekends.

4 Rationale: The client should avoid the intake of alcohol. Alcohol could interact with the client's seizure medications, or the alcohol could precipitate seizure activity. The client should take all medications on time to avoid decreases in therapeutic drug levels, which could precipitate seizures. The client should not bathe in the shower or tub without someone nearby and should not smoke alone, to minimize the risk of injury if a seizure occurs.

The nurse is assessing a child with increased intracranial pressure. On assessment, the nurse notes that the child is now exhibiting decerebrate posturing. The nurse should modify the client's plan of care based on which interpretation of the client's change? 1.An insignificant finding 2.An improvement in condition 3.Decreasing intracranial pressure 4.Deteriorating neurological function

4 Rationale: The progression from decorticate to decerebrate posturing usually indicates deteriorating neurological function and warrants health care provider notification. Options 1, 2, and 3 are inaccurate interpretations.

The home health nurse has been discussing interventions to prevent constipation in a client with multiple sclerosis. The nurse determines that the client is using the information most effectively if the client reports which action? 1.Drinking a total of 1000 mL/day 2.Giving herself an enema every morning before breakfast 3.Taking stool softeners daily and a glycerin suppository once a week 4. Initiating a bowel movement every other day, 45 minutes after the largest meal of the day

4 Rationale: To manage constipation, the client should take in a high-fiber diet, bulk formers, and stool softeners. A fluid intake of 2000 mL/day is recommended. The client should initiate the bowel program on an every-other-day basis. This should be done approximately 45 minutes after the largest meal of the day to take advantage of the gastrocolic reflex. A glycerin suppository, bisacodyl suppository, or digital stimulation may be used to initiate the process. Laxatives and enemas should be avoided whenever possible because they lead to dependence.

A client who has a spinal cord injury that resulted in paraplegia experiences a sudden onset of severe headache and nausea. The client is diaphoretic with piloerection and has flushing of the skin. The client's systolic blood pressure (BP) is 210 mm Hg. What should the nurse immediately suspect? 1.Return of spinal shock 2.Malignant hypertension 3.Impending brain attack (stroke) 4.Autonomic dysreflexia (hyperreflexia)

4. Autonomic dysreflexia (hyperreflexia) results from sudden strong discharge of the sympathetic nervous system in response to a noxious stimulus. Signs and symptoms include pounding headache, nausea, nasal stuffiness, flushed skin, piloerection, and diaphoresis. Severe hypertension can occur, with a systolic BP rising potentially as high as 300 mm Hg. It often is triggered by thermal or mechanical events such as a kinking of catheter tubing, constipation, urinary tract infection, or any variety of cutaneous stimuli. The nurse must recognize this situation immediately and take corrective action to remove the stimulus. If untreated, this medical emergency could result in stroke, status epilepticus, or possibly death. * Note the strategic word immediately. Focus on the client's diagnosis, spinal cord injury, and recall the complications that can occur after this type of injury. Noting the client's signs and symptoms and recalling these complications will direct you to the correct option.

The nurse is performing an assessment on a client with a diagnosis of thrombotic brain attack (stroke). Which assessment question would elicit data specific to this type of stroke? 1."Have you had any headaches in the past few days?" 2."Have you recently been having difficulty with seeing at nighttime?" 3."Have you had any sudden episodes of passing out in the past few days?" 4."Have you had any numbness or tingling or paralysis-type feelings in any of your extremities recently?"

4. Cerebral thrombosis (thrombotic stroke) does not occur suddenly. In the few days or hours preceding the thrombotic stroke, the client may experience a transient loss of speech, hemiparesis, or paresthesias on one side of the body. Signs and symptoms of this type of stroke vary but may also include dizziness, cognitive changes, or seizures. Headache is rare, but some clients with stroke experience signs and symptoms similar to those of cerebral embolism or intracranial hemorrhage. The client does not complain of difficulty with night vision as part of this clinical problem. In addition, most clients do not have repeated episodes of loss of consciousness. * Focus on the subject, the signs and symptoms of a thrombotic stroke. It is necessary to know that cerebral thrombosis does not occur suddenly, and in the few days or hours preceding the thrombotic stroke, the client may experience a transient loss of speech, hemiparesis, or paresthesias on one side of the body.

The nurse has a prescription to begin aneurysm precautions for a client with a subarachnoid hemorrhage secondary to aneurysm rupture. The nurse would plan to incorporate which intervention in controlling the environment for this client? 1.Keep the window blinds open. 2.Turn on a small spotlight above the client's head. 3.Make sure the door to the room is open at all times. 4.Prohibit or limit the use of a radio or television and reading.

4. Environmental stimuli are kept to a minimum with subarachnoid precautions to prevent or minimize increases in intracranial pressure. For this reason, lighting is reduced by closing window blinds and keeping the door to the client's room shut. Overhead lighting also is avoided for the same reason. The nurse prohibits television, radio, and reading unless this is so stressful for the client that it would be counterproductive. In that instance, minimal amounts of stimuli by these means are allowed with approval of the health care provider. * Focus on the subject, aneurysm precautions, noting the words controlling the environment. Then analyze each of the options in terms of how the intervention could affect the intracranial pressure.

The nurse is preparing a plan of care for a client with a brain attack (stroke) who has global aphasia. The nurse should incorporate communication strategies into the plan of care because the client's speech will be characteristic of which finding? 1.Intact 2.Rambling 3.Characterized by literal paraphasia 4.Associated with poor comprehension

4. Global aphasia is a condition in which the affected person has few language skills as a result of extensive damage to the left hemisphere. The speech is nonfluent and is associated with poor comprehension and limited ability to name objects or repeat words. The client with conduction aphasia has difficulty repeating words spoken by another, and speech is characterized by literal paraphasia with intact comprehension. The client with Wernicke's aphasia may exhibit a rambling type of speech. * Focus on the subject, global aphasia. Knowledge of the characteristics associated with global aphasia is required to answer this question. Remember that global aphasia is associated with poor comprehension.

The nurse is developing a plan of care for a client with a stroke (brain attack) who has right homonymous hemianopsia. Which should the nurse include in the plan of care for the client? 1.Place an eye patch on the left eye. 2.Place personal articles on the client's right side. 3.Approach the client from the right field of vision. 4.Instruct the client to turn the head to scan the right visual field.

4. Homonymous hemianopsia is a loss of half of the visual field. The nurse instructs the client to scan the environment and stands within the client's intact field of vision. The nurse should not patch the eye because the client does not have double vision. The client should have objects placed in the intact fields of vision, and the nurse should approach the client from the intact side. * Focus on the subject, a visual problem, and recall the definition of homonymous hemianopsia. Recalling that the client loses half of the visual field will assist in directing you to the correct option.

The nurse is preparing a plan of care for a client with a diagnosis of brain attack (stroke). On reviewing the client's record, the nurse notes an assessment finding of anosognosia. The nursing care plan should address which manifestation related to this finding? 1.The client will be easily fatigued. 2.The client will have difficulty speaking. 3.The client will have difficulty swallowing. 4.The client will exhibit neglect of the affected side.

4. In anosognosia, the client neglects the affected side of the body. The client either may ignore the presence of the affected side (often creating a safety hazard as a result of potential injuries) or may state that the involved arm or leg belongs to someone else. Options 1, 2, and 3 are not associated with anosognosia. * Focus on the subject, care of the client with a brain attack (stroke). Knowledge of the definition of anosognosia and the associated manifestations is required to answer this question. Remember that in anosognosia the client neglects the affected side of the body.

The nurse in the neurological unit is caring for a client with a supratentorial lesion. The nurse assesses which measurement as the most critical index of central nervous system (CNS) dysfunction? 1.Temperature 2.Blood pressure 3.Ability to speak 4.Level of consciousness

4. Level of consciousness is the most critical index of CNS dysfunction. Changes in level of consciousness can indicate clinical improvement or deterioration. Although blood pressure, temperature, and ability to speak may be components of the assessment, the client's level of consciousness is the most critical index of CNS dysfunction. * Note the strategic word most in the question. Focusing on the subject of the question, a neurological problem, will assist in directing you to the correct option. Also, the correct option is the umbrella option.

The nurse is teaching a client hospitalized with a seizure disorder and the client's spouse about safety precautions after discharge. The nurse determines that the client needs more information if he or she states an intention to take which action? 1.Refrain from smoking alone. 2.Take all prescribed medications on time. 3.Have the spouse nearby when showering. 4.Drink alcohol in small amounts and only on weekends.

4. The client should avoid the intake of alcohol. Alcohol could interact with the client's seizure medications, or the alcohol could precipitate seizure activity. The client should take all medications on time to avoid decreases in therapeutic drug levels, which could precipitate seizures. The client should not bathe in the shower or tub without someone nearby and should not smoke alone, to minimize the risk of injury if a seizure occurs. * Note the strategic words needs more information. This phrase indicates a negative event query and the need to select the incorrect client statement. The word safety guides you to think about appropriate safety measures that should be used in the home. Use general medication guidelines to assist in answering correctly, and remember that alcohol is restricted, not limited.

The nurse is caring for a client with a diagnosis of right (nondominant) hemispheric stroke. The nurse notes that the client is alert and oriented to time and place. On the basis of these assessment findings, the nurse should make which interpretation? 1.Had a very mild stroke 2.Most likely suffered a transient ischemic attack 3.May have difficulty with language abilities only 4.Is likely to have perceptual and spatial disabilities

4. The client with a right (nondominant) hemispheric stroke may be alert and oriented to time and place. These signs of apparent wellness often suggest that the client is less disabled than is the case. However, impulsivity and confusion in carrying out activities may be very real problems for these clients as a result of perceptual and spatial disabilities. The right hemisphere is considered specialized in sensory-perceptual and visual-spatial processing and awareness of body space. The left hemisphere is dominant for language abilities. * Think about the anatomical location of the stroke. Eliminate option 3 first because of the closed-ended word only. Regarding the remaining options, it is necessary to recall that perceptual and spatial disabilities occur in the client with a right hemispheric stroke.

The nurse is providing instructions to the client with trigeminal neuralgia regarding measures to take to prevent the episodes of pain. Which should the nurse instruct the client to do? 1.Prevent stressful situations. 2.Avoid activities that may cause fatigue. 3.Avoid contact with people with an infection. 4.Avoid activities that may cause pressure near the face.

4. The pain that accompanies trigeminal neuralgia is triggered by stimulation of the trigeminal nerve. Symptoms can be triggered by pressure such as from washing the face, brushing the teeth, shaving, eating, or drinking. Symptoms also can be triggered by stimulation by a draft or cold air. Options 1, 2, and 3 are not associated with triggering episodes of pain. * Focus on the subject, measures to prevent pain. Focus on the pathophysiology associated with this disorder, and think about the precipitating factors to answer this question. Recalling the anatomical location of the trigeminal nerve and that the pain is triggered by stimulation of the trigeminal nerve will direct you to the correct option.

The nurse is performing an assessment on a client suspected of having trigeminal neuralgia (tic douloureux). Which assessment question would elicit data specific to this disorder? 1."Have you had any facial paralysis?" 2."Have you noticed that your eyelid has been drooping?" 3."Have you had any numbness and tingling in your face?" 4."Have you had any sharp pain or any twitching in any part of your face?"

4. Trigeminal neuralgia is characterized by spasms of pain that start suddenly and last from seconds to minutes. The pain often is described as either stabbing or similar to an electric shock. It is accompanied by spasms of the facial muscles that cause twitching of parts of the face or mouth, or closure of the eye. * Focus on the subject, the manifestations of trigeminal neuralgia. Note the word tic in the name of the disorder. Recalling that a tic is a nervous twitch will assist in directing you to the correct option.

The nurse is planning care for the client with a neurogenic bladder caused by multiple sclerosis. The nurse plans for fluid administration of at least 2000 mL/day. Which plan would be most helpful to this client? 1. 400 to 500 mL with each meal and 500 to 600 mL in the evening before bedtime 2. 400 to 500 mL with each meal and additional fluids in the morning but not after midday 3. 400 to 500 mL with each meal, with all extra fluid concentrated in the afternoon and evening 4. 400 to 500 mL with each meal and 200 to 250 mL at midmorning, midafternoon, and late afternoon

400 to 500 mL with each meal and 200 to 250 mL at midmorning, midafternoon, and late afternoon Spacing fluid intake over the day helps the client with a neurogenic bladder to establish regular times for successful voiding. Omitting intake after the evening meal minimizes incontinence or the need to empty the bladder during the night.

A mother arrives at an emergency department with her 5-year-old child and states that the child fell off a bunk bed. A head injury is suspected, and the nurse checks the child's airway status and assesses the child for early and late signs of increased intracranial pressure (ICP). Which is a late sign of increased ICP? 1.Nausea 2.Irritability 3.Headache 4. Bradycardia

4Rationale: Head injury is the pathological result of any mechanical force to the skull, scalp, meninges, or brain. A head injury can cause bleeding in the brain and result in increased intracranial pressure (ICP). In a child, early signs include a slight change in level of consciousness, headache, nausea, vomiting, visual disturbances (diplopia), seizures. Late signs of increased ICP include a significant decrease in level of consciousness, bradycardia, decreased motor and sensory responses, alterations in pupil size and reactivity, posturing, Cheyne-Stokes respirations, and coma.

The nurse is preparing to administer tissue plasminogen activator (t-PA) to a patient who weighs 132 lb. The order reads 0.9 mg/kg t-PA. The nurse understands that 10% of the calculated dose is administered as an IV bolus over 1 minute, and the remaining dose (90%) is administered IV over 1 hour via an infusion pump. How many milligrams IV bolus over 1 minute will the nurse initially administer?

5.4 The patient is weighed to determine the dose of t-PA. Typically two or more IV sites are established prior to administration of t-PA (one for the t-PA and the other for administration of IV fluids). The dosage for t-PA is 0.9 mg/kg, with a maximum dose of 90 mg. Of the calculated dose, 10% is administered as an IV bolus over 1 minute. The remaining dose (90%) is administered IV over 1 hour via an infusion pump. First, the nurse must convert the patient's weight to kilograms (132/2.2 = 60 kg), then multiply 0.9 mg × 60 kg = 54 mg. Next, the nurse figure out that 10% of 54 mg is 5.4 (54 ×.10). The nurse will initially administer 5.4 mgs IV bolus over 1 minute.

Because of symptoms experienced after a cerebrovascular accident (CVA), the nurse discovers that a client needs assistance using utensils while eating. What would the nurse do to support this activity of care? a) Encourage participation in the feeding process to the best of the client's abilities. b) Feed another patient and wait in the dining room until the client can accomplish feeding. c) Request that the client's food be pureed by dietary staff. d) Have the family feed the client at every meal to reduce staffing limitations.

A

The nurse is caring for a client following a supratentorial craniotomy, in which a large tumor was removed from the left side. In which position can the nurse safely place the client?

A Clients who have undergone supratentorial surgery should have the head of the bed elevated 30 degrees to promote venous drainage from the head. The client is positioned to avoid extreme hip or neck flexion and the head is maintained in a midline neutral position. If a large tumor has been removed, the client should be placed on the nonoperative side to prevent displacement of the cranial contents. A flat position or Trendelenburg's position would increase intracranial pressure. A reverse Trendelenburg's position would not be helpful and may be uncomfortable for the client.

The nurse is assisting the neurologist in performing an assessment on a client who is unconscious after sustaining a head injury. The nurse understands that the neurologist would avoid performing the oculocephalic response (doll's eyes maneuver) if which condition is present in the client? 1. Dilated pupils 2. Lumbar trauma 3. A cervical cord injury 4. Altered level of consciousness

A cervical cord injury In an unconscious client, eye movements are an indication of brainstem activity and are tested by the oculocephalic response. When the doll's eyes maneuver is intact, the eyes move in the opposite direction when the head is turned. Abnormal responses include movement of the eyes in the same direction as that for the head and maintenance of a midline position of the eyes when the head is turned. An abnormal response indicates a disruption in the processing of information through the brainstem. Contraindications to performing this test include cervical-level spinal cord injuries and severely increased intracranial pressure.

A 6-month-old infant is admitted with a diagnosis of bacterial meningitis. The nurse would place the infant in which room? a) A room with an 8-month-old infant with failure to thrive b) A private room near the nurses' station c) A room with a 12-month-old infant with a urinary tract infection d) A two-bed room in the middle of the hall

A private room near the nurses' station Correct Explanation: A child who has the diagnosis of bacterial meningitis will need to be placed in a private room until that child has received I.V. antibiotics for 24 hours because the child is considered contagious. Additionally, bacterial meningitis can be quite serious; therefore, the child should be placed near the nurses' station for close monitoring and easier access in case of a crisis.

An 84-year-old client in an acute state of disorientation was brought to the emergency department by the client's daughter. The daughter states that this is the first time that the client experienced confusion. The nurse determines from this piece of information that which is unlikely to be the cause of the client's disorientation?

Alzheimer's disease Alzheimer's disease is a chronic disease with progression of memory deficits over time. The situation presented in the question represents an acute problem. Medication use, hypoglycemia, and impaired cerebral circulation require evaluation to determine if they play a role in causing the client's current symptoms.

The nurse overhears the term sundowning used to describe the behavior of a client newly admitted to the nursing unit during the previous evening shift. Of which diagnosis is sundowning a symptom?

Alzheimer's disease The term sundowning or sundown syndrome refers to a pattern of disorientation in which the client is more oriented during the daytime hours and more disoriented at night. It is seen often in clients with Alzheimer's disease. It is not a characteristic of the conditions noted in the other options.

A client with myasthenia gravis is having difficulty with airway clearance and difficulty with maintaining an effective breathing pattern. The nurse should keep which most important items available at the client's bedside? 1. Oxygen and metered-dose inhaler 2. Ambu bag and suction equipment 3. Pulse oximeter and cardiac monitor 4. Incentive spirometer and cough pillow

Ambu bag and suction equipment The client with myasthenia gravis may experience episodes of respiratory distress if excessively fatigued or with development of myasthenic or cholinergic crisis. For this reason, an Ambu bag, intubation tray, and suction equipment should be available at the bedside.

The nurse is assisting with caloric testing of the oculovestibular reflex in an unconscious client. Cold water is injected into the left auditory canal. The client exhibits eye conjugate movements toward the left, followed by eye movement back to midline. The nurse understands that this finding indicates which situation? 1. Brain death 2. A cerebral lesion 3. A temporal lesion 4. An intact brainstem

An intact brainstem Caloric testing provides information about differentiating between cerebellar and brainstem lesions. After determining patency of the ear canal, cold or warm water is injected into the auditory canal. A normal response that indicates intact function of cranial nerves III, VI, and VIII is conjugate eye movements toward the side being irrigated, followed by eye movement back to midline. Absent or dysconjugate eye movements indicate brainstem damage.

The nurse is assisting with caloric testing of the oculovestibular reflex in an unconscious client. Cold water is injected into the left auditory canal. The client exhibits eye conjugate movements toward the left, followed by eye movement back to midline. The nurse understands that this finding indicates which situation? 1. Brain death 2. A cerebral lesion 3. A temporal lesion 4. An intact brainstem

An intact brainstem Caloric testing provides information about differentiating between cerebellar and brainstem lesions. After determining patency of the ear canal, cold or warm water is injected into the auditory canal. A normal response that indicates intact function of cranial nerves III, VI, and VIII is conjugate eye movements toward the side being irrigated, followed by eye movement back to midline. Absent or dysconjugate eye movements indicate brainstem damage.

A nurse is performing a complete neurological examination of a 7-year-old boy. She will now test his cerebellar function. Which of the following tests would be appropriate for this purpose? a) Ask the boy to close his eyes and then touch his skin with a cotton wisp; ask him to point to where he was touched b) Ask the boy who he is, where he is, and what day it is c) Measure the circumference of the calves and thighs with a tape measure d) Ask the boy to touch each finger on one hand with the thumb that hand in rapid succession

Ask the boy to touch each finger on one hand with the thumb that hand in rapid succession Correct Explanation: Tests for cerebellar function are tests for balance and coordination, such as asking the child to touch each finger on one hand with the thumb of that hand in rapid succession. Motor function is measured by evaluating muscle size, strength, and tone. Begin by comparing the size and symmetry of extremities. If in doubt about either of these, measure the circumference of the calves and thighs or upper and lower arms with a tape measure. If children's sensory systems are intact, they should be able to distinguish light touch, pain, vibration, hot, and cold. Have a child close his eyes and then ask him to point to the spot where you touch him with an object. Orientation, which is one measure of cerebral function, refers to whether children are aware of who they are, where they are, and what day it is (person, place, and time).

The nurse is ambulating a client with a known seizure disorder. The client says, "I'm seeing those flashing lights again," then loses consciousness and develops a clonic-tonic seizure. Which would be the nurse's initial action?

Assist the client to the floor. Assisting the client to the floor is the initial action to prevent client injury. Inserting an oral airway may actually cause harm to the client and no item should be inserted into the client's mouth during a seizure. Administering a dose of phenytoin requires a health care provider's prescription and would not be the first action. Stat paging the health care provider would not be the first action from the options provided

The nurse is creating a plan of care for a client with a stroke (brain attack) who has global aphasia. The nurse should incorporate communication strategies into the plan of care because of which expected characteristic of the client's speech? 1. Intact 2. Rambling 3. Characterized by literal paraphasia 4. Associated with poor comprehension

Associated with poor comprehension Global aphasia is a condition in which the affected person has few language skills as a result of extensive damage to the left hemisphere. The speech is nonfluent and is associated with poor comprehension and limited ability to name objects or repeat words. The client with conduction aphasia has difficulty repeating words spoken by another, and speech is characterized by literal paraphasia with intact comprehension. The client with Wernicke's aphasia may exhibit a rambling type of speech.

The nurse is preparing a plan of care for a client with a brain attack (stroke) who has global aphasia. The nurse should incorporate communication strategies into the plan of care because the client's speech will be characteristic of which finding? Intact 2. Rambling 3. Characterized by literal paraphasia 4. Associated with poor comprehension

Associated with poor comprehension Rationale: Global aphasia is a condition in which the affected person has few language skills as a result of extensive damage to the left hemisphere. The speech is nonfluent and is associated with poor comprehension and limited ability to name objects or repeat words. The client with conduction aphasia has difficulty repeating words spoken by another, and speech is characterized by literal paraphasia with intact comprehension. The client with Wernicke's aphasia may exhibit a rambling type of speech.

Which of the following, if left untreated, can lead to an ischemic stroke?

Atrial fibrillation Atrial fibrillation is the most frequently diagnosed arrhythmia in the United States. If left untreated, it can lead to an ischemic stroke. Cerebral hemorrhage, arteriovenous malformation, and cerebral hemorrhage can lead to a hemorrhagic stroke. Cerebral aneurysm, arteriovenous malformations, and ruptured cerebral arteries can lead to hemorrhagic stroke.

A client with myasthenia gravis arrives at the hospital emergency department in suspected crisis. The health care provider plans to administer edrophonium to differentiate between myasthenic and cholinergic crises. The nurse ensures that which medication is available in the event that the client is in cholinergic crisis? 1. Atropine sulfate 2. Morphine sulfate 3. Protamine sulfate 4. Pyridostigmine bromide

Atropine sulfate Clients with cholinergic crisis have experienced overdosage of medication. Edrophonium will exacerbate symptoms in cholinergic crisis to the point at which the client may need intubation and mechanical ventilation. Intravenous atropine sulfate is used to reverse the effects of these anticholinesterase medications. Morphine sulfate and pyridostigmine bromide would worsen the symptoms of cholinergic crisis. Protamine sulfate is the antidote for heparin.

A 45-year-old client is admitted to the facility with excruciating paroxysmal facial pain. He reports that the episodes occur most often after feeling cold drafts and drinking cold beverages. Based on these findings, the nurse determines that the client is most likely suffering from which neurologic disorder? a) Migraine headache b) Trigeminal neuralgia c) Angina pectoris d) Bell's palsy

B

A client has short-term memory loss. To help the client cope with memory loss, the nurse should: a) tell the client in the morning what activities will be expected to be performed that day. b) place a single-date calendar where the client can view it. c) ask the client to try harder to remember things. d) instruct family members to ignore the behavior.

B

A client who is prescribed by the health care provider (HCP) to take aspirin daily in order to prevent thrombus formation reports having ringing in the ears. The nurse advises the client to take which measure? a) Use acetaminophen instead. b) Contact the HCP. c) Increase fluid intake. d) Stop taking the aspirin.

B

A client with a head injury regains consciousness after several days. When the client first awakes, what should the nurse say to the client? a) "Can you tell me your name and where you live?" b) "You are in the hosipital. You were in an accident and unconscious." c) "I will get your family." d) "I will bet you are a little confused right now."

B

The client arrives in the emergency department following a bicycle accident in which the client's forehead hit the pavement. The client is diagnosed as having a hyphema. The nurse should place the client in which position? a) side-lying on the affected side b) semi-Folwer's c) side-lying on the unaffected side d) supine

B

Which statement indicates the client understands the expected course of Ménière's disease? a) "Continued medication therapy will cure the disease." b) "Control of the episodes is usually possible, but a cure is not yet available." c) "The disease process will gradually extend to the eyes." d) "Bilateral deafness is an inevitable outcome of the disease."

B

A child has been diagnosed with a basilar skull fracture. The nurse identifies ecchymosis behind the patient's ear. This would be documented as which of the following? a) Raccoon eyes b) Otorrhea c) Battle sign d) Rhinorrhea

Battle sign Correct Explanation: Two signs of basilar skull fracture include Battle sign (bruising or ecchymosis behind the ear) and "raccoon eyes" (blood leaking into the frontal sinuses causing an edematous and bruised periorbital area). Rhinorrhea is CSF leakage from the nose. Otorrhea is CSF leaking from the ear.

A child has been diagnosed with a basilar skull fracture. The nurse identifies ecchymosis behind the child's ear. This would be documented as:

Battle sign. Two signs of basilar skull fracture include Battle sign (bruising or ecchymosis behind the ear) and "raccoon eyes" (blood leaking into the frontal sinuses causing an edematous and bruised periorbital area). Rhinorrhea is CSF leakage from the nose. Otorrhea is CSF leaking from the ear.

The nurse in the neurological unit is monitoring a client for signs of increased intracranial pressure (ICP). The nurse reviews the assessment findings for the client and notes documentation of the presence of Cushing's reflex. The nurse determines that the presence of this reflex is obtained by assessing which item? 1. Blood pressure 2. Motor response 3. Pupillary response 4. Level of consciousness

Blood pressure Cushing's reflex is a late sign of increased ICP and consists of a widening pulse pressure (systolic pressure rises faster than diastolic pressure) and bradycardia. The remaining options are unrelated to monitoring for Cushing's reflex.

A client accidentally splashes chemicals into one eye. The nurse knows that eye irrigation with plain tap water should begin immediately and continue for 15 to 20 minutes. What is the primary purpose of this first-aid treatment? a) To serve as a stopgap measure until help arrives b) To hasten formation of scar tissue c) To prevent vision loss d) To eliminate the need for medical care

C

A client experienced a stroke that damaged the hypothalamus. The nurse should anticipate that the client will have problems with: a) thinking and reasoning. b) visual acuity. c) body temperature control. d) balance and equilibrium.

C

A client is admitted with a cervical spine injury sustained during a diving accident. When planning this client's care, the nurse should assign highest priority to which nursing diagnosis? a) Dressing or grooming self-care deficit b) Disturbed sensory perception (tactile) c) Ineffective breathing pattern d) Impaired physical mobility

C

A client is diagnosed with a brain tumor. The nurse's assessment reveals that the client has difficulty interpreting visual stimuli. Based on these findings, the nurse suspects injury to which lobe of the brain? a) Parietal b) Temporal c) Occipital d) Frontal

C

A client was running along an ocean pier, tripped on an elevated area of the decking, and struck his head on the pier railing. According to his friends, "He was unconscious briefly and then became alert and behaved as though nothing had happened." Shortly afterward, he began complaining of a headache and asked to be taken to the emergency department. If the client's intracranial pressure (ICP) is increasing, the nurse should expect to observe which sign first? a) Pupillary asymmetry b) Irregular breathing pattern c) Declining level of consciousness (LOC) d) Involuntary posturing

C

A client who is paraplegic cannot feel the lower extremities and has been positioned on the side. The nurse should inspect which area that is a potential pressure point when the client is in this position? a) sacrum b) occiput c) ankles d) heel

C

A client with a head injury regains consciousness after several days. When the client first awakes, what should the nurse say to the client? a) "I will get your family." b) "I will bet you are a little confused right now." c) "You are in the hosipital. You were in an accident and unconscious." d) "Can you tell me your name and where you live?"

C

A nurse is caring for a client with dementia. A family member of the client asks what the most common cause of dementia is. Which response by the nurse is most appropriate? a) "Depression may manifest as dementia in elderly clients." b) "Drug interactions are the most common cause of dementia in the elderly." c) "The most common cause of dementia in the elderly is Alzheimer's disease." d) "Dementia is a terrible disease of the elderly."

C

A nurse on a neurologic unit is working on performance improvement with a stroke-management team. The nurse identifies a gap between the time a client enters the emergency department (ED) and the time he's admitted to the intensive care unit (ICU) for aggressive treatment. She's meeting with the team to develop a change strategy using indicators. Which statement by a team member indicates a need for further teaching regarding performance management? a) "We can collaborate with staff from the ED and the ICU to formulate strategies and implement change." b) "We can review ED staffing to see if shortages affect ICU admission." c) "We can discipline the ED staff for not getting the clients to the ICU fast enough." d) "We can use statistics gathered in the ED during triage to determine the average time for admission to the ICU."

C

A nurse, a licensed practical nurse (LPN), and a nursing assistant are caring for a group of clients. The nurse asks the nursing assistant to check the pulse oximetry level of a client who underwent laminectomy. The nursing assistant reports that the pulse oximetry reading is 89%. The client Kardex contains an order for oxygen application at 2 L/min should the pulse oximetry level fall below 92%. The nurse is currently assessing a postoperative client who just returned from the postanesthesia care unit. How should the nurse proceed? a) Complete the assessment of the new client before attending to the client who underwent laminectomy. b) Immediately go the client's room and assess vital signs, administer oxygen at 2 L/minute, and notify the physician. c) Ask the LPN to obtain vital signs and administer oxygen at 2 L/min to the client who underwent laminectomy. d) Ask the nursing assistant to notify the physician of the low pulse oximetry level.

C

The nurse has asked the unlicensed assistive personnel (UAP) to ambulate a client with Parkinson's disease. The nurse observes the UAP pulling on the client's arms to get the client to walk forward. The nurse should: a) assist the UAP with getting the client back in bed. b) have the UAP keep a steady pull on the client to promote forward ambulation. c) explain how to overcome a freezing gait by telling the client to march in place. d) give the client a muscle relaxant.

C

A 76-year-old client is brought to the clinic by his daughter. The daughter states that her father has had two transient ischemic attacks (TIAs) in the past week. The physician orders carotid angiography, and the report reveals that the carotid artery has been narrowed by atherosclerotic plaques. What treatment option does the nurse expect the physician to offer this client to increase blood flow to the brain?

Carotid endarterectomy If narrowing of the carotid artery by atherosclerotic plaques is the cause of the TIAs, a carotid endarterectomy (surgical removal of atherosclerotic plaque) is a treatment option. A balloon angioplasty, a procedure similar to a percutaneous transluminal coronary artery angioplasty, is performed to dilate the carotid artery and increase blood flow to the brain. Options A, B, and C are not surgical options to increase blood flow through the carotid artery to the brain.

To detect complications as early as possible in a child with meningitis who's receiving I.V. fluids, monitoring for which condition should be the nurse's priority? a) Cerebral edema b) Renal failure c) Left-sided heart failure d) Cardiogenic shock

Cerebral edema Correct Explanation: The child with meningitis is already at increased risk for cerebral edema and increased intracranial pressure due to inflammation of the meningeal membranes; therefore, the nurse should carefully monitor fluid intake and output to avoid fluid volume overload. Renal failure and cardiogenic shock aren't complications of I.V. therapy. The child with a healthy heart wouldn't be expected to develop left-sided heart failure.

Cerebral thrombosis does not occur suddenly. In the few hours or days preceding a thrombotic brain attack (stroke), the client may experience a transient loss of speech, hemiplegia, or paresthesias on one side of the body. Signs and symptoms of thrombotic brain attack (stroke) vary but may include dizziness, cognitive changes, or seizures. Headache is rare, but some clients with brain attack (stroke) experience signs and symptoms similar to those of cerebral embolism or intracranial hemorrhage. Sudden loss of consciousness occurred. 2. Signs and symptoms occurred suddenly. 3. The client experienced paresthesias a few days before admission to the hospital. 4. The client complained of a severe headache, which was followed by sudden onset of paralysis.

Cerebral thrombosis does not occur suddenly. In the few hours or days preceding a thrombotic brain attack (stroke), the client may experience a transient loss of speech, hemiplegia, or paresthesias on one side of the body. Signs and symptoms of thrombotic brain attack (stroke) vary but may include dizziness, cognitive changes, or seizures. Headache is rare, but some clients with brain attack (stroke) experience signs and symptoms similar to those of cerebral embolism or intracranial hemorrhage. 3. The client experienced paresthesias a few days before admission to the hospital.

The nurse reviews the health care provider's (HCP's) prescriptions for a client with Guillain-Barré syndrome. Which prescription written by the HCP should the nurse question? 1. Clear liquid diet 2. Bilateral calf measure 3. Monitor vital signs frequently 4. Passive range-of-motion (ROM) exercises

Clear liquid diet Clients with Guillain-Barré syndrome have dysphagia. Clients with dysphagia are more likely to aspirate clear liquids than thick or semisolid foods. Passive ROM exercises can help prevent contractures, and assessing calf measurements can help detect deep vein thrombosis, for which these clients are at risk. Because clients with Guillain-Barré syndrome are at risk for hypotension or hypertension, bradycardia, and respiratory depression, frequent monitoring of vital signs is required.

The nurse is assessing a client's muscle strength and notes that when asked, the client cannot maintain the hands in a supinated position with the arms extended and eyes closed. How should the nurse correctly document this finding on the medical record? 1. Client is demonstrating ataxia. 2. Client is exhibiting pronator drift. 3. Client appears to have nystagmus. 4. Client examination reveals hyperreflexia.

Client is exhibiting pronator drift. Pronator drift occurs when a client cannot maintain the hands in a supinated position with the arms extended and eyes closed. This assessment may be done to detect small changes in muscle strength that might not otherwise be noted. Ataxia is a disturbance in gait. Nystagmus is characterized by fine, involuntary eye movements. It can occur with neurological disease or as a side effect of selected medications. Hyperreflexia is an excessive reflex action.

The nurse is caring for a client diagnosed with a hydrocephalus. Which should the nurse anticipate as being the cause of this disorder? 1. Closure of cranial sutures 2. Small aqueduct of Sylvius 3. Enlarged foramen of Monro 4. Increased number of arachnoid villi

Closure of cranial sutures The closure of cranial sutures during childhood prevents expansion of the cranial vault when hydrocephalus occurs in the adult. This leads to increased neurological changes with lesser degrees of hydrocephalus compared with hydrocephalus during early childhood. The other structures identified in the remaining options are associated with cerebrospinal fluid formation and circulation, but are not responsible for hydrocephalus.

A lumbar puncture is performed on a child suspected to have bacterial meningitis, and cerebrospinal fluid (CSF) is obtained for analysis. The nurse reviews the results of the CSF analysis and determines that which results would verify the diagnosis?

Cloudy CSF, elevated protein, and decreased glucose levels Meningitis is an infectious process of the central nervous system caused by bacteria and viruses; it may be acquired as a primary disease or as a result of complications of neurosurgery, trauma, infection of the sinus or ears, or systemic infections. Meningitis is diagnosed by testing cerebrospinal fluid obtained by lumbar puncture. In the case of bacterial meningitis, findings usually include an elevated pressure; turbid or cloudy cerebrospinal fluid; and elevated leukocyte, elevated protein, and decreased glucose levels.

A child is diagnosed with bacterial meningitis. The nurse would suspect which abnormality of cerebrospinal fluid (CSF)?

Cloudy appearance In the CSF of clients diagnosed with bacterial meningitis, the pressure is elevated, the appearance is cloudy, and the leukocytes are elevated. A decreased sugar content is noted.

A child is diagnosed with bacterial meningitis. The nurse would suspect which abnormality of cerebrospinal fluid (CSF)? a) Cloudy appearance b) Elevated sugar c) Decreased leukocytes d) Decreased pressure

Cloudy appearance Correct Explanation: In the CSF of clients diagnosed with bacterial meningitis, the pressure is elevated, the appearance is cloudy, and the leukocytes are elevated. A decreased sugar content is noted.

While making your initial rounds after coming on shift, you find a client thrashing about in bed complaining of a severe headache. The client tells you the pain is behind his right eye which is red and tearing. What type of headache would you suspect this client of having?

Cluster A person with a cluster headache has pain on one side of the head, usually behind the eye, accompanied by nasal congestion, rhinorrhea (watery discharge from the nose), and tearing and redness of the eye. The pain is so severe that the person is not likely to lie still; rather he or she paces or thrashes about. The symptoms in the scenario do not describe a sinus headache.

Antibiotic therapy to treat meningitis should be instituted immediately after which event? a) Identification of the causative organism b) Collection of cerebrospinal fluid (CSF) and blood for culture c) Initiation of I.V. therapy d) Admission to the nursing unit

Collection of cerebrospinal fluid (CSF) and blood for culture Explanation: Antibiotic therapy should always begin immediately after the collection of CSF and blood cultures. After the specific organism is identified, bacteria-specific antibiotics can be administered if the initial choice of antibiotic therapy isn't appropriate. Admission and initiation of I.V. therapy aren't, by themselves, appropriate times to begin antibiotic therapy.

A client has a halo vest that was applied following a C6 spinal cord injury. The nurse performs which action to determine whether the client is ready to begin sitting up?

Compares the client's pulse and blood pressure when both flat and sitting Clients with cervical spinal cord injuries may lose control over peripheral vasoconstriction, causing postural (orthostatic) hypotension when upright. A drop of 15 mm Hg in the systolic pressure or 10 mm Hg in the diastolic pressure accompanied by an increase in heart rate when the head is elevated may indicate autonomic insufficiency that can cause dizziness or syncope in the upright position. Assessment of skin integrity of the pin sites is important but does not affect sitting readiness. Hip range of motion is not affected initially in this type of cord injury. The halo vest is not loosened by the nurse. The vest provides trunk stability for sitting.

The client with a cervical spine injury has Crutchfield tongs applied in the emergency department. The nurse should perform which essential action when caring for this client?

Comparing the amount of prescribed weights with the amount in use Crutchfield tongs are applied after drilling holes in the client's skull under local anesthesia. Weights are attached to the tongs, which exert pulling pressure on the longitudinal axis of the cervical spine. The nurse ensures that weights hang freely and that the amount of weight matches the current prescription. The client with Crutchfield tongs is placed on a Stryker frame or Roto-Rest bed. The nurse does not remove the weights to administer care or change the level of tension or traction based on client comfort level.

The father of a 7-year-old boy reports to the nurse that two or three times over the past weeks he has observed his son seemingly staring into space and rubbing his hands. The behavior lasts for a minute or so, followed by an inability of the child to understand what's being said to him. When the nurse asks the child about his experience, he says he doesn't know what his father is talking about. What type of seizure do these symptoms indicate the child is experiencing? a) Complex partial seizures b) Simple partial sensory seizures c) Simple partial motor seizures d) Absence seizures

Complex partial seizures

The father of a 7-year-old boy reports to the nurse that two or three times over the past weeks he has observed his son seemingly staring into space and rubbing his hands. The behavior lasts for a minute or so, followed by an inability of the child to understand what's being said to him. When the nurse asks the child about his experience, he says he doesn't know what his father is talking about. The symptoms this child is exhibiting might indicate the child is having a) Absence seizures b) Complex partial seizures c) Simple partial motor seizures d) Simple partial sensory seizures

Complex partial seizures Explanation: Complex partial seizures, also called psychomotor seizures, begin in a small area of the brain and change or alter consciousness. They cause memory loss and staring. Nonpurposeful movements such as hand rubbing, lip smacking, arm dropping, and swallowing may occur. Following the seizure the child may sleep or be confused for a few minutes. The child is often unaware of the seizure. Simple partial sensory seizures may include sensory symptoms called an aura (a sensation that signals an impending attack) involving sight, sound, taste, smell, touch, or emotions (a feeling of fear, for example). The child may also have numbness, tingling, paresthesia, or pain. Simple partial motor seizures cause a localized motor activity such as shaking of an arm, leg, or other part of the body. Absence seizures rarely last longer than 20 seconds. The child loses awareness and stares straight ahead but does not fall. The child may have blinking or twitching of the mouth or an extremity along with the staring. Immediately after the seizure, the child is alert and continues conversation but does not know what was said or done during the episode.

The nurse caring for a client following a craniotomy monitors for signs of increased intracranial pressure (ICP). Which indicates an early sign of increased ICP?

Confusion Early signs/symptoms of increased intracranial pressure are subtle and may often be transient, lasting for only a few minutes in some cases. These early clinical signs/symptoms include changes in level of consciousness, including episodes of confusion and drowsiness, and slight pupillary and breathing changes. Clinical signs/symptoms of later increased intracranial pressure include decreasing levels of consciousness, a widened pulse pressure, and bradycardia. Cheyne-Stokes respiratory pattern, or a hyperventilation respiratory pattern, and sluggish and dilating pupils appear in the later stages.

A client who has been treated for chronic open-angle glaucoma (COAG) for 5 years asks the nurse, "How does glaucoma damage my eyesight?" The nurse's reply should be based on the knowledge that COAG: a) is caused by decreased blood flow to the retina. b) results from chronic eye inflammation. c) leads to detachment of the retina. d) causes increased intraocular pressure.

D

A client with an inflammatory ophthalmic disorder has been receiving repeated courses of a corticosteroid ointment, one-half inch in the lower conjunctival sac four times a day as directed. The client reports a headache and blurred vision. The nurse suspects that these symptoms represent: a) common adverse reactions to corticosteroid therapy. b) incorrect ointment application. c) expected drug effects that should diminish over time. d) increased intraocular pressure (IOP).

D

A client with quadriplegia is in spinal shock. What finding should the nurse expect? a) Positive Babinski's reflex along with spastic extremities b) Hyperreflexia along with spastic extremities c) Spasticity of all four extremities d) Absence of reflexes along with flaccid extremities

D

A nurse caring for a client who had a stroke is using the unit's new computerized documentation system. The nurse uses the information technology appropriately when she: a) determines a client's identity from a computer chart. b) documents medications before administration. c) e-mails information about a client to a friend at home. d) documents medications after administration.

D

A nurse completing her management rotation in the intensive care unit (ICU) is working with an experienced ICU nurse. One client's work supervisor calls to "check up" on the client. The nurse offers to transfer the call to the client's family members. The experienced ICU nurse recognizes this action as: a) passing the buck to avoid work. b) a violation of privacy laws. c) a clever way of avoiding the supervisor. d) protection of the client's privacy.

D

A nurse is assisting during a lumbar puncture. How should the nurse position the client for this procedure? a) Prone, with the head turned to the right b) Supine, with the knees raised toward the chest c) Lateral, with right leg flexed d) Lateral recumbent, with chin resting on flexed knees

D

A nurse is caring for a client admitted to the unit with a seizure disorder. The client seems upset and asks the nurse, "What will they do to me? I'm scared of the tests and of what they'll find out." The nurse should focus her teaching plans on which diagnostic tests? a) Electrocardiography, TEE, prothrombin time (PT), and International Normalized Ratio (INR) b) Transesophageal echocardiogram (TEE), troponin levels, and a complete blood count c) X-ray of the brain, bone marrow aspiration, and EEG d) EEG, blood cultures, and neuroimaging studies

D

As a first step in teaching a woman with a spinal cord injury and quadriplegia about her sexual health, the nurse assesses her understanding of her current sexual functioning. Which statement by the client indicates she understands her current ability? a) "I cannot have sexual intercourse because it causes hypertension, but other sexual activity is okay." b) "I will not be able to have sexual intercourse until the urinary catheter is removed." c) "I should be able to participate in sexual activity, but I will be infertile." d) "I can participate in sexual activity but might not experience orgasm."

D

Sodium polystyrene sulfonate is prescribed for a client following crush injury. The drug is effective if: a) the pulse is weak and irregular. b) the ECG is showing tall, peaked T waves. c) there is muscle weakness on physical examination. d) the serum potassium is 4.0 mEq/L (4.0 mmol/L).

D

The client has a sustained increased intracranial pressure (ICP) of 20 mm Hg. Which client position would be most appropriate? a) left Sims position b) the head elevated on two pillows c) Trendelenburg's position d) the head of the bed elevated 15 to 20 degrees

D

Which action is contraindicated for a client with seizure precautions? a) allowing the client to wear his or her own clothing b) encouraging the client to perform his or her own personal hygiene c) encouraging the client to be out of bed d) assessing the client's oral temperature with a glass thermometer

D

The nurse is monitoring a client with a head injury and notes that the client is assuming the posture shown in the figure. What is the client exhibiting that would require the nurse to notify the registered nurse immediately? Refer to the figure.

Decorticate posturing In decorticate posturing, the upper extremities (arms, wrists, and fingers) are flexed with adduction of the arms. The lower extremities are extended with internal rotation and plantar flexion. Decorticate posturing indicates a hemispheric lesion of the cerebral cortex. In decerebrate posturing, the upper extremities are extended stiffly and adducted with internal rotation and pronation of the palms. The lower extremities are extended stiffly with plantar flexion. The teeth are clenched and the back is hyperextended. Decerebrate posturing indicates a lesion in the brainstem at the midbrain or upper pons. Flaccid quadriplegia is complete loss of muscle tone and paralysis of all four extremities, indicating a completely nonfunctional brainstem. Opisthotonos is prolonged arching of the back with the head and heels bent backward. Opisthotonos indicates meningeal irritation.

The nurse is performing an assessment on a child with a head injury. The nurse notes an abnormal flexion of the upper extremities and an extension of the lower extremities. What should the nurse document that the child is experiencing? Decorticate posturing 2. Decerebrate posturing 3. Flexion of the arms and legs 4. Normal expected positioning after head injury

Decorticate posturing Rationale: Decorticate posturing is an abnormal flexion of the upper extremities and an extension of the lower extremities with possible plantar flexion of the feet. Decerebrate posturing is an abnormal extension of the upper extremities with internal rotation of the upper arms and wrists and an extension of the lower extremities with some internal rotation.

Acetazolamide is prescribed for a client with a diagnosis of a supratentorial lesion. The nurse monitors the client for effectiveness of this medication, knowing which is its primary action?

Decrease cerebrospinal fluid production Acetazolamide is a carbonic anhydrase inhibitor. It is used in the client with, or at risk for, increased intracranial pressure to decrease cerebrospinal fluid production. Options 1, 2, and 4 are not actions of this medication.

A preschool-age child has just been admitted to the pediatric unit with a diagnosis of bacterial meningitis. The nurse would include which recommendation in the nursing plan? a) Decrease environmental stimulation b) Take vital signs every 4 hours c) Encourage the parents to hold the child d) Monitor temperature every 4 hours

Decrease environmental stimulation

A preschool-age child has just been admitted to the pediatric unit with a diagnosis of bacterial meningitis. The nurse would include which recommendation in the nursing plan?

Decrease environmental stimulation A child with the diagnosis of meningitis is much more comfortable with decreased environmental stimuli. Noise and bright lights stimulate the child and can be irritating, causing the child to cry, in turn increasing intracranial pressure. Vital signs would be taken initially every hour and temperature monitored every 2 hours. Children with bacterial meningitis are usually much more comfortable if allowed to lie flat because this position doesn't cause increased meningeal irritation.

A preschool-age child has just been admitted to the pediatric unit with a diagnosis of bacterial meningitis. The nurse would include which recommendation in the nursing plan? a) Take vital signs every 4 hours b) Encourage the parents to hold the child c) Monitor temperature every 4 hours d) Decrease environmental stimulation

Decrease environmental stimulation Correct Explanation: A child with the diagnosis of meningitis is much more comfortable with decreased environmental stimuli. Noise and bright lights stimulate the child and can be irritating, causing the child to cry, in turn increasing intracranial pressure. Vital signs would be taken initially every hour and temperature monitored every 2 hours. Children with bacterial meningitis are usually much more comfortable if allowed to lie flat because this position doesn't cause increased meningeal irritation.

The nurse is admitting a client with Guillain-Barré syndrome to the nursing unit. The client has ascending paralysis to the level of the waist. Knowing the complications of the disorder, the nurse should bring which most essential items into the client's room? 1. Nebulizer and pulse oximeter 2. Blood pressure cuff and flashlight 3. Flashlight and incentive spirometer 4. Electrocardiographic monitoring electrodes and intubation tray

Electrocardiographic monitoring electrodes and intubation tray The client with Guillain-Barré syndrome is at risk for respiratory failure because of ascending paralysis. An intubation tray should be available for use. Another complication of this syndrome is cardiac dysrhythmias, which necessitates the use of electrocardiographic monitoring. Because the client is immobilized, the nurse should assess for deep vein thrombosis and pulmonary embolism routinely. Although items in the incorrect options may be used in care, they are not the most essential items from the options provided.

The nurse is caring for a client who sustained a spinal cord injury. During administration of morning care, the client begins to exhibit signs and symptoms of autonomic dysreflexia. Which initial nursing action should the nurse take? 1. Elevate the head of the bed. 2. Examine the rectum digitally. 3. Assess the client's blood pressure. 4. Place the client in the prone position.

Elevate the head of the bed. Autonomic dysreflexia is a serious complication that can occur in the spinal cord-injured client. Once the syndrome is identified, the nurse elevates the head of the client's bed and then examines the client for the source of noxious stimuli. The nurse also assesses the client's blood pressure, but the initial action would be to elevate the head of the bed. The client would not be placed in the prone position; lying flat will increase the client's blood pressure.

The nurse is caring for a child after surgical removal of a brain tumor. The nurse should assess the child for which sign that would indicate that brainstem involvement occurred during the surgical procedure? Inability to swallow 2. Elevated temperature 3. Altered hearing ability 4. Orthostatic hypotension

Elevated temperature Vital signs and neurological status are assessed frequently after surgical removal of a brain tumor. Special attention is given to the child's temperature, which may be elevated because of hypothalamic or brainstem involvement during surgery. A cooling blanket should be in place on the bed or readily available if the child becomes hyperthermic. Inability to swallow and altered hearing ability are related to functional deficits after surgery. Orthostatic hypotension is not a common clinical manifestation after brain surgery. An elevated blood pressure and widened pulse pressure may be associated with increased intracranial pressure, which is a complication after brain. surgery, but is not related to brainstem involvement

A client who had cranial surgery 5 days earlier to remove a brain tumor has a few cognitive deficits and does not seem to be progressing as quickly as the client or family had hoped. The nurse plans to implement which approach as most helpful to the client and family at this time? 1. Emphasize progress in a realistic manner. 2. Set high goals to give the client something to "aim for." 3. Tell the family to be extremely optimistic with the client. 4. Inform the client and family of standardized goals of care.

Emphasize progress in a realistic manner. The most helpful approach by the nurse is to emphasize progress that is being made in a realistic manner. The nurse does not offer false hope but does provide factual information in a clear and positive manner. The nurse encourages the family to be realistic in their expectations and attitudes. The plan of care should be individualized for each client.

The nurse is planning care for a client who displays confusion secondary to a neurological problem. Which approach by the nurse would be least helpful in assisting this client?

Encouraging multiple visitors at one time Clients with cognitive impairment from neurological dysfunction respond best to a stable environment, which is limited in the amounts and type of sensory input. The nurse can provide sensory cues and give clear, simple directions in a positive manner. Confusion and agitation can be minimized by reducing environmental stimuli (such as television, multiple visitors) and keeping familiar personal articles (such as family pictures) at the bedside.

A client recovering from a head injury is participating in care. The nurse determines that the client understands measures to prevent elevations in intracranial pressure if the nurse observes the client doing which activity? 1. Blowing the nose 2. Isometric exercises 3. Coughing vigorously 4. Exhaling during repositioning

Exhaling during repositioning Activities that increase intrathoracic and intraabdominal pressures cause an indirect elevation of the intracranial pressure. Some of these activities include isometric exercises, Valsalva's maneuver, coughing, sneezing, and blowing the nose. Exhaling during activities such as repositioning or pulling up in bed opens the glottis, which prevents intrathoracic pressure from rising.

The nurse is trying to help the family of an unconscious client cope with the situation. Which intervention should the nurse plan to incorporate into the care routine for the client and family? 1. Discouraging the family from touching the client 2. Explaining equipment and procedures on an ongoing basis 3. Ensuring adherence to visiting hours to ensure the client's rest 4. Encouraging the family not to "give in" to their feelings of grief

Explaining equipment and procedures on an ongoing basis Families often need assistance to cope with the illness of a loved one. The nurse should explain all equipment, treatments, and procedures and should supplement or reinforce information given by the health care provider. Family members should be encouraged to touch and speak to the client and to become involved in the client's care to the extent they are comfortable. The nurse should allow the family to stay with the client to the extent possible and should encourage them to eat and sleep adequately to maintain strength. The nurse can help family members of an unconscious client by assisting them to work through their feelings of grief.

The nurse is trying to help the family of an unconscious client cope with the situation. Which intervention should the nurse plan to incorporate into the care routine for the client?

Explaining equipment and procedures on an ongoing basis Families often need assistance to cope with the sudden, severe illness of a loved one. The nurse can help the family of an unconscious client by assisting them to work through their feelings of grief. The nurse should explain all equipment, treatments, and procedures, and supplement or reinforce information given by the health care provider. The family should be encouraged to touch and speak to the client and to become involved in the client's care to the extent that they are comfortable. The nurse should allow the family to stay with the client as much as possible and should encourage them to eat and sleep adequately to maintain their strength.

The nurse is assisting in gathering data on cranial nerve XII of a client who sustained a brain attack (stroke). The nurse understands that the client should be asked to perform which action?

Extend the tongue. To assess the function of cranial (hypoglossal) nerve XII, the nurse would assess the client's ability to extend the tongue. Options 1, 3, and 4 are unrelated to assessing this cranial nerve.

The nurse is assisting in caring for a client who sustained a traumatic head injury following a motor vehicle crash. The nurse documents that the client is exhibiting decerebrate posturing. The nurse bases this documentation on which observation?

Extension of the extremities and pronation of the arms Decerebrate posturing (abnormal extension), which is associated with dysfunction in the brainstem area, consists of extension of the extremities and pronation of the arms. Posturing is a late sign of deterioration in the client's neurological status and warrants immediate health care provider notification.

Which sign/symptom is observed in the clonic phase of a seizure?

Extension spasms of the body The clonic phase of a seizure is characterized by violent extension spasms of the entire body interrupted by muscular relaxation and accompanied by strenuous hyperventilation. The face is contorted and the eyes roll. There is excessive salivation resulting in frothing from the mouth, biting of the tongue, profuse sweating, and a rapid pulse. The clonic jerking subsides by slowing in frequency and losing strength over a period of 30 seconds. Options 1, 2, and 3 identify the tonic phase of a seizure.

The nurse is performing a neurological assessment on a client and is assessing the function of cranial nerves III, IV, and VI. Assessment of which aspect of function will yield the best information about these cranial nerves? 1. Eye movements 2. Response to verbal stimuli 3. Affect, feelings, or emotions 4. Insight, judgment, and planning

Eye movements Eye movements are under the control of cranial nerves III, IV, and VI. Level of consciousness (response to verbal stimuli) is controlled by the reticular activating system and both cerebral hemispheres. Feelings are part of the role of the limbic system and involve both hemispheres. Insight, judgment, and planning are part of the function of the frontal lobe in conjunction with association fibers that connect to other areas of the cerebrum.

A client has clear fluid leaking from the nose following a basilar skull fracture. Which finding would alert the nurse that cerebrospinal fluid is present?

Fluid separates into concentric rings and tests positive for glucose. Leakage of cerebrospinal fluid (CSF) from the ears or nose may accompany basilar skull fracture. CSF can be distinguished from other body fluids because the drainage will separate into bloody and yellow concentric rings on dressing material, called a halo sign. The fluid also tests positive for glucose.

A client has clear fluid leaking from the nose following a basilar skull fracture. Which finding would alert the nurse that cerebrospinal fluid is present? 1. Fluid is clear and tests negative for glucose. 2. Fluid is grossly bloody in appearance and has a pH of 6. 3. Fluid clumps together on the dressing and has a pH of 7. 4. Fluid separates into concentric rings and tests positive for glucose.

Fluid separates into concentric rings and tests positive for glucose. Leakage of cerebrospinal fluid (CSF) from the ears or nose may accompany basilar skull fracture. CSF can be distinguished from other body fluids because the drainage will separate into bloody and yellow concentric rings on dressing material, called a halo sign. The fluid also tests positive for glucose.

The client with a head injury opens eyes to sound, has no verbal response, and localizes to painful stimuli when applied to each extremity. How should the nurse document the Glasgow Coma Scale (GCS) score? 1. GCS = 3 2. GCS = 6 3. GCS = 9 4. GCS = 11

GCS = 9 The GCS is a method for assessing neurological status. The highest possible GCS score is 15. A score lower than 8 indicates that coma is present. Motor response points are as follows: Obeys a simple response = 6; Localizes painful stimuli = 5; Normal flexion (withdrawal) = 4; Abnormal flexion (decorticate posturing) = 3; Extensor response (decerebrate posturing) = 2; No motor response to pain = 1. Verbal response points are as follows: Oriented = 5; Confused conversation = 4; Inappropriate words = 3; Responds with incomprehensible sounds = 2; No verbal response = 1. Eye opening points are as follows: Spontaneous = 4; In response to sound = 3; In response to pain = 2; No response, even to painful stimuli = 1. Using the GCS, a score of 3 is given when the client opens the eyes to sound. Localization to pain is scored as 5. When there is no verbal response the score is 1. The total score is then equal to 9.

The nurse knows that the heads of infants and toddlers are large in proportion to their bodies, placing them at risk for what problem?

Head trauma A larger head size in relation to the rest of their body size gives young children a higher center of gravity, which causes them to hit their head more readily, thus placing them at risk for head trauma. Fragile capillaries in the periventricular area of the brain put preterm infants at risk for intracranial hemorrhage. Congenital hydrocephalus may be caused by abnormal intrauterine development or infection. Positional plagiocephaly is caused by an infant's head remaining in the same position for too long.

The nurse observes the unlicensed assistive personnel (UAP) positioning the client with increased intracranial pressure (ICP). Which position would require intervention by the nurse?

Head turned to the side The head of the client with increased ICP should be positioned so that the head is in a neutral, midline position. The nurse should avoid flexing or extending the neck or turning the head side to side. The head of the bed should be raised to 30 to 45 degrees. Use of proper positions promotes venous drainage from the cranium to keep ICP down.

The nurse is collecting neurological data on a poststroke adult client. Which technique should the nurse perform to adequately check proprioception?

Hold the sides of the client's great toe, and while moving it, ask what position it is in. Proprioception is tested by holding the sides of the client's great toe and, while moving it, asking the client what position it is in. Option 1 identifies the assessment for gastrocnemius muscle contraction, and option 2 tests two-point discrimination. The plantar reflex is elicited in option 3. Normally, the toes plantar flex, but when abnormal, the toes dorsiflex and fan out.

The nurse notes that a client who has suffered a brain injury has an adequate heart rate, blood pressure, fluid balance, and body temperature. Based on these clinical findings, the nurse determines that which brain area is functioning properly? 1. Thalamus 2. Hypothalamus 3. Limbic system 4. Reticular activating system

Hypothalamus The hypothalamus is responsible for autonomic nervous system functions, such as heart rate, blood pressure, temperature, and fluid and electrolyte balance (among others). The thalamus acts as a relay station for sensory and motor information. The limbic system is responsible for emotions. The reticular activating system is responsible for the sleep-wake cycle.

The nurse is developing a plan of care for a client with a diagnosis of brain attack (stroke) with anosognosia. To meet the needs of the client with this deficit, the nurse should include activities that will achieve which outcome? Encourage communication. 2. Provide a consistent daily routine. 3. Promote adequate bowel elimination. 4. Increase the client's awareness of the affected side.

Increase the client's awareness of the affected side. In anosognosia, the client exhibits neglect of the affected side of the body. The nurse will plan care activities that remind the client to perform actions that require looking at the affected arm or leg, as well as activities that will increase the client's awareness of the affected side. Options 1, 2, and 3 are not associated with this deficit.

The nurse is creating a plan of care for a client with a diagnosis of stroke (brain attack) with anosognosia. To meet the needs of the client with this deficit, the nurse should include activities that will achieve which outcome? 1. Encourage communication. 2. Provide a consistent daily routine. 3. Promote adequate bowel elimination. 4. Increase the client's awareness of the affected side.

Increase the client's awareness of the affected side. In anosognosia, the client exhibits neglect of the affected side of the body. The nurse will plan care activities that remind the client to perform actions that require looking at the affected arm or leg, as well as activities that will increase the client's awareness of the affected side. The remaining options are not associated with this deficit.

The nurse is caring for a client with a diagnosis of brain attack (stroke) with anosognosia. To meet the needs of the client with this deficit, which action does the nurse plan?

Increase the client's awareness of the affected side. In anosognosia, the client neglects the affected side of the body. The nurse should plan care activities that encourage the client to look at the affected arm or leg and that will increase the client's awareness of the affected side. Options 1, 2, and 3 are not associated with this deficit.

The nurse assigned to the care of an unconscious client is making initial daily rounds. On entering the client's room, the nurse observes that the client is lying supine in bed, with the head of the bed elevated approximately 5 degrees. The nasogastric tube feeding is running at 70 mL/hr, as prescribed. The nurse assesses the client and auscultates adventitious breath sounds. Which judgment should the nurse formulate for the client? 1. Impaired nutritional intake 2. Increased risk for aspiration 3. Increased likelihood for injury 4. Susceptibility to fluid volume deficit

Increased risk for aspiration Increased risk for aspiration is a condition in which an individual is at risk for entry of gastrointestinal (GI) secretions, oropharyngeal secretions, or solids or fluids into tracheobronchial passages. Conditions that place the client at risk for aspiration include reduced level of consciousness, depressed cough and gag reflexes, and feeding via a GI tube. There is no information in the question indicating that the remaining options are a concern.

The nurse is caring for the client with increased intracranial pressure. The nurse would note which trend in vital signs if the intracranial pressure is rising? Increasing temperature, increasing pulse, increasing respirations, decreasing blood pressure 2. Increasing temperature, decreasing pulse, decreasing respirations, increasing blood pressure 3. Decreasing temperature, decreasing pulse, increasing respirations, decreasing blood pressure 4. Decreasing temperature, increasing pulse, decreasing respirations, increasing blood pressure

Increasing temperature, decreasing pulse, decreasing respirations, increasing blood pressure Rationale said the correct answer but didn't give explanation

The nurse has made a judgment that a client who had a craniotomy is experiencing a problem with body image. The nurse develops goals for the client but determines that the client has not met the outcome criteria by discharge if the client performs which action? 1. Wears a turban to cover the incision 2. Indicates that facial puffiness will be a permanent problem 3. Verbalizes that periorbital bruising will disappear over time 4. States an intention to purchase a hairpiece until hair has grown back

Indicates that facial puffiness will be a permanent problem After craniotomy, clients may experience difficulty with altered personal appearance. The nurse can help by listening to the client's concerns and by clarifying any misconceptions about facial edema, periorbital bruising, and hair loss (all of which are temporary). The nurse can encourage the client to participate in self-grooming and use personal articles of clothing. Finally, the nurse can suggest the use of a turban, followed by a hairpiece, to help the client adapt to the temporary change in appearance.

The home health nurse has been discussing interventions to prevent constipation in a client with multiple sclerosis. The nurse determines that the client is using the information most effectively if the client reports which action? 1. Drinking a total of 1000 mL/day 2. Giving herself an enema every morning before breakfast 3. Taking stool softeners daily and a glycerin suppository once a week 4. Initiating a bowel movement every other day, 45 minutes after the largest meal of the day

Initiating a bowel movement every other day, 45 minutes after the largest meal of the day To manage constipation, the client should take in a high-fiber diet, bulk formers, and stool softeners. A fluid intake of 2000 mL/day is recommended. The client should initiate a bowel movement on an every-other-day basis and should sit on the toilet or commode. This should be done approximately 45 minutes after the largest meal of the day to take advantage of the gastrocolic reflex. A glycerin suppository, bisacodyl suppository, or digital stimulation may be used to initiate the process. Laxatives and enemas should be avoided whenever possible because they lead to dependence.

The nurse is caring for a client with a diagnosis of right (nondominant) hemispheric stroke. The nurse notes that the client is alert and oriented to time and place. On the basis of these assessment findings, the nurse should make which interpretation? 1. Had a very mild stroke 2. Most likely suffered a transient ischemic attack 3. May have difficulty with language abilities only 4. Is likely to have perceptual and spatial disabilities

Is likely to have perceptual and spatial disabilities The client with a right (nondominant) hemispheric stroke may be alert and oriented to time and place. These signs of apparent wellness often suggest that the client is less disabled than is the case. However, impulsivity and confusion in carrying out activities may be very real problems for these clients as a result of perceptual and spatial disabilities. The right hemisphere is considered specialized in sensory-perceptual and visual-spatial processing and awareness of body space. The left hemisphere is dominant for language abilities.

A postoperative craniotomy client who sustained a severe head injury is admitted to the neurological unit. What nursing intervention is necessary for this client? 1. Take and record vital signs every 4 to 8 hours. 2. Prophylactically hyperventilate during the first 24 hours. 3. Treat a central fever with the administration of antipyretic medications such as acetaminophen. 4. Keep the head of the bed elevated at least 30 degrees, and position the client to avoid extreme flexion or extension of the neck and head.

Keep the head of the bed elevated at least 30 degrees, and position the client to avoid extreme flexion or extension of the neck and head. Avoiding extreme flexion and extension of the neck can enhance venous drainage and help prevent increased intracranial pressure. As a general rule, hyperventilation is avoided during the first 24 hours postoperatively because it may produce ischemia caused by cerebral vasoconstriction. Vital signs need to be taken and recorded at least every 1 to 2 hours. Central fevers caused by hypothalamic damage respond better to cooling (hypothermia blankets, sponge baths) than to the administration of antipyretic medications.

A 9-year-old girl who is suspected of having an infection of the central nervous system is undergoing a lumbar puncture to withdraw cerebrospinal fluid for analysis. The nurse knows that the needle will be introduced into the subarachnoid space at the level of which of the following vertebrae? a) T3 or T4 b) L1 or L2 c) C1 or C2 d) L4 or L5

L4 or L5

A 9-year-old girl who is suspected of having an infection of the central nervous system is undergoing a lumbar puncture to withdraw cerebrospinal fluid for analysis. The nurse knows that the needle will be introduced into the subarachnoid space at the level of which of the following vertebrae? a) L1 or L2 b) L4 or L5 c) T3 or T4 d) C1 or C2

L4 or L5 Correct Explanation: Lumbar puncture, the introduction of a needle into the subarachnoid space (under the arachnoid membrane) at the level of L4 or L5 to withdraw CSF for analysis, is used most frequently with children to diagnose hemorrhage or infection in the CNS or to diagnose an obstruction of CSF flow.

A client has experienced an ischemic stroke that has damaged the lower motor neurons of the brain. Which of the following deficits would the nurse expect during assessment?

Lack of deep tendon reflexes Damage to the occipital lobe can result in visual agnosia, whereas damage to the temporal lobe can cause auditory agnosia. If damage has occurred to the frontal lobe, learning capacity, memory, or other higher cortical intellectual functions may be impaired. Such dysfunction may be reflected in a limited attention span, difficulties in comprehension, forgetfulness, and lack of motivation. Damage to the lower motor neurons may cause decreased muscle tone, flaccid muscle paralysis, and a decrease in or loss of reflexes.

A client is admitted with weakness, expressive aphasia, and right hemianopia. The brain MRI reveals an infarct. The nurse understands these symptoms to be suggestive of which of the following findings?

Left-sided cerebrovascular accident (CVA) When the infarct is on the left side of the brain, the symptoms are likely to be on the right, and the speech is more likely to be involved. If the MRI reveals an infarct, TIA is no longer the diagnosis. There is not enough information to determine if the stroke is still evolving or is complete.

A family member comes to the clinic to talk to the nurse about a client who has had a stroke on the right side of the brain. The family member is concerned because of the deficits the client is exhibiting. The nurse knows that when a client experiences a stroke on the right side of the brain, common deficits include what? Select all that apply.

Left-sided hemiplegia Tendency to distractibility Neglect of objects and people on the left side Left-sided hemiplegia (stroke on right side of brain) may have the following neurologic deficits: spatial-perceptual defects; disregard for the deficits of the affected side require special safety considerations; tendency to distractibility; impulsive behavior, unaware of deficits; poor judgment; defects in left visual fields; misjudge distances; difficulty distinguishing upsidedown and rightsideup; impairment of short-term memory; and neglect left side of body, objects and people on left side.

The nurse in the neurological unit is caring for a client with a supratentorial lesion. The nurse assesses which measurement as the most critical index of central nervous system (CNS) dysfunction? 1. Temperature 2. Blood pressure 3. Ability to speak 4. Level of consciousness

Level of consciousness Level of consciousness is the most critical index of CNS dysfunction. Changes in level of consciousness can indicate clinical improvement or deterioration. Although blood pressure, temperature, and ability to speak may be components of the assessment, the client's level of consciousness is the most critical index of CNS dysfunction.

A client has experienced an ischemic stroke that has damaged the frontal lobe of his brain. Which of the following deficits does the nurse expect to observe during assessment? a) Hemiplegia or hemiparesis b) Limited attention span and forgetfulness c) Visual and auditory agnosia d) Lack of deep tendon reflexes

Limited attention span and forgetfulness Damage to the frontal lobe may impair learning capacity, memory, or other higher cortical intellectual functions. Such dysfunction may be reflected in a limited attention span, difficulties in comprehension, forgetfulness, and a lack of motivation

A patient who has suffered a stroke begins having complications regarding spasticity in the lower extremity. What ordered medication does the nurse administer to help alleviate this problem?

Lioresal (Baclofen) Spasticity, particularly in the hand, can be a disabling complication after stroke. Botulinum toxin type A injected intramuscularly into wrist and finger muscles has been shown to be effective in reducing this spasticity (although the effect is temporary, typically lasting 2 to 4 months) (Teasell, Foley, Pereira, et al., 2012). Other treatments for spasticity may include stretching, splinting, and oral medications such as baclofen (Lioresal).

A client with myasthenia gravis is having difficulty speaking. The client's speech is dysarthric and has a nasal tone. The nurse should use which communication strategies when working with this client? Select all that apply.

Listening attentively Asking yes and no questions when able Using a communication board when necessary Repeating what the client said to verify the message The client has speech that is nasal in tone and dysarthric because of cranial nerve involvement of the muscles governing speech. The nurse listens attentively and verbally, verifies what the client has said, asks questions requiring a yes or no response, and develops alternative communication methods (e.g., letter board, picture board, pen and paper, flash cards). Encouraging the client to speak quickly is an ineffective communication strategy and is counterproductive.

Which of the following is accurate regarding a hemorrhagic stroke? a) It is caused by a large-artery thrombosis. b) One of the main presenting symptoms is numbness or weakness of the face. c) Main presenting symptom is an "exploding headache." d) Functional recovery usually plateaus at 6 months.

Main presenting symptom is an "exploding headache." One of hemorrhagic stroke's main presenting symptom is an "exploding headache." In ischemic stroke, functional recovery usually plateaus at 6 months; it may be caused by a large artery thrombosis and may have a presenting symptoms of numbness or weakness of the face.

The nurse is caring for a patient diagnosed with a hemorrhagic stroke. The nurse recognizes that which of the following interventions is most important? a) Maintaining a patent airway b) Elevating the head of the bed at 30 degrees c) Administering a stool softener d) Monitoring for seizure activity

Maintaining a patent airway Maintaining the airway is the most important nursing intervention. Immediate complications of a hemorrhagic stroke include cerebral hypoxia, decreased cerebral blood flow, and extension of the area of injury. Providing adequate oxygenation of blood to the brain minimizes cerebral hypoxia. Brain function depends on delivery of oxygen to the tissues. Administering supplemental oxygen and maintaining the hemoglobin and hematocrit at acceptable levels will assist in maintaining tissue oxygenation.

The nurse is planning to put aneurysm precautions in place for the client with a cerebral aneurysm. Which item should be included as part of the precautions?

Maintaining the head of the bed at 15 degrees Aneurysm precautions include placing the client on bed rest with the head of the bed elevated in a quiet setting. Lights are kept dim to minimize environmental stimulation. Any activity such as pushing, pulling, sneezing, coughing, or straining that increases blood pressure or impedes venous return from the brain is prohibited. The nurse provides all physical care to minimize increases in blood pressure. For the same reason, visitors, radio, television, and reading materials are prohibited or limited. Stimulants such as caffeine and nicotine are prohibited; decaffeinated coffee or tea may be given

A client with a spinal cord injury expresses little interest in food and is very particular about the choice of meals that are actually eaten. How should the nurse interpret this?

Meal choices represent an area of client control and should be encouraged as much as is nutritionally reasonable. Depression is frequently seen in the client with spinal cord injury and may be exhibited as a loss of appetite. The client should be allowed to choose the types of food eaten and to eat as much as is feasible because it is one of the few areas of control that the client has left.

While observing a child, the nurse notes that the child's arms and legs are extended and pronated. The nurse interprets this as indicating damage to which of the following? a) Midbrain b) Meninges c) Cerebral cortex d) Cranial nerves

Midbrain Correct Explanation: The observations indicate decerebrate posturing, which occurs with damage to the midbrain. Decorticate posturing as evidenced by arm adduction, elbow flexion with arms over the chest, and wrist flexion with fisted hands indicates damage to the cerebral cortex. Damage to the cranial nerves would be manifested by defects in motor and/or sensory function depending on the cranial nerves affected. Meningeal irritation as with bacterial meningitis is manifested by opisthotonus in an infant. With this position, the head and neck are hyperextended to relieve discomfort.

The nurse is reviewing the medical record of a client diagnosed with amyotrophic lateral sclerosis (ALS). Which initial sign/symptom of this disorder supports this diagnosis?

Mild clumsiness The initial manifestation of ALS is a mild clumsiness usually in the distal portion of one extremity. The client may complain of tripping and may drag one leg when the lower extremities are involved. Mentation and intellectual function are usually normal. Diminished gag reflex and muscle wasting are not initial clinical signs/symptoms.

The nurse is reviewing the record for a client seen in the health care clinic and notes that the health care provider has documented a diagnosis of amyotrophic lateral sclerosis (ALS). Which initial clinical manifestation of this disorder should the nurse expect to see documented in the record? 1. Muscle wasting 2. Mild clumsiness 3. Altered mentation 4. Diminished gag reflex

Mild clumsiness The initial symptom of ALS is a mild clumsiness, usually noted in the distal portion of one extremity. The client may complain of tripping and drag one leg when the lower extremities are involved. Mentation and intellectual function usually are normal. Diminished gag reflex and muscle wasting are not initial clinical manifestations.

A client was seen and treated in the hospital emergency department for a concussion. The nurse determines that the family needs further teaching if they verbalize to call the health care provider (HCP) for which client sign or symptom? 1. Vomiting 2. Minor headache 3. Difficulty speaking 4. Difficulty awakening

Minor headache A concussion after head injury is a temporary loss of consciousness (from a few seconds to a few minutes) without evidence of structural damage. After concussion, the family is taught to monitor the client and call the HCP or return the client to the emergency department for signs and symptoms such as confusion, difficulty awakening or speaking, one-sided weakness, vomiting, and severe headache. Minor headache is expected.

The client was seen and treated in the emergency department (ED) for a concussion. Before discharge, the nurse explains the signs/symptoms of a worsening condition. The nurse determines that the family needs further teaching if they state they will return to the ED if the client experiences which sign/symptom?

Minor headache A concussion after head injury is a temporary loss of consciousness (from a few seconds to a few minutes) without evidence of structural damage. After concussion, the family is taught to monitor the client and call the health care provider or return the client to the emergency department if certain signs and symptoms are noted. These include confusion, difficulty awakening or speaking, one-sided weakness, vomiting, or severe headache. Minor headache is expected.

The nurse caring for an infant with craniosynostosis, specifically positional plagiocephaly, should prioritize which activity?

Moving the infant's head every 2 hours Positional plagiocephaly can occur because the infant's head is allowed to stay in one position for too long. Because the bones of the skull are soft and moldable, they can become flattened if the head is allowed to remain in the same position for a long period of time. Massaging the scalp will not affect the skull. Measuring the intake and output is important but has no effect on the skull bones. Small feedings are indicated whenever an infant has increased intracranial pressure, but feeding an infant each time he fusses is inappropriate care.

The nurse is monitoring a child for bleeding after surgery for removal of a brain tumor. The nurse checks the head dressing for the presence of blood and notes a colorless drainage on the back of the dressing. Which intervention should the nurse perform immediately? Reinforce the dressing. 2. Notify the health care provider (HCP). 3. Document the findings and continue to monitor. 4. Circle the area of drainage and continue to monitor.

Notify the health care provider (HCP). Colorless drainage on the dressing in a child after craniotomy indicates the presence of cerebrospinal fluid and should be reported to the HCP immediately. Options 1, 3, and 4 are not the immediate nursing intervention because they do not address the need for immediate intervention to prevent complications

A client is recovering at home after suffering a brain attack (stroke) 2 weeks ago. A home caregiver tells the home health nurse that the client has some difficulty swallowing food and fluids. Which nursing action would be appropriate?

Observe the client feeding himself or herself. It is not uncommon for a client to have difficulty swallowing after having a brain attack (stroke). Often the client has hemiplegia. The client's arm may be paralyzed, and the client has to learn to use an opposite arm to feed himself or herself. Using a different arm may require rehabilitation and retraining. Also a client may have partial paralysis of the mouth, tongue, or esophagus. To best assist the client, the nurse should first assess the situation by watching the client feed himself or herself. Perhaps the problem lies in the feeding technique, the type of feeding tool used, the types of foods being served, or a combination of problems. Having someone else feed the client may be necessary if the client is determined to be unable to feed himself or herself, but this action does not promote independence in the client. A feeding syringe is not recommended for feeding most clients.

The nurse is collecting data on a client with a diagnosis of meningitis and notes that the client is assuming this posture. (Refer to figure.) The nurse contacts the health care provider and reports that the client is exhibiting which?

Opisthotonos Opisthotonos is a prolonged arching of the back with the head and heels bent backward. Opisthotonos indicates meningeal irritation. In decorticate rigidity, the upper extremities (arms, wrists, and fingers) are flexed with adduction of the arms. The lower extremities are extended with internal rotation and plantar flexion. Decorticate rigidity indicates a hemispheric lesion of the cerebral cortex. In decerebrate rigidity, the upper extremities are stiffly extended and adducted with internal rotation and pronation of the palms. The lower extremities are extended stiffly with plantar flexion. The teeth are clenched and the back is hyperextended. Decerebrate rigidity indicates a lesion in the brainstem at the midbrain or upper pons. Flaccid quadriplegia is complete loss of muscle tone and paralysis of all four extremities, indicating a completely nonfunctional brain stem.

The nurse is preparing for the admission to the unit of a client with a diagnosis of seizures and is preparing to institute full seizure precautions. Which item is contraindicated for use if a seizure occurs? Oxygen source 2. Suction machine 3. Padded tongue blade 4. Padding for the side rails

Padded tongue blade Full seizure precautions include bed rest with padded side rails in a raised position, a suction machine at the bedside, having diazepam (Valium) or lorazepam (Ativan) available, and providing an oxygen source. Objects such as tongue blades are contraindicated and should never be placed in the client's mouth during a seizure.

The nurse is preparing for the admission to the unit of a client with a diagnosis of seizures and asks the nursing student to institute full seizure precautions. Which item if noted in the client's room would need to be removed and warrants the need to review seizure precautions with the student? 1. Oxygen source 2. Suction machine 3. Padded tongue blade 4. Padding for the side rails

Padded tongue blade Full seizure precautions include bed rest with padded side rails in a raised position, a suction machine at the bedside, having diazepam or lorazepam available, and providing an oxygen source. Objects such as tongue blades are contraindicated and should never be placed in the client's mouth during a seizure.

The clinic nurse is reviewing the record of a client scheduled to be seen in the clinic. The nurse notes that the client is taking selegiline hydrochloride. The nurse suspects that the client has which disorder? 1. Diabetes mellitus 2. Parkinson's disease 3. Alzheimer's disease 4. Coronary artery disease

Parkinson's disease Selegiline hydrochloride is an antiparkinsonian medication. The medication increases dopaminergic action, assisting in the reduction of tremor, akinesia, and the rigidity of parkinsonism. This medication is not used to treat diabetes mellitus, Alzheimer's disease, or coronary artery disease.

A nurse is caring for a 3-year-old girl with microcephaly. Which of the following actions is appropriate for the nurse to take? a) Playfully ask the child to touch her nose. b) Teach the parents about ventriculoperitoneal (VP) shunts. c) Administer antipyretics as ordered. d) Prepare the child for the experience of cranial surgery.

Playfully ask the child to touch her nose. Correct Explanation: Having the child touch her nose will assist the nurse in assessing probable neurologic and cognitive deficits. A VP shunt may be necessary for hydrocephaly, not this disorder. Surgery is often an intervention for craniosynostosis but cannot correct microcephaly. Hyperthermia is not a complication with microcephaly.

The nurse is planning to put aneurysm precautions in place for a client with a cerebral aneurysm. Which nursing measure would be a potentially unsafe component of the precautions? Provide physical aspects of care. 2. Prevent pushing or straining activities. 3. Maintain the head of the bed at 15 degrees. 4. Limit caffeinated coffee to one cup per day.

Provide physical aspects of care. Aneurysm precautions include placing the client on bed rest (as prescribed) in a quiet setting. Stimulants such as caffeine and nicotine are prohibited; decaffeinated coffee or tea may be used. Lights are kept dim to minimize environmental stimulation. Any activity that increases the blood pressure or impedes venous return from the brain is prohibited, such as pushing, pulling, sneezing, coughing, or straining. The nurse provides physical care to minimize increases in blood pressure. For the same reason, visitors, radio, television, and reading materials are prohibited or limited.

The nurse is providing diet-related advice to a male patient following a cerebrovascular accident (CVA). The patient wants to minimize the volume of food and yet meet all nutritional elements. Which of the following suggestions should the nurse give to the patient about controlling the volume of food intake?

Provide thickened commercial beverages and fortified cooked cereals. Patients with CVA or other cerebrovascular disorders should lose weight and therefore should minimize their volume of food consumption. To ensure this, the nurse may provide thickened commercial beverages, fortified cooked cereals, or scrambled eggs. Patients should avoid eating high-fat foods, and serving foods hot or tepid will not minimize the volume consumed by the patient. Foods such as peanut butter, bread, tart foods, dry or crisp foods, and chewy meats should also be avoided because they cause choking.

The nurse is planning to institute seizure precautions for a client who is being admitted from the emergency department. Which measure should the nurse avoid in planning for the client's safety?

Putting a padded tongue blade at the head of the bed Seizure precautions may vary somewhat from agency to agency, but they generally have some commonalities. Usually airway, oxygen, and suctioning equipment are kept available at the bedside. The side rails of the bed are padded, and the bed is kept in the lowest position. The client should have an IV access in place to have a readily accessible route if anticonvulsant medications must be administered. The use of padded tongue blades is no longer best practice, and they should not be kept at the bedside. Forcing a tongue blade into the mouth during a seizure will more likely harm the client who bites down during seizure activity. Other risks include blocking the airway from improper placement, chipping the client's teeth, and subsequent risk of aspirating tooth fragments. If the client has an aura before the seizure, it may give the nurse enough time to place an oral airway before seizure activity begins.

The nurse cares for a client immediately following a lumbar laminectomy procedure. The client reports numbness and tingling down the left lateral thigh and knee. What is the next action for the nurse to take? 1. Document the assessment. 2. Contact the health care provider (HCP). 3. Inform the client that this is to be expected. 4. Question the client about preoperative symptoms.

Question the client about preoperative symptoms. After spinal surgery the client requires frequent neurological assessments. Movement of the arms and the legs and assessment of sensation should be unchanged when compared with the preoperative status. Thus, the correct option is 4. Although the assessment finding should be documented, option 1 is incorrect, as that is not the next thing the nurse should do. Option 2 is incorrect, as more assessment data are needed before calling the HCP. Option 3 is incorrect because this is an unexpected finding except if these findings were present before surgery; however, that preoperative information hasn't been gathered yet.

NO LONGER USE FOR ICP The nurse has a prescription to give dexamethasone (Decadron) by the intravenous (IV) route to a client with cerebral edema. How should the nurse prepare this medication? 1. Diluting the medication in 500 mL of 5% dextrose 2. Preparing an undiluted direct injection of the medication 3. Diluting the medication in 1 mL of lactated Ringer's solution for direct injection 4. Diluting the medication in 10% dextrose in water and administering it as a direct injection

Rationale: Dexamethasone may be given by direct IV injection or IV infusion. For IV infusion, it may be mixed with 50 to 100 mL of 0.9% sodium chloride or 5% dextrose in water. It is not mixed with lactated Ringer's solution or 10% dextrose in water.

An infant with a diagnosis of hydrocephalus is scheduled for surgery. Which is the priority nursing intervention in the preoperative period? Test the urine for protein. 2. Reposition the infant frequently. 3. Provide a stimulating environment. 4. Assess blood pressure every 15 minutes.

Reposition the infant frequently. An infant with a diagnosis of hydrocephalus is scheduled for surgery. Which is the priority nursing intervention in the preoperative period?Rationale: Hydrocephalus occurs as a result of an imbalance of cerebrospinal fluid absorption or production that is caused by malformations, tumors, hemorrhage, infections, or trauma. It results in head enlargement and increased intracranial pressure. In infants with hydrocephalus, the head grows at an abnormal rate, and if the infant is not repositioned frequently, pressure ulcers can occur on the back and side of the head. An egg crate mattress under the head is also a nursing intervention that can help prevent skin breakdown. Proteinuria is not specific to hydrocephalus. Stimulus should be kept at a minimum because of the increase in intracranial pressure. It is not necessary to check the blood pressure every 15 minutes.

The nurse is caring for a client who begins to experience seizure activity while in bed. Which action by the nurse would be contraindicated?

Restrain the client's limbs. Nursing actions during a seizure include providing privacy; loosening restrictive clothing; removing the pillow and raising the padded side rails in bed; and placing the client on one side with the head flexed forward, if possible, to allow the tongue to fall forward and facilitate drainage. The limbs are never restrained because the strong muscle contractions could cause the client harm. If the client is not in bed when seizure activity begins, the nurse lowers the client to the floor, if possible; protects the head against injury; and moves furniture that may injure the client.

The nurse is assessing the client's level of consciousness and documents that the client has delirium. On the basis of this documentation, the nurse should determine that there is damage to which area of the nervous system? 1. Temporal lobe and frontal lobe 2. Hippocampus and frontal lobe 3. Limbic system and cerebral hemispheres 4. Reticular activating system and cerebral hemispheres

Reticular activating system and cerebral hemispheres Insomnia, agitation, mania, and delirium are caused by excessive arousal of the reticular activating system in conjunction with the cerebral hemispheres. The temporal lobe, hippocampus, and frontal lobe are responsible for memory. The limbic system is responsible for feelings and affect.

The nurse has given instructions to the client with Parkinson's disease about maintaining mobility. The nurse determines that the client understands the directions if the client states that he or she will perform which activity?

Rock back and forth to start movement with bradykinesia. The client with Parkinson's disease should exercise in the morning, when energy levels are highest. The client should avoid sitting in soft, deep chairs because getting up from them can be difficult. The client can rock back and forth to initiate movement. The client should buy clothes with Velcro fasteners and slide-locking buckles to allow for easier dressing.

The nurse is caring for a patient having a hemorrhagic stroke. What position in the bed will the nurse maintain this patient? a) Semi-Fowler's b) Supine c) High-Fowler's d) Prone

Semi-Fowler's The head of the bed is elevated 15 to 30 degrees (semi-Fowler's position) to promote venous drainage and decrease intracranial pressure.

The nurse is assisting with caring for a client after a craniotomy. Which is the best position for the client to be placed?

Semi-Fowler's position After a craniotomy, the head of the bed is elevated 30 to 45 degrees (semi-Fowler's to Fowler's position), and the client's head is maintained in a midline, neutral position to facilitate venous drainage.

The nurse is caring for a client following a supratentorial craniotomy, in which a large tumor was removed from the left side. In which position can the nurse safely place the client? 1. Semi-Fowler's 2. Trendelenburg's 3. Reverse Trendelenburg's 4. Flat

Semi-Fowlers Rationale: Clients who have undergone supratentorial surgery should have the head of the bed elevated 30 degrees to promote venous drainage from the head. The client is positioned to avoid extreme hip or neck flexion and the head is maintained in a midline neutral position. If a large tumor has been removed, the client should be placed on the nonoperative side to prevent displacement of the cranial contents. A flat position or Trendelenburg's position would increase intracranial pressure. A reverse Trendelenburg's position would not be helpful and may be uncomfortable for the client.

A client had a transsphenoidal resection of the pituitary gland. The nurse notes drainage on the nasal dressing. Suspecting cerebrospinal fluid (CSF) leakage, the nurse should look for drainage that is of which characteristic? 1. Serosanguineous only 2. Bloody with very small clots 3. Sanguineous only with no clot formation 4. Serosanguineous, surrounded by clear to straw-colored fluid

Serosanguineous, surrounded by clear to straw-colored fluid CSF leakage after cranial surgery may be detected by noting drainage that is serosanguineous (from the surgery) and surrounded by an area of clear or straw-colored drainage. The typical appearance of CSF drainage is that of a "halo." The nurse also would further verify actual CSF drainage by testing the drainage for glucose, which would be positive.

A client had a transsphenoidal resection of the pituitary gland. The nurse notes drainage on the nasal dressing. Suspecting cerebrospinal fluid (CSF) leakage, the nurse should look for drainage that is of which characteristic? Serosanguineous only 2. Bloody with very small clots 3. Sanguineous only with no clot formation 4. Serosanguineous, surrounded by clear to straw-colored fluid

Serosanguineous, surrounded by clear to straw-colored fluid Rationale: CSF leakage after cranial surgery may be detected by noting drainage that is serosanguineous (from the surgery) and surrounded by an area of clear or straw-colored drainage. The typical appearance of CSF drainage is that of a "halo." The nurse also would further verify actual CSF drainage by testing the drainage for glucose, which would be positive. Rationale: CSF leakage after cranial surgery may be detected by noting drainage that is serosanguineous (from the surgery) and surrounded by an area of clear or straw-colored drainage. The typical appearance of CSF drainage is that of a "halo." The nurse also would further verify actual CSF drainage by testing the drainage for glucose, which would be positive.

The nurse is assessing fluid balance in a client who has undergone a craniotomy. The nurse should assess for which finding as a sign of overhydration, which would aggravate cerebral edema? 1. Unchanged weight 2. Shift intake 950 mL, output 900 mL 3. Blood urea nitrogen (BUN) 10 mg/dL (3.6 mmol/L) 4. Serum osmolality 280 mOsm/kg H2O (280 mmol/kg)

Serum osmolality 280 mOsm/kg H2O (280 mmol/kg) After craniotomy the goal is to keep the serum osmolality on the high side of normal to minimize excess body water and control cerebral edema. The normal serum osmolality is 285 to 295 mOsm/kg H2O (285 to 295 mmol/kg). A higher value indicates dehydration; a lower value indicates overhydration. Stable weight indicates that there is neither fluid excess nor fluid deficit. A difference of 50 mL in intake and output for an 8-hour shift is insignificant. The BUN of 10 mg/dL (3.6 mmol/L) is within normal range and does not indicate overhydration or underhydration.

The nurse is teaching a client with paraplegia measures to maintain skin integrity. Which instruction will be most helpful to the client? 1. Shift weight every 2 hours while in a wheelchair. 2. Change bed sheets every other week to maintain cleanliness. 3. Place a pillow on the seat of the wheelchair to provide extra comfort. 4. Use a mirror to inspect for redness and skin breakdown twice a week.

Shift weight every 2 hours while in a wheelchair. To maintain skin integrity, the client should shift weight in the wheelchair every 2 hours and use a pressure relief pad. A pillow is not sufficient to relieve the pressure. While the client is in bed, the bottom sheet should be free of wrinkles and wetness. Sheets should be changed as needed and more frequently than every other week. The client should use a mirror to inspect the skin twice daily (morning and evening) to assess for redness, edema, and breakdown. General additional measures include a nutritious diet and meticulous skin care.

Which nursing assessment data should be given the highest priority for a child with clinical findings related to meningitis? a) Occurrence of urine and fecal contamination b) Degree and extent of nuchal rigidity c) Onset and character of fever d) Signs of increased intracranial pressure (ICP)

Signs of increased intracranial pressure (ICP) Correct Explanation: Assessment of fever and evaluation of nuchal rigidity are important aspects of care, but assessment for signs of increasing ICP should be the highest priority due to the life-threatening implications. Urinary and fecal incontinence can occur in a child who's ill from nearly any cause but doesn't pose a great danger to life.

The nurse has applied a hypothermia blanket to a client with a fever. The nurse should inspect the skin frequently to detect which condition that is a complication of hypothermia blanket use? 1. Frostbite 2. Skin breakdown 3. Arterial insufficiency 4. Venous insufficiency

Skin breakdown When a hypothermia blanket is used, the skin is inspected frequently for pressure points, which over time could lead to skin breakdown. The hypothermia blanket decreases the blood flow to pressure areas and can cause numbness, making it so that the client is not aware of damage to the skin. The temperature of the blanket is not cold enough to cause frostbite. Arterial insufficiency and venous insufficiency are not complications of hypothermia blanket use.

The nurse is reviewing a discharge teaching plan for a postcraniotomy client that was prepared by a nursing student. The nurse would intervene and provide teaching to the student if the student included which home care instruction? Sounds will not be heard clearly unless they are loud. 2. Obtain assistance with ambulation if client is lightheaded. 3. Tub bath or shower is permitted, but the scalp is kept dry until the sutures are removed. 4. Use a check-off system for administering anticonvulsant medications to avoid missing doses.

Sounds will not be heard clearly unless they are loud. Rationale: The postcraniotomy client typically is sensitive to loud noises and can find them excessively irritating. Control of environmental noise by others will be helpful for this client. Seizures are a potential complication that may occur for up to 1 year after surgery. For this reason, the client must diligently take anticonvulsant medications. The client and family are encouraged to keep track of the doses administered. The family should learn seizure precautions and should accompany the client during ambulation if dizziness or seizures tend to occur. The suture line is kept dry until sutures are removed to prevent infection.

The nurse is reviewing a discharge teaching plan for a postcraniotomy client that was prepared by a nursing student. The nurse would intervene and provide teaching to the student if the student included which home care instruction? 1. Sounds will not be heard clearly unless they are loud. 2. Obtain assistance with ambulation if the client is lightheaded. 3. Tub bath or shower is permitted, but the scalp is kept dry until the sutures are removed. 4. Use a check-off system for administering anticonvulsant medications to avoid missing doses.

Sounds will not be heard clearly unless they are loud. The postcraniotomy client typically is sensitive to loud noises and can find them excessively irritating. Control of environmental noise by others will be helpful for this client. Seizures are a potential complication that may occur for up to 1 year after surgery. For this reason, the client must diligently take anticonvulsant medications. The client and family are encouraged to keep track of the doses administered. The family should learn seizure precautions and should accompany the client during ambulation if dizziness or seizures tend to occur. The suture line is kept dry until sutures are removed to prevent infection.

The nurse has just received report on a client in the ED being transferred to the acute stroke unit with a diagnosis of a right hemispheric stroke. Which findins does the nurse understand is indicative of a right hemispheric stroke?

Spatial-perceptual deficits Clients with right hemispheric stroke exhibit partial perceptual deficits, left visual field deficit, and paralysis with weakness on the left side of the body. Left hemispheric damage causes aphasia, slow, cautious behavior, and altered intellectual ability.

A client has suffered damage to Broca's area of the brain. Which priority assessment should the nurse perform? 1. Speech 2. Hearing 3. Balance 4. Level of consciousness

Speech Broca's area in the brain is responsible for the motor aspects of speech, through coordination of the muscular activity of the tongue, mouth, and larynx. The term assigned to damage in this area is aphasia. The items listed in the other options are not the responsibility of Broca's area.

The nurse is assigned to care for an 8-year-old child with a diagnosis of a basilar skull fracture. The nurse reviews the health care provider's (HCPs) prescriptions and should contact the HCP to question which prescription? Suction as needed. 2. Obtain daily weight. 3. Provide clear liquid intake. 4. Maintain a patent intravenous line.

Suction as needed. basilar skull fracture is a type of head injury. Nasotracheal suctioning is contraindicated in a child with a basilar skull fracture: Because of the nature of the injury, there is a possibility that the catheter will enter the brain through the fracture, creating a high risk of secondary infection. Fluid balance is monitored closely by daily weight determination, intake and output measurement, and serum osmolality determination to detect early signs of water retention, excessive dehydration, and states of hypertonicity or hypotonicity. The child is maintained on NPO status or restricted to clear liquids until it is determined that vomiting will not occur. An intravenous line is maintained to administer fluids or medications if necessary.

The nurse develops a plan of care for a child at risk for tonic-clonic seizures. In the plan of care, the nurse identifies seizure precautions and documents that which item(s) need to be placed at the child's bedside?

Suctioning equipment and oxygen A seizure results from the excessive and unorganized neuronal discharges in the brain that activate associated motor and sensory organs. A type of generalized seizure is a tonic-clonic seizure. This type of seizure causes rigidity of all body muscles, followed by intense jerking movements. Because increased oral secretions and apnea can occur during and after the seizure, oxygen and suctioning equipment are placed at the bedside. A tracheotomy is not performed during a seizure. No object, including a padded tongue blade, is placed into the child's mouth during a seizure. An emergency cart would not be left at the bedside, but would be available in the treatment room or nearby on the nursing unit.

A client is anxious about an upcoming diagnostic procedure. The client's pupils are dilated, and the respiratory rate, heart rate, and blood pressure are increased from baseline. The nurse determines that the client's clinical manifestations are due to what type of physiologic response? 1. Vagal 2. Peripheral nervous system 3. Sympathetic nervous system 4. Parasympathetic nervous system

Sympathetic nervous system The sympathetic nervous system is responsible for the so-called fight or flight response, which is characterized by dilated pupils, increases in heart rate and cardiac output, and increases in respiratory rate and blood pressure. The sympathetic nervous system response affects some type of change in most systems of the body. The responses stated in the other options do not produce these effects.

The nurse is teaching a client with myasthenia gravis about the prevention of myasthenic and cholinergic crises. Which client activity suggests that teaching is most effective? 1. Taking medications as scheduled 2. Eating large, well-balanced meals 3. Doing muscle-strengthening exercises 4. Doing all chores early in the day while less fatigued

Taking medications as scheduled Clients with myasthenia gravis are taught to space out activities over the day to conserve energy and restore muscle strength. Taking medications correctly to maintain blood levels that are not too low or too high is important. Muscle-strengthening exercises are not helpful and can fatigue the client. Overeating is a cause of exacerbation of symptoms, as is exposure to heat, crowds, erratic sleep habits, and emotional stress.

Which intervention should the nurse include in a postoperative teaching plan for a client who underwent a spinal fusion and will be wearing a brace? 1. Tell the client to inspect the environment for safety hazards. 2. Inform the client about the importance of sitting as much as possible. 3. Inform the client that lotions and body powders can be used for skin breakdown. 4. Instruct the client to tighten the brace during meals and to loosen it for the first 30 minutes after each meal.

Tell the client to inspect the environment for safety hazards. The client must inspect the environment for safety hazards. The client is instructed in the importance of avoiding prolonged sitting and standing. Powders and lotions should not be used because they may irritate the skin. The client should be taught to loosen the brace during meals and for 30 minutes after each meal. The client may have difficulty eating if the brace is too tight. Loosening the brace after each meal will allow adequate nutritional intake and promote comfort.

The nurse is caring for an 8-year-old girl who was in a car accident. What would lead the nurse to suspect a concussion? a) The child has vomited and has bruising behind her ear. b) The child is weak and has blurry vision. c) The child is easily distracted and can't concentrate. d) The child is bleeding from the ear and draining fluid from the nose.

The child is easily distracted and can't concentrate.

The nurse is interviewing the caregivers of a child brought to the emergency unit. The caregiver states, "She has a history of seizures but this time it lasted more than 30 minutes and she just keeps having them." The most accurate description of this child's condition would be which of the following? a) The child may begin to have absence seizures every day. b) The child is having generalized seizures. c) The child is in status epilepticus. d) The child's history indicates she has infantile seizures.

The child is in status epilepticus. Correct Explanation: Status epilepticus is the term used to describe a seizure that lasts longer than 30 minutes or a series of seizures in which the child does not return to his or her previous normal level of consciousness. The child likely is having generalized seizures, but the most accurate description of what is happening is status epilepticus. With infantile spasms, muscle contractions are sudden, brief, symmetrical, and accompanied by rolling eyes. With absence seizures the child loses awareness and stares straight ahead but does not fall.

A mother has brought her 5-month-old son to the clinic because he has been drowsy and unresponsive. The child has hydrocephalus and had a shunt placed about a month previously. Which of the following symptoms indicate that the shunt is infected? a) The fontanels are bulging or tense. b) The child is not responding or eating well. c) The child's pupil reaction time is rapid and uneven. d) The child has a high-pitched cry.

The child is not responding or eating well. Correct Explanation: Poor feeding and decreased responsiveness are signs of an infection. The nurse might also observe localized inflammation along the shunt tract. Bulging or tense fontanels suggest a shunt malfunction that is causing increased intracranial pressure. A high-pitched cry suggests increased intracranial pressure due to a shunt malfunction. Decreased and uneven pupil reaction times are symptoms of a shunt malfunction that is causing increased intracranial pressure.

The nurse is caring for a child admitted with complex partial seizures. Which of the following clinical manifestations would likely have been noted in the child with this diagnosis? a) The child was dizzy and had decreased coordination. b) The child had shaking movements on one side of the body. c) The child was rubbing the hands and smacking the lips. d) The child had jerking movements and then the extremities stiffened.

The child was rubbing the hands and smacking the lips.

In caring for a child with a seizure disorder, the primary goal of treatment is:

The child will be free from injury during a seizure. Keeping the child free from injury is the highest priority goal. The other choices are important, but keeping the child safe is higher than the anxiety or knowledge deficit concerns. The physical always is a priority over the psychological.

The nurse is reviewing the medical records of a client admitted to the nursing unit with a diagnosis of a thrombotic brain attack (stroke). The nurse would expect to note that which is documented in the assessment data section of the record? 1. Sudden loss of consciousness occurred. 2. Signs and symptoms occurred suddenly. 3. The client experienced paresthesias a few days before admission to the hospital. 4. The client complained of a severe headache, which was followed by sudden onset of paralysis.

The client experienced paresthesias a few days before admission to the hospital. Cerebral thrombosis does not occur suddenly. In the few hours or days preceding a thrombotic brain attack (stroke), the client may experience a transient loss of speech, hemiplegia, or paresthesias on 1 side of the body. Signs and symptoms of thrombotic brain attack (stroke) vary but may include dizziness, cognitive changes, or seizures. Headache is rare, but some clients with stroke (brain attack) experience signs and symptoms similar to those of cerebral embolism or intracranial hemorrhage.

The nurse is assigned to care for a client with complete right-sided hemiparesis. Which characteristics are associated with this condition? Select all that apply. The client is aphasic. 2. The client has weakness in the face and tongue. 3. The client has weakness on the right side of the body. 4. The client has complete bilateral paralysis of the arms and legs. 5. The client has lost the ability to move the right arm but is able to walk independently. 6. The client has lost the ability to ambulate independently but is able to feed and bathe himself or herself without assistance.

The client is aphasic. 2. The client has weakness in the face and tongue. 3. The client has weakness on the right side of the body. Hemiparesis is a weakness of one side of the body that may occur after a stroke. It involves weakness of the face and tongue, arm, and leg on one side. These clients are also aphasic: unable to discriminate words and letters. They are generally very cautious and get anxious when attempting a new task. Complete bilateral paralysis does not occur in this hemiparesis. The client with right-sided hemiparesis has weakness of the right arm and leg and needs assistance with feeding, bathing, and ambulating.

The nurse is caring for a client with a diagnosis of right (nondominant) hemispheric brain attack (stroke). The nurse notes that the client is alert and oriented to time and place. Based on these findings, the nurse makes which determination?

The client may have perceptual and spatial disabilities. The client with a right (nondominant) hemispheric stroke may be alert and oriented to time and place. These signs of apparent wellness often result in interpretations that the client is less disabled than is the case. However, impulsive actions and confusion in carrying out activities may be very much a problem for these clients, as a result of perceptual and spatial disabilities. The right hemisphere is considered specialized in sensory-perceptual and visuospatial processing and awareness of body space. The left hemisphere is dominant for language abilities

The nurse is assessing a client for meningeal irritation and elicits a positive Brudzinski's sign. Which finding did the nurse observe? 1. The client rigidly extends the arms with pronated forearms and plantar flexion of the feet. 2. The client flexes a leg at the hip and knee and reports pain in the vertebral column when the leg is extended. 3. The client passively flexes the hip and knee in response to neck flexion and reports pain in the vertebral column. 4. The client's upper arms are flexed and held tightly to the sides of the body and the legs are extended and internally rotated.

The client passively flexes the hip and knee in response to neck flexion and reports pain in the vertebral column. Rationale: Brudzinski's sign is tested with the client in the supine position. The nurse flexes the client's head (gently moves the head to the chest) and there should be no reports of pain or resistance to the neck flexion. A positive Brudzinski's sign is observed if the client passively flexes the hip and knee in response to neck flexion and reports pain in the vertebral column. Kernig's sign also tests for meningeal irritation and is positive when the client flexes the legs at the hip and knee and complains of pain along the vertebral column when the leg is extended. Decorticate posturing is abnormal flexion and is noted when the client's upper arms are flexed and held tightly to the sides of the body and the legs are extended and internally rotated. Decerebrate posturing is abnormal extension and occurs when the arms are fully extended, forearms pronated, wrists and fingers flexed, jaws clenched, neck extended, and feet plantar-flexed.

The nurse is caring for a client with a head injury and is monitoring the client for signs of increased intracranial pressure (ICP). Which sign if noted in the client should the nurse report immediately?

The client vomits. The client with a closed head injury is at risk of developing increased ICP. This is evidenced by symptoms such as headache, dizziness, confusion, weakness, and vomiting. Options 2, 3, and 4 are expected occurrences. Option 1 may be an indication of increased ICP, requiring notification of the registered nurse and health care provider.

The nurse is caring for a client who was admitted for a stroke (brain attack) of the temporal lobe. Which clinical manifestations should the nurse expect to note in the client? 1. The client will be unable to recall past events. 2. The client will have difficulty understanding language. 3. The client will demonstrate difficulty articulating words. 4. The client will have difficulty moving 1 side of the body.

The client will have difficulty understanding language. Wernicke's area consists of a small group of cells in the temporal lobe, the function of which is the understanding of language. The hippocampus is responsible for the storage of memory (the client will be unable to recall past events). Damage to Broca's area is responsible for aphasia (the client will demonstrate difficulty articulating words). The motor cortex in the precentral gyrus controls voluntary motor activity (the client will have difficulty moving one side of the body).

CSF leakage after cranial surgery may be detected by noting drainage that is serosanguineous (from the surgery) and surrounded by an area of clear or straw-colored drainage. The typical appearance of CSF drainage is that of a "halo." The nurse also would further verify actual CSF drainage by testing the drainage for glucose, which would be positive. 1. The left side of the body 2. The right side of the body 3. Both sides of the body equally 4. Cranial nerves only, such as speech and pupillary response

The left side of the body Motor responses such as weakness and decreased movement will be seen on the side of the body that is opposite an area of head injury. Contralateral deficits result from compression of the cortex of the brain or the pyramidal tracts. Depending on the severity of the injury, the client may have a variety of neurological deficits.

After a difficult birth, the nurse observes that a newborn has swelling on part of his head. The nurse suspects caput succedaneum based on which of the following? a) The infant had a low birthweight when born at term. b) The infant had low-set ears and facial abnormalities. c) The swelling crosses the midline of the infant's scalp. d) The swelling is limited to one small area without crossing the sagittal suture.

The swelling crosses the midline of the infant's scalp. Correct Explanation: The fact that the swelling crosses the midline of the infant's scalp indicates caput succedaneum. If the swelling is limited and does not cross the midline or suture lines, it would suggest cephalohematoma. Low birthweight does not suggest caput succedaneum. Low-set ears may be seen in infants with chromosomal abnormalities. Facial abnormalities may accompany encephalocele.

Which of the following age groups of children have the highest actual rate of death from drowning? a) School-age children b) Toddlers c) Preschool children d) Infants

Toddlers

Which of these age groups has the highest actual rate of death from drowning?

Toddlers Toddlers and older adolescents have the highest actual rate of death from drowning.

Which of the following age groups of children have the highest actual rate of death from drowning? a) School-age children b) Infants c) Preschool children d) Toddlers

Toddlers Correct Explanation: Toddlers and older adolescents have the highest actual rate of death from drowning.

Which of the following is the chief cause of intracerebral hemorrhage (ICH)? a) Diabetes b) Uncontrolled hypertension c) Migraine headaches d) Hypercholesterolemia

Uncontrolled hypertension Primary intracerebral hemorrhage (ICH) from a spontaneous rupture of small arteries or arterioles accounts for approximately 80% of hemorrhagic strokes and is caused chiefly by uncontrolled hypertension.

The nurse is educating the parents of a 7-year-old girl with epilepsy about managing treatment of the disorder at home. Which intervention is most effective for eliminating breakthrough seizures? a) Treating the child as though she did not have epilepsy b) Understanding the side effects of medications c) Instructing her teacher how to respond to a seizure d) Placing the child on her side on the floor

Understanding the side effects of medications

The nurse is educating the parents of a 7-year-old girl with epilepsy about managing treatment of the disorder at home. Which of the following interventions is most effective for eliminating breakthrough seizures? a) Instructing her teacher how to respond to a seizure b) Understanding the side effects of medications c) Placing the child on her side on the floor d) Treating the child as though she did not have epilepsy

Understanding the side effects of medications Correct Explanation: The most common cause of breakthrough seizures is noncompliance with medication administration, which may occur if the parents do not understand what side effects to expect or how to deal with them. Treating the child as though she did not have epilepsy helps improve her self-image and self-esteem. Placing the child on her side on the floor is an intervention to prevent injury during a seizure. Instructing the teacher on how to respond when a seizure occurs will help relieve anxiety and provide a sense of control.

A client who is experiencing an inferior wall myocardial infarction has had a drop in heart rate into the 50 to 56 beats/minute range. The client is also complaining of nausea. Which cranial nerve damage should the nurse expect that the client is experiencing? 1. Vagus (CN X) 2. Hypoglossal (CN XII) 3. Spinal accessory (CN XI) 4. Glossopharyngeal (CN IX)

Vagus (CN X) The vagus nerve is responsible for sensations in the thoracic and abdominal viscera. It is also responsible for the decrease in heart rate because approximately 75% of all parasympathetic stimulation is carried by the vagus nerve. CN IX is responsible for taste in the posterior two thirds of the tongue, pharyngeal sensation, and swallowing. CN XI is responsible for neck and shoulder movement. CN XII is responsible for tongue movement.

The nurse is caring for a client with trigeminal neuralgia (tic douloureux). The client asks for a snack and something to drink. The nurse should offer which best snack to the client? 1. Cocoa with honey and toast 2. Hot herbal tea with graham crackers 3. Iced coffee and peanut butter and crackers 4. Vanilla wafers and room-temperature water

Vanilla wafers and room-temperature water Because mild tactile stimulation of the face can trigger pain in trigeminal neuralgia, the client needs to eat or drink lukewarm, nutritious foods that are soft and easy to chew. Extremes of temperature will cause trigeminal nerve pain. Therefore, the options that include cocoa, hot herbal tea, and iced coffee are incorrect.

The nurse is assessing a toddler for motor function. Which of the following activities will be most valuable? a) Give the child some potato chips. b) Watch the child playing with a pull-toy. c) Let the child look at a picture book. d) Have the child catch a ball.

Watch the child playing with a pull-toy. Correct Explanation: Watching the child playing with a pull-toy would be most valuable for assessing motor function. Catching a ball is too advanced for a toddler to accomplish. Looking at a picture book would help assess visual acuity and eye movement. Eating potato chips would help assess sensor function for taste.

The nurse has instructed the family of a client with stroke (brain attack) who has homonymous hemianopsia about measures to help the client overcome the deficit. Which statement suggests that the family understands the measures to use when caring for the client? We need to discourage him from wearing eyeglasses." 2. "We need to place objects in his impaired field of vision." 3. "We need to approach him from the impaired field of vision." 4. "We need to remind him to turn his head to scan the lost visual field."

We need to remind him to turn his head to scan the lost visual field."

The nurse is providing information about strokes to a community group. Which of the following would the nurse identify as the primary initial symptoms of an ischemic stroke?

Weakness on one side of the body and difficulty with speech The main presenting symptoms for an ischemic stroke are numbness or weakness of the face, arm, or leg, especially on one side of the body; confusion or change in mental status; and trouble speaking or understanding speech. Severe headache, vomiting, early change in level of consciousness, and seizures are early signs of a hemorrhagic stroke. Footdrop and external hip rotation are things that can occur if a stroke victim is not turned or positioned correctly.

A nurse is performing a neurological examination of a preschool girl. She is testing her remote memory. Which of the following would be an appropriate type of memory to ask the girl to recall? a) Where the girl and her family went on vacation last year b) What the girl had for dinner last night c) A string of three digits that the nurse has just spoken to her d) The name of an object that the nurse showed her 5 minutes ago

What the girl had for dinner last night Explanation: Immediate recall is the ability to retain a concept for a short time such as being able to remember a series of numbers and repeat them (a child of 4 years can usually repeat three digits; a child older than 6 years can repeat five digits). Recent memory covers a slightly longer period of time. To measure this, show the preschool child an object such as a key and ask him to remember it, because later you will ask him to tell you what it was. After about 5 minutes, ask whether he remembers what object you showed him. Ask older children what they ate for breakfast to test recent memory. Remote memory is long-term recall. Ask preschoolers what they ate for breakfast that morning, or dinner the night before as, for them, that was a long time ago; ask older children what was the name of their first-grade teacher as most people remember that their whole life.

The nurse is testing the spinal reflexes of a client during neurological assessment. Which assessment by the nurse would help to determine the adequacy of the spinal reflex? 1. Cough reflex 2. Withdrawal reflex 3. Munro-Kellie reflex 4. Accommodation reflex

Withdrawal reflex The withdrawal reflex is one of the spinal reflexes. It is an abrupt withdrawal of a body part in response to painful or injurious stimuli. The cough reflex is a brainstem-associated reflex. Accommodation reflex is associated with cranial nerve III and is part of the ocular motor system. Munro-Kellie is not a reflex; it is a doctrine or a hypothesis addressing the cerebral volume relationships among the brain, the cerebrospinal fluid, and intracranial blood and their cumulative impact on intracranial pressure.

The nurse is caring for a client after a craniotomy and monitors the client for signs of increased intracranial pressure (ICP). Which finding, if noted in the client, would indicate an early sign of increased ICP? Confusion 2. Bradycardia 3. Sluggish pupils 4. A widened pulse pressure

confusion Rationale: Early manifestations of increased ICP are subtle and often may be transient, lasting for only a few minutes in some cases. These early clinical manifestations include episodes of confusion, drowsiness, and slight pupillary and breathing changes. Later manifestations include a further decrease in the level of consciousness, a widened pulse pressure, and bradycardia. Cheyne-Stokes respiratory pattern, or a hyperventilation respiratory pattern; pupillary sluggishness and dilatation appear in the late stages.

A client with stroke symptoms has a blood pressure of 240/124 mm Hg. The nurse prepares the prescribed nicardipine intravenous (IV) infusion solution correctly to yield 0.1 mg/mL. The nurse then administers the initial prescription to infuse at 5 mg/hr by setting the infusion pump at 50 mL/hr. What is the nurse's priority action at this time? a. Assess hourly urinary output b. Increase pump setting to correct administration rate to 100 mL/hr d. Keep systolic blood pressure above 170 mm Hg e. Monitor for a widening QT interval

d. Keep systolic blood pressure above 170 mm Hg

Dexamethasone is often prescribed for the child who has sustained a severe head injury. Dexamethasone is a(n) a) steroid. b) diuretic. c) antihistamine. d) anticonvulsant.

steroid

Dexamethasone is often prescribed for the child who has sustained a severe head injury. Dexamethasone is a(n):

steroid. A steroid may be prescribed to reduce inflammation and pressure on vital centers.

Dexamethasone (Decadron) is often prescribed for the child who has sustained a severe head injury. Decadron is a(n) a) diuretic. b) anticonvulsant. c) steroid. d) antihistamine.

steroid. Correct Explanation: A steroid may be prescribed to reduce inflammation and pressure on vital centers.

In understanding the nervous system, the nurse recognizes that the central nervous system is made up of: a) the brain and spinal cord. b) fluid that flows through the brain. c) a protective cushion for nerve cells. d) nerves throughout the upper body.

the brain and spinal cord.

The meningococcal vaccine should be offered to high-risk populations. If never vaccinated, who has an increased risk of becoming infected with meningococcal meningitis? Select all that apply. a) 18-year-old student who is preparing for college in the fall and has signed up to live in a dormitory with two other suite mates b) 12-year-old child with asthma c) 8-year-old child who is in good health d) 5-year-old child who routinely travels in the summer with her parents on mission trips to Haiti e) 9-year-old child who was diagnosed with diabetes mellitus when he was 7 years old

• 18-year-old student who is preparing for college in the fall and has signed up to live in a dormitory with two other suite mates • 12-year-old child with asthma • 5-year-old child who routinely travels in the summer with her parents on mission trips to Haiti • 9-year-old child who was diagnosed with diabetes mellitus when he was 7 years old

A 9-year-old diagnosed with neurofibromatosis is being evaluated for the presence of a brain tumor. What tests may be ordered to diagnose this condition? Select all that apply. a) Radiology b) Lumbar puncture c) Positron emission tomography d) Computed tomography e) Magnetic resonance imaging f) Electroencephalogram

• Computed tomography • Magnetic resonance imaging

A 9-year-old diagnosed with neurofibromatosis is being evaluated for the presence of a brain tumor. What tests may be ordered to diagnose this condition?

• Computed tomography • Magnetic resonance imaging Computed tomography is used for visualization of tumors, ventricles, brain tissue, CSF, hematomas, and cysts. Magnetic resonance imaging is also useful in tumor identification. Lumbar puncture is used to measure CSF pressure and collect CSF samples for laboratory tests. Electroencephalograms detect and locate abnormal electrical discharges produced in the brain. Radiology identifies the presence of fractures, widened skull sutures, calcifications, bone erosion, or skeletal anomalies

A nurse is providing care to a child with status epilepticus. Which medications would the nurse identify as appropriate to give in this situation? Select all that apply. a) Lorazepam b) Gabapentin c) Fosphenytoin d) Carbamazepine e) Diazepam

• Diazepam (Valuim) • Lorazepam (Ativan) • Fosphenytoin treating status epilepticus include lorazepam, diazepam, and fosphenytoin. Gabapentin and carbamazepine are anticonvulsants used to treat and prevent seizures in general.

The young child has been diagnosed with bacterial meningitis. Which nursing interventions are appropriate? Select all that apply. a) Identify close contacts of the child who will require post-exposure prophylactic medication b) Initiate seizure precautions c) Monitor the child for signs and symptoms associated with decreased intracranial pressure d) Administer antibiotics as ordered e) Initiate droplet isolation

• Initiate droplet isolation • Identify close contacts of the child who will require post-exposure prophylactic medication • Administer antibiotics as ordered • Initiate seizure precautions

A nurse is providing care to a child with status epilepticus. Which medications would the nurse identify as appropriate to give in this situation? Select all that apply. a) Lorazepam b) Diazepam c) Fosphenytoin d) Gabapentin e) Carbamazepine

• Lorazepam • Diazepam • Fosphenytoin Correct Explanation: Commonly used medications for treating status epilepticus include lorazepam, diazepam, and fosphenytoin. Gabapentin and carbamazepine are anticonvulsants used to treat and prevent seizures in general.

The nurse is using the pediatric Glasgow Coma Scale to assess a child's level of consciousness. What would the nurse assess? Select all answers that apply. a) Motor response b) Posture c) Eye opening d) Verbal response e) Fontanels

• Motor response • Eye opening • Verbal response

When assessing a neonate for seizures, which of the following would the nurse expect to find? Select all that apply. a) Tonic-clonic contractions b) Tachycardia c) Ocular deviation d) Elevated blood pressure e) Jitteriness

• Tachycardia • Ocular deviation • Elevated blood pressure • Jitteriness Correct Explanation: Neonatal seizures may be difficult to recognize but may be manifested by tremors, jitteriness, tachycardia and elevated blood pressure, and ocular deviation. Tonic-clonic contractions typically are more common in older children.

The young boy was involved in a motor vehicle accident and was admitted to the pediatric intensive care unit with changes in level of consciousness and a high-pitched cry. Which are late signs of increased intracranial pressure? Select all that apply. a) The sclera of the eyes is visible above the iris. b) The child's pupils are fixed and dilated. c) The child's toes are pointed downward, his head and neck are arched backwards, and his arms and legs are extended. d) The child's heart rate is 56 beats per minute. e) The child states that he feels a little "dizzy."

• The child's toes are pointed downward, his head and neck are arched backwards, and his arms and legs are extended. • The child's heart rate is 56 beats per minute. • The child's pupils are fixed and dilated.

A nurse is providing information to the parents of a child diagnosed with absence seizures. What information would the nurse expect to include when describing this type of seizure? Select all that apply. a) You might have mistaken this type of seizure for lack of attention. b) This type of seizure is more common in girls than it is in boys. c) The child will commonly report a strange odor or sensation before the seizure. d) This type of seizure is usually short, lasting for no more than 30 seconds. e) You might see a blank facial expression after a sudden stoppage of speech. f) Your child will probably sleep deeply for ½ to 2 hours after the seizure.

• This type of seizure is more common in girls than it is in boys. • You might see a blank facial expression after a sudden stoppage of speech. • This type of seizure is usually short, lasting for no more than 30 seconds. • You might have mistaken this type of seizure for lack of attention.

The nurse is reinforcing teaching with the caregivers of a child being discharged from the urgent care setting following a mild head injury that occurred in an in-line skating accident. What should the caregivers be instructed to do? Select all that apply. a) Observe and report any vomiting that occurs within six hours. b) Check the pupil reaction to light every 15 minutes for two hours. c) Wake the child every one to two hours to check level of consciousness. d) Observe for and report to provider any double or blurred vision. e) Administer acetaminophen for headache.

• Wake the child every one to two hours to check level of consciousness. • Observe and report any vomiting that occurs within six hours. • Observe for and report to provider any double or blurred vision.

The nurse is reinforcing teaching with the caregivers of a child being discharged from the urgent care setting following a mild head injury that occurred in an in-line skating accident. The caregiver should be instructed to do which of the following? Select all that apply. a) Administer acetaminophen for headache. b) Observe for and report to provider any double or blurred vision. c) Check the pupil reaction to light every 15 minutes for two hours. d) Wake the child every one to two hours to check level of consciousness. e) Observe and report any vomiting that occurs within six hours.

• Wake the child every one to two hours to check level of consciousness. • Observe and report any vomiting that occurs within six hours. • Observe for and report to provider any double or blurred vision. Explanation: The caregiver should observe the child for at least six hours for vomiting or a change in the child's level of consciousness. If the child falls asleep, he or she should be awakened every one to two hours to determine that the level of consciousness has not changed. No analgesics or sedatives should be administered during this period of observation. The child's pupils are checked for reaction to light every four hours for 48 hours.

A nurse is providing information to the parents of a child diagnosed with absence seizures. Which of the following would the nurse expect to include when describing this type of seizure? Select all that apply. a) You might have mistaken this type of seizure for lack of attention. b) This type of seizure is more common in girls than it is in boys. c) This type of seizure is usually short, lasting usually for no more than 30 seconds. d) Your child will probably sleep deeply for ½ to 2 hours after the seizure. e) You might see a blank facial expression after a sudden stoppage of speech. f) The child will commonly report a strange odor or sensation before the seizure.

• You might have mistaken this type of seizure for lack of attention. • This type of seizure is more common in girls than it is in boys. • This type of seizure is usually short, lasting usually for no more than 30 seconds. • You might see a blank facial expression after a sudden stoppage of speech. Correct Explanation: Absence seizures are more common in girls than boys and are characterized by a sudden cessation of motor activity or speech with a blank facial expression or rhythmic twitching of the mouth or blinking of the eyelids. This type of seizure lasts less than 30 seconds and may have been mistaken for inattentiveness because of the subtle changes. Absence seizures are not associated with a postictal state.


Related study sets

Chapter 12: Some Lessons from Capital Market History

View Set

Lifetime Fitness and Wellness Chapter 9

View Set